You are on page 1of 332

Study Anurag Mishra with www.puucho.

com

-~-J.
Ba\a ! Er. Anurag Mishra
-

Main & Advanced

www.puucho.com
Study Anurag Mishra with www.puucho.com

www.puucho.com
Study Anurag Mishra with www.puucho.com

/
/

Heat&
Thermodynamics
for JE E (Mo'. & Adva.ced)

by:
Er. Anurag Mishra
BTech (Mech. Engg.)
HBTI Kanpur

SHRI BALAJI PUBLICATIONS


(EDUCATIONAL PUBLISHERS & DISTRIBUTORS}
AN ISO 9001-2008 CERTIFIED ORGANIZATION

Muzaffarnagar (U.P.) - 251001

'.
www.puucho.com
Study Anurag Mishra with www.puucho.com

11 Published by: 'ti!


SHRI BAlAJl PUBLICATIONS
lEDUCATIONAL PUBLISHERS & DISTRIBUTORS)

6, Gulshan Vihar, Gali No.1,


Opp. Mahalaxmi Enclave, ,,
Jansath Road, Muzaffarnagar (U.P.)
Phone: 0131-2660440 (0), 2600503 (R) , ,,,
website: www.shribalajibooks.com
email: sbjpub@gmail.com

11 First edition : 2010


11 Third edition: 2013
11 Reprint : 2018

, ,

11 @AII Right Reserved with Author

11 Price: ~ 275.00 /

11 Typeset by :
Sun Creation
Muzaffarnagar

III All the rights reserved. No part of this publication


may be reproduced. stored in a retrieval system
or transmitted, in any form or by any means,
electronic. mechanical. photocopying, recording
or otherwise, without the prior permission of the
author and publisher. Any violation/breach shall
be takAn into legal action,

www.puucho.com
Study Anurag Mishra with www.puucho.com

Preface
I am very thankful to all those readers who gave warm welcome to Mechanics-l & II
and Electricity & Magnetism ~ I & II. These books have got whelming response to the
regular demand of the students which compelled me to complete this volume in very
short span of time. "Heat and Thermodynamics" is one of the most scoring topic in the
IIT.JEE syllabus.

This book will help the students in building analytical and quantitative skills,
addressing key misconceptions and developing confidence in problem solving.

I sincerely wish that this book will fulfill all the aspirations of the readers.
Although utmost full care has been taken to make the book free from error but some
errors inadvertently may creep in. Author and Publisher shall be highly obliged if
suggestions regarding improvement and errors are pointed out by readers. I am
indebted Neeraj Ii for providing me an opportunity to write a book of this
magnitude.

I am indebted to my father Sh. Bhavesh Mishra, my mother Smt. Priyamvada


Mishra, my wife Manjari, my sister Parul, my little kids Vrishank and Ira for giving
their valuable time which I utilized during the writing of this book and people of
Moradabad, who supported me throughout my career.

I am also thankful to Mr. T. Konda)a Rao, Mr. Abhishek Sinha (Ranchi), Mr.
Sunil Manohar, Mr. S.P. Sharma, Mr. Sudhir Sharma and Mr. P. Narendra Reddy
fortheir valuable suggestions in improving the book.

In the last, I also pay my sincere thanks to all the esteemed members of
Shri Balaji Publications in bringing out this book in the presentform.

Anurag Mishra

www.puucho.com
Study Anurag Mishra with www.puucho.com

..
, '

','

,,

','.
. ...
"

. .. ,

" ,
, ,

,
, .
.' ,

www.puucho.com
Study Anurag Mishra with www.puucho.com

~--- How to face the challenge?

Following are some doubts which arise in the mind of almost all the students
but may face them by taking some care.

1. / can not solve numerical because my concepts are notc1ear. In fact numerical
solving itselfis an exercise to learn concepts.

2. / can not study because I am in depression, I fell into it because 1 was not
studying! Depression is escape mechanism of people afraid offacing failures.
Failure is integral part oflearning.

I understand everything in class but can not solve on my own. WRITI NG work
is vital. It is a multiple activity. initially idea comes in mind then we put into
language to express it, we are focussed in hand eye coordination, eyes create
visual impression on brain which is recorded there. WRITING WORKS ARE
EMBOSSED ON BRAIN LIKE CARVINGS OF AJANTA CAVES.

4. In exams my brain goes blank, but I can crack them at home. Home attempt is
your second attempt! you are contemplating about it while home back. You
do nDt behave differently in exam YDUreplicate your instincts. Once a fast
bDwler was bDwling no balls. His cDach placed a stump Dn crease, in fear Df
injury he got it right. CONCEPTUALIZATION, WRITING EQUATION,
SOLVING, THEN PROBLEM GETS TO CONCLUSION!

S. I am an average student. It is a ratiDnalizatiDn used by peDple afraid of hard


wDrk. In their reference frame Newton's first law applies "if I have a
misconceptiDn I will continue with it unless pushed by an external agent
even I will surround him in my web Df misconceptiDn yielding zero
resultant." AVERAGE IS NOT DUE TO CAPACITY LACUANE BUT DUE TO
LACKOF DETERMINATION TO SHED INERTNESS.

6. A famous cliche "I do not have luck in my favour" PRINCIPLE OF CAUSALITY:


CAUSE OF AN EVENT OCCURS IN TIME BEFORE OCCURRENCE OF THAT
EVENT i.e., cause Dccurs first then event Dccurs. SHINING OF LUCK IS NOT
AN INSTANTANEOUS EVENT IT IS PRECEDED BY RELENTLESS HARD
WORK. Sow a seed Dfaspiration in mind, water it with passion, dedication it
will bear fruit, luck can give you sweeter fruit .

www.puucho.com
Study Anurag Mishra with www.puucho.com

Useful Tips I
1. Do not take study as a burden actually its a skill like singing and dancing. It
has to be honed by proper devotion and dedication.

2. Without strong sense of achievement you can't excel. Before entering the
competitive field strong counselling by parents is must. Majority do not
know what for they are here. No strategic planning, they behave like a tail
ender batting in frontof5teyn's bouncers.

3. Science is not a subject based on well laid down procedures or based on


learning some facts, it involves very intuitive and exploratory approach.
Unless their is desire and passion to learn you can not discover new ideas. It
requires patience and hard work, whose fruits may be tangible later on.

4. Some students realize very late that they are studying for acquiring skills and
honing them. Their is a feeling that they can ride at the back of instructor and
achieve excellence. Study comes as torturous exercise enforced on them and
theiris some mechanism that can take this burden of them.

• S. Science is not about gaining good marks, up to Xth by reading key points
good marks are achieved but beyond that only those survive who have
genuine interest in learning and exploring. Self study habit is must.

6. IF YOUWANT TO GAIN LEAD START EARLY.Majority of successful students


try to finish major portion elementary part of syllabus before they enter
Coaching Institute. Due to this their maturity level as compared to others is
more they get ample time to adjust with the fast pace. They are less
traumatized by the scientific matter handed over. For those who enter fresh
must be counselled to not get bullied by early starters but work harder
initially within first two months initial edge is neutralized.

7. Once a student lags behind due to some forced or unforced errors his mind
begins to play rationalization remarks like I am an average student, my mind
is not sharp enough, I have low IQ etc. These words are mechanisms used to
avoid hard work. These words are relative terms a person who has early start
may be intelligent relative to you.

Intelligence means cumulative result of hard work of previous years, that


hard work has eventually led to a development of instinct to crack things
easily.

www.puucho.com
Study Anurag Mishra with www.puucho.com

"--------------....;;;; .•
-

1. Temperature, Heat and the Eguation of State, Heat Transfer

Thermal Equilibrium and the Zeroth Law of Thermodynamics (1); Thermometers and
Temperature Scales (2); The Triple Point of Water (2); Constant-Volume Gas Thermometer
(3); Constant-pressure Gas Thermometer (3); Thermal Expansion (4); Thermal Stress
(18); Determination of the Coefficient of Expansion of Liquid (20); Calorimetry (25);
Ideal-Gases (33); The Ideal-Gas Law (34); Gas Laws (35); Microscopic View of
Thermodynamics (36); The Eqoipartition of Energy Theorem (38); Ideal Monoatomic Gas
(39); Ideal Diatomic Gas (39); Specific Heat Capacity of Solids (41); Maxwell-s Law of
Distribution of Velocities (42); HeatTr?nsfer (59); Conduction (59); Why is Conductive
Heat Flux (60); Radiation (75); Kirchhoff's Law (77); Temperature of the Sun (79);
Subjective Problems (100).
Level- I: Only One Alternative is Correct (II 4); Solutions (129).
Level.2: More than One Alternative is/are Correct (135); Solutions (139);
Leve!-3: Comprehension Based Problems (140); Matching Type Problems (144);
Assertion and Reason (147); Solutions (150).

2. Thermodynamics ~2-32Y
Thermodynamic Processes (153); Atoms and Moles (153); Ideal-Gas Processes (154);
Heat Transfer to a System (162); The First Law of Thermodynamics (162); Conversion of
Graphs (167); Mixing of Gases (186); Condition for the Molar Heat Capacity is Negative
(193); Analysis of Thermodynamic Cycles (213); The Second Law of Thermodynamics
and Heat Engines (218); Absolute Zero and the Third Law of Thermodynamics (219).
Level-I: Only One Alternative is Correct (227); Solutions (246).
Level-2: More than One Alternative is/are Correct (250); Solutions (259);
Level-3: Comprehension Based Problems (260); Matching Type Problems (268);
Assertion and Reason (271); Solutions (273).

l!,' Previous Year Problems with Solutions

• www.puucho.com
Study Anurag Mishra with www.puucho.com

GROW GReeN

Save NaTURe

www.puucho.com
Study Anurag Mishra with www.puucho.com

THERMAL EQUILIBRIUM AND THE ZEROTH thermal equilibrium with a third system are in
LAW OF THERMODYNAMICS thermal equilibrium with each other.
Two systems are said to be in thermal contact if heat
Example 1
transfer can take place
between the systems.
When two systems at A thennometer (system 3) consists of a quantity of alcohol in
the same temperature a glass tube. When the thermometer is in thennal contact
are placed in thermal with soft ice cream in a box (system 1), the length of the
contact with each other, alcohol column is 5 em. When placed in a freezer (system 2),
there is zero heat the alcohol column remains 5 cm long. What happens to the
transfer between them, ice cream when it is put into the freezer 7
Fig. 1.1
the systems are said to A cup of cooling coffee illustrates the Solution: The length of the alcohol column measures
be in thermal Idea of a thermodynamic system. temperature.
equilibrium. When
When they are in thermal contact, the ice cream and
two systems in thermal equilibrium are put in contact
thermometer come to thennal equilibrium. Similarly, the
through a diathermic wall, their variables of state do not
thermometer comes to thermal equilibrium with the freezer
change. A diathermic boundary between lWa systems allows
when placed inside. Since the alcohol column has the same
thermal interaction, whereas an adiabatic wall inhibits
length in the freezer and in the ice cream, the thermometer
thermal interaction.
is simultaneously in thennal equilibrium with both ice
For a given process, system choice can became that cream and freezer.
allows the three different ways to describe a cup of coffee.
By the zeroth law, the ice cream and freezer are in
(a) The system is the coffee. Its environment is the cup the
equilibrium with each other. There is no net exchange of
room and everything else.
energy and the ice cream neither melts nor freezes hard.
(b) The system is the coffee and the cup. Its
We may restate the zeroth laws as follows.
environment is the room and the rest of the world. (c) The
system is the coffee, the cup, and the room. The 1\vo systems are in thermal equilibrium with each other if
environment is everything outside the room. and only if they have the "ame temperature.
'TWosystems can be in thermal equilibrium even if they In this form, the zeroth law defilles temperature as the
are not in direct contacr. If systems A and B each are in quantity that must be the same in two systems in thermal
thennal equilibrium with a third system C, then A and Bare equilibrium.
in thermal equilibrium \'iith each other. This result is called
zeroth law of thermodynamics. Two systems in

www.puucho.com
Study Anurag Mishra with www.puucho.com

2 THERMODYNAMICS

Thermometers and Temperature Scales and the steam point (lOO°C for a mixture of water and
A thermometer relies on some physical property of water vapour at the boiling point). The two points (Li, Dec)
matter, known as a thermometric property that changes and (i,. lOOCC) are used to determine the slope and
with temperature. Operation of the familiar intercept of the line T = ai + b.
mercury-in-glass thermometer depends on the empirical
observation that the volume occupied by a fixed mass of
Example 2~
mercury v<Jries with temperature. The mercury expands
\•...
hen it is warmed and contracts when cooled. There are On the Fahrenheit scale the icepoint and steam point are 32"
many other such physical quantities such as electrical F and 217' F respectively. A Fahrenheit thermometer reads f.
resistance of a metal wire; the colour of a hal, glowing body, Determine the corresponding reading t of a Celsius
the pressure of a fixed mass of gas confined in a fixed thermometer at the same location.
volume, etc.
Solution: Fig. lE.2 shows the linear relation
A thermometer is a device with a measurable property between Fahrenheit and Celsius scale. The temperatures on
that varies in a known way with temperature. the two scales can be related by equating the ratios
In a mercury thermometer, the property is the length of 1-32"F t-DOC
~-----
the mercury column. As temperature varies" the length 212°F - 32°F lOO°C - D"C
changes in proportion to the change in temperature. To use
a thermometer, we put it in thermal contact with the system 212°F 100'C
to be measured. Once thermal equilibrium is established, the
f t
thermometer and the original system have the same
temperature.
With a specific mercury thermometer, we could measure 32°F
temperature in units of centimeters of mercury.
A common thermometer is calibrated by bringing it Fahrenheit Celsius
successively into thermal contact with a large thermometer thermometer
thermodynamic system at two different temperatures, Fig.1E.2
known as fixed points.
Following rules are observed in calibration: On solving for I, we get
1. A system is chosen in which a particular phenomenon 1= IBDoP t + 32°F = 9°F t + 32"F
always occurs at the same temperature, for example the lOD"C SoC
melting of pure ice when the pressure is 1 atm. Immerse the For example 37"C, the normal human body temperature
thermometer in the system and mark the reading. This on the Celsius scale, can be expressed on Fahrenheit scale as
reading is the first fixed point. 9°F
f ~-)( 37°C + 32°F
2. Similarly a second system is chosen in which a SoC
particular phenomenon is known to always occur at the = 9B.6°F
same temperature. For
example, the boiling of pure T The Triple Point of Normal
water at 1 atm pressure. T,=100.C Water melting point

Immerse the thermometer in Water exists in three 101


the system and mark the
T
Tj=O'
L,' L.,
, forms: ice, liquid water P" k Pa ,,
: Vapour
reading. This reading is the and steam, called
Triple:point solid.
second tlxed point. phases. lWo phases, 0.61 liquid l"nd vapour
3. Divide the interval solid and liquid can phas!ts are in
equili~rium
bet\veen two fixed point marks coexist in equilibrium at
Fig. 1.2 0.000.01 100.00
on the thermometer into a a temperature called the
number of equal parts. Each of these parts is called a melting point. Fig. 1.3
degree (0). Similarly, the liquid and
vapour (steam) phases are in equilibrium at the boiling
Consider a celsius temperature scale in which T is a
point. The sublimation point corresponds to the solid
linear function of L (length of mercury in a glass tube).
and vapour in equilibrium. There is a unique temperature
T=aL+b and pressure at which all the three phases are in
The two fixed points are the ice point (O"C for a equilibrium. At this state the melting point, boiling point
mixture of water and ice in equilibrium at 1 attn pressure)

www.puucho.com
Study Anurag Mishra with www.puucho.com

TEMPERATURE, HEAT AND THE EQUATION OF STATE, HEAT TRANSFER


- ---- 3

and sublimation point arc the same. This state is called the 273.16 K, in which the imercept in the Fig. 1.S is a K, called
triple point of water. absolute zero.
Triple point pressure = 6.11 x 102 Pa A degree on the Celsius scale is the same size as a degree
on the Kelvin scale, but the zeros of each scale are differem.
=6x lO~3 atm
= (,,,,him + 273.15 K
T
Tripk point tC1l1p<:n1tur~' = a.Oloe
In accordance with the ideal temperature scale the gas
ConstantNolume Gas Thermometer in a constam volume gas has a pressure P when it is in
Fig. 1.4 shows a thermal equilibrium with a system at temperature T. The
constant volume gas temperature T can be determined from
thermometer. A constant T = 273.16 P
volume is maintained by P"
raising or lowering the
right limb of the where Prp is the pressure of the gas in the same
Mercury
manometer tube. In this reservoir thermometer when in thermal equilibrium with water at its
case the pressure P of the triple poim.
B
constant volume of the Bath or
Flexible
Constant-Pressure Gas Thermometer
environment
gas is used to establish a to be measured hose Fig. 1.6 shows the constant pressure gas thermometer.
temperature scale. The The gas in the bulb has the same pressure as the pressure in
Fig. 1.4
linear relation between T the pressure control chamber. A scale behind the tube is
and P can be expressed as calibrated to read the volume V of the gas trapped in the
T=aP+b bulb as a function of the position of the mercury drop.
The slope and intercept of this relation are campmed Merc\Jry-in-glass
from two fixed points, ice point and steam point. thermometer Glass tuba

Experiments show that the temperature of a system


indicated by the constant volume gas thermometer is
independent of the particular gas as long as the gas is far p
Bulb V.t.P
from the liquefaction temperature. This means that the Mercury
intercept b is same for all gases. When we extend the lines of c.:: c=> drop
To
our temperature scales to P = 0, they all intersect at the same Variable-temperature Pressure vacuum
G" pump
point: T =-273.15"C. A new temperature scale may be Stirrer bath gauge reservoir

Fig. 1.6

T
Fig. 1.7 shows the graph of ratio VIVlpofthe gas trapped
in the bulb of a constant pressure gas thermometer as a
1000C~-._-- ,
function of the Celsius temperature t.
'T=aP+b
,
OOG ' P :L
PI Ps V.
(.1

.
(°1
T ...:;(::]
TS=373.t5K~
1 ;.
TJ=273,16K
TI = 273.15K --- ~ P
t{OC)
0\.
P,
Fig. 1.7
('I
Fig. 1.5 The slope of the straight line is
p = _d_C_V,_V_,,_l = _1__ dV
devised that has the advantage of requiring only one fixed de Vip dt
point. The ideal gas temperature scale is defined by
setting the temperature of the triple point of water to be

www.puucho.com
Study Anurag Mishra with www.puucho.com
4 ... ',' THERMODYNAMICS

Here J3is called the volume coefficient of thermal


or { =
x r -X0 x 1000C
expansion at constant pressure. The unit of P is XlOo -Xo
(Corl, it is analogous to compressibility.
In modern thermometry,
K =(~:) T
-=-
TIp
X
Xtp
The coefficient of expansion P is the same for all gases.
X
p= I
273.16
(Corl .."or T=-xTrp
X'P

p=_1 "V,
for helium at low pressure
=[XX~ x 273.16] K

VII'M ~ For a mercury scale,

{II~ =[ Lr -Lo
LillO - Lo
x 100] °c

or T =[~ L,p
x 273.16] K

Setting V = 0 and using ~ For a constant volume thermometer,


p = _1_
273.16
(Corl, we have {gas =[ PI -Po
PlOO -Po
X 100]OC

or T = [.!:!-
L,p
X 273.16] K

•. For a constant pressure thermometer,


We use the value
tIP = O.OFC. t
gas
=[ V
Vr -VoV
tOO-o
x 100] °c
I
to=-p+tlp

= (- 273.16+ O.OI)°C
or T =[~ V,p
x 273.16] K

to = 273.1S"C ~ For a platinum resistance thermometer,


Thus we have obtained the temperature at which the R,
= ----x
tplatinum
[R
-R-Ro I 00]0 C
volume of such a thermometer goes (Q zero. Strikingly, we lOO o
use a single fixed point to obtain it.
General Method of Measuring Temperature
Suppose a thermometric property is proportional to
or T =[~ R,p
x 273.16]K

temperature. At ice point (DoC), steam point and unknown ~ If a faulty thermometer has measurements X'I' X' 0
temperature of the properties are Xo• X1OO' Xt respectively. and X'tOO'then
Then Xt-Xo X1-Xo
=
Xr = Xo{l + at] ..,(I) X\oo-X'o X100 -Xo

XlOO = Xo[l + 1000.] ... (2) THERMAL EXPANSION


From eqn. 0), Fig. 1.8 (a). (b) and (c) show molecules of solid, liquid
Xr-Xo=Xoat ... (3) and gas in their random motions. Atoms in a solid always
From eqn. (2), have the same neighbours. held near home by forces
represented here by springs. The atoms are essentially in
XlOO -Xo =XolOOa ... (4) contact with one another. A rock is an example of a solid. It
From eqns. (3) and (4), can stand alone because of the forces holding irs atoms
XI - Xo { together.
-~--=-
XlOO - Xo 100

www.puucho.com
Study Anurag Mishra with www.puucho.com

TEMPERATURE, HEAT AND THE EQUATION Of STATE, HEAT TRANSfER 5

Atoms in a liquid are also in dose comart but can slide neighbour lies between 1"1mill and rl max' the average
separation is r\ a" The U(r) is not symmetrical, it is flatter to
the right at larger r-values. At higher temperature the total
energy E is higher; the particle spends morc time at r.values
towards less steep portion of the curve. The average
separation r2a, increases at higher temperatures. Because
r2"" > r1.", the average separation ofthe atoms or molecules
in the solid increases with increasing temperature.
Linear Expansion
---~ Consider a one-dimensional solid of length I at
temperature T. If the temperature changes by l'.T the change
(.J (bJ (oJ in length is directly proportional to (1) the original length 1,
Flg.1.8
(2) the change in temperature, i. e.,
6.1 = ul6.T
over one another. Forces benveen them strongly resist \vhere a is coefficient of thermal expansion, a
attempts to push them closer together and also hold them in proportionality constant.
dose contact. Water is an example of a liquid. It can flow, At a particular temperature
but it also remains in an open container because of the 1 ill 1 dl
forces between its atoms. a= lim --=--
.1T->O 1 ilT I dT
Atoms in a gas are separated by distances that are
considerably larger than their diameters, they freely move Area Expansion
about throught the container enclosed they are. Gas must be A two-dimensional object, such as a thin metallic plate,
held in a closed container to prevent it from moving out changes area when their temperature is raised or lowered.
freely. For an isotropic material the area expansion may be
expressed in terms of the linear expansion coefficient a.
Most substances expand when their temperature is
raised and contract when cooled. There is exception to this Consider a rectangular area of dimensions II and 12, The
statement: water contracts wilen its temperature is increased area is
from o"'C to 4~C.Thus water has its minimum volume, hence A = It x 12
maximum density, at 4'C. dA = I dl2 + I dlt
Atoms in solids are in dose contact; the forces between dT IdT 2dT

them allow the atoms to vibrate but not to move freely. =11(al2)+ (ulj)12
These forces can be thought of as springs that can be
= 2aI}12
stretched or compressed. An individual molecule's motion
can be modelled as a point-like particle oscillating in a If a is constant over the temperature range of under
potential well caused by the interatomic forces, which is consideration, this equation can be integrated
parabolic for a Hooke's law SPring( U(x) = ~ kx
2
). The mass f Ai
A,
dA = 201;12 f 'J
1;
dT

oscillates in simple M = (2a)A6.T = I3AtlT


U(r)
hannonic motion Parabola where ~ is coefficient of superficial expansion,
between maximum ,, nvice the coefficient of linear expansion.
and minimum , rlmax:/
,, Volume Expansion
positions. The ,
minimum of the E2 Objects expand along all dimensions as shown in Fig.
symmetrical 1.10; their areas and volumes increase with temperature as
potential energy well as their lengths. If we cut a hole in a metal plate, the
High ,' ...._ ....;
function locates the
average position.
temperature. T 2 ,, ,
'[ill-- -- -- -- ----, , ,,.::."._----,'-- ,, :
The actual potential
Low
temperature, T1 , '0---",. :
,, ,, , , ,,
,
,,
,
energy curve is not
Fig. 1.9
:, " ...' :, : ,-,'
, ,
symmetrical, as ~ _-_
.. ..... _.' t. ••••.... ,','
shown in Fig. 1.9. The variable r is the separation benveen a (.J (b) (oJ
panicle and its nearest neighbour. At temperature T} the
Fig. 1.10
total energy is £} and its separation from its nearest

www.puucho.com
Study Anurag Mishra with www.puucho.com

6 -----,,'1..: ...}. THERMODYNAMICS


remaining material will expand exactly as it would if the Effect of Temperature on Density
plug were still in place.
Density of a substance is
Consider a rectangular parallelopiped of sides II' 12• 13, m
11=-
V=I,lzI3 V
For a temperature change
or ~=~= V
dV dt] dll dll P Vr V(I+y6.T)
- =1,12 -+ I,-I, +-1213
dT dT dT dT I
=
Now dl] _ al dl2 _ al dll _ al 1 + Y liT
dT - 3. dT - 2. dT - 1

dV
or PI =p(l + Y tiT )-1
- = l112(al3) + 11Cul2)/J + (al,)l213
dT ~p(l-y"TJ
= (3a)I}12/3 Effect of Temperature on The Reading of a Scale
If a is constant over the temperature range under Consider a scale designed (Q give correct reading at
consideration, temperature T. When the temperature rises, due to linear
expansion of scale the scale reading will be less than the
V! dV = 3aIl/213fT! dT
f VJ 1; actual reading. Consider a unit reading on scale, it will
become (l + ClL\T). Thus
,),V = (3a)VdT
Actual reading =- Scale reading (l + a I1T )
~v = y(V~T )
Similarly when temperature is decreased, the scale will
where "( is the coefficient of volume expansion,
contract and the actual reading will be less than the scale
which is three times the coefficient of linear expansion.
reading.
Effect of Temperature on the Time Period of a
Pendulum o x,
X expands 10X'
.• XX'
- - -
X' contracts to X

XX
Time period of a pendulum is given by __ 'I 'I
T = 27tH I- -I
Actual reading
otT
I"
Scale reading
alr (r>T)
-I I- _,
Scale reading
alr (r<T)
AR=SR AR>SR AR<SR
Fractional charge in time period,
dT Idl Idg Fig. 1.11
-=-----
T 21 2g
Effect of temperature on Buoyant force
If acceleration due to gravity is uniform, dg = 0 The buoyant force on a completely submerged body of
g
volume V in a liquid of density a is
dT Idl
-=-- B =- Vag
T 2 1
dl 1-10 With the increase in temperature,
As I = lo(l+a.6.T), - =-- = atJ.T V' = V(l + YsI1T)
1 10
a
Thus we have and a'~------
(l + YLI1T)
dT Idl I
- =-- =-C( tiT Hence new buoyant force is B' = V' a'g
T 2 1 2
a
If temperature is increased, the length of pendulum will =V(l + Ys .6T) -----g
increase, consequently time period will increase. The (l + YL tiT )
pendulum clock will run slow. The time lost in rime t, when Thus B' = V'a'g =(1 + YstiT)
actual time period is T and apparent time period is T " is B Vag 1 + YLI1T
M =(~:)r The apparent weight of a body in a liquid is
Wapp. =W-B=W-aVg
Similarly time gained when temperature is decreased is
also given by Apparent weight when temperature increases,
W' 'PP. =W-("J'V'g

www.puucho.com
Study Anurag Mishra with www.puucho.com

TEMPERATURE, HEAT AND THE EQUATION OF STATE, HEAT TRANSFER 7

Generally Y5 > YL

B' > Band W' apr. > W.rr


Thus with the rise in tempertlture buoyant force The clock wil1100se
decreases and apparent weight will increase. ilT =N(7' -1'0)
Effect of Temperature on The Reading of a 24 x 60)< 60 :t aloilT
Barometer with a Brass Scale
Let true height of mercury column and density of
.
=-----
27t r;; g
\ g
F
\ g
mercury be ho and Po respectively, and 11 and p be the
=12x 60x 60ut s
corresponding values after temperature change t:..T.
Neglecting any significant change in atmospheric pressure, Method 2:
we have
hoPo =hp =p
T = 2J't f!.
\g
= K ,'7 (g is constant 1

or 110 =[~} d1'


-=--
l'
T' - T
2 I
1 dl

1 L' - L 1
r 1 or -- =--- = - (1 + MT)
where -=------ T 2 L 2
Po (l + "fllg 8.'[' )
1
,-\T =- MT
_ 11 2
Thus h 0-
(l + YIl~AT) Time lost in [\'ienty-four hours
If is coefficient of expansion of brass scale on, the 1
(l =-x24x60x60x(l{ilT)
barometer, 2
~ The clock will become slow if
T1i",,1 > Tim1i"I' i. e., ilT > 0, will loose time.
3 _ The clock will bewme fast if 1"t;",,1 < T,mhal'
Example
i. e., ilT < 0, will gain time.
At a temperature 'fo, the pendulumof a clock has length Land
The loss or gain of time is independent of time period
the clock then goes accurately. The coefficient of linear 1'.
expansion of the pendulum material is a = 1.85x 10-5• How 4 _
Example
much the dock wil/loose or gain in nventy-four hours if the
ambient temperature is 1O"C higher than To?
A steel rod with a cross-sectional area A is set lengthwise
Solution: Time period of pendulum is between fwo rigidly secured massive steel plates. With what

To 211:
II;;
I~
force wiII the rod press against each plate if the temperature of
= the rod is increased by ~T? The Young's modulus a/steel is Y
\ g and the coefficient of linear expansion of steel is a.
Number of oscillations performed in nventy-four hours,
Solution: If the rod is free to expand, the change in
N=24x60x60
its length is L - La = aLoilT , From the given conditions,
2rr..•.jloig boundary' of rod is rigid, it cannot expand. Hence the plates
When the temperature changes by t degrees, the new will exert compressive stress corresponding to a
length is L =Lo(1 + MT) compression of L - La. From the definition of Young's
Change in time period of oscillations of the pendulum is modulus,

1'-1'0 =2n[~~ -fjJ or


y= FL
AM

F=YA-
t.L
27t L ~Lo L
= g (i: + if," =YAu..1T
\g fgc
_lIL-Lu
-g- ..,I1 ,'g0

I www.puucho.com
Study Anurag Mishra with www.puucho.com

8 THERMODYNAMICS

Example d
--!. = (l + 3a6T)
d,
An optical engineering firm needs to ensure that the d _ d,
separation between two mirrors is unaffected by temperature 2 - (l + 3M T)
changes. The mirrors are auached to the ends of two bars of
different materiaLs chat are welded together at one end as = dj (1 + 3a6Tr'
shown in Fig. 1£.5. The surfaces of the bars in COntact are "d, (I - 3aAT)
lubricated. Show chat the distance 1 does not change with
temperature if aliI'" uzl2• where uj and u2 are the Example
respective thennal coefficients of temperature. Also detennine
the required lengths IJ and fz, in terms OfU"U2 and 1. An aluminiUm cup of volume V, is filled with a liquid at
Mirrors temperature T. How much liquid will spill out if the
temperature of both, the cup and liquid, is increased by 6T?
~I Assume coefficient of expansion of aluminium aa is less than
coefficient of expansion of liquid y.

Lubricaling Solution: The change in the volume of the cup,


oil
'1-.----1, 6V, =3aaV,!J.T

Fig_ 1E.5 The change in volume of the liquid, llVI = yV, !J.T
Nare that the original volume of liquid is same as the
Solution: At any temperature t, the lengths of the two original volume of cup.
bars are The volume of liquid that spills is
1'1 =1](1+u1t), 1'2 =12(1+(120 ... (1) flVI -6.V, = (yV, -3aaV,)6.T
From the given conditions. •. Volume of liquid after temperature change,
['2-1\=[ and 12-11=[ ...(2) V'I = V(l + yl!J.T )
From eqos. (1) and (2), Volume of container after temperature change,
1'2 -l'1 = (12 -iI) + l20.2t -I, o.lt V', = V(l + y,llT)
12u2 = 1,0., ... (3) Change in volume of liquid relative to container,
12 Ct.) V'I - V', = V(YI - y,)llT
-"- ... (4)
I, 0.2 YI - Yc is called apparent coefficient of expansion. Level
From eqns. (2) and (4), of liquid in the container will rise or fall depends on whethet
YI >Y, or 11 <yc. If YI ='(" the level of liquid in the
I) = -~~-
10.2 lu,
12 = -~~_ container will not change on temperature change.
u2-a) 0.2-0.)

Example
Example
A piece of metal floats on mercury. The coefficients of
The density of a substance is d) at temperature Tj" If expansion of the metal and mercury are 11 and '(2
coefficient of linear expansion is a, find the density at respectively. If the temperatures of both mercury and metal
temperature T2. are increased by flT, by what factor the fraction of the volume
of the metal submerged in mercury changes?
Solution: Since mass of a sample is constant with
temperature change, Solution: Let the total
V)d, = V2d2 volume of metal in air and mercury be
dt V2 V and V. respectively, p and C1 be the
-=- ...(1) densities of metal and mercury
d2 V)
respectively. When the metal floats in
Volume after temperature change 6T = T2 - T" equilibrium,
V2 = VI(l + 3a6T) ... (2) Weight of metal'" Buoyant force of the
"9
Eqns. (1) and (2) yield liquids
pVg = crV.g ... (1) Flg.1E.8

www.puucho.com
Study Anurag Mishra with www.puucho.com

• TEMPERATURE, HEAT AND THE EOUATION OF STATE, HEAT TRANSFER 9

Fraction of the volume submerged, p'


1'.::: d =~(l+IilT)
V, P
ol=V"=-;;
f ... (2)
:::f. (1 + yilT)
When the temperature changes, the fraction of volume Expansion of solid has been ignored.
submerged changes as densities change. When the solid is completely submerged,
~I, =1,' -I, =1,'_1 f'., =1 =1,0 + y~T)
I, I, I, or ilT=--'
I-I
a
p'
=-x--l ... (3)
y f.
0' p (b) I" = I, (l + '(AT,)
Density of liquid decreases with temperature increase. f,~:::f,(l + I!:J.T2)

p' = P I" =(I+y~T'J


1 + Yl~T f'2 1 + I!:J.T2
a
d= On solving for I, we have
1 + Y2t1T
y::: f'l -f'2
which on substitution in eqo. (3) yields f521lTI - f'l iJ.T2
t1f, l+Yzt:.T 1
p f' r'
Is I+YIA1' (e) A; I, = and .<
=
d
_(1+Y2Sr)-(l+YI~T)
(1 + Ytl1T ) =( P
1+111';'1'
JO+Y26T)
0
(12 - Yl')D.'1'
0+111\1') =1[1 + Y2~TJ
I + Ylc"T
:(12 - 11 )l<.T(l - YlaT )
'::«"12 - "II )!'>.T
If Y2 > /1' the solid sinks.
If Y2 :::11' no effect on submergence,
[As 1 =(1+y1t.T)-1 ~(l-Ylt:.T)
1 + Yt!':..T If12 <II' the solid lifts up.

and 11 x 12 is a small number, it has been neglected.]

Concept: Let their initial deruities be PI and Pz' The


There are two rods of lengths IJ and 12 and coefficients of
fraction of the metal inside mercury =.!l = f. thermal expansions (l.J and (1.2' What should be the relation
p,
con/leeting II' 12,(1.1 and U2 so that the difference of their
dt = dPt _ dpz. We have dp =-p.y.dT
f PI Pz
lengths remains a constant with temperature variations ?
df
hence, 7 =(y, -y,),dT Solution: Let the temperature changes through dT.
The respective length increments of rods will be given by
dl1 =11.u1.dT and dl2 =12,(l.2,dT. 'If the increments in the
9 _
two lengths are equal, their difference remains a constant',
11.12 =a1.o.2
(a) In the previous example, determine the temperature rise
at which the solid i.s completely immersed; neglect expansion .E:><arnpJe 11 ~
of solid? (b) If fs) and f'2 are the fractions submerged at
temperatures, !:J.TIand llT2, detennine y of the liquid. (c) A linear scale of brass (al) measuring accurately at To °c, is
Under what conditions will the sphere sink, lift up considering used to measure the length of a wooden pencil (a. 2 > Ul) at
expansion of solid? (TJ > T2 re.
Find the percentage error in the reading shown.

Solution: (a) From previous problem, Solution: When the scale itself expands, observed
reading is smaller than true reading. The fractional changes
f. :::£.
a ~I ::: (t. dT is given by, therefore, the % error in the reading
When the temperature is increased by llT, will be :

www.puucho.com
Study Anurag Mishra with www.puucho.com

10 THERMODYNAMICS

Solution: The period t 'X J(Geometricallength)


dt 1 dl
~=2'T
d, 1
or -=-a.dT
A unifonn di$k is spinning in free space with an angular , 2
veloell)' ro. It has a mass m and radius r. The coefficient of
thermal expamion is a. Now the temperature is increased
Example
through dT. Find the fractional changes in the following
parameters: (a) Its radius of gyration (K), (b) Its moment of An aluminimum 'can' of cylindrical shape contains Vocm3 of
inertia (J), (e) Its angular velocity, (d) the speed a/its rim (v), water. The area of the inner cross-section of the 'can' is
(e) Its angular momentum (L), (f) Its circumference, (g) Its Aocm2• (All measurements refer to To "'C) Find the rise in the
geometrical radius, (h) Irs area. and COIts rotational kinetic water leve~ if the temperature increases to (Tj "'C > To). a AI
eoergy(Ek}. ',,' and Y waltr are given.

Solution:
When a disc spinning in a free space Solution: Height of the water column inside [he
expands thermally, is conserved angular momentum (L). vessel = h =-
v
A
Hence = dL '" 0 Since the disc is expanding thermally, all its
L errors
linear dimensions will have a fractional increase a.de. i.e.,
~ =~ (where L is the circumference = 2rrr) = a. dT. and

Not only the linear dimensions but also the radius of


gyration (k) has the same fractional increase elK = a. dT. Its
/(
Example
area, being L:2 in its dimension will increase by a fraction
y. dT = 20.' dT. Since the mass remains a constant, dm = a An aluminium cube floats OT! mercury at temperature 300 K.
m
hence, the moment of inertia will also have proportional How much farther will the block sink if temperature is raised
increase 2adT to 350 K. PI/g =13.6 glee, PA]=2.7 glce,
'(Ilg = 1.B><IO-4/",C, Y AI = 23>< w-6/"C?
Since the angular momentum is conserved,
dL dJ dw
-=0=-+- Solution: Method 1: Let V be the volume of the
L I w cube; A the area of the cube face; h is the extent to which it
dw dI is submerged. In equilibrium,
Hence, - = -- = (-20.. dT)
w I
PAl Vg =PHg gAh ... (1)
for the rotational kinetic energy for instance,
L' As V = L3, A = e, from eqn. (1) we obtain
Ek -. Krot. = 2/' h = PAlL
dL dl PH,
E. -+K
Tot.
=2---
L I
With the change in temperature all three quantities in
=-2a.dT the expression for h change.
v =rw. PHg (Ah}g
dv dr dw
-=-+-
v r w
= a.dT - 2a.dT =-a.dT ......----F
Example

A pendulum rod has a material of thennal expansion


Fig.1E.15
coefficient a. Find the fractional loss in its period, when the
temp'erature rises through dT.

www.puucho.com
Study Anurag Mishra with www.puucho.com

~ TEMPERATURE, HEAT AND THE EQUATlDN DF STATE, HEAT TRANSFER

8h
------- On substinuing expression of 11 in eqn. eZ), we get
11

£111 = -- .1p AI
i"PAI j,h =h' -h = PAl L(y - 2u)fl.T
L P"
=--.1.P ..•.
1
P Hg Same expression as obtained before.
First we consider the change in density r AI
Example 16V--
M
PAl"" V
The area A of a rectangular window is LILz. Its coefficient of
ll.PAI
= dPAI:-.V linear expansion is a. After a temperature rise t.T, side LI is
dV longer by j,ll and side L2 by M,2' What is the change ill area
M 1'.V of the rectangular window?
=--C.V=-PA-
V' V
Solution: Initial area of the window A1 =L\L2.
l'.V = 3aVl1T After expansion, area of the window,
6.PAI =-3uAJ PAI1'.T
Similarly for mercury,

Finally, M ...= eJL6T


The expres!:iion for t..h becomes Flg.1E.16

L ( -3UPAlj,T ) --,-p,,,L ( -YP 6T


~h =-- )
Hg A2 = (I'I + UI )(L2 + M,2)
PHg Pllg
= LIL2 + LlfiL2 + L2AL1 + M,I M,2
+ PAl CoL:..T) + LIM,2 + L2MI
"'" LIL2
PlIg
when the term M1hL2 has been omitted because it is
= r,-\LL(y~2a)6T much smaller in comparison to other tenns.
P lIg Thus the change in area,

= ( 2.7
13.6
J (20)[1.80 x 10-4 - 2(3 x 10-6)] x SO
IV. = A2 - Al
If Sf is temperature change, then
= L1b.1.2 + LzM1

= 3.45 x 10-2 ALI = u.LlhT and hL2 = uL2!1T

Method 2: where a is the coefficient of linear expansion.


PA1L3g =PIIg(L2h)g =P'llg ([,2 h')g Hence IV. = a(L1L2 + LIL2) t.T
= 2aLIL2!1T
p' Hg > L' > h' denote density, length and depth submerged
at increased temperature.

~ =(~,J'(;J(l) The temperature of a liquid in a barometer is changed by t.T


and the pressure is constant, The liquid's height is h, y is
['=[(1 + nAT)
coefficient of volume expansion. Neglecting the expansion of

cr' =( 1 + ~AT J
glass tube, find the change in height of the

Solution: The change in volume


liquid.

of liquid
h' 1 + "fl'..T If cross-section area of the tube is A and initial
l'.V = "'IVt\T.
-=
h (1 + (ciT)2 height is h. V = Ah. Since the cross-sectional area of the tube
~1 + (y - 2u)l-\'/'
is unchanged, therefore,
tt.V = Att.h =)'Ahtt.T
Hence h' - 11 = hey - 2a)AO ... (2)
hh = yhtJ.T
From initial condition of equilibrium,

h =(~:;:JL ... (3)

www.puucho.com
Study Anurag Mishra with www.puucho.com

12 THERMODYNAMICS

Exarnple Solution: The liquid is in hydrostatic equilibrium.


p,ghr =Pogho
The height of liquid in a tube is ho at temperature To. If y is . where, p ( is density of liquid in hot bath, Po is density of
the coefficient of expansion of liquid Qnd a is the coefficient of liqui? in cold bath.
expansion of glass, find the change in the height of the liquid .. ' Volumes of a given mass M of liquid at temperatutes t
after a temperature rise !J.T. and aoc are related by
Solution: The height of the liquid in the tube Vr = Von + it)
Volume of the liquid Since PrVI =PoVo
=
Cross- sectional area of the tube PoVo
p{ =--
With the increase in temperature the cross-sectional v,
area and the volume occupied by the liquid both increase.
=
Po
Let V be the original volume of the liquid. Volume after
expansion is V + yV.6.T. Let A be the area of the tube. Area
n + yt)
after expansion is A + 2aA.L\T. The height of the liquid in the Since
tube after temperature increase is
h = V(l +y~T) p,
A(l + 2eu.T)
= hon + yt)
Original height of the liquid ho = VI A. which on solving for y, yields
h = 1 + YAT h (hr - ho)
y=----
1+2a.6T 0
hoi
The change in height is •. With this experiment the correct value of y of liquid
h-h
o
=( 1l+y!'J.T
2a.6T+
-l)h
0
can be determined without the knowledge of y for the
container. If we measure y of liquid in a vessel by
change in its volume, what is actually obtained, is y
_ (y-2a)&T h
relative to the material of which the container is
- 1+2att.T 0
made .
•. Expression for h - ho can be further simplified.
1 = (1 + 2att.T rl Exarnple
1 + 2att.T
zl-2a..&T A bar with a crack at it.~ centre buckles as a result of
temperawre rise of 32°C. If the fixed distance Lo is 3.77 m
h -ho = (y-2a)(1-2a..&T)l'.Tho
and the coefficient of linear expansion of the bar is
Ex~rnple 2.5 x 10--6joc, find the rise x of the centre.

In Fig. 1E.19 shown, left ann of a U-tube is immersed in a hot


water bath at temperature T, and right arm is immersed in a
bath of melting ice; the height of manometric liquid in
respective columns is hr and ho' Determine the coefficient of
e.:'(pansion of the liquid.
Water at Fig.1E.20
temperature
t"C Meltingice
Solution: We consider one-half of the bar. its
original length 10 = Lo/2. and its length after temperature
increase, 1 = 10 + alol'.T. From Pythagoras theorem,
x2 =12 -lJ =IJn + a..&T)2 -l~
z lJ[1 + 2a.l'.n -15 = 2/5att.T
Hence x = lo.J2att.T

_._- Fig.1E.19
= 3.77 ,/2(25
2
=7.SxlO-2m
x 10-6)(32)

www.puucho.com
Study Anurag Mishra with www.puucho.com

TEMPERATURE, HEAT AND THE EQUATIQN Qf STATE, HEAT TRANSfER

Example 21 _
--------- 13

l'
,
2d
Tile temperottlre compen.mted pendulum L~ designed to Iron
D, ~'
compensate for the c1lQllge in length due to temperature rise.
Copper
Fig. 1E.21 -,1Io\\'s one such pendulum. It consists of an' I" 10
L~oscelestriangular frame as shown in rile figure. The (.) Ibl
pendulum is supported at mid-point of sideAB, and it remains Fig. 1E.22
horizolltal. Determine the ratio 11112'.~othat the lengtll of the
pendulum remains same at allrernperawres.
Solution: The length of the leaves at temperature
T + !iT, will be
I' = R' 0 = 100 + ucuf:.T) ... (I)
t = R"O == 100 + (l~~AT) ... (2)
Where 1', R', I", R" are lengths of copper and iron
leaves respectively.
Also, (R' - ROO) = 2d ... (3)
c
Subtracting second equation from first, we have
Fig.1E.21
(R' -R")O ==(u{'u -urello.1T
0== (ucu -uf~)loll.T
Solution: According to condition of the problem, or
2d
-height of the isosceles triangle ABC is unchanged. The
doned lines show configuration after a temperature rise. Now we add the eqns. (1) and (2).
Increase in length of rod AB, OCR' +R")=2lo+(ucu +uhllOSf
D.ll =llU],iT The mean radius of curvature is
Thus A4.'=~llalL\T
R = R' + R" 210 + (ucu + (lfc)lotlT
2 2 29
We drm'll a normal from A to A'C (the final length of _ 210 + (ucu + ul'e )10ll7'
AC). Increase in length of AC is A' N. 2(ucu -o.Fe)loliT 2d
A'N =120.2liT _ [2 ..•..
(ucu + ufc)llnd
Considering increase ill angle 0 to be very small, (o.cu - uf~).1T
A'N:::-M' cos 0 As (ucu +ufc)«2,
I, 2d
Where cos 8 =- R~
21, ((leu -ufe).1T

Thus, we have /2o.2li7' =(~ Ijo.]liT )(;:J Ex~",F"",


,...--,
23
Hence !!. = 2/0. 2 Fig. 1£.23 shows a temperature compensated pendulum
12 \ o.} clock. A light rod AB supports a heary bob by means of a
regulating screw S. Differential expansion of rod and bob is
Example 22~ used to keep the centre of oscillation of the pendulum at a
fixed distance below the point of suspen8ion. Find the
A bimetallic strip is made by bonding thin 8traight leaves of dimensions of the pendulum if it is to tick off seconds.
copper and irOIl together at temperature To. The thickness of
ho = Length at O'C
each leaf is d and the length of each is La. At some higher (L, = Coefficient of
temperanlre the bimetallic bends ill an arc so that t1Vo linear expansion
electrical cOllfacts tOl/eh and an electrical control system is fO = Radius at DOC
aetivuted. Show that the bimetallic strip bends into an arc of (J~ = Coefficient of
linear expansion
angle 8 given by
I,
8=-(0.,. -u<")(T~Tr.)
2d"re v Fig. 1E.23

www.puucho.com
Study Anurag Mishra with www.puucho.com

14 THERMODYNAMICS

Solution: As the rod is light, we can take centre of


COS8=1
/2 +12_12
2 J
the bob as the centre of oscillation. 211/2
The effective length of the pendulum is
or 211/2 cosO = 112+ Ii -Ii ... (I)
l=h-r
Differe~tiating eqn. (1), get
The height and radius at temperature Tare
ZII cosOdl2 + 212 cosOdl1 -21112sin8d8
h = ho(l + u1t.T)
= 21jdlj + 2l2dl2 -21JdIJ ... (2).
r = roO + 0.26.T )
The changes in length of respective rods due to
Then l=ho-ro+(houj-rouz).6.T
temperature rise are
The length I will be independent of temperature if
dlj =llajl!.T
haul-raUl =0 d12 = l2ajAT
Also lo=ho-ro dlJ = lJa2l!.T ... (3)
On solving these equations simultaneously, we have For an equilateral triangle 11 = 12 = IJ = I and 8 = 60°. So
0.210 from eqns. (2) and (3) we get
h ,_- 0.2-0.1 212 cos600u1l!.T + 21 2 cos60oulM _ Z12 sin 600do

alIa = 212u]AT + Zl2uIAT - 2l2a2l!.T


and TO =---
0.2 -a]
or
al a]
-~T +-8T--dO = u]l!.T + a1l!.T - a2l!.T
..J3
If the clock is (0 tick seconds, time period is 2 seconds. 222
J3 =2a .:iT - u .:iT
T=2rtft =2 or u]l!.T --de l
2
2

or (a2 -al)AT =- dO
J3
10 =..f.. 2
n'
d8=2(a2-aJ).:iT
or
J3
Thus the dimensions of the pendulum are
0.210
, Example
h ,- _
0.2 -(1,
«z(g/it)
0.2 -U1

2 The apparatus shown in Fig. 1£.25 consists of four glass


UI(g/7t )
and TO =---- columns connected by horizontal sections. The height of the
Uz -0.1 two central columns Band Care 49 cm each. The two outer
columns A and D are open to the atmosphere. A and Care
Example maintained at a temperature of 95°C while the columns B
and D are maintained at 5°C. The height, of the liquid in A
Three rods A, Band ClOT an equilateral triangle at DOC.Rods and D measured from the base line are 52.8 cm and 51 cm
AB and Be have same coefficient of e.xpan.sionell and rod AC respectively. Detennine the coefficient of thermal expansion of
has 0.2, ljtemperature a/the system is increased to PC, what the liquid.
is the change in angle e fanned by rods AB and Be?
.. ,
''
T T T
.• A'' '
.0 ••••••••

..
, '' E ::
o E
, o 5
:0
, .
/'
'
"l\ ~.
A
9S"C ~*
<:QuB

S'C
C

9S"C S'C ~

.'--..," '" '


'

--~
''
'
j
B 12, a, C
Flg.1E.25
Fig. 1E.24
Solution: We start from [he liquid surface of column
Solution: After increase of temperature, the A, as we move along the tube to liquid surface in column D.
triangle will not be equilateral. Let the lengths of the three When we move down, pressure increases and decreases as
rods be 11' 12 and lJ• we move up.
From cosine law, we have

_w'" ,-
www.puucho.com
Study Anurag Mishra with www.puucho.com

TEMPERATURE, HEAT AND THE EQUATION Of STATE, HEAT TRANSfER 15

Po + PAg!lA -PBghB + Pcghe -PVghD =Po Example 27 _


P AKhA -Pagha + Pcghc - PnghD =: a
as PA =Pc =P95"C Calculate the variation in time period of physical pendulum
Pa ""PD =Ps'(" when temperature changes by b.T.
P9sc(hA t/lc;)=PsocChn tho) Solution: The time period of a physical pendulum is
Pst.' =!JA+hc =528+49=1.018 given by
P9S"C lin + liD 49 + S1

as PS"C"
t = 21t) ~gd
P95',(" -----
[1 + y(90'C)]
where. I is the moment of inenia of the pendulum about
PS"C =(1 +y(90°C)]=1.01B the axis of oscillation and L is the distance of centre of mass
P9S"C of pendulum from the point of suspension. The moment of
inertia of a body composed of large number of particles can
Hence 'l =( 1.018-1)
90'C
= 2x 10~4/C be written as

Example 26~
where, rj is the distance of point mass trli from the axis of
rotation
Two vertical tubes filled with a liquid are connected at their
lower ends by a horizontal capillary tube. The tube is When temperature changes, both distances ri (hence 1)
surrounded by a bath containing ice and water in equilibrium and L change. Let us first calculate change in I w.r.t. T.
at Dee, and the other by a hot water bath (t0C). Due to change in temperature by b.T, change in momcm
of inertia is
The difference in height of the liquid in the two columns is I1h,
and ho is the height of the column at 0"(;. Show how this dI =(1 +dI)-[
apparatus can be used to measure the true coefficient of = Iml(r, + !'.r,)2 - I m,T?
i i
volume expansion of the liquid after neglecting the respective
= InJi(r,(l + MT)2 -Im,T,2
quality of tubcs. Determine g if t = 16OC, ha = 126 cm and , ,
!'.h = 1.50.
Neglecting the terms containing a 2, which is extremely
small, we get
dI '" r, m,r/Za L1T = 2al L1T

T The change in length L is given by


l' M=Lu.L1T
Therefore, change in time period can be written as
Fig.1E.26 M=(t+M)-t

Solution: Equating pressures in the two limits, we


: )~g [J~:~-H]
hlP]g = h2P2g
= ~ 1I[(l+2ab.T)120+attTrI2-11
h]Plg = (hi + !'.h)P2g ..;Mg ~L
Bur, PI =P2(1+yb."]"), Using binomial approximation and retaining only first
h1P2(1 + y b.T )g = (h] + b.h)P2g order terms in a, we get
b.h =h1y b.T
M=.!atttT
or y=
,h
__ 2
h1b.T Thus, variation in time period of a physical pendulum is
same as that of a simple pendulum.

www.puucho.com
Study Anurag Mishra with www.puucho.com

16 THER'' M' 'O"'DYNAM='' IC''S' 'I

Example ""Example ,~V-

A steel tube, whose coefficient of linear expansion is 18x 10-6 The brass scale attached to a barometer reads correctly at
per "C, contains mercury, whose coefficient of absolute 20"C. The barometer height is read as 75.34 cm of mercury
expansion is 1BOx 10--6 per "c. The volume of mercury when the temperature is 25cC. What is the true height at
contained in the tube is 10-5 mJ at O"C, and it is desired that DOC? The coefficients of volume expansion of mercury and
the length of the mercury column should remain constant at of linear expansion of brass are IlHg = 18x 1O-5;oC and
all normal temperatures. This is achieved by inserting into the Us =1.8x 1O-5;oC, respectively.
mercury column a rod of silica, whose thennal apansion is
negligible. Calculate the volume of the silica rod. (See Fig. Solution: The brass scale reads correctly at 20"C.
lE.28) but at 2S"C the scale expands and therefore indicates a
smaller length than the true length of the measured object.
Mercury Hence, the true length is given by the measured length plus
v the expansion of the brass scale due to the temperature rise

I"1
from 20"C to 2S"C. Note that. the brass scale readings always
give the true length of the scale at 20"C. The true length, I, of
\ the mercury then becomes
S'ee' ,"b. 1=75.34 + (75.34)U8(25 - 20)
Fig.1E.28
=75.340+9x 10-5) em
The height 10 of the mercury column at QOC will be
Solution: At O<t:, let the volume of the silica rod be smaller than 1 since the density increases with decreasing
Va. the volume of mercury be V. and the cross-sectional area . temperature and the same mass of mercury occupies a,
and length of the column be Ao and 1o, respectively_ Then at smal1er volume. The volume at O"Cis
t == Dee, Vo : V - VPllg (25 - 0)
loAo = V + Va ... (1) ~ V(l-PH, .25)
At any temperature t, V and Ao will change to their new where V is the volume at 25"C. The cross-section A, of
values as a result of thermal expansion. These new values the glass tube containing the mercury remains practically
are. respectively, V' and A, where constant as the temperature changes. Hence, the change of
V'~ V(l + PO the volume of the mercury is reflected as the change in its
length.
and A ",AoO + 2at)
V, V
Here, and p are the coefficient of linear expansion of
(l "'A = A 0-I3H~ x25)
steel and the coefficient of absolme expansion of mercury.
Note that we have imposed the constraint that the column 10 '" 1(1 -IlHg x 25) = 10- 4.5 x 10-3)
length, 10, be constant. Hence, at temperature t. we may = 75.34(1 + 9 x 10-5)(1_ 4.5 x to-3)
write
= 75.01 em
loA'" V' + Vo
or 10Ao(1 + 2at) '" VO + pt) + Vo ... (2) Example
Using (1) in (2)
The volume of the bulb of a mercury themometer at O"C is
(V + Vo)O + 2at) = VO + pt) + Vo
Vo, and the cross-section of the Cflpillary is Ao' The
VoO + 2at -l)=VO+l3t -1-2at) coefficient of linear expansion of the glass is a g per "C, and
V, = V(P - 2.)< ~ V(P - 2.) the coefficient of cubical expansion of mercury is tlHg per "C. If
2ur 20. the mercury just fills the bulb at O"C, what is the length of the I

to- x 10-6 - 36x 10--6)


5080 mercury column in the capillary at a temperature of t"C?
v------------
o- 36x 10-6
Solution: An exaggerated view of the expansion is
10-s x 144 3 shown in Fig. IE.30 (b). Fig. lE.30(a) represents the initial.
~----m
36 situation. When exposed to a temperature change iiT, the
cross-section of the capiJIary, the volume of the bulb, and the
:4xl0-S m3•
volume occupied by the mercury all change (sec figures).

www.puucho.com
Study Anurag Mishra with www.puucho.com

TEMPERATURE, HEAT AND THE EQUATION Of STATE, HEAT TRANSfER


~------ The volume of the displaced water equals the volume of
17

A the bulb since the bulb is completely immersed in the water


while being weighed. The change in the volume of the bulb
is due to the change in the volume of the glass since the gas
inside the bulb cannot appreciably enlarge the glass. If the
v, v specific weights of water at l' and 1'} are Pw and P ••~,
respectively, then
(.) (bJ W ... (1)
VB =-
Flg.1E.30 Pw
W,
The final volume occupied by the mercury is VB] = Vll +tJ.VIJ =-- ... (2)
PWI
Vllg :=\'O(1+1lHgC,T) ... (1)
where illig is the coefficient of volume expansion of are respectively the volumes of the bulb at T and 1'1'The
mercury. The new cross-section of the capillary will be specific weight of water will decrease as a result of the
thermal expansion of its volume since its weight remains
A=AoCl+2ugc,T) ... (2)
constant. If the weight of water is WI" and its volume at l' is
where ug is the coefficient of linear expansion of glass. V••., we can write
Similarly, the new volume of the bulb is
W ••. = VwP ••. = (VlI' + bVW)PWI
Vg := VoO + 3ugtlT) ... (3)
where b V is the volumetric expansion of water. Hence
II'

(Note that, initially; the volume of IIg = the volume of


p," Vw
----Pw 1
----Pw (3
...)
the bulb.)
V••. + bV... 1 + bYw
Now, the volume of mercury outside the bulb in Fig. (b) Vw
will be
The volumetric expansions bVB and b.Vw are given as
Vllg - vg voO
:= + 111l~t1T -1 - 3ugtl.T)
W
VHg - Yg = VOCflHg-3ug )AY ... (4) b.VB =PBVBb.1'=PB-(T1 ~T) ... (4)
Pw
If the length of the mercury column in Fig. (b) is h, then .. (5)
b.Vw =PwVwtJ.1' =PwV",(T] -1')
Yill.! -Vg =hA = hAo(l + 2ugc,T) ... (5)
From eqns. (1) and (2), we get
where we have used (2). Equating (5) and (4), W, W
b.Vll =---
hAoO + ZUg DT) = VOCPllg - 3ag )tJ.T
PII'j PI"
or h =
Ao
VO(PHS -3Ug
1 + 2ugb.T
J~T or, using eqn. (3),

Since
6.V8 = WI (1 + b.Ww J_ W
P... VII' Pw
W} b.V••.
1
=-(W]~W)+--
PI" PI" V",
Substituting the expressions (4) and (5) for b.VB and
b.Vw in the above equation, we get
W WI WI ~W
A glass bulb with volumetric e.>.pansioncoefficient PB is P. -(1'1 -T) =-P",(T1 -T) + ~--
weighed in water at temperatures T and T1. The weights of the Pw P... P,.
displaced water are Wand WI' respectively. Find the WI WI -W
volumetric expansion coefficient Pw of the water in the
or J3B =WP,. + W(T -1')
I
temperature interval from l' to 1'1'
The volumetric expansion coefficient for water is
Solution: The volumetric expansion coefficient P W W-WI
Pw =W-PR + W (T -T)
relates the change bV in the volume of a substance to a small
change tJ.1'in the temperature of that substance:
, "
tJ.V =PtJ.1'
= J311 + W - WI (J3JJ +.....!...J
W] b.T
V
where V is the initial volume.

www.puucho.com
Study Anurag Mishra with www.puucho.com

18 THERMODYNAMIC!

The <lbove relarion will hold for small ,\T "" (T1 - T ). . L\I~t'il
Thermal stram"" --- =0
Thi_~corresponds to a small voiumNric change for the bulb I
in the sense then Thermal stress developed in a rod clamped
rigid between fixed support
A metal rod having coefficient
I
THERMAL STRESS
When a metal rod is hc •.lted or cooled, it expands or
contracts. If it is completely or partially constrained from the
of linear expansion n, length i,
Young's modulus of elasticity Yand -T -T
area of cross section A is rigidly Fig. 1.13
T -
clamped between the two fixed
expansion or contraction. then themlal strain and a support. It is not Clllowed to expand or contract at all on
compressi\'c or tensile stress is developed in it. Compressive changing the temperature. Thermal strain developed is
stress is developed when rod is hemed and tensile stress given by,
when the rod is cooled.
. 6/-0 Al
Imagine th<lt what would have happened in absence of Thermal stram "'-- "'- "'fl..i.\O
I I
support. Would have expanded contmented but due ro
stress
constraint of suPPOrt it is not allowed to expand or contract. And, = Young's modulus (Y) =--
strain
length in absence length in absence
of support So, Thermal stress = Ya t'iO
or support ./
Compressive
....................
I.. "" YnliO
stress :- A
-' .l;::=::;,~o=:::l .. O-
~ -- .: Tensiel
stress Where T is the tension developed in the rod.

r---/(1 + cUT)-----i
" Hence, T '" Y a AAO
l(l-uAT) Shifting of the junction of two rods clamped
Healing rod Cooling rod
between two fixed supports.
Fig. 1.12 Consider the two uniform rods having the same
cross-sectional area' A and joined at the junction J'. Now
The magnitude of the [ensile or compressive force y
produced is independent of the length of the rod.
Thermal Strain
• A A

Thermal strain produced corresponds [0 length of


expansion or contraction that is not allowed due to presence
of support. ,,,
" ,, "
y ,,
If a metal rod of length / is prevented completely or
partially from expansion or contraction, when heated or • ,,
,

<' I ------.,
cooled by 6.8. It 6.1is change in length corresponding to the
change in temperature 6.8; 6.1== a /6.8.Then thermal strain is
defined as, ,- ---->, --
.• un - - - ~i
.."---
-
'2-~
Therma I stram. == ------
(l+6.I)-(l+x) 6./-x 11"~
=--
I+M 1+1i/ Fig. 1.14: let us find In. the sift In the junction.
Ii/-x
Thermal strain (61 « I)
== --
their temperature is increased by fie so that one rod vvith
I .
greater coefficient of expansion expands and the other
x is change in length that actually occurred. contracts. Consequently the junction shifts along the length.
If the rod is fixed between perfectly rigid support and When the temperature is increased, in the final
does not expand or contract at all on changing the equilibrium condition. When the force developed in the t\Vo
temperature, then x = 0, rods become equal, the shifting of junction stops. Let the
Thi. 61
erma stram == T shift be x,
At equilibrium
If the rod is allowed to expand or contract without any or (Stress)} x A = (Stress)2 x A
constraint changing the temperature, then x == iiI and,
or (Stress)} x Y1 '" (Stress) 2 x Y2

www.puucho.com
Study Anurag Mishra with www.puucho.com

TEMPERATURE, HEAT AND THE EQUATlQN Qf STATE, HEAT TRANSfER 19

=~12 -x 32 _
or
x y, Example
I~ -
as, L\/1 = 11ul~fl An ani:>otropic body Ila,~ iB principal lincar e>.-Pallsit'itie"
along the three dircetiollS as u l' u: 2 alld (LJ re.,pectivcly,
and. ;<,1::'" 12u2~\O
Defenl1inc its averagc ,wpcrfieial c.xparl.';it'iIY,
So, /I't],""O-x x y 1=-l:,)u2'\0-.
--xYl
Consider a cube of edge I. made tip of tlte abr)\'c material, willi
II 12
irs edges lying aloll!: the three principal direClion."
or, x(Y~1 ~-\i]=(YIUI
2 II
-Yz(.(z),\O Solution: A temperalLlrc change of .1T, would render
its new t'dge as 10 rtl.\T),I(l-
-4- (t2,\T) and l(l + u3;\T)
Thus, the change in it.~sur/act' area will be
o~ X= l Y, "l::-Y,~
y y 1 2 [l(l + u1,\T)IO + rtli\T),~ 1(1- (l\,.\T) I (1 .•.u~~T)

I:
'- 1]' +1 (l + (L 2~n I (l + rt3L\T)] ~ 612

't Z12[Z(u\ .•.u2 + (3)~T]


If the junction does not shift at nil, then x ::= a
:. Change in its area per unit original area per unit
or, YlUl = YZClZ
change in temperature.
Shifting of centre of mass of the system
2/'
=--[2(01 +a? +(1)],\t
Due to the cxtern<ll force acting from supports the 6{2,1.T -
position of eM shifts.
2
Position of the centre of lllass of the system before ="3(°1-(/2 +(3)=1~
he<tting initially.
Expansion of Liquids
mll~ +mzx(11 +1~J, Like solids, liquids also, expand 011 heating; but, their
R: em = ~ __ cx
expansion is much large compared to solids for the same
TIl] + /Ilz
temperature rise. A liquid is always contained in a container
where TIl] <Jnd1112 are the masses of the rods. hence the expansion of the vessel also comes into
Nter heating. in final equilibrium position here position consideration. Linear or superficial expansion in case of a
is. liquid is not.

R'em =
TIlle!; .\")+ Ill z x (II + X+ 1 2
;~) •
x
Consider a liquid contained in
a spherical flask fitted with a long z
narrow stem as shown in figure. Let
Tn] +mz
the initial level of the liquid be X.
Shift in the position of the centre of mass of the system, When it is heated the level falls
mix mzx initially to Y
--+mzx--
2 2 oX However, after sometime, the
liquid level eventually rises to Z.
x Upon being hearted, the container
-(mt +mz) gets heated first and hence
2 " x_ Fig. 1.15
= x=-x expands. As a result the liquid level
m1 + mz 2
falls.
Where x is the shift in junction, After sometime, the heat gets conducted from the vessel
to the liquid and hence liquid also expand thereby rising its
Shift=[Y'U'
zil"+Y'}
-y,u,],," level eventually to Z. Since coefficient of the volume
expansion of liquids, are greater than solids, so the level Z
will be above the level X.
1/ 1 1 2
Real expansion = apparent expansion + expansion of
the container

www.puucho.com
Study Anurag Mishra with www.puucho.com
20

If \'t and Vc be the original volume of the liquid and the


'.' . , .
(say, TI and T2 where T2 > TI). Let the weights be W} and W2
THERMODYNAMICS

temperature rise is !!T, then, respectively. The mass of the liquid filling the apparatus at
The real volume expansion of the liquid will be the two temperatures will be WI - W = ml and W2 - W = m2
(say).
(8.V)=\'l' -v, =YrV1L\T,

where Yr is the coefficient of real expansion.


Similarly, the volume expansion of container, will be
.
lIII
(dV)c = v~ - Vc = ycVch.T,
Where Ie is the coefficient of cubical expansion
of the container.
!ll!!ll!II!!!m .!!!!!!!!I :::::_ B
......
The apparent expansion of the liquid = Relative
Fig. 1.16
expansion of the liquid with respect to the container
I.e. (.1.V)g = (IlV}1 -(6V)c Let V and V2 be the respective volumes of the apparatus
j

or YaVI"'T = (YlIVI -YcVC>.1.T, P I and P l be the density of the liquid at temperatures TI and
T,.
Where Ya is the coefficient of apparent expansion of the
liquid.

OT la ='~r -Yc(~~) or
:: ~(~:J(~:J ...(0
V,
=> '1=- Ifyp and 'Ie be the respective volume expansivities of the
Vc
liquid and the glass apparatus, then
If the liquid fills the entire. vessel then" = 1
V2 =Vt(l+y,T)andpt andp2 (l+yT)
la =Y. -Y,
where T =T2 -TI
If Ya is positive, upon heating the liquid seems to expand
relative to the container. And, If Ya is negative upon heating, or VI = 1 and £.!. = 1 + 'I,T
the liquid seems to contract relative to the container. Vl l+y,T P2

If there is no apparent expansion of liquid relative to or From Eq. (0 we get


container, then 'I a "'" D m
l l+y,T
-~---
V, 'Ie m2 l+y,T
and 11 =-~-
Ve Y, =1+h,+'I,)T
Anomalous Expansion 'of Water m -m
(Y,+Y,)T= I 2
When pure water at DOCis heated slowly then initially, m,
water is found to contract, till a temperature of 4°C is m -m
or y,+y'=Y(J= 1 2 ... (ii)
obtained. Beyond that water expands for a further rise in
temperature. The density of water increases from DOCto 4°C
m,
and decreases, beyond 4°C. Such a behaviour of water is
referred to as anomalous (abnormal) expansion of water.
or y,=[m\':2m2]+y, ...(iii)
DETERMINATION OF THE COEFFICIENT OF also T=T2-T1= n1j-m2
EXPANSION OF LIQUID m2(Yr -y,)
1. Weight. Thermometer Method
The apparatus has a glass bulb B with a narrow outlet or T =[n1:tY~~~cJ+TI
2 ...(iv)
pipe P. A vessel containing the liquid for coefficient of
expansion is to be determined, is connected to bulb B. Equation (ii) yields the coefficient of apparent
(Figure 1.16) expansion of the liquid relative to the material of the
apparatus (glass) . Equation (iii) gives the coefficient of real
The glass bulb is cleaned and weighed. Let it be W. Next, expansion of the liquid which is obtained by weighing the
the bulb alongwith the stem is completely filled with the liquid. The method is known as weight thermometer
liquid.
method. The appartus is reffered as thermometer because
The bulb and the outlet pipe completely filled with the knowing Y the coefficient of apparent expansion, any
(J

liquid is separately weighed at two different temperatures

www.puucho.com
Study Anurag Mishra with www.puucho.com

TEMPERATURE, HEAT AHD THE EQUATION OF STATE, HEAT TRANSFER 21

unknown temperature (Tz) can be obtained from equation hz -hI


y=-----
Ov). (htTZ -1I2Tl)
Dulong and Petifs Method 33 _
Example
Fig. 1.17 shows the experimental arrangement
consisting of a glass U tube filled with the experimental An isosceles triangle is formed with a rod of length II and
coefficient of linear expansion aI for the base and nvo thin
rods each of length 12 and coefficient of linear expansion (12
for the two pieces, if the distance benveen the apex and the
midpoint of the base remain unchanged as the temperatures
varied show that .!.L= 2 a 2
'2
!
\a J
.

"m" I"u,
Fig. 1E.33

Fig. 1.17

liquid to the same height in the two limbs. Both the limbs are
enclosed in two different jackets, maintained at steam point
(lOOCC) and ice point (aCC) by means of steam and ice cold
water circulated in the two jackets respectively.
dl~ increment of length with increment in dt ~
The two jackets contain accurate thermometers to give
temperature of system
their respective temperatures. When the two thermometers
yield thermal equilibrium readings, the levels of the liquid in Given,
two limbs are recorded (say hI and hz). Since, the base of Because I remains constant with rise in temperature
the liquid columns in the twO limbs are at the same therefore dl = 0
horizontal level, pressure, at those points will be the same. If de
Po and Pwo be the densities of the liquid at DoC and lOooe So,
and Po the atmospheric pressure, then
Po + h1Pog = Po + hZPlOog dll = IJaldt, dl2 = 12 x 0.2 x dt
or hlpo = hZPlOO 11>< l}atdr = 412 >< lza2dr
p,
112a} = 41;0.2
PIOO = l+y.l00

So, hz == hI (l + lOOy)
h2 -iiI
~=2/u.z
12 \ 0.1
or y ==~-~
100hl ,
Example 34 V--
The experiment can be carried, even with temperatures
other than the ice and the steam point. IfTI °C and T2 °C be A brass rod of mass m = 4.25 kg and a cross sectional area 5
the temperatures of the liquid columns in the two limbs, cm2 increases its length by 0.3 mm upon heating from DoC.
corresponding to heights h] and liz respectivel}~ then What amount of heat is spent for hearing the rod ? The
Po + htPIg = Po + hzp2g coefficient of linear expansion for brass is 2>< 1O-5/K. Its
hIP I h2P2== specific heat is 0.39 kJ/kg-K and the density of brass is
hI 1 + yT2 8.5>< 103 kg/m3.
01 -=---
li2 I +yTI
Solution:
htpo = hzpo Given III =0.03cm, 0.= 2>< 10-5,
l+yTI 1+1T2
m = 4.25 kg, A = 5cm2

www.puucho.com
Study Anurag Mishra with www.puucho.com

22 THERMOOYNAMICS

Initial length of the rod is PI\'


p~.=
I = ~ =_
(rn_r_l ~ _rn_ O+y",M)'
A A Ap 4
1'1\' =2x 10- /oC,'l .• =3a.,
I~ 4.25 fJ.T = SoC,
5x 10-4 x 8.50x 103
1'$ =3x1.2x lO-s/oe, mg =?
~ 1
5)(10-1)(2 P,,'voO + ysfJ.T) = 109 + m ... (3)
(I + y",~J')
= 1 m =IOOem
So if increment in length is At = o.3mm ""D.03cm then Putting P,,'vo from eqn. (1) in (3), we get
temperature rises by
li/=/aAO
All = L\l = 0.03 = 150C
La 100)(2xlO-$
li(J = 15°C
So amount of heat supply to rise the [emperature from
Mg=109g=p'.V'og
acc to lS~C
Mg
Q = 4.25x 0.39)( 103 x IS J Fig. 1E.35 (b)

Q = 24.86x 103J= 24.86kJ =25kJ 11 x lOBx 0 +ysfJ.T) = 109 + m


(I + yw"T)
Example 35y--
[1+5)(3.6)(1O-sJ 10'
11 x lOB x = +m
A submarine made of steel weighing 109g has to take 10IJg of [1 + 5x 2)( 1O-4J
water in order to submerge when the temperature of the sea is
11 x lOB x (1+ 1.8 x 10-4) 10'
woe. How much less water it will have to take in when the => ---~. = +m
sea is at 15"C ? (Coefficient of cubic expansion of sea water 0+10x10-4)
= 2 x 10-4 joc, coefficient of linear expansion of steel 11x lOB + 18x 11x 104 = 109 + 106 + m + mx 10-3
~1.2xl0-5/oC) 1011 _ 80.2 x 10" = m(1 + 0.001)

Solution: p,,'vQ8" = M ",g + Mg[Mg = Ps Vo8"l 104 (l0000 - 80.2) = m(1001)


8
P",.voS = 10 g + 109g m = 99098901.1 gm
P".vo = 11 x 108 ... (1) So at this position it takes 99098901.1 mass and wiJI be
99098901.1 gm. So finally reduction in the weight will be
PwVo9
=lOB -99098901.1 == 901098.9gm
or = 9.01 x lOSgm
v. Example 36

A U.tube filled with a liquid of volumetric coefficient of


Mg=PsVo9"'109g
8
M",g= 10 9 ','
1O-51°c lies in a venical plane. The height of liquid column
in the left vertical limb is 100 em. The liquid in the left vertical
Fig. 1E.35 (a) limb is maintained at a temperature = DGe while the liquid in
the right limb is maintained at a temperature = lODGe. Find
Note: Initially it takes in 108 g weight water then mass will be 108 the difference in levels in the two limbs.
in gm (gram)
Condition when temperature rised from 10°C to ISoC Solution: _ Po
Pt"C - (l + yfJ.O)
p;"VQg =p~VQg[Mg]+mg
at point C the pressure will be equal, PI = Pz
p;" Vog = 109 g + mg ... (2)

www.puucho.com
Study Anurag Mishra with www.puucho.com

TEMPERATURE, HEAT AND THE EQUATlDN DF STATE, HEAT TRANSFER 23

1/'-1/ = V' 1l~~VII~

I
100"C AV = AVUl'
~,,=V ~ 3u,;:/ ~ ~r
<-\V1lg = VH;: ~ YHg )<, ~\T
OT y
\' x 3(1.s1x ,H' '= Vll~)<, YH;: x :\T
100cm
V x '(
V = Il~ Hg

3 x ugl
300x 1.8 x 1O"~
- 2000 em 3
Fig. 1E.36 3x9xlOll
PI =rO'l: xg x 100 Example 38__
P2 =rWD'C xg xy
Po'c xg x 100 =PIOO C xg xy A clock pendulum mude of iTll'ar has a period of 0,5 sec at
20°C. If (lie clock is used in a climate where average
PocxgxlOO= PO"c xgxy
(l+yL\O) temperature is 3QoC, approximately. How much fast or sloll'
will tlie clock run ill 106 sec. (a illvar = 1x 10 -6 Joe).
100(1 + 1.:'.0) =Y

y :: 100(1 + 10-5 x 100)


Solution: '['-2
- it ~
-
y = 1000 + 10<1) = 100 x 1001 = 100.1 \g
So change in length = 100.1 -100 = 0.1 em T' = 2n ~ = 2n ILO + a.lO)
.1.y =0.1 em \ g \ g

Example 37__ T'


rooO-dl.M)) " = ( 1+ ,,",0 J
2
A glass flask contains some mercury at room temperature. It r=T(l+~O)
is found that at different temperatures the volume of air
inside rheflask remains the same. If the volume of mercury in T'=T+e:t.t..OT
rhef/ask is 300 cm3• then find volume of the flask (given that 2
coefficient of volume expansion of merwl)' and coefficient of r-T = MOT
linear expansion of glass are 1.8x 10-4 (Ocr I and 2
9 x lo-{,COcrJ respectively], 1
tJ.T = - a.~OT
2
Change in time period will be
tJ.T = .!.~OT =..!. x 10.6 x lOx 0.5 = 25 x 10-6 sec. So due to
2 2
increase in length time period will increase by
air tJ.T = 2 5 x 10-6. So clock will slow and in one time period
means in 0.5 sec it slow by 2.5 x 10-6 sec. Then in lsec it will
slow by 5 x 10-6 sec. So in 106 sec it will slow by
5 x 10-6 x 106 = 5sec.

Flg.1E.37
Exarnple 39

Solution: Suppose if initially volume of the container An iron bar (Young's modulus = 1011 N/
m2 ,ex = 10-6 /oC )
3 2
1 m long and 10- m in area is heated from O°C to 100°C
is V then
without being allowed to bend or expand. Find the
v = YAir + VH~ ... (1)
compressive force developed inside the bar.
And if VAir remain constant then on heating V t and
VHgi and V-t V', VlIg-t V'Hg
V' = YAir + V' Hg ... (2)

www.puucho.com t
Study Anurag Mishra with www.puucho.com

24 THERMODYNAMICS

Solution: ... y = F/A Example


I1U
61
F=Yx-xA A thennostated chamber at small height h above earth's
I surface maintained at 30°C has a clock fitted in it with an
and 61=lxaxl!.T uncompensated pendulum. The clock designer correctly
F=Yx«xIiTxA designs it for height h, but for temperature of 20°C. If this
= lOll )( 10-6 x 102 x 10-3 = 104 N chamber is taken to earth's surface, the clock in it would click
correct time. Find the coefficient of linear expansion of
Example material of pendulum. (earth's radiw is R)
-. .'. -
Solution: The variation of g with height from earth's
Three conducting rods of same material and cross-section are surface
shown in figure. Temperature of A, D and C are maintained at g' = g(l- 2h/R)
20"C, 90°C and DOC. Find the ratio of length BD and Be if
there is no heat flow in AB. According to given problem the time period T of clock is
correct at height h and temperature 20°C if suppose the
ABC
length of pendulum at 20°C is L then
20.C I O.C
T = 2, f, __ L ... (1)
90.C 0 ¥g(l-2hR)
Flg.1E.4D But if at h height from earth's surface temperature 30°
then
Solution: Because the heat is not flowing from A to B T = 2it LO + a)( 10)
so both should be at the same temperarureTA '" TB = 20 e. C
g(l- 2h'R)
... (2)

And so rate of heat flow from D to B should be equal to


theBtoC In correct time period and clock will become fast
because Li.Ti
lOB =lac
KA(90 - 20) KA(20 - 0) But if clock with chamber at 30°C at earth surface is
=---- giving correct time period then
lOB lBe -----
Here KOB =KBC =K [for same material] T = 21'"
.\
ILO + ga x 10) ...(3)

ADa = ABC = A
Equating (1) and (3)
50 lOB =?.
lBe 2 0+ ax 10) = ---- 1
(I-2h/R)
Example o ••
(I + a x 10) = (1_2h/R)O',
h
If two rods of layer Land 2 L having coefficients of linear 0+ a x 10) = 0 + 2h/R), a=-
5R
expansion a and 2« respectively are connected so that total
length becomes 3L, detennine the average coefficient a/linear Example
expansion of the composite rod.

•L 2• The coefficient of volume expansion of mercury is 20 times the


2L coefficient of linear expansion of glass. Find the volume of
mercury that must be poured into a glass vessel of volume V so
Flg.1E.41 that the volume above mercury may remain constant at all
temperature.
Solution: If temperature of the whole system is
increased by 1i0 then total composite rod of length 3L will Solution: Volume of vessel is
increase by 61.: and it will be /lL'::: 3Lutqli0 V = VHg + constant volume ..,(1)
3Lu.q1iO = LCL10 + 2L x 2a x.10 And if Hg and glass vessel both are heated up [0 same
5 temperature then temperature increased of glass vessel and
a =-u Hg by 6T.
" 3

www.puucho.com
Study Anurag Mishra with www.puucho.com

_ TlMP!RATUR!, H!AT AND TH! EQUATION OF STATI, H!A.TT


•••• RA
•• N_S"'FE
••R •••• _ 25

So V-..V', VHg----. Vilg The heat transfer is from the warmer body to the colder
V. = V~g+ constant volume ... (2) body.
From eqn. (1) and (2) The amount of heat required to raise the temperature of
V'-V = Vilg -VHs' 6V =.6.VHg ..,(3) an object by one kelvin is called the object's heat capacity
C'.
Given YHg = 20agl
If Q is the amount of heat required to raise the
1'.V=Vx3aglxt.r, temperature of an object by 1':1T, then the heat capacity is
f!.VHg = VHg x ZOog1><!:J.T given by
Putting chese in eqn. (3) and get C' =..9.. ... (1)
3 6T
VH =-V.
, 20 The specific heat is the heat capacity per unit mass,
C'/m
1 Q
C=-- ...(2)
m 6T
A metal rod A of 25 em length expands by 0,050 em. When its
temperature is raised from DOC to lOO"'C. Another rod B of a The molar heat capacity C is the heat capacity per
different metal of length 40 em expands by 0,040 emfor the mole of substance, C 'In, when II is the number of moles.
same rise in temperature. A third rod C of 50 em length is C =.!...9.. ... (3)
made-up of pieces of rods A Qnd B placed end to end expands n 6T
by 0.030 em on heatingfrorn DOCto 50°C. Find the lengths of The above definitions hold if the heat capacity is
each portion of the composite rod. independent of temperature; otherwise
Solution: HeatcapacitvC' = dQ ... (4)
. dr
Given LA = 25cm, LB = 40cm, Le '" SOcm
SpecificheatC =~ dQ .(5)
MA=O,OSOcm, ME'" 0.040 em, !lie == 0,030 em Tn dT
tJ,T '" lOOoe iloT '" IOO~C !:J.T'=50°C
IT
Moarspecllc heat=---1 dQ ... (6)
j,IA '" LAf.l.At.T !liB'" LBuBt>.T LC=/A+/II n dT
0,050 0,040 When the specific heat is not constant with temperature,
a - -2x10-5 a - 50=1,\+111
A - 25xlOO 11 - 40 xlOO we find the total heat required for a given temperature
change from Ti to Tf by integration of relation expressed by
UA = 2xlO-s uB = 10-5 4J =(50-11\) eqn. (5).
1':11, =1':11'A+1':1I'B dQ = mC(T) dT

1':1le = lAaA1':1T'+lBaB1':1T' Q =m f Tf
T,
Cel') dT
5 5
O.030=IA x 2x 10- x sO+(sO-IA)X 10- x 50
Changes of Phase
3000 = SOIA x 2 + 2s00-s01A
When a substance undergoes a change of phase from
3000 - 2500 = 501A
solid to liquid or a liquid to a gas, a certain amount of energy
500 = SOIA is involved. During a pha.~e change, temperature of a
IA = lOcm substance does not change as heat i.> added to it. During a
In = 40cm phase change heat transferred does not increase kinetic
energy of the molecules; it is expended in work to overcome
CALORIMETRY intermolecular attractive forces, so that separation benveen
In calorimeuy experiments, one thermodynamic system molecules increases.
at high temperature is placed in thermal contact with
another thermodynamic system at a lower temperature. As a
tesult heat transfer takes place between two systems until
the combined system attains a common temperature and is
in thermodynamic equilibrium.

www.puucho.com
Study Anurag Mishra with www.puucho.com

26 THERMODYNAMICS

1. 'C the temperature of the water at the surface reaches 4°C. this
100
process stops. As the temperature decreases further, the
cooler water at the surface has a lower density than that
80 below, and therefore stays at the top. Finally it starts to
freeze. If the water is deep enough, it remains liquid below
60 the frozen top layer. This peculiar behaviour of water allows
fish and other aquatic life to survive through the winter.
40

o What is the heat required to convert 40.0 g of ice at O.OOC to


convert it all to steam at 1 DO'"C?
-10
Ice ~----------
Ice and
Water water and Steam
AQ"5.5 cal water ",Q=loo cal steam t.'lQ:4.8 cal
mlKture m'xlure
-----t.a,cal

Solution: Ice at O'C changes [0 steam at lOO'C in


11.0=79.7
cal ",Q"540cal three steps:
1. Change of phase from a'C ice to a'C water. Heat
fig. 1.18
required Q1 = mL!
The heat transfer required to change the phase of one 2. The temperature of the water is raised from O"Cto
kilogram of a substance is called the latent heat. 100"C,
If the phase change is from solid to liquid (or vice-versa), Heat required Q2 = mC ••.(Tf - T j)

the latent heal is called latent heat of fusion eLf). 3. Change of phase from 100'C water to 10O"Csteam:
If the phase change is from a liquid to a gas (or Heat required Q3 = mL1.
vice-versa), the latent heat is called the latent heat of Total heat required
vaporisation (Lv).
Q = mLf + mC..,6T + mL,.
Fig. 1.18 shows the temperature variation of one gram
0:: m{Lj + C,.,6T + L,.)
of ice initially at -lO'C, until it reaches a temperature of
110'C as steam. To change the phase of a mass m. the heat " (40.0)[79.7 + (1.00)(100 - 0) + 540)
transfer to the mass is = 28.8 kcal
Q=ml

~.
r::::::l Heat transfer Q

Heat transfer
= mL"

a = mL"
.~ ~

What will be the final temperature T of the mixture if


L,.....• Lalent heat ofvaporisation 200 cm3 of rea at 95°C is poured into a 150 g glass cup
initially at 25°C? Assume the system to be isolated and that
~ HealtransferQ ;; m4 tea mainly consists of water. Specific heats of tea and glass
~. HeallransferQ ;; m4
-I Water I are 4186J/kg °C and 840 J/kg °C respectively.

Solution: As tea is mainly water its mass is


4 -4 latent heat offusion equivalent to mass of water
m =pV
Anomalous Expansion of Water
= (l.0 x 103){200 x 10-6)
The thermal expansion of Volume(em3) of
water is interesting because it 1 9 ofwater = 0.20 kg.
exhibits anomalous expansion Applying conservation of energy, we get
bet\'Jeen a and 4°C. As Fig. 1.0002 heat lost by tea = heat gained by cup
1.19 shows. bet\'Jeen these 1.0001
tw'o temperatures its volume 1.0000 m1eaC1",,{95 -T) =mc""Cc"l'{T - 25)
decreases as the temperature where T is the final temperature.
o 2 4 6 a 10
rises. The maximum density of Temperature(~C) (0.20)( 4IB6)(95 - T) "(0.15)(B40)(T - 25)
water is 1.000 g/cm3 at 4.a°c. which on solving for T yields
This explains why water starts Fig. 1'.19

to freeze at the top of a lake. T = 89°C


As the air temperature drops from some value above 4°C, The tea drops in temperature by 6'C by coming into
the cooler water at the lOp is denser and so it sinks. When equilibrium with the cup.

www.puucho.com
Study Anurag Mishra with www.puucho.com

TEMPERATURE, HEAT AND THE EQUATION OF STATE, HEAT TRANSFER

Method 2: Since total hem transferred into or out of


-_... Energy required to change the ice to water at O'C,
27

he isolated system is zero, Q3 = m;c"Lj = (0.50)(333) = 167 kJ


IQ, =0 A total of 10.5 kJ + 167 kJ = 177 kJ energy is required
Each term is written as Q = me(T; - Tf ). where Ti and Tf to bring ice at -10'C to water at O'C, whereas energy
He the initial and final temperatures. required to bring water at 20'C down to O'C is 250 kJ. Thus
the mixture must end up all water somewhere betvveen O'C
"fQ = l1l,c.,C Ie. (95 - T ) + moure cUI'(25 - n =a and 2D'C. Let the final temperature be T. Now applying
The second term is negative because T will be greater
.han 25"C. On solving the above equation we obtain the
;arne result. e[ne:~it:;S:f:tJn~ [He~t.~~ ~gangeJ + [ H~~~~O:;ise ]

Example 47__ from -10"Cto of Ice of water


DOC to water from O"C to T
A 0.150 kg sample of an alloy, after being heated to 540°C, is Heat 10>[ by ]
dropped in 400 g of water at woe, which is contained in a
3.0 kg of
200 g aluminium calorimeter cup. The final temperature of
= water cooling
tile mixture is 3D.SOC. Calculate the specific heat of the alloy [
given C ,at = 900 Jlkg. from 20"C to T

Solution: We apply the conservation of energy and Hence,


write 10.5 + 167 + 0.50(4186)T = (3)(41B6)(20'C -T)
Heat lost bY) = (Heat gained). (Heat g~ined by ) which on solving yields T = S.l"C
( sample by water calonmeter
Example 49 _
msC.,AT. = m",C", AT", + m,'alC"al~TC"1
(0.150X', (540 - 30.5) = (0040)(4186)(30.5 - 10) A calorimeter contains 4.00 kg of water at 20.0"C. What
+ (0.20)(900)(30.5 -10) amount of ice at -1OOCmust be added to cause the resulting
mixture to reach thernlal equilibrium at 5.0"C. Assume that
76.4C, = (34,325 + 3690)
heat transfer occurs only benveen the water and ice.
C, = SOOl/kg °c
Solution: Let m be the mass of ice needed.
Example ~8_~ lIeat of raise ] [Heat to rhange]
m kg of ice from + m kg of ice
A 0.50 kg ice cube at -w"e is placed in 3.0 kg of coffee at [ -lOoC to DUC to water
20"C. What will be the final temperature and composition of
mixture? Assume thar specific heat of tea is same as that of Heat to raise ] [Heat 10>[ by 4 kg]
water. m kg of water = of water cooling
{
Solution: In this situation there are three from DOCto 5.DOC from 20°C to SoC
.possibilities regarding the final state of the mixture:
mic"C;ccAT + m;c.Lj + mioeCwAT = nl",C",AT
L All ice
m(2100)(D"C - (-lOOC)] + m(333) + m(4186)
2. A mixture of ice and water at O'C
(5.0'C - 0.0° C)
3. All water
= 4(4186)[5.0'C - 20.0'C]
Energy release required to bring the 3.0 kg of water
m(21DO)(1O) + 333m + m(210) = 251D kJ
at 20"C down to O"C,
Method 2: Heat required to raise temperature of ice
QI =m",C",(20"C-0"C)
from -lO'C to DOC,
= (3)(4186)(20)
QI :: mi""C ;ceAT
= 2S1 kJ
:: m(210D)[D"C - (-lOOC)]
Energy required to change the ice from -10'C to O'C,
= m(21DD) (lO) kJ/kg
Qz =m;"cC,cc[O"C-(-lOOC)]
Hear required for phase change of ice,
= (0.50)(2100)(10)
Qz :: m j""L j = m( 333) kJ/kg
= 10.5 kJ

www.puucho.com
Study Anurag Mishra with www.puucho.com
-,
THERMOOYNAMICS

Heat required to raise water (formed from ice) to final Thus nearly 87% of the original mass of the water can.
temperature of S.O"C, be frozen by intensive evaporation.
Q3 = nJi....,Cw.6.T
Example
"m(4186)(S"C - o.O"C)
= 21m kJ/kg
An aluminium container of mass 100 gm contains 200 gm of
Heat lost by water in cooling from 20.0"C to S.O"C, ice at -20"C. Heat is added to the system at the rate of
Q, "mwCwH" (4)(4186)(S.0"C - 20.0"C) 100 call sec. Find the temperature of the system after 4
~-2S1 k.J minutes. (specific heat of ice = 0.5 and L = 80 eal/gm, specific
heat of Al = 0.2 cal/gm/oC)
Since system is isolated from surroundings,
Q1 +Q2 +Q3 +Q4 =OJ Solution: Total heat supplied in 4 min = 240sec
m(2100)(lO) -+ 333m + 210m - 2510 = 0 at the rate of
Note that equations formed are mathematically same. 100cal/sec_ 100 x 240 ""24 x 103 cal
and heat required to rise the temperature of ice from
Example -200C to DoCand then melt it into warer
= 200x 0.5x 20 + 200x 80"" 18x 103cal
A vessel from which the air is rapidly being pumped out
contains a small amount of water at oec. The intensive So out of 24 x 103 cal, 18 x 103 cal will consume and rest
evaporation causes a gradl/a/freezing of the water. What part 6x 103 will utilize for increase the temperature of the
of the original water can be converted into ice by this method? system from DOC to ODe.
Solution: At any temperature, a layer of vapour is 6x 103"" 100x0.2x.68 + 200x 1x tl.8
formed over the surface of the liquid. An equilibrium exists 6x 10 3
"" (220)tl.8
between vapour and liquid. The vapour pressure at this
stage is termed saturated vapour pressure. When air is tl.0 =< 6000 =< 27.270C
pumped out, panial pressure of the vapour falls and funher 220
liquid evaporates to saturate it. The rate of evaporation can
be increased by anificially removing vapour from the
Example ~~
surface of the liquid.
A volume of 120 mL of drink (half alcohol + half water by
The latent heat required for evaporation comes from the mass) originally at a temperature of 25°C is cooled by adding
water itself thus freezing it. During freezing, latent heat of
20 gm ice at o°c. If all the ice melts, find the final
fusion is liberated which is used up for evaporation.
temperature of the drink. (density of drink = 0.833 gm/cc,
Suppose there is initially m kg of water present out of specific heat of alcohol =< 0.6caljgm(OC)
which ml kg evaporates and m2 kg freezes. If Lll and Lf are
respectively the latent heats of evaporation and freezing, Solution: 120 mL drink = 120 cm3
then we have ' Mass of the drink = 120 x 0.833 =< 100gm
miL" == m2Lf 100
And M alcohol = 2 =< 50gm
miLt,
or m2 ==--
L, M Water = 50gm
Since ml + m2 = m, the fraction of water that would SOx 0.6x (25-0) + SOx 1 x (25-0)
finally be frozen is =< 20x 80+ 20x 1 X 0

m2 = L" 30(25 - 8) + 50(25 - 0) =< 1600 + 200


m Lv + Lf 750 - 308 + 1250 - 500 = 1600 + 20e
Latent heats of vaporisation and fusion are Lt. = 539 2000 -1600 = 1000
caVg and Lf = 80 caVg, respectively. 400" 100e
m2 "" 0.87
e = 4°C
m

www.puucho.com
Study Anurag Mishra with www.puucho.com
TEMPERATURE, HEAT AND THE EQUATION OF STATE, HEAT TRANSFER
------- 29
Example 53 _

A $olid receives heat by radiation over its surface at the rate of


L\hA ",r~-1) Pk~
om
Pw
... (2)

where A is the cross-section area of the calorimeter.


4 kw. 71le heat convection ratefro11l the surface of solid to the
Substituting expression for tom from eqn. (2) in eqn. (0
surrounding i$ 5.2 kW, and heat is generated at a rate of 1.7
and using I1l1l' '" (h/3)p ••,A and mi,',> = (/lj3)Pi~ •.A, we get
kW over the volume of tile solid .. Tile rate of cliange of the
average temperature of the solid is O.5°es-J• Find the heat h T
C '"A ~r",... I h Picei',," C h
+ 'fAt.\ --- - i<"I.,PiceAT j",
capacity of the solid. 3 P", -Pice 3

Solution: Net heat gained by the sphere Hence T,~ =_!.L 30.h p", _ C,!:.. p", T
~'

= 4kW - 5.2kW + 17kW Ci,'~ It PI\' -Pke Cke Pice

= O.5kW = soow On substituting numerical values, we get


Tke -:::--54°e
And if SOOWheat generate in the sphere per second
mSdO =Q = 500 Example 55 _
dt
500
C "OmS =~- A copper calorimeter of mass 100 gm colltains 200 gm of a

(~~) ml"'(ture of ice and water. Steam at 100°C under normal


pressure is passed into the caiorimecer and the temperature of
= 500 = lOOOJ/oC the mixture is allowed co rise co 50~C. If the mass of the
0.5 calorimeter and its contents is now 330 gm, what was the
C = 1115= lOOOJ;oC ratio of ice and water in the beginning? Neglect heat losses.

Example 54 _ Given: Specific heat capacity of copper


=0.42x103Jkg-1K-1

Two iden6cal thermally insulated cylindrical calorimeters of Specific heat capacity of warer
height h = 75 em are filled to one third of their capacity. One = 4.2 x 103 J kg -1 K ~1
calorimeter is filled with ice formed as a result of freezing Specific heat of fusion of ice
water poured in it, and the second is filled with water at ==3.36xlOsJkg-1
T", = 1O"C. Water from the second calorimeter is poured into
the first one, and as a result it becomes to be filled to Latent heat of condensation of steam
two-thirds. When tile temperature is stabilized in the first =22.5x lOs J kg-I
calorimeter, its level of water increases by toh = 0.5 cm. The
density of ice is P,a = 0.9P"'Qfrr> the latent heat of fusion of
Solution: Suppose in the starting the mass of the ice
was m in 200 gm mixture of ice and water i. e.,
ice is L = 340 kJ/kg, the specific heat of ice is 2.1 kJ/(kg. K)
and the specific heat of water is C = 4.2 kJ/(kg. K).
Il' Mass of ice = m gm, mass of water = (200 - m) gm and
Determine the initial temperature of ice in the first finally the mass increases by 30gm that means 30 gm steam
calorimeter. condensed and temperature of 30 gm condensed water falls
from IOooe to 50~e so heat lost by 30 gm steam for
Solution: Water shows anomalous expansion; the condensed and then drop it's temperature by 50 e will 0

volume of water increases during freezing. If the entire utilize for melt m gm ice and rise the temperature of
mlume of water would have frozen then its volume would ice-water mixture by 50~e and also rise the temperature of
have increased by a factor of ~ ::::1.1 and the level of water calorimeter of copper by 50~e initially ice-water mixwre
P'c< was oce.
lin the calorimeter would have increased by (i) Heat required for melt the m gm ice '" 336 x m
:!l/3)(1.1-1)-:::-2.5clll. But according to the problem,
(ii) Heat required for rise the temperature of m gm
,\h '" 0.5 em, it implies thar a part of water has frozen. Thus
water (from ice) by 50~e = m x 4.2 x 50
(he calorimeter has ice mixture at o°e.
Ileat lost by water = Heat gained by ice (iii) Heat required for (200 -Ill) gm water to rise
temperature by 50 e '" (200 - m) x 4.2 x 50
0

m ••.C",(1~. - 0) + .1mLf ",Cie ••11lic'e(O-Tice) ... (1)


(iv) Heat required for rise temperature of 100 gm
where tom is the mass of the water tbat free;:es. As the
copper calorimeter by 50 e = 100 x 0.42 x 50
0

volume of water changes by a factor of p", Ipir~ on freezing,


we have

www.puucho.com
Study Anurag Mishra with www.puucho.com

30 IHERMODYNAMICS

Net required heat 880=10xs\ + 2)+ lOx L+ lOx52 x (I-Om)


X (Om
= 336m + nIX 4.2)( 50+ (200-m) Here Om is the mehing point so first after gi\ing 880 call
x4.2x 50+ 100 x 0.42)( 50 [10x.5\ x (0", + 2)] heat will increase temperature from _2°e
to Om °C and then it will melt so out of 880 cal some wm
= 336m + 200x 4.2x 50+ 100 x 0.42)( 50
utilize [0 melt it and then if Om °C is the melting point. Mter
= 336m + 42000+ 2100 melting it's temperature will start to rise from Om 'e to 1°C so
= 336m + 44100 some energy out of 880 cal will utilize to rise the
Net heat gained by steam temperamre of melting substance.
= 30)( 2250+ 30x 4.2 x 50 880=lOxsl" (Om +2)+ 10><L+ lOx 52 X (1-0",)

= 73800J 880= 1O"Q8x (0", + 2)+ lOx L + lOx 1 X (I ~nm)


854+ 20
Heat supplied = Heat gained 10L = 854+ 20m, L = '" ca1;gm
10
And value of L in callkg will be
336m + 44100 = 73800
::::> 336m = 29700
L = 85400 + 2000m

=> m = 88.39 Example 57,...--


If m = 88.39(ice)
then water (200 - m) = 111.60 gm A steel drill making 180 rpm is used to drill a hole in a block
Ice : Water = m : (200 - m)
of steel. The mass of the steel block and the drill is 180 gm. If
the entire mechanical work is used up in producing heat and
-8839: lIL61-1: L26 the rate of raise in temperatw'e of the block and the drill is
9.5°C/sec. Find
Example 56~
(a) the rate of working of the drill in waW, and
A solid substance of mass 10 gm at _woe was heated to (b) the torque required to drive the drill.
-2"C (still in the solid state). The heat required wetS 64 Specific heat of steel = 0.1 and J = 4.2 Jleu!.
calones. Another 880 calories was required to rai.~e the Use:P=nll
temperature a/the substance (now in the liquid state) to loe.
while 900 calories was required to raise the temperaturejrom Solution: (a) If temperature of 180 gm steel system
-2"C to 3°C. Calculate the specific heat capacities of the [Drill + Block] increases by 0.5°C per sec then heat
substances in the solid and liquid state in calories per supplying or creating by work will be
kilogram per kelvin. Show that the larent heat affusion L is W = H = 180 x 0.1)( 4.2 X 0.5 = 37.8Jjsec
related to the melting point temperature t In by
Here specific heat of steel is 0.1 caVgm- °C and in joule
L = 85400 +200 tm•
it will be OA2J/gm-OC so work done by drill will convert
Solution: According to problem. 10 gm solid require into the heat will be 37.8watt.
64cal to rise temperature 8°C then specific heat 51 will be (b) Torque required to drive the drill_ because 37.e
(of substance in solid state).
watt power is obtained by the drill so 37.8 J work has to de
10xslx8=64 the drill per second therefore drill is delivering the power a;
~ 5\ = 0.8cal/gm-Oe the rate of 37.8 watt.
s\ = 800eal/kg-Oe ...•• --> •.•.• •.•.•
And P = F. V same P = t. coso for delivering the P power
And if 880 cal require to rise temperature _2°C to 1°C ~
but at 1°C it is in liquid and heat require _2°C to 3°C, 900 t will be t=P/w

cal. Here drill rotating by 180 rpm = 180 revolution per


So heat require PC to 3°C, 20 cal in the solid state minute, if 180 revolution done in 60 sec (l min) then in 1
20 =10x S2 x 2° sec 3 revolution means 6n.
(Specific heat of substance in liquid state) w = B/t so (0 = 61trad/ sec
S2 = 1eal/gm-Oe = 1000~ L = 37.8 - 2.006N-m
kg_Oe 6x 3.14
After giving heat 880 cal some heat will be utilize for
rise the temperature, and some of heat will melt the
substance (it will utilized as a latent heat of fusion)

www.puucho.com
Study Anurag Mishra with www.puucho.com

TEMPERATURE, HEAT AND THE EQUATION OF STATE, HEAT TRANSFER 31

Example 58 _ VI =11O[1+(25-100)xlO-3)=101.75cm3
and 300cm:1 toluene \vill increase it's volume and it will
A jIow calorimeter is ll,~ed to measure the specific heat of a become
liquid. Heat is added at a knuwn rate to a scream of the liquid V2 = 300[1 + y(25 - O)J
as it pmses through the calorimeter at a knowll rate. Then a
= 300[1 + 25x 10-3] = 307.5c013
measurement of the resulting temperature difference between
the inflow and the Otllj101V poinr.~ of the liquid stream enables And now net volume V = VJ + V2 = 409.25 em 3

1I.~to compute the specific heac o/the liquid. A liquid of density 3


So final volume = 409.25cm = Vf
0.2 glcm3 flows through a colorimeter at the rate of
IDcm3 Isec. H('ot is added by meQns of a 250 W electric And total initial volume = 41Ocm3 = VI
heating coil, Qnd a temperature difference of 25°C is
Net reduction in the volume /,\V = (V,- - Vi)
establisl1ed in steady-stare conditions between the inflow and
the outflow points. Find the .~pecific hear of the liquid. = 409.25 - 410
= -0.75cm3
Solution: Rate of flow of liquid in gm/scc is
O.2gmjcm3 x lOcm3 sec = 2 gm/sec and if 2 gm liquid Example 60 _
entering per sec then 2 gm liquid will out per sec after
receiving 250 J heat energy in one sec. e
lee at -20o is filled upto height h = 10 em in a unifonn
230=2>0:5>0:25, cylindrical vessel. Water at temperature O°C is filled in
-' = 5J/gm-CC another identical vessel upto the same height h = 10 em. Now,
water from second vessel is poured into firs( vessel and it i.~
= 500 J/kg-OC
found that level of upper surface falls through /'\h = 0.5 em
Example 59 _ when thermal equilibrium is reached. Neglecting thermal
capacity of vessels, change in density of water due to change in
temperature and lOS$ of heat due to radiation. Calculate
Toluene liquid of volume 300 cmJ at ooe is contained il1 a
initial temperature 0 of water.
beaker an another quantity of toluene of volume 110 cmJ at
100°C i-, in another beaker. (The combined volume i5 410 Given, Density of water, P ••. = 1 gm em -3,
J
em] ). Determine the total l'olume of the mixture of the Density of ice, Pi =0.9gm/cm
toluene liquid" when they are mixed together. Given the Specific heat of water, Sw = 1callgm-OC
coefficient of volume expa1l$ion'f = 0.00 Ire and allforms of
Specific heat of ice si = 0.5 eal/gm-OC
heat losses can be ignored. Also find the final temperature of
the mixture. Specific latent heat of ice, L = 80 callgm

Solution: If suppose density of toluene of O"C is Po' Solution: When the \','ater at SOC is powered into the
3
Then volume of 110 cm toluene at 100°C will be change ice vessel then when the thennal equilibrium reach the
and become Vo at E)°C then temperature of wClter will down from ijQC and due to this
temperature change if there is no change in density of water
110 = Vo(l + 'I x 100)
therefore \'olume of water remain same but at this stage
= \"0[1 + 10-3 x: 100)
total fall in height /'\11 =0.5cm it is only due to that some of
110 = Vo[l.l] =::> Vo = 100cm3 the ice will melt and convert into water.
So ice column height (h = IDem) will change to (h -""II)
Total mass will be (300po + 100Po) = 400po at O°C and
= 9.5 em and at this moment not the complete column of ice
mass will remain constant with temperature
will melt but some column height (y) out of h will melt y =?
So if (lOOpo) mass toluene will rise the temperature
from O°C to 100°C lfvessel cross-section Clrea is A then mass ofy height ice
Then mass will also remain (lOOpo) but volume from M icc = 1\1 waler
100c013 at O°C will change to 110c013 at 100°C. AXYXPicc =Ax(y-o.5)xflw

at mixing =:> pD.9oCy-D..5)


100po x sx (l00-8) = 300po x sx 0 ~ y = 5cm
(lOa - e) = 30 => 40 = 100 => e = 25°C Means 5 cm height ice will melt out of 10 cm height ice
and it will convert into 4.5 em height water.
temperature of [he mixture
So at equilibrium temperature 110 em 3 toluene which is
at 100°C will reduce it's volume at it will be

www.puucho.com
Study Anurag Mishra with www.puucho.com

THERMODYNAMICS I
If (A x 10 x P IV) gm water powered into ice finally it will $0total heat required to melt ice from -ISOto DoC
come to DOC temperature and it will be equilibrium water
temperature. :=750+ 8000:= 8750cal
total heat lost by ice = heat used by ice to temperature And heat can consume from 250 gm of water at 25°C is
gain + to melt ice :=250x 1 x 25 = 6250 cal
Axl0xp",x(O-O) $0 6250 cal is less the required heal so all the ice can not

=Si x lOx A x Pi(O+ 20) + 5>< Ax PiX L melt into the water. Out of 6250 cal heat 750 cal will use to
IOAp",O =Si x lOx A xPi x 20+ 5x A xPi xL rise the temperature upto DoCfrom -15°C, and rest of 550(}
lOx 1 x 0 = 0.5x lOx 0.9x 20+ 5><0.9>< 80 cal will utilize for melt the ice
tOn = 9><10+ 360= 90+ 360 = 450 5500:=mx80
8 = 45"C 275
m=--
4
This is the amount of ice which will be melt from DOCice
to O°C water so remaining ice in the mixture
Two identical calorimeter A and B contain equal quantity of
water at 20ce. A 5 gm piece of metal X of specific heat 0.2 100- 275:= 125gm
cal g -I (OCr' is dropped into A and a 5 gm piece of metal Y 4 4
into B. The equilibrium temperature in A is 22"C and in and water 250 + 275 "" 1275 gm
8 is23°C. The initial temperature of both the metaL~is 4D"C. 4 4
Find the specific heat of metal Y in cal g -1 l
. coer And the mixture will be at the DoC.
Solution: Given, nJx::: Sgm, my = Sgm, Example
._.--~. - .. -- -. -
Sx =O.2cal/gm-OC, Sy =?
Water is heated from lOOC to 90°C in Q residential hot water
T" =20'C Ta '" 20'C heater at a rate of 70 litre per minute. NatJ.Iralgas with a
T.=40'C Tv'" 40"C density of 1.2 kg/m3 is used in the heater, which has a
transfer efficiency of 32%. Find the gas consumption rate in
TeQu~lb(",m T~;Drium
cubic meters per hour. (heat combustion for natural gas is
= 22"C = 23'C 8400 kcaVkg)

A B Solution: 70 litre water = 70 kg water = 70x 103gm


Fig.1E.61 water and to rise the temperature of 70 kg water from 10"C
to 90"C. Heat required
mx xsx x (40-22) :=Mw xSw x 2° ...(1) 70x 103 x 1 x 80= 56x 105 cal = 5600 kcal
My xSy x(40-23):=Mw xswx3° ...(2) $0 5600 kcal heat required per minute to rise the
(Mw)A :=(MwJs :=Mw temperature from 10°C to 90°C.
From eqn. (1) and (2)on solving 5600kcal -t 5600kcaljmin
27 $0 amount of gas which has to be combustion to
sy:=-caljgmOC
85 produce 5600 kcal. heat if only 32% of heat can transfer to
water
Example 100
5600x -kcal = 1.75x 104 kcaljmin heat will.
32
Two 50 gm ice cubes are dropped into 250 gm of water in a extracted from natural gas and if density 1.2 kglm sand 1
glass. If the water was initially at a temperature of 25°C and kg produce 8400 kcal heat then heat produce in (kcal/ m 3)
the temperature of ice -15°C. Find the final temperature ofi will be 10080 kcaJ/m3 so amount of gas (ms/min) will be
water. (Specific heat of ice = 0.5 caVgmOC and L= 80
caVgm) consume to produce 1.75 x 104 kcal/min.
4
Solution: Heat required to rise the temperature of Heat per min = 1.75 x 10 m 3/min :=1.73m 3/min
10080
100 gm ice from -15°C to O°C:= 100 x 0.5 x 15:= 750ca1
S
And if 1.73 m gas consume in one minute then in 6Q
And heat required to melt the 100 gm ice at DoC, in
water at O°C is minute the gas amount will consume 1.73 x 60:= 104.16
mS/hr.
= 100x 80:= 8000ca1

www.puucho.com
Study Anurag Mishra with www.puucho.com

TEMPERATURE, HEAT AND THE EQUATION OF STATE, HEAT TRAI/SFER 33


-----
Example 64__ Then rate of heat supplying per sec will be = ~ caVsec
9
A substance is in the solid form at O°C. Tile amount of heat So total heat supplied in 10 hours
added to this substance and its temperature are plotted in the 2
= lOx 60x 60><- = 8000 cal
following graph. If the relative specific heat capacity of the 9
solid .'iubstance is 0.5. find fturn tile graph And 8000 cal melts same amount of ice (l00 gm) then
heat required to melt I gm of ice will be 80 caVgm . It is
c
latent heat of fusion of ice
I
120
100 SOcal/gm = BOkealjkg
A B
E 80
IDEAL GASES
- 60
! 40
20
Solids and liquids are nearly incompressible. Atoms are
fairly "hard" and can not be pressed together once they
come into contact with each other. Atoms also resist being
200 400 &00 800 'OOO1200WlO 1600'6002000 2200
Q_(calories)
pulled apart. In solids the atoms held together by attractive
forces. These attractive forces are responsible for the
Fig.1E.64 strength of solids-It is far easier to break a solid or a liquid
than it is to compress it, so these attractive forces must be
(i) the mass of the substance;
weak in comparison to the repulsive forces that occur when
(ii) the specific latem heat of the melting process, and we push the atoms close together.
(iii) the specific heat of the substance in the liquid state. An atom is a small particle that is attracted to other
nearby atoms but strongly repelled by them if it gets too
Solution: (a) M x O,Sx 80 = BOO,
close. Fig. 1.20 shows the potential-energy diagram of two
m = 20gm = O.02kg. atoms separated by resistance r. The force exerted by one
Here 0.5 is the specific heat of substance in the solid atom on the other is the negative of the slope of this graph.
state because if 0.5 is relative specific heat then it is relative The slope is large and negative for a separation of r less than
\V.r.t.water and specific heat of water is 1 caljgm-OC so of the equilibrium valUe req, so the force for r < req is large and
substance 0.5 cal. gm-OC resistive. For r just slightly greater than req, the positive
(b) A ~ B heat required = 800 cal slope indicates a weak attractive force. The ,slope has
become nearly zero by r ""0.4 nm, hence attractive force is
SOO=20xLj
restricted to atoms within about 0.4 nm of each other. An
::::> 40cal/gm = 40000cal/kg
Lj = atom separated by more than about 0.4 nm do not interact.
(c) Temperature increases from B to C is 40°C and so
• ,,,',,
heat supplied from B ~ C is 600 cal. ,
600 = 20 x sl x 40 :01.,."
,, ''
u ,
,
'
',
sl = ~ eal/gm-OC
4 : : large repulsive force
: _,.-~"--'forr<req
= 750eal/kg _°C ~-
, .'
:
sl = 750ea~lkg - K ,, '
,, '
'
: : _' _- Weak attract,ve force
Example 65~ : :,..- forr<req
,, ''
, :,
A vessel containing 100 gm water at DoC is sw.pended in tile ,,
middle o/a room. In 15 minutes the temperature of the water ,,
rises by 2°C. When an equal amount of ice is placed in the Zero force
vessel, it melts in 10 IlOurs. Calculate the specific heat of for r <0.4 nm
r(nm)
fusion of ice. 00 02 0.4 06
Equllibrium position req
Solution: The amount of heat required to rise the
temperature of 100 gm of water from DoC to Fig. 1.20: The potential energy diagram
2°C = 100x 1 x 2 = 200 cal. for the interaction of two atoms.

And 200 cal heat is supplied in 15 min (900 sec)

www.puucho.com
Study Anurag Mishra with www.puucho.com

34 THERMODYNAMICS

Solids and liquids are systems in which the atomic pV(J) The graph of pV against nT
separation is very close to 1",'<1' thus the attractive and is a straight line with slope
R" 8.31 J/mol K.
repulsive atomic forces are balanced. If you try to pull the
atoms closer together, the repulsive forces resist. If you try to
pull them apan the auractive forces resist.
In A gas IS less dense and the average spacing of atom is
much larger than r,,'1' Consequently, the atoms are usually
not interacting with each other at all. Instead, they spend
most of their time moving freely through space, only
occasionally coming close enough to another amrn to
interact with each other. nT(mo! K)
An ideal gas is a collection of small, hard, r<Jndomly
moving atoms that occasionally collide and bounce off each Fig. 1.22: A graph of pV •••
ersus nTfor an ideal gas.

other but otherwise do nOl interact. Non~theless,


It is customary to write the relationship in a slightly
experiments show that the ideal-gas model is quite good for
different form, namely
gases if tw"oconditions are mee
U PV = nRT (ideal-gas law) ... (1)
EqUation (1) is the ideal gas law.

Atoms bounce when their


The constant R, which is determined experimentally as
centers are -0.1 nm apart. the slope of the graph in Fig. 1.22 is called the universal
gas c~nstant. Its value, in 51 units, is
R = 8.31 J.mol K
All gases have he same graph and the same value of R. A
very simple noble gas such as helium should the same slope
." Atoms do not interact as a more complex gas such methane (CH4). Both have the
at all if r > rcont!",,1 same value for R. The Ideal gas law, within its limits of
r(nm) validity,describes all gases \•...
ith a single value of constant R.
0.0 0.2 0.4 0.6
(n this book, we will l'Onsider only gases in sealed
Fig. 1.21: An idealized hard-sphere model of the containers. The number of moles (and number of
interaction potentia! energy of two atoms, molecules) will not change during a problem. In that case,
PV
1. The density is low (i.e., the atoms occupy a volume = nR = constant
-
much smaller than that of the container), and T
2. The temperature is well above the condensation If the gas is initially in state i, characterizes by the stare
point. variables PI' V; and Tj and at some later time in a final state I,
the state variables for these two states are related by
If the density gets tw"ohigh, or the temperature too low,
Pi Vi P-V,
then the attractive forces betw"een the atoms came -- • _'_1 (ideal gas in a scaled container)
interaction. These are the forces that are responsible under Ti 1';
the right conditions, for the gas condensing into a liquid.
The mathematical telation between the state variables
As for as rotational motion is concerned, the ideal-gas of a system is called the equation of state.
model does not distinguish between a mono-atomic gas and
Equation of State for an Ideal Gas
a diatomic gas; both are considered as simply as hard
spheres. Hence the terms atoms and molecules can be used PV=nRT ... (1)
interchangeably to mean the basic constituents of the gas. In Eqn. (1), n is the number of moles of the gas and R is
The Ideal-Gas Law a constant called the universal gas constant.
Experiments show that there is a specific relationship R ==8.314J/moVK
between the four state variables. == 0.0821 L atm/moVK
There is a very dear proportionality between the == 1.986 caVmoVK
quantity PVand the quantity nT. Ifwe designate the slope of
the line in this graph as R, then we can write the relationship
as
PV == R x (nT)

www.puucho.com
Study Anurag Mishra with www.puucho.com

TEMPERATURE, HEAl AND IHE EQUATION OF SlATE, HEAl lRANSFER 35

GAS LAWS V

Boyle's Law: Relation Between Pressure and Slope = Yon


Volume of a Gas
At constant tempenlture, volume of a fixed mas~ of a gas n =Yo
1 [.W]
tit

is inversely proportional to its pressure. 1


= 273

pL
Isotherms
-273 0 tOG
P
o 273 TK

Fig. 1.24

~~;T, Thus VI = Vo + Voal = Vo(l + vt)

V IN u. is called coefficient of expansion. It is approximately


1
(a) PV = cons!. (b) p = CO~SI. - for all the gases.
273

V=Vo (1 + -'-)
273
Thus, an increase the temperature of a fixed
volume of a gas at constant pressure increases the
I
1/P logY volume by -- of the volume at O"C.
273
(e) V = co~st. (d) log P = const. -logY
V Const. P V Gonst. T

V
Fig. 1.23

1~-
1
P
"':t<
.... n,.
~'.'
P
l~~."." V •

T T T
PV '" const.
Fig. 1.25
P1 VI = P2V2 CT, n arc con~lanls)
Charles' Law: Relation Between Temperature and
Volume of a Gas
VI = Vo [2~37;r];
when a gas is healed at constant pressure, its volume is Gay-Lussac's Law
a linear function of the temperature and can be expressed by
When the temperature of a gas is changed keeping the
the equation for a straight line
volume constant, the pressure of the gas changes. Similar to
V = mt + C volume, the pressure changes by 1/273 of its value
where t is the temperature in °C and III and Care at O"C for each one degree change of temperature.
constants. The intercept on the vertical axis, C, is Vo which is PI =Po(l+J3t)
the volume at t = O"C. The slope of the line is III '" hoV where p is called pressure coefficient of expansion, and
6'
it is approximately 1/273.
Thus
P Const. V P Isothermal
=Vo+(~~}
Vr

-L'!.V.= Increase .In voIume per degree


M

u. = - 1 -OV = re I..
Vo L'!.t
Cn,pareconstants)

. vo Iwnc per degree


atlve mereao.e In
Pl== T
Fig. 1.26
T T

p=
I
Po
[M]
M
I
= 273

www.puucho.com
Study Anurag Mishra with www.puucho.com

36 THERMODYNAMICS

P =P,[1+_'_J:p(273+t)
0273°273
f
N/m 2) and volume in cubic metres, the product PV has units
of newton metres, or joules, and R has the value

or
P, = T R "" 8.315 Jjmol K
Po To . If the pressure is expressed in atmospheres and the
volume in litres (lL = 103 cm3 = 10-3 m3), then R has the
Dalton's Law of Partial Pressures
value
This law states that the total pressure exerted by a
mixture of non-reacting gases is equal [Q the sum of the R = 0.08214 L atm/mol K
pressures which each component would exert if placed Using this value of R we find that the volume occupied
independently in the container. by 1 mol of any gas at atmospheric pressure and at orc (273
Ptmal = PI + Pz + P3 + ... CT, Vare constant) K) is 22.4 L.

Ploral = L, Pi MICROSCOPIC VIEW OF THERMODYNAMICS


where the symbol I stands for the summation over all An ideal gas is a collection of point-like particles obeying
ideal gas equation of state
the components presenf in the mixture. "
PV = nRT
Nitrogen The following assumptions apply to an ideal gas;
"d 1. The volume V containing the gas is large as compared to
Oxygen
total volume actually occupied by the gas particles themselves.
It implies that separation of the particles in the gas is large
Fig. 1.27 compared with their size and that long range forces benveen
the particles are negligible; particles are point-like.
The partial pressure P; of component i is defined as
2. Newton's second law of motion is applicable. The
the pressure that the gas would exert if it were present alone
particles move at constant velocity between collisions with
in the same volume and at the same temperature.
either walls of the container or each other.
nlRT
PI =-V 3. The particles are equally likely to be moving in any
direction.
n2RT
Pz =-V- 4. The gas particles interact only via collisions.
5. The gas is in thermal equilibrium with its surroun-
The total pressure of the system can be written as dings.
PIOla!
,
= I Pi 6. Particles are identical and indistinguishable.
Kinetic Theory
Fig. 1.28 shows a piston in a cylinder of length Land
cross-sectional area A. The pressure exerted on the piston is
and partial pressure of irh component can be written as the result of the large number of elastic collisions on it by
n gas molecules each and every second. The motion of
Pi = PIOlal --'-
ntotal

when is the mole fraction of the respective

component
Equation of State (-(
The equation of state for an ideal gas:
PV =nRT
In this expression, known as the ideal gas law, R is a
universal constant that is the same for all gases and T is the
absolute temperature in kelvins. Experiments on numerous
>----L--O<
gases show that as the pressure approaches zero. the
(OJ (oj
quantity PVJnT approaches the same value R for all gases.
Fig. 1.28
For this reason, R is called the universal gas constant. In
51 units, in which pressure is expressed in pascals (l Pa = 1

www.puucho.com
Study Anurag Mishra with www.puucho.com

TEMPERATURE, HEAT AND THE EQUATION OF STATE, HEAT TRANSFER

molecules can be resolved into its X-, yo, z-components. The


------ Nm(v2)avg
37

Thus, p=---
collisions are perfectly elastic (KE is conserved) and so each 3LA
time a particle bounces off the piston, its velocity's y- and
z-component parallel to face of the piston remains Nm(I,2)avg
=
unal[(~red; only the x-component changes reversing itself 3V
from +V" to -V ..•. where V = L A is the volume of the cylinder.
Change in momentum of single particle, I1P~~ 2rnvx Since Nm is the total mass of the gas, the density
Each molecule moves along the length of the cylinder at p = NmjV and
a speed t'.~'which is constant evell after collision with side
P =.!.p(1'2)
3
walls. Hence, time taken to travel a distance 2L (to the far avg
2L
face and back), M = - PV ==.!.3 Nm(I,2) ayg
l'x

This is the time between collisions. The reciprocal of this


time is collision frequency (number of collisions per The tr~nslational kinetic energy is given by.! mv2•
2
second). 2
Rate of change of momentum, PV =- N(KElayg
3
11P~ = change of momentum per collision x number of 2N
P =- - (KEtnns )avg
v
'"
collisions per second
3
The pressure of the gas is proportional to the number of
== 2m!'x == ml~; particles per unit volume and the average rranslational
2L l'x L kinetic energy of rhe molecules.
From Newton's second law, this equals the average force 2
as, PV=-N(KEtrans)avg
exerted by a single panicle on the piston. Assuming that all 3
the molecules in the cylinder behave in a similar manner, the combining this equation with the ideal gas equation of
total average force due to N particles is stare PV = nRT, we have
Nnw; 2
Fav8 =-L- nRT == - N(KE[rans )avg
3
By definition, the force per unit area is the pressure 2
= '3 nN A (KE[ranslavg
NmL,2
P=--' where n is number of moles, N A = Avogadro's number
LA
= 6.02x 1023 particles/mol.
Total speed of the particles is lJ2 = v2, + v2Y + v2, R 2
N T='3(KE[ranslavg
Applying assumption (3) of the kinetic theory A
222
v .•. =vy=vs'
(KEnans) ayg = 2"3 kT
and so, L,2
,
=.:!.3 1,2 The constant RjN A is called Boltzmann's constant.
Nmp2 Thus, the temperature of the gas is a direct measure of
Accordingly, P = -- the average translational kinetic energy of the ideal gas
3LA
molecules.
This would be pressure if each molecule moved with the The total energy for N panicles is the sum of their
same speed, which is not true. Maxwell and Boltzmann
individual kinetic energies
showed that there is a temperature dependent distribution
of speeds. U = N (KE trans l avg
3
We will use an average speed in the pressure equation in NkT
=-
place of v2. The sum of the squares of the speeds of each of 2
the N particles divided by N is the average or mean of the It follows from this equation that
speed squared. 2 3kT
(v lavg =--
m

www.puucho.com
Study Anurag Mishra with www.puucho.com

38 .,.. . , THERMODYNAMICS
which is the mean of the speed squared. If we take its We can interpret this result as follows:
square root for we get the root of the mean of the square Energy associated with the x-component, y-component
(rms for short)
and z-components of motion is same; there is equal
lJrms =~(v2)avg distribution of energy for each independent motion of a
molecule.
=~3~T The term degrees of freedom implies restrictions or
limits. In classical physics the term degree of freedom refers
The Equipartition of Energy Theorem: to number of generalized coordinates to specify the motion
A good model for an atom in a monoatomic ideal gas is a of all mass points of a system.
rigid spherical ball: a hard, round, structureless object, "Degree of Freedom" is the correct technical term, but
capable only of linear motion. The atoms continually collide "Mode of energy storage" is better way to describe it.
with each other, changing the directions of their velocities A point-like bead on a wire has 1 degree of freedom. A
and exchanging energy. As a result, the averag~. kinetic large bead that can spin about and slide along the wire has 2
energy per atom,
degrees of freedom. Because (KEavg) = ~ kY, it follows that a
K =.!. m (v 2) 2
2 mean energy ..!. kT
is associated with three principal
1 2 2 2 2
=-m (vx> + (1!y) + (vz),
2 directions or three degrees of freedom. The three mean
energies for X-, y., z-motions are equal; there is equal
consists of three equal parts:
division of mean energy among the degrees of freedom; this
..!m{t,2) =.!.m{v2) is called equipartition.
2 x 2 y
The contribution to the internal energy of a gas is not
1
= - m(v;) = Kj3 solely due to translation. The internal energy of a gas
2 includes contributions from the translational, vibrational
Since the total kinetic energy K of each atom has an and rotational motions of the molecules.
average value of.:! kT, each of the three parts has an average A billiard ball gas is a good ~
2 model for helium or argon but
value of.!. kT, Each part is a mode of energy storage, or degree other common gases have
2 more complex molecules. For
of freedom, for the gas and internal energy is distributed example, oxygen and nitrogen
equally among the modes. molecules are diatomic and are Fig. 1.29: An ammonia molecule
is pyramidshaped
Previously we have discussed that more like dumb-bells than like
spheres. Ammonia molecules are pyramids (Fig. 1.29).
(v;)avg =(v~)avg =(v~)avg
Rotation and vibration of such molecules offer additional
2
(v )al'g = 3(v; )avg ways to store energy.

m(v; )avg 1 A polyatomic molecule rotates about its center of mass


---=-kT and its rotational energy is also the sum of three terms:
2 2
1 2 2 2
as ..!2 m(v2) = ~ kT Ero! = '2 Ux oox +lyW.I' +Izoo;)
avg
2
Just as for translational energy, each of the three terms
-1 m. 3(vx2)avg =-
3 kT
corresponds to a possible mode of energy storage.
2 2
We model a
or %m(V;)avg=~kT diatomic molecule as
two billiard balls
1 2 1 oscillating on the
Similarly '2m(vy)avg ='2kT
ends of a spring (Fig.
1 1.30). As the balls
and 2
-m(vz)avg =_1k T
2 2 oscillate, they have a
~
relative displacement Fig. 1.30
Thus the total kinetic energy may be expressed as
1212123 S and corresponding The energy of the molecule includes the ki-
netic energy of the vibratingatoms and the
-m(vx)avg +-m(vy)avg +-m(L':)avg =-kT relative velocity
2 2 2 2 potential energy of the "spring"joiningthem.

www.puucho.com
Study Anurag Mishra with www.puucho.com

TEMPERATURE, HEAT AND THE EQUATION OF STATE, HEAT TRANSFER 39

sufficiently energetic 1() excite the Illode and vcry little


ds'/ dt = v',. The molecule's vihrational ener~y varies
energy is 5lOred in lh,lt mode.
between kinetic and potential forms
1 ~ 1 , ., Concept: III t!lermal equilibrium, each gas molecule
2
E'Lh = 2b;- ~ 1111'.;
Iws, VIl QI'crage, ! kr of (,1lt'n~y ill cacll exciled mode of ener&.'Y
2 .
Each of the two terms here corresponds to ail el1el~Y sloruge.
storage mode. More complicated ll10lenlks vibrate \'Ii'i,h
I~'quiparlirion (~ff'ltergy tileorem states that there is
combinations of such simple oscillations. Since each
oscillation provides two storage modes, the number of associated an aveJ'age energy of! kT per molecules
2
vibrational energy storage modes of a polyatomic lllo1l'cu!c
with each degree offreedom of the molecules in the
is twice the number of ways it can oscilbte.
system.
Concept: Collisions transfer energy between modes and
ensure that, on average, no mode srores more energy than any Ideal Monoatomic Gas
other. Off-center collisions, for example, Catlse the molecules The 1'Ilergy of point.likc p.micles of a mono<ltomic ideal
to rotate a.~shown in Fig. 1.31. Similarly, a rotating 1I1oleCilIe gas is solely due 10 translation.
causes another molecule to rebound like a ba._eball Mnlck by a 1 ., ~ 2
U = - IJI(I'~ .~,.- -+- I")
rotating hat. 2 ' -" '
Polyatomic molecules al.~o rotate as a ,esult of off-cemer Therefore. according tIll'
equipartition
10 energy
collisiO/1$ among them. Diatomic molewles do not rotate
about their longitudinal a.:ris because it is almost impossible
theorem, the <lvct"<lgeenergy per molecule 15
. '1 )
kT For a 3l'2
to hit them at points Tlor on this (uis. While rhese models help gas of N molecules, the total energy is
us to visualize [he idem, quantum mechanics goverm the
U = ~ NkT
be1lavio, of lhe molecules and delermine$ those modes that 2
are excited. For one mole, N A = Avogadro number, number of
An individual moleclile may have any amount of energy panicles
in translation and in rotation, but averaged over all the u =~ RT
molecules, there is an equal amount of energy in each mode. 2
A molecule has three modes of translational kinetic energy, Ideal Diiatomic Gas
accounting for an internal energy Unan, =~ kT, or! kT per The shape of an ideal diatomic molecule is like a
2 2 dumb-bell. The centre of mass of the molecule can translate
mode for each molecule. At high enough temperatures, in the X-. y. and z- directions. The molecule can rotate about
rotational and vibrational modes also store, on average. three mutually perpendicular axis. Because we have
.!. kT each per molecule. modelled the individual atoms as point masses, the moment
2 of inertia about the symmetry axis through the line joining
the masses is zero; thus we ignore rotation about this axis.
So, if M is the total mass of the diatomic molecule. the
expression for the total kinetic energy of the molecules is

U:o ~M(t,; + v; v;)


-+- + ~(llwf + I 2{()~)

Fig. 1.31 There are five degrees of freedom of a rigid diatomic


molecule. For a rigid diatomic gas of N molecules the total
A molecule does not always exhibit all possible modes of energy is
motion (we say that some modes are not "excited").
According to quantum theory, there is a smallest amount of U =~NkT
2
energy, called a quantum that can exist in any mode. Each
mode has a different size quantum. For vibrations, it is 5
U = - RT (for one mole of gas)
proportional to the frequency of the vibration and for 2
rotations, it is inversely proportional to the molecule's Some diatomic molecules. such as NO and e12• are not
rotational inertia. The quantum of translational energy is very rigid; their equilibrium separation oscillates about an
negligible. If the size of the quantum for a particular mode average distance even at room temperature. They are
exceeds.!:. 2 kT, only• a ven; modelled by a two particle system connected with a spring.
., small fraction of collisions are
In addition to five degrees of freedom needed to describe its
translational and rotational kinetic energies we need two

www.puucho.com
Study Anurag Mishra with www.puucho.com

40 .0;,'. ~ •• ,I ,
THIRMODYNAf!lICS

more: one to designate the velocity of one molecule relative For a Vibrating Diatomic Ideal Gas
to other and other for a relative separation coordinate 7 7
required to express the potential energy between the two U=-NkT=::-nRT
2 2
a[Qrns. These are called vibrational degrees of freedom.
Therefore, the total number of degrees of freedom is 7 and
average energy for N molecules in the gas is
so that, C
"
>='.!. .!!..-(Z
ndT 2
nRT) = Z2 R
7 7
U=-NkT =-(8.314)
2 2
=:: 29.1 J/moVK
U ",,?RT (for one mole of the gas)
2 7 9
a~ Cp=::Cv+R>='-R+R=::-R
Another Look at Specific Heat 2 2
The molar specific heat at constant volumeCv is defined 9
=-(8.314)
by the equation 2
C =.!dU =:: 37AJ/moVK
" n dT In general, for a polyaromic gas' with f degrees of
and C p can be computed from the equation freedom,
Cp=C,,+R
c" =L2R
For a Monoatomic Ideal Gas
Recall that k =RINA and N =nNA, and C
p
=LR+R=(/+2)R
2 2
Nk=(nNA)(N~)=nR
In addition y=::CP=::(l+~JR
3 3 C1• f
U =-NkT =-nRT
2 2 The predicated values of molar heat capacities

C
to
=.!.~(~nRT)=~R
ndT 2 2
sometimes do not agree with the experiment. As an example
C v for H2 is (5/2)R from about 250 K to 750 K and then
increase steadily to (7/2)R well above 750 K, see Fig. 1.32,
= ~ (8.314) which suggests that much more significant vibrations occur
2
at very high temperatures. At temperatures well below 250
= 12.5 J/moVK
K, C" has a value of about (312)R, suggesting that the
Cp =ev +R molecule has only translational energy at low temperatures.
3 5 At very high temperatures when the vibrational degrees of
=-R+R=-R
2 2 freedom are beginning to participate, the molecules
5 dissociate before these additional degrees of freedom
= - (8.314) = 20.8 J/mollK
2 pecome fully active. Thus at a given temperature some of the
For Rigid Diatomic Ideal Gas degrees of freedom (vibrational, e.g.,) do not participate
5 S. effectively in the sharing of energy transferred between
U=-NkT=-nRT molecules in collisions. Vibrational degrees of freedom are
2 2
assumed to be inactive in all cases at normal temperatures.
so that, C =..!..!!-(~nRT)=~R
ndT 2 2
u Hydrogen, a diatomic gas, has C v (k / m) at low =:: ~
2
= ~ (8.314) temperature. The three translational modes are excited but
2
rotation and vibration are not. As temperature increases, C,.
=:: 20.8 J/moVK
increases to ~ (k / m). corresponding to five excited modes.
and Cp=::Cv+R 2
5 7 The two additional modes correspond to rotation about the
=::-R+R=::-R
2 2 two axes perpendicular to the symmetry axis of the
7 molecule. Because of the small rotational inertia about the
=-(8.314)
2 symmetry axis, the energy quantum for rotation about that
=:: 29.1 J/moVK axis is very much larger than for the other two, and the

www.puucho.com I
Study Anurag Mishra with www.puucho.com

TEMPERATURE, HEAT AND THE EQUATION Of STATE, HEAT TRANSfER


--. 41

mode is not excited. At higher temperatures, the specific as rotational eller~'. The essential feature of interal
"heat increases again as molecules begin to vibrate. The thermal energy is its random distribution among the
-molecules dissociate into atomic hydrogen before many are molecules.
excited to higher vibrational stares and the value 2. The internal energy U of a system is tile total energy of
Ct, =?'Ck/m) is never reached. At high tcmpcra~ures, its molecules in a referenceframe in which the .~ystem'scemre
2 of mass is at rest.
nitrogen and chlorine have C" = ~ (kim), corresponding to 3. In a monatomic ideal gas, the structure of the atoms is
not important. Only translational kinetic energy comributes:
-three translational, n,;,o rotational and tWO vibrational 2
U =NK =Nm(v )
modes. 2
3 3
or U==-NkT==-nRT
30 2 2
25 Vibr<ltion 4. The internal energy is proportional to the number of
;< atoms in the system. The temperature measures energy per
~ 20 atom and is independent of the total number of atoms. The
! 15 internal energy of a system is distinct from any bulk kinetic
d 10 energy the system may have if it moves as a whole.
Translation
5
Conceptual Example 1: A cylinder containing III kg
0
10 20 50 100 200
Temperature
500100020005000
(K)
10,000
-
of argon gas at temperature T is carried on train moving at
vo. Find the internal energy of the argon gas. Show that the
FIg. 1.32 total kinetic energy of the gas may be divided into rwo
parts-bulk kinetic energy and internal kinetic energy- and
Specific Heat Capacity of Solids that the internal kinetic energy is the same as when the gas
is at rest. Compare the magnitudes of the two parts.
Fig. 1.32 shows the variation of molar heat capacity of a
solid with temperature. According to quantum theory for The internal energy is determined by the temperature of
-low temperatures C t, varies as (T/TD)3. The parameter Tn is the gas
-called Debye temperature. At low temperatures C I' 3
U =-NkT
approaches zero and a[ high temperatures it approaches a 2
value 3R. This phenomenon is called the law of Dulong
and Petit. Each atom in the solid executes simple harmonic =~(MJ(kn.
2 m
motion in three directions about its equilibrium position. Its
where M is the toral mass of the sample and m the mass
total energy is, therefore,
of a single atom.
U == .!.2x."o2x
m(v2 + 1)2 + t'~) + .!. (k x2 + k y 2 + k z 2)

-
y z
where kx• k" and k: represent bond strengths in three
directions. Hence there are six degrees of freedom. The total
velocity -
If the train velocity is ~oand
Vr , the atom's total velocity is
a gas atom has random
Vo +~
Vr. The mean
square magnitude of the atoms total velocity is :
internal energy of solid of N atoms is
(112) ==«(\,'0+ V'r)2)
U =6N(~kT)
== 3NkT == 3nRT
2-"-02
1 d == Vo + 2vo. (Vr) + (v r)
and C,,=--(3nRT)=3R
n dT
Since the average value of the random velocity IS

Concept: 1. The molecular motions that cause the (V,) = 0


pressure of an ideal gas are random in direction and do not
result in any net motion of the gas. The total energy (1'2) == v~+ (I'~)
associated with these random motion~ is called internal The total kinetic energy is :
thermal energy. In general, internal energy may include other 1 , ,
forms of kinetic and potential energy of tlie molecules, such K==ZM(I'O+(Vr))==KcM +U

www.puucho.com
Study Anurag Mishra with www.puucho.com

THERMODYNAMICS

The first term is the kinetic energy due to mmion of the Since N is the total number of molecules in the gas, then
center of mass (bulk motion). The second term is the we must have
thermal energy due to random motion. The internal energy
is independent of the velocity;o of the train. In particular,
L" fev)dv =N

when the container of argon is at rest, l!0 = 0, and : .' 'The Maxwell's distribution for a given gas depends only
on the absolute temperature. As temperature increases the
K"~M(v2) entire curve shifts to the right at higher temperatures.
Three different "average" speeds are associated with the
"~M(Vn "U Maxwell-Boltzmann speed distribution
Root mean square speed,
The ratio of the two energies is :
1 2 U nru = ~3~T
-Mvo 2
K Clll 2 ml'o

U= ~NkT = 3kT' Average speed,


2
vav =~8kT =O.9221'nm
where m = MIN is the average mass of the gas nm
molecules. Most probable speed,

MAXWELL'S LAW OF DISTRIBUTION


urn.p. = ~ z:T = O.817vnru

OF VELOCITIES vrms > Vav. > vm.p.


Maxwell showed that at any temperature the velocity of Mean Free Path
a molecule can lie between zero and infinity. Very small
number of molecules have very low velocity or vel)' high Average distance travelled between successive collision-
velocity. The Maxwell-Boltzmann speed distribution is defined as the mean free path (J..). Consider one
function is molecule of radius r1 moving
with speed v through a region of '-'.
2/2kT
)_ 4 ( m )32 1 _ml. stational)' molecules. The
f( v---- v,
.r,; 2kT molecule will collide with
f(v)
another molecule of radius r2 if
the centres of the two molecules
come within a distance
d = r1 + r 2 from each other. The
molecule will collide with any
molecule that comes in the range
d. In some time interval t, the
molecule moves a distance vt
Djamete~ _vt
d~

/~
'~:~:"
Area nd2
o
0
.••
-~.
()

;"

and collides with every molecule


in the cylindrical volume rrd 2vt. fIg. 1.34
Fig. 1.33 If n is the number of molecules
per unit volume, the number of molecules in cylindrical"
In a gas of N molecules, the number that has speeds in volumes is nrrd 2Vt. After each collision the path of the
the range v and I) + dv is dN, given by molecule becomes zig- zag. The mean free path is the total
dN " Nf(v)dv distance divided by the number of collisions.
The shaded area in Fig. 1.33 denotes the fraction " vt 1
A"---"--
dN nrrd. 2vt nrrd. 2
- "f(v)dv
N Considering the motion of all the molecules, the correct
The most probable speed vm.p• is the speed for which expression is given by
J(v) is maximum. 1
f.=---

vm.p. = ,/~T 12 nrrd2

www.puucho.com
Study Anurag Mishra with www.puucho.com

TEMPERATURE, HEAT AND THE EQUATlDN DF STATE, HEAT TRANSFER 43


The average time taken between successive collisions is Example 67 _
collision time or relaxation time r. The reciprocal of
relaxation time is approximately the average number of
collisions per second or collision frequency. Considering A parallel beam of nitrogen molecules moving with velocity v
= 400 m/s impinges on a wall at an angle 0 = 30<>to irs
average speed I'a,'_' the average distance between successive
collisions is nonnal. The concentration of molecules in the beam
no = 0.9 x 1019 cm-3• Find the pressure exerted by the beam
1.=Pav,' on the wall w;suming the collision of molecules with the wall
are peifectly elw;tic.
. Example 66y---

(a) . Calculate (i) root mean square speed and (ii) the mean W'".~
kinetic energy of one mole of hydrogen at S. T,P. (given that
density of hydrogen is 0.09 kg / m3• .:. '0
..•.:- ..6"'30'
(b) Given that the mw;s of a molecule of hydrogen is
3.34 x 1O-27kg. Calculate Avogadro's number.
(e) Calculate Boltzmann's constant. .'~~~~:"
Fig.1E.67
Solution: (a) (i) We know that the pressure of a gas is
given as
1 2nn, Solution. If rna is the mass of each nitrogen molecule,
P=-pt> momentum of each molecule is

=m
3
Po = mol!

"=, During collision momentum imparted to wall by each


molecule is
II =1 ~(-3-'-O-.7-6-'-1-3-.6-'-1-O-2-'-9-.8-) l<.Po = 2mull cos 0
=, vl 009 If A is the cross-sectional area of beam and v be the
velocity of molecules in the beam then number of molecules
= 1837m / sec incident on wall per second are
= 1.837 km / sec. N = TlovA [no = moleculardensiry]
( •.•.
) K"metlc energy = -1 mv 2 mlS
Thus total momentum imparted to the wall per second
2 by the beam which is force exerted
Here M =2m =2>< 10-3kg F = APo '" 2mot' cos e x "ot'A
M
or K.E.",.!>< 2><10-3 x (1837)2
2 Thus the pressure on wall by beam is
F ,
= 3374.56joule P =- =2moL' cosO.no
A
(b) Mass of one molecule of Hz '" 3.34><10-27 kg
28x 10-3
Molecular mass of hydrogen = 2 x 10-3 kg = 2>< . x 9><1024 x (400)2 x cos2(30<»
6.023><.1023
The Avogadro's number NAt which is the number of = lOsPa;;latrn
molecules in one gram molecule of hydrogen is given by
2x 10-3
,--
NA "'--~--
Example 68~
3.34x 10-27
'" 5.988x 1023 molecules A collapsible container contains a gas that obeys Boyle's law,
which is maintained at a constant temperature. When the
(c) We know that k =~ '" 8.3 2- volume' of the chamber is 550 cm3 and the pressure is
NA 5.988x103 14 kPa, it is being shrunk in volume. If the pressure is
'" 1.37 x 10-23 J / mol -K changing at a rate of60.0kPcVmin, what is the rate at which
the volume is diminishing?

www.puucho.com
Study Anurag Mishra with www.puucho.com

THERMOOYNAMICS

Solution: Since the gas obeys Boyle's law, Solution: On a PoT graph, constant volume curve is
PV = constant a straight line as can be seen from the ideal gas equation
On differentiating this equation w.r,t. time, we get P=(":)T p

P(~~)+V(~)=O Th~. constant volume line


corresponding to the small volume will
Hence, dV __ (V) dP form a birge angle with the T-axis on the
dt Pdt poT gtaph. For a particular temperature T
550x 10-6 a given mass of gas will exert a pressure
=----(1.00) Fig,1E.71
(140) which will increase as the volume
decreases. When the gas is heated in a
= 3.93x 10-6 ro3/s
small vessel, the pressure will increase faster than during
heating in the large vessel.
Exampl.~.
Example
Consider the lung capacity to be 500 em3 and the pressure
therein to be equivalent of 761 mm of Hg; estimate the A mass of gas is enclosed in a cylinder by afree moving piscon.
number of molecules per breath. plot the volume versus temperature cun'e:

Solution: From ideal gas equation, (a) when the gas is heated with small load on the piston; and
PV = NkT (~) with a large load.
Normal body temperature is 98.6cF :: 37°C = 310 K. " Solution: For a given v
N =PV temperature, the gas will occupy a
kT smaller volume at high pressure.
(101.46)(500)( 10-6) From ideal gas equation,
=
(1.3807
= 1.19 x 10
x
19
10-23 x

molecules
310)
V=(~)T Fig.1E.72
Constant pressure curve is a
straight line; the higher the pressure at which the constant
pressure process occurs, the smaller is the slope of the
Taking temperature and pressure to be -55CC and 19.4 kPa at constant pressure line.
an altitude of 12.5 km, find the mass of the gas to be put in a
balloon to fi.Il it to 2000 m3 at that altitude.
Example

Solution: From ideal gas equation, Fig. 1£.73 shows the constant temperature (isothennsJ on a
PV P-V diagram. Which curve corresponds to a lower
n=-
RT temperature?
(19.4x 10')(2000) P
=
(8.314)(218)
= 21.4 x 103 moles
Each mole has a mass of 0.002 kg; hence the total mass
is (0.002)(21.4)( 103) "" 43 kg.
~, V
Fig.1E.73
Example
Solution: For an isothermal process,
A mass of gas is heated in two vessels, first in a smaller one PV = constant
and then in a larger one. During heating the volumes of the
The p.v curve is a hyperbola; for a lower value of
vessels remain constant. How will the pressure-temperature
constant the hyperbolas are nearer to the origin. Thus the
graphs difter in the first and the second case?
curve -2 corresponds to lower temperature, T1 > T2•

www.puucho.com
Study Anurag Mishra with www.puucho.com

TEMPERATURE, HEAT AND THE EQUATION OF STATE, HEAT TRANSFER:

Example 74 _
---------v 45

A gas Lsenclosed in u cylinder with a freely movable pLston.


The load on the piston is gradually decreased. The
temperature of the gas can be changed by placing the cylinder
/' V) :d~----
on hot and cold heat reservoirs. 71le Fig. 1£.74 shows' p-v L----_T ~T
graph of such a cylinder. What inference can be c1rai,'n'the Fig.1E.76
1lOlllre of change ill tile temperature of the gas? '
p
P
1

" The slope of the curve is ( n: ) . The slope is small for

~, T,
, T,
larger pressure. Here point 1 lies on the constant pressure
line which forms a smaller angle with the x-axis than the
V
0
v constant pressure line passing through point 2. Hence
(.) (b) pressure of the gas decreases while heating.
Fig. 1E.74
Example 77~
Solution: We draw nvo isotherms representing
A certain mass of a gas was heated in a constant volume
constant temperatures '/1 and T2, such thatT2 passes through
vessel; its P- T curve u 1; similarly another mass of the gas was
linitial and final points 1 and 2, and TI passes through certain
heated in the same vessel; its P-T curve is 2. If tanll =: 2tan u.,
lintermediate point 3. The curve closer to origin represents
what Lsthe ratio of masses of gas in the two experiments?
:lower temperature. Hence the gas is heated in the section
1-3 and cooled in the section 1-2,

Example 75 _ p~
/' " P 1
Fig. 1£.75 (a) shows poT curve of an ideal gas during a '.' a T
process. Does compression or expansion takes place when the
gas is heated? Flg.1E.77

p
p Solution: From ideal gas equation,

P~(~JT~(:~JT
The constant volume curves are straight lines with slope
T mRIMV. The slope of lines is proportional to the mass of the
o gas.
('I (b)
(Slope)} ml tan a 1
Fig.1E.75 ----~-~--~-
(Slopeh m2 tan ~ 2
Solution: We draw constant volume lines Hence the mass m2 is twice of mI'
:isochores) through the initial and final points 1 and 2. (See Illustration
~xample 71). The \'olume V2 is greater than VI' Hence (1) The curve shown in Fig. 1.35 is a
Juring heating of the gas expansion took place. straight line with positive slope and
Example 76 _ negative imercept with V-axis. Its equation
will be T-
V =: aT - b Fig. 1.35
'lise the V-T curve shown in Fig. 1£.76 to find the nature of
change in the pres.wre of a gas during heat. So from the ideal gas equation PV = ~LRT, we have

Solution: Draw constant pressure lines through


P~ _,'R__T ~ _,_",_T_ ~ __ '"'__
V aT-b a-(bT)
JOints 1 and 2. From ideal gas equation,

www.puucho.com
Study Anurag Mishra with www.puucho.com

46 , THERMODYNAMICS

NowfromFig.l.3S,Tz >T1,soPZ <Pj,i.e.,whenrhegas dP ... (1)


~ =-pg
dy
is !leated, its pressure decreases.
(2) The curve shown in Fig. 1.36 is a From ideal gas equation, we have
straight line with positive slope and positive pRT
P=-- ... (2)
intercept with P~axis.Its equation will be M
P = aT + b . ,We eliminate p from eqns. (l) and (2),
So from gas equation PV = J.lRT, we T- dP Mg P
-=---
have Fig. 1.36 dy R T
v = "RT dP =_Mg dy
P
P R T
,oRT
=---=
"R =_ Mg tty
aT+b a+(biT)
R To(1-cy)
Now from Fig. 1.36, T2 > Til so V2 > VI' i.e., wilen the gas
Integrating the above equation, we obtain
is heated its volume increases. Mg
log~
P = --Iog~ To(1-0') +C
(3) The curve shown in Fig. 1.37 is a RcTo
straight line with negative slope and positive
intercept with P-axis. Its equation will be where C is a constant of integration.
P=-aV+b By applying initial condition at y = 0, P = Po,
So from the gas equation PV = IillT, v~ Substituting for C, we get
(-aV+b)V="RT ... (1) Fig. 1.37
logP-IogPo = Mg 10ge(1-0')
This is the equation of a parabola RTo'
as shown in Fig. 1.38. So during
expansion
first
tile temperature
increases, reaches a maximum
of the gas t log!..- = 10g(1 - cy)n
Po
Mg
and then decreases. T where II =--
RToc
T will be maximum when (b/2a) (bla)
p=po(1_0')n
(dTjdV) = 0 V~

-2av+b=O Fig. 1.38 =Po(l_cy)MgRToc

i.,., V = (bj2a)
Example
and substituting this value in Eqn. (1),
Tmu =...!-Jc[-a(Jc)+b]
~R 2a 2a
1Wo thermally insulated bulbs, filled with air and connected
by a short tube containing a valve, initially closed. The
b' pressures, volumes and temperatures in the two vesseLs are
=-- PI' VI' T1 and P2' V2, T2 respectively. Find the P, T values
4"Ra
after opening the valve.
Example Solution: When the valve is opened, the air flows
from the bulb at higher pressure to the bulb at lower
In the lower atmosphere the temperature decreases with pressure. In equilibrium both the vessels have the same'
altitude. Given that the temperature variation is linear, pressure.
T = To (1 - 0'), where the reference level is at the surface of the
earch. Find an expression for the variation of pressure with
altitude, assuming an ideal gas for which PV = mRT .
M

Solution: We consider the equilibrium behaviour of Flg.1E.79


a disc of fluid of thickness dy, experiencing a pressure
difference arising from the fluid. Total volume of the air. V = V1 + V2
(P+ dP)A+ pgAdy =PA After mixing of air, total number of moles, n = nl + "2

dP = -pg dy Let the common temperature attained be T.

www.puucho.com
Study Anurag Mishra with www.puucho.com

TEMPERATURE, HEAT AND THE EQUATIDN DF STATE, HEAT TRANSFER

Hence P(VI + V2) == + n2)Rf


----- 47
(Ill Example 80 _
P == (nl + 1l2)RT
... (I)
(VI + V2) Fig. 1£.80 shows a cylindrical tube of radius r and length I,
The combined system is thermally insulated; hence fitted with a cork. The friction coefficient between the cork
2 = 0; system does no mechanical work, since dV = 0, and the tube is ~. 1"1Jetube contains an ideal gas at
V = P dV = O. From first law of thermodynamics, temperature T, and atmospheric pressure Po' rhe tube is
2==dU+W. slowly heated; the cork pipe out when temperature is doubled.
lIence dU = 0. There is no change in internal energy. What is nonnal force per unit length exerted by the cork on
the periphery of tube? Assume uniform temperature
The internal energy U of an ideal gas is given by throughout gas at any im;larlt.
U = ne,:r dI
nRT PV
=--=--
(-I-I) (y-I) dN

fl1RTj 1l RT
Uinilial =--+-- 2 2
(y-I) (y-I)

=(~J y -1
(niTI + n2T2) ... (2)

(nl + 112)RT
UfinaJ ... (3)
(y-I)

Uinitial =Ufinal
Fig. 1E.60
n}TJ + 112T2 =(nl +n2)7
T =(I1ITI + n2T2 J ... (4) Solution: Since volume of the gas is constant,
III + 112 Pi == PI
PIVj+P2V2 Tl T.r
=

[(~~~'Jfi;' J] Pf =~(T~ J ==2P, =2Po


The forces acting on the cork are shown in the figure, in
equilibrium.
From eqn. (1), Po x A + IJ.N == 2PoA
P = (lIj + n2)RT N = PoA
VJ + V2

=
R
(n} + n2)T
"
N is the total normal force exened by the tube on the
VI + V2 cork; hence contact force per unit length is
From eqn. (4), dN N PeA
-=-=--
dT 2rcr 2rcW
(nl + n2)T = nlTI + n2T2
R Example 81~
P - (niTI + n2T2)
(VI + V2)

(PI Vj + P2V2) A capillary tube of constant cross-sectional area is filled with


(V1+V2) an ideal gas. The temperature of the gas vades linearly from
one end (x= 0) to the other (x= L) according to the equation
as PIV) =njRTj.
T =To + (T L -T
LOx )
and P2 V2 = n 2R"li
The volume of the capillary is V and the pressure P is uniform
throughout. Determine the /lumber of moles of the gas in the
capillary.

www.puucho.com
-I
Study Anurag Mishra with www.puucho.com

THERMODYNAMICS
48
Solution: Let A be the area of cross-section of the (b) Work done is given by,
tube. Consider a differential volume A dx of the capillary at W~fdW~fpdV
a distance x from left end. The temperature at this
cross-section is From eqn. (1),
dV=_I Vo(-a) dh
2(l_ah)12

= PoVoa dh ... {3,


2(l-ah)~'2
x =0 X=l
Substituting eqn. (3) in (2),
Flg.1E.81

T =1'0 +(1'L ~TO )x W ~ fo h


dW

= PoVoo. fh dh
Volume of the capillary V = AL, is consumt. Applying 2 0 (l_ah)32
ideal gas equation to this differential volume, we have
= _ 2Po Vo [,/0 _ all)J
peA dx) ~ dn RT
2
peA dx) = dnR [1'0 + (T L
~ 1'0 ) x J =-PoVo,/l-olI
(c) Change in internal energy,
Rearranging the expression, we have dU = nCl,dT

f' dx
, To+ (1'1-1'0) =
f"Rd
0PA ~nC"T,(~l-nh -1)
----
L
x
II
=n(~R )To(..)l-o.h-1)
From first law of thermodynamics,
TL:TolnITo+(TL~To)xl: ;~
dQ=dU+W
L tn(To+TL-1'o)=nR ~ ~PoV,(Jl-nh -I) -P,V,~l-nh
TL -To To PA 2
1 r.---T 3
n ~(~).!.tn(T,)
1'r -To R To
=-PoVovl-ah --PoVo
2 2

Example
PV I (TL)
= (Tr -To)R n To
Two glass bulbs of equal volume are connected by a narrow
tube and are filled with a gas at OOC and a pressure of 76cm_
of mercury. One of the bulbs is then placed in melting ice and-
the other is placed in a water bath maintained at 62"C. What
A monatomic gas filled balloon is rising. Volume of the is the new value of the pressure inside the bulbs? The volume
balloon at the ground is Va, pressure and temperature as a of the connecting tube is negligible.
function of height are Pa(l- ah) and Ta.J(1-ah)
respectively, where a is a constant. Find: Solution: The gas fills the entire volume Vo of th
(a) volume of gas as a function of height, system uniformly at pressure Po and temperature To
Number of moles of gas in each bulb is given by
(b) work done by gas as function of height,
PoVo
(c) change in intemol energy as function of height. n =-- 0,
...
RT,
Solution: (a) We can apply ideal gas equation to Let the final temperatures of the t'NObulbs be T} and 1'2
determine volume at height h. respectively. As the temperature changes, pressure will
Po(l-ah)V Po Va change and gas molecules will redistribute in such a wa~
T,JO-nh) ~r;;- that pressure remains same throughout in order to maintai,-
mechanical equilibrium. The system will not attain therma-
V~ ~ ... (1) equilibrium, i. e., the temperature will not be sam,
( l-nh) throughout. In steady state, temperature of the bulbs shal
remain constant at 1', and T2, heat will continuously flov

www.puucho.com
Study Anurag Mishra with www.puucho.com

TEMPERATURE, HEAT AND THE EQUATION OF STATE, HEAT TRA",H",S~Fi'liR


•••• __ 49

from source (water bath) at T2 to sink (melting-ice) at P=Po+lpg


T1 (T2 :>T1). where Po is atmospheric pressure and p is the density of
Assuming the gas to be ideal and applying the gas mercury. Hence, from eqo. (1)
equation, we have nRT = [A(Po + Ipg) ... (2)
PVo Assuming that the tube is raised isothermally tlbove the
Tl =-
1 RT , mercury in the vessel. If I ' is the length of the air column
PVo when the level of mercury in the tube coincides with that in
and 112 =-- the vessel, see Fig. lE.B4(b), then we have
RT,
p'= nRT
where, III and "2 are the number of moles of gas in the
I'A
bulbs at temperatures T] and T2, respectively. Final pressure
is P, while volume Vo does not change. and also, P'= Po
Since TIl -+ nz = 2n, we get P' is the pressure of air in the tube, which is equal to
P P 21'0 atmospheric pressure Po' because at equilibrium, pressure at
-+-=- level 0' must be same.
T1 T2 To
P = 21'0 TITz
Hence, l'=nRT ... (3)
or PoA
To T1 + T2
Substituting for nRT from (2), we obtain
On substituting numerical values, Po = 76 em of Hg,
1'= [(Po + Ipg)
To :: T1 = 273 K and T2 = 335 K, we get
Po
p 2:<76x335
Mass of the air in the tube is equal to
273 + 335
PoAI'M
= 83.8 em of Hg m =nM =---
RT
where, M is the molecular weight of air.
On substituting numerical values I = 0.1 m, T = 300 K
A glass tube, which is closed at one end. is completely and taking m = 29 x 10-3 kg and p = 13.5 x 10-3 kg/m].
submerged with open end downward in a vessel with mercury, 1'= 11.3 em
so that an air column of length 10 em is trapped inside the
tube. To what height must the upper end of the tube be raised and m = 13.7 mg
above the level of mercury in the vessel so that the level of
mercury in the rube coincides with that in the vessel? Also
EX<;1mpJe
calculate mass of the air inside the rube if the temperature
remains constant at 27OC. The area of cross-section of the What is the number of molecules per cc of air at STP? What is
tube is equal to 1 cm2• the mean free path of its molecules at STP? At what
temperaTUre is the RMS-velociry of air molecules equal to the
t
...,.'
t
1;0'
escape velocity from the swface of the earth?

C"""' Solution: (D Assuming air to be an ideal gas, one


mole of any ideal gas has nearly 6.02 x 1023 number of
0
molecules. 1 mole gas occupies 22.4 Htres of volume at STP.
Hence, number of molecules per unit volume at STP is
(-) (b)
No 6.02>< 1023
flg.iE.54
n" = V= 22.4>< 103

Solution: The pressure P of the air trapped inside = 2.7 x 1019 molecules/em 3

the tube when it is completely submerged in mercury, is TIl" the number of molecules per cc, is called
given by Loschmidt's number.
P = nRT (1)
IA , .. The average distance, I between two successive
collisions is called mean free path and is given by
where, I is the length of the air column.
I = 1
Equating the pressure at level 0, in equilibrium [Fig.
J2rr.1l I' d2
lE.B4(a)],

www.puucho.com
Study Anurag Mishra with www.puucho.com

50 )",~,', ' .. THERMODYNAMICS

where. d is the diameter of (spherical) molecule and n" 20-4


or X=--
is the number of molecules per unit volume. Taking effective 200
molecular diameter of air, d = 2)( 10-8 em, mean free path 16
=-=0.08=8cm
of air molecules at STP is, 200
1= 1 .Thus in equilibrium, energy stored in spring is
1t../Z(2.7 x 1019)(2 x lO-Bl
U =.!kx2
:o;2xlO-scm 2

(iii) According to kinetic theory of gases, temperature is =.! x 200x (0.08)2 = 0.64 joule
measure of average translational kinetic energy of irs 2
molecules. i. e., When the mass fall down to the bottom of vessel the
potential energy stored in spring and the net gravitational
!m1'2 . =~kT potential energy of block is lost released and used in heating
2 ~ 2
the water and block. Thus we have
vrms = \!3~T 0.64 + mg(mg -l ••.rg) (004) = m'tl )( s,o )( !!.T + m8 )( S8 x !!.T
or 0.64 + (20 - 4) x 004 = (0.3)( 4200 + 2)( 250] t"T
The escape velocity on eanh is given by the expression
or !!.T = 7.09 = 0.00490C
J2gRo = 11 kmis
vescap" '" 1760
where, Ro is the radius of earth.
Air molecules can escape from eanh's gravitational field
Example
if their velocity (vmls) is equal to or more than vmape-
Therefore, A vertical cylinder of height 100 cm contains air at a constant
3kT 2 temperature and its top is closed by a frictionless piston at
---;;- = Vescape atmospheric pressure (76 cm of Hg) as shown in Fig. 1E.87. If
mercury is slowly poured on the piston, due to its weight air is
or m 2
compressed. Find the maximum height of the mercury column
T = 3k t' escape
which can be put on the piston.
Taking average molecular weight M of air as 29 glmol,
Palm (76 em of Hg) Palm (76 em 01 Hg)
as there are No = 6.02)( 1023 molecules per mole, average
mass of an air molecule is

m=--
M
No
29x 10-3
6.02)( 1023
Therefore, required temperature is
kg
100em
i f,
t
T = 29)( 10-3 )( _1_1_'_1_1_'_1_0_6 10O-x
3)( 6.02)( 1023
= 140,794 K
1.38)( 10-23
1 1-)
1 Ibl
Fig.1E.87
Example
Solution: On pouring mercury poured on the top of
A metal block of density 5000 kg/m3and mass 2kg is the piston, the volume of air will decrease according to
suspended by a spring of force constant 200 N/m. The spring Boyle's Law. If final mercury column of height x is poured on
block system is submerged in a wat~r vessel. Total mass Oh the piston then gas pressure in equilibrium is given by
water in it is 300g and in equilibrium the block is at a height PI =(76+x)cmofHg
40cm above the bottom of vessel. If the support is broken. Find
If A be the area of cross-section of cylinder then we have
the rise in temperature of waler. Specific heat of the material according to Boyle's Law
of block is 250 joule/kg K and that of water is 4200 J/kg K.
Neglect the heat capacities of the vessel and the spring. PlVl =P2V2
or (76cm)(l00A)~(76+x)(100-x)A
Solution. For equilibrium of block in water, we have or 7600"" 7600 + 24x_x2
kx + Buoyant force = weight of block
or x=24cm
2
200x + -- x 1000x 10 = 2x 10
5000

www.puucho.com
Study Anurag Mishra with www.puucho.com

TEMPERATURE, HEAT AND THE EQUATION OF STATE, HEAT TRANSFER 51

Example 88__ Example 89__

A vertical hoIlOl": cylinder of height 1.52 m is fitted witlt a Figure 1.89 shows a horizontal cylindrical container of length
movable piston of ineligible mas$ and thickness. The lower 30 cm, which is partitioned by a tight fitting separator. The
parr of cylinder conrains an ideal gas and the upper part is separator is diathermic but conducts hear very slowly.
filled with mercury as shown in figure lE.88. Initially the Initially the separator is in the stare shown in figure. The
temperature of system is 300K and rile lengths of gas and temperature of left part of cylinder is lOOK and that on right
mercury column are equal. Find the temperature to which part is 400 K. Initially the separator is in equilibrium. A~heat
system is raised so Chat half of mercury overflows. Take is conducted from right to left part, separator displaces to the
atmospheric pressure is 76cm of Hg and ncglect thermal right. Find the displacement of separator after a long two
expansion of mercury. when gases on the parts of cylinder are in thermal
equilibriwn.
f
f
h/2
h/2 _10cm-~'~'.'---
20cm

11 t
'hi'
1
Flg.1E.88 Fig. 1E.89

Solution. After half of mercury overflows let its Solution. Initially the separator is in equilibrium thus
temperature be T. Then we have pressure on both sides of gas are equal let it be PI. Let A be
PlV! ""PzVz the area of cross-section of cylinder, number of moles of gas
T1 Tz in left a:ld right pan are respectively n I and n2
PI (IDA)
Here p] =76+-
I> ", --~---
R(400)
2
= 76+ 76 ': 152 em of Hg and 11 = PI (20A)
, R(400)
h
V\=-A=76A
2 Finally when separator is displaced to right by a
T1 :: 300K distance x, we have
Pf(lO+ x)A
Similarly after heating n, ~------
h RTf
Pz =76+-
4 Pj (20- x)A
and 112 =-----
=76+ 38 = 114 em of Hg RTf
31>
Vz=-A=114A If Pj and Tf be the final pressure and temperature on
4
both sides after a long time. Since no of moles on each side
Thus from gas law, we have remains unchanged.
152x76A 114x 114A
We equate the ratio of moles!!.l. in initial and final state,
300 T2 n,
or T2 = 337.sK we get
III (lOA 1100) 00+ x)A
-=-----=
112 (20A I 400) (20-x)A
2(20-x)=10+x
lOcmx =

Thus, the piston is displaced to 10cm right from its


initial position.

www.puucho.com
Study Anurag Mishra with www.puucho.com

52 'i~...;.,'.""_' THERMODYNAMICS
'1' mg
Example 900.--- PI =---
11'-1 A
A freely moving pi.~toTl divides a vertical cylinder, closed at To calculate, the ratio ofT' and T, we use
both ends, into two parIs each having 1 mole of air. In PI V] T'
equilibrium, at T"" 300K, volume of the upper parI is 11 "" 4 ---=-
P,V, T
rimes greater rhall that of the lower parr. Ar whar
temperarure will the ratio of these volumes be equal to \1' = 31 PzVz
=---
PzVz
Solution: In Fig. lE.90 shown piston PQ divides 1" p] VI
volume of the cylinder ABeD inw two parts. Initially, let the -=---
volume of the lower part be VI so that the volume of the T PIV]
upper part is equal to Vz = f1VI' '1 '1-1 VI
=-_._---
Since the two gases in the two parts Ar--, B '1'-1 fI VI
are in thermodynamic equilibrium:
Ratio or V I and VI is obtained from the condition
(a) Temperatures of both the systems 2
VI + V2 = V't + V'z = Total volumeor cylinder"
are same. Applying ideal gas equation to
P' ------ ••• 0' or VI(l+'1)=V\ (1+'1')
each part, we have
pf--'---1 a or V'I
-= 1+'1
__
PlVI =RTl ... (1) c '---'_..J D
VI 1+'1'
PzVz=RTz
Fig.1E.90
with, Tz =T], Vz =TlYl 1" '1' '12_1
Thus, we get =_._--
(b) Resultant pressure at the interface PQ is zero, piston T '1 '1,2- 1
is stable at position PQ. If m is the mass of the piston and A is On substitming numerical values,
the area of cross-section of the cylinder, then we have
T=300K,
mg
Pl =Pz +- ... (2) fI =4, 11'",,3
A
we get 1""" 421.9 K.
When the temperature of the two systems is changed,
their volume and pressure change. When thermodynamic
equilibrium is attained, it occupies a new position P'Q'. If
volume and pressure of the lower and upper parts are now
(V'l' P l) and (Vz , Pz) respectively, then we have A factory chimney of height h = 50 m carries off smoke at a
temperature T1 = 60OC. Find the static pressure M' producing
the draught in the chimney. The air temperature is To ""DoC.
PI VI =RTI The density of air is do = 1.29 x 10-3 g cm-3 at DOC.
Pz V'2 =Riz ... (3}
Solution: Let us assume that initially the chimney
with, V'z =11'V'1
has no smoke. The pressure A (see Fig. lE.9l) is the sum of
and PI =Pz+mg ... (4) the air pressure PB at B plus the pressure at A due to the air
A column in the chimney
From eqn. 0), we get PA ""PB +gdoh
P1VI =PzVz ="pzVI where g is the gravitational acceleration and do is the
or PI = TjPz density of air at To = DOC.
Substituting in eqn, (2), we have
'I" fB
P =_l_mg
z
Tj-1A
~nh
",,~UL
~ mg
PI =----
f1-1A
Similarly; from eqn. (3) and eqn. (4), we get
-
Fig.1E.91

When hot smoke fills the chimney, the temperature


A

p __ l_mg
z-'1'-IA inside becomes TI. We can take the density of the smoke-air
mixture as approximately equal to that of the air dl at TI•
The pressure at A in this case, is

www.puucho.com
Study Anurag Mishra with www.puucho.com

TEMPERATURE, HEAT AND THE EQUATION OF ITATE, HEAT TRANSFER .. 53

P'A =PH + gJj 11 or = (980) x (50" 102) x 0.29" 10-3)


Right before A, where hot air is being produced,
pressure is equal to the air pressure PA, Since P'A < P~. hot -< (
1
-
273
(273 + 60)
1
air is pushed upward by the difference = 1.14" 103 dynes/cm~
M=PA-P'A
Example 92 _
= gil (do - u1)
We can arrive at this result in a simper manner if we A cylinder containing gas at 27''C i.\ divided inco two parts of
realize that the warm air in the chimney is less dense than equal volume, each of 100 em:>, and at equal pressure, by a
the surrounding cold air and is in touch with the cold air at piscon of cross-sectional area 15 cm2. The gas in aile part is
the two ends of the chimney. Hence, as a result of raised in temperature to lOO~C; rhe other vallime is
Archimedes' principle for fluids, hot air will be buoyed up maintained al rhe original tempcratllrc. The piMon and walls
with a force equal to the weight of the cold air displaced by are perfea i/l,~u.lators.How far lI'illrlle piston move during the
the warm air. The net force M acting on the warm air change ill telllperawre?
column therefore equals the difference benvecn this
Solution: The heating of one side of the cylinder
buoyant force and the weight of the warm air increases the pressure of the gas in that portion. If the piston
were fixed, the volume on the two sides would stay equal
I1F =gdoV-gd]1/
and there would be a pressure difference across the piston.
= gAh(du - d1 ) Since the piston is movable. it alters its position until there is
no pressure differente between its two sides: the hotter gas
The resulting draught pressure tiP is
expands and thus drops in pressure, and the cooler gas is
M compres~ed find thus increases in pressure. When
tl,P =A =gh(do -dl)
equilibrium has been readJed. the two pressures are equal at
Let us assume that air can be thought of as an ideal gas. Po; the cooler gas now occupies a volume smaller by an
We do not expect the pressure inside the chimney to differ amount dV and the hotter gas a volume greater by a
appreciably from the outside pressure since it is mainly the corresp0:lding amount ,lV. The idc<li gas equations for both
higher temperature that determines the volume of the warm compartments are
air: - Po(V - dV) = nRT (1)
V '" Constant x temperature Po(V + tlV) = nRT' (2)
Under the assumption that pressure is approximately where n is the number of moles in each compartment, T
constant, the volume of a gas at two different temperatures and T' are the temperatures. Dividing (1) by (2), we get
T1 and To obeys the relation V -dV T
~--"-
Vo = To V +dV T'
VI Tj
VT' - dVT' = VT + dVT
The mass of air in both cases is the same VeT' - T) -= dV(T + T' )
Mass = Vodo = Vldl dV T'-T
- --
giving Vo =~ V T'+T
VJ do Therefore, the change in the volume of each
The density of warm air at Tj "C is related to the density compartment is
of air at O"C by dV '" (373 - 300) " 100
To (373 + 300)
dt =do-
T, = 10.85 cm3
The final expression for M' is therefore The piston has an area of 15 cm2. Hence it moves a

M' "ghdo( 1 - ::J distance of 10.85 em 3/15 em 2= 0.723 em.

"ghdo(l-~ )

t. www.puucho.com
Study Anurag Mishra with www.puucho.com

THERMODYNAMICS

Example Example 94~


A capillary tube of length 50 em is closed at both ends. It Two bulbs of equal volume joined by a narrow tube of
contains dry air at each end separated by a merCllry column negligible volume contain hydrogen at OOC and 1 atm
10 em long. With the rube horizontal, the air columru; are pressure.
"'oth 20 em long, bur with the rube vertical. the columttS are
(aJ What is the pressure of the gas when one of the bulbs is
15 em and 25 em long. What is the pressure in the capillary
immersed in steam at 1OO"Cand the other in liquid oxygen
wbe when. it is horizontal? at -190"C?
Solution: When (b) The volume of each bulb is 1O~3 m3 and the density of
T hydrogen is 0.09 kg m~3 at O"C and 1 atm. What mass of
the mercury column is
" c P,
1.
vertical, the pressure on P, hydrogen passes along the connecting tube?
[he gas in the two parts is h
( c I
as shown in Fig. lE.93, I+lo~
Solution: (a) When the two bulbs are at different
where PI is the pressure at T. B P, temperatures, one bulb contains n] moles at temperature TI
1. occupying volume V, and the other n z moles at temperature
the foot of the mercury Flg.1E.93
column and Pz the Tz also occupying volume V. Once equilibrium has been
pressure at the top. But the difference in pressure at two attained, both must be at the same pressure P. Originally, the
levels in a vertical column of liquid is known from the laws gas in the bulbs was at To = OOC = 273 K with the pressure
of hydrostatic pressure. Thus measuring distance from the Po "" 1atm. It had (nl + nz) moles in a volume of 2V. The gas
top of the mercury column, equation in this case is PV = nRT where R is the gas constant.
PI =Pz Then Po(2V) = (n1 + n2}RTo or
and PI = pg(h) + Pz
nl + n2
PoV =--- RTo ... (l)
2
whence Pz - PI = - pgh
When the bulbs are immersed in steam and liquid
Here, p is the density of mercury.
oxygen, the gas equation for each bulb is
Applying Boyle's law to section C of the gas when the
PV=n]RT1, T1 =lOO°C=373K ... (2)
capillary is in its 2 positions, we obtain
PV""n2RT2, T2 =-190°C=83K ... (3)
PIAI, =PoAlo
Dividing (2) by (l), we get
Similarly, for section B,
~ nlRT1
PzAlz = PoAlo p I
where Po and 10 refer to conditions when the tube is o _ (n1 + n2) RTo
2
horizontal. Thus
2n1T1 ZTI
=-----= ... (4)
(nl +n2}To ( 1+-'
") T,

and
. ",
From (2) and (3), we see that
PV = nlRTI = n2RT2

or n2 = TI
... (5)
nl T2
Substituting in (4),
p 2T,
-=
Po (1 ~-)To
+

2x 373 _ 0.497
(1 + 3:33) 273
Hence, P = 0.497Po = 0.497 atm.

www.puucho.com
Study Anurag Mishra with www.puucho.com

TEMPERATURE, HEAT AND THE EQUATION OF STATE, HEAT TRANSFER 55


(b) At the initial temperature To. one bulb contained change the state of the air inside so that the air pressure on
..!.. (nl + 112) moles and at temper<Hure '1'\ it had Til moles. the mercury imide the tube is still P" [Fig. lE.95(b)1. When
2 the tube is in nir with irs top still closed, the mercury f1ow~
Since Tj = lOO?(' > To = Dee, gas in this bulb expands and out until the pressure at the bottom of the tube equals air
some of it flows into the other bulb. The number of moles pressure [Fig. IE.95(c)J. If Pis tbc pressure of the <lir in the
that pass along the connecting tube is tube anu Ii is the length of the remnining mercury, we cau
1 1 write
-en
2 ~
+Il,)-Il, =-(TlZ-lll)
2 p =p+ weight of the mercury =p + WH~
" cross- sectional are<l A
The equation Il~ = T1 from eqn. (5) [s identical to
n. '1'2 PI!~A '/1
=P+---~=P •.hpIl .~.(l)
!..'.-c!!..L 1'1 - "1i A '
f1j+flZ "'2~"'1 where PH~ is the specific weight of mercury.
We;.:an find P by using Uoylc's law for the air trapped in
the rube. Since the temperaturc relPains constant. we have
or
(Pa) x (Volume in Fig. b) = (P) x (Volume in Fig. c)
1
or Pa - L4. = P(l, -It )A
373 - 83 290 2
=----=
373 + 83 456
giving P = L1>u ... (2)
2(L - h)
Thus 290 of the mass in the bulb at D't: passed along the
456 Substituting eqn. (2) ill equ. (I), \ve gct
tube as the temperature varied from 0"<: to lOOee. But each L
bulb held 10.3 In 3 at 0"(;, corresponding LO <l mass of Pa = P" 2(L -It) + hPH~ ... (3)
3 3
10- m xU.09kgm-3=9x IO~ kg
We are told that Pa is equal to the weight of H em long
The mass passing along the tube is thus mercury column with unit cross.sectional arca,

290)( 9 x 10-5 kg = 5.72 x Hr' kg = 0.0572 g~ Pa "" HPHy;


456 - Hencc, (3) becomes
L
Example 95:.--- HrHlI = HPllg --- -.-IJPHg
2(L-h)
L
A cylindrical glass tube of length L i5 half submerged in or H = H ----+ IJ. ... (4)
merwry, as shown in Fig. 1£.95. The tube is closed byafinger 2(L-1I)
and withdrawn. Part of the mercury flows out. What length of If we solve (4) for h, we get
mercury column remains in the tube? [Figs. 1£.95 (b) and
2H(L -h) = HL + 2lt(L -h)
(d). Take the atmospheric pressure Pa to be H cm of Hg.
2HL-2Hh=HL-2hL+2h2 =0
Ip,
p,
L-h
i p 2h 2 _ 2h(H + L) + LH

h = (H +L)_..JH2 +L2
= a
~.T.
2L
2
1 2
~~.L
Example 96:.---
(OJ
Fig.1E.95 Two identical vessels are connected by a tube with a' valve
letting the gas pm;s from one vessel into the other if the
Solution: When the tube is submerged in mercury pressure difference I1P ~ 1.10 atm. Initially there was vacuum
wirh both ends open, mercury rises in the lUbe until the in one vessel while the other contained ideal gm; at a
levels of mercury in the tube and in the container are the temperature Tj = 270C and pressure Pj = 1atm Then both
same [Fig. IE.95(a)]. We expect no change in the level of vessels were heated to a temperature Tf = 107OC. Up to what
the mercury inside the tube if we close the top of the tube value will the pressure in the first vessel (which had vacuum
since the air trapped inside was previously in equilibrium initially) will increase?
with the atmospheric pressure Pa. Closing the top does not

www.puucho.com \
Study Anurag Mishra with www.puucho.com

56 .(.,' THERMODYNAMICS

Solution: No. of moles of gas in vessel B is Pz(V + !:iv) == PIV ... (2)

nB

Due to heating of vessels


=--
P,V
RT
the
or P,=--=P
P,V
(V
o
+ 6v)
(V- V + 6"
l'
B A
gas pressure in B increases and
rises by more than 1.1 atm,
if it
the
Similarly, P,=P (_V]' o
V+t.v
valve opens and the gas into the
vessel A. In equilibrium
pressure difference between
the
the
I ~ ---
Flg.1E.96
I After Nth stroke, P =PN o(
V+t.v
V IN ... (3)

two vessels is
(P'n-P'A)$tlP ... (1) In (!#,' 1
In equilibrium let the number of moles of the gas in or ... (4)
vessels A and B be n'", n'B respectively. As 'the system is
closed,
N= In(v:~.,)
nB =0'" + n'B Note that the mass of the gas ejected in each stroke is
PBV p' ..••V P'B V not the same
-=--+-- ...
(2)
From ideal gas equation, we have
RTj RTf RTf
mN_I
The valve will open when P'B - P'A = 6P, or PN_IV ==-- RT
P'B = P'A + I1P M
[mass after N - 1 strokes]
Hence ego. (2) becomes
PB = ZP'A + tlP PM V == - '"N RT [mass after N strokes]
. M
T, Tf Mass of gas ejected in the Nth stroke is
MV
or PA=~H;,J-~] TnN_1-mN == RT (PN-1-PN) ... (5)

After Nth stroke, we have from Boyle's law,


On substituting numerical values,
PN_1V==PN{V+6v)
Tj=300K, Tf =380K
or (PN_I-PN)V==PN{6v) ... (6)
PH = 1 atm, dP '= 1.1 arm
From eqn. (5) and (6), we have
Thus P'", :::-0,08 arm
M(6v)
mN_I -mN ==--- PN
Example RT
6t'
A vessel of volume V is evacuated by means of a piston air
=-'"N
V
pump as shown in Fig. 1£.97. Valve A opens (B closed) when or
V V liv V
----=---- ... (7)
the piston moves outward and A closes (B opens) when the mN mN_I V mN-l
piston moves inward. Each stroke of the piston captures the
volume 6 V. How many strokes are needed to redu.ce the The specific volume of gas ~ is volume of a unit mass of
pressure in the vessel T) times? The process is assumed to be
isothennal and ideal. gas. '"
V V
Solution: When the piston moves ourward,
volum~ of the gas is (V + v) and pressure PI' such that
the
=- "vV == constant
PI(V+6v)==PoV ... (1)
mN_l
During return stroke of piston
volume 6v is ejected. The moment the The fractional change in specific volume is constant in
return stroke begins, valve A is closed, this process.
the remaining gas has pressure PI and
Flg.1E.97
volume V. For second stroke, we have

www.puucho.com
Study Anurag Mishra with www.puucho.com

TEMPERATURE, HEAT AND THE EQUATION OF STATE, HEAT TRA",N",SF"E",R _ 57

Example 98y-- Exampl.e 99 _

A smooth vertical cylinder has two different cross-sections Find the pressure of air in a vessel being evacuated as a
open from both ends and equipped with two pistons of funerion of evacuation time t. The vessel volume is V, the
different cross-section areas. Each piston slides within a initial pressure is Po, The process is assumed to be isothermal
respective section. One mole of an ideal gas is enclosed and the evacuation rate is continuous. The evacuation rate is
between the pistons tied with a non-stretchable thread. The the gas volume being evacuated per unit time, with that
cross-section area of the upper piston is M greater than that volume being measured under the gas pressures attained by
of the lower one. The combined mass of the ovo pistons is that moment.
equal to m The atmospheric pressure outside is Po' By how
many kelvins must the gas between the pistons be heated to Solution: In the example 97 the mass ejection was
shift the piston through I units. in discrete strokes, whereas in this case it is continuous.
From the previous example we can see that volume ejected
Solution: Let A}. A2 denote the cross-section area of in each stroke,
the lower and upper piston and 11.12 the lengths of string in V
~lJ=-(m""_l-mN)
respective sections. Initial volume of the gas, mN
Vi'=' lJAJ + 12A2
Similarly in the present case, the evacuation rate,
If the piston shifts upward by I, the final volume is C=~v=_ V m(t+ll.t)-m(t)
Vj = (II -I)AI + (12 + I)A2 M met + M) ll.t
The change in volume,
In the limits, C =~ dm(t) ... (I)
Po met) dt
From ideal gas equation, we have
P(t)V = met) RT
M
dP(t) RT dm(t)
-~=--- ... (2)
MV dt
de
From eqns. (1) and (2), we have
(.) dP(r) = _ "- mIt )RT
Flg.1E.98 dt V.MY
~V=tAl -tA2 =IM ... (1) C
= --pet)
Now we consider the equilibrium of the upper and lower V
'pistons, we have dP(t) = _ ~dt
PAl + mIg = T + PoAI [Lower piston] ... (2) PIt) V
C
PA2 = PoA2 + T + m2g [Upper piston] ... (3) or InP(t) =--t +K
From eqns. (2) and (3), we get V
where K is constant of integration.
peAl -A2) = Po(AI -A2l + (ml + m2)g
As att =0, P =Po, we have
or Pll.V=(PoM+ mg)/
when m = ml + m:.!is the combined mass of the piston
K = In Po
Ct
After temperature rise by ll.T, from ideal gas equation Thus InP(t)=--+lnPo
V
>forone mole we have
or pet) =Poe-(C,Vll
ll.T = P~V = (PoM + mg)l
R R

www.puucho.com
Study Anurag Mishra with www.puucho.com

58 THERMOOYNAMICS

Method 2. We consider a differential time interval de 0«' Cross-section area


, A
in which the piston moves out. In this time interval we can
fairly approximate that mass is constant.
pV~(V+Cdt)(p+dp)
PA (P+dP)A
or pV=pV+Vdp+pCdr+Cdpdt
I--r dr
or pC de '" -v dp [neglecting dp dr] ... (I)
As the process is isothennal, Fig.1E.100

P =p RT PM ,
dP "'-00 rdr
M RT
RT dP ",MolfTrdr
dP~dp- l'
M or
f Pop RT 0

or
dP ~ dp
... (2) P Mw2 r2
P P or In-=---
Po RT 2
From eqn. (1) and (2), we get
or
dp dP
Cdt =-v-=-v-
p P Example 101!---,,,
or Io,Cdt =-v I'
Po
-dPP Assuming that air is under standard conditions dose to the
C P earth's surface and the temperature and the molar muss of air
or --[ =In- are independent of height, detennine the air pressure at a
V Po
height h from the surface.
p -(4'V)t
or -=e
Po Solution: Considering the equilibrium of a
differential layer of thickness dh and height h, we have
or P = Poe-(Ctv)t
(P + dP)A + dmg '" PA
Example or AdP "'-gdm "'-gpAdh ... (I)
From ideal gas equation, we have
An ideal gas of molar mass M is filled in a horizontal cylinder pRT PM
closed at one end. The cylinder rotates with a constant
P=Mor p= RT ... (2)

angular velocity Cl) abour a vertical axis passing through the From eqns. (I) and (2), we get
open end of the cylinder. The pressure at the axis of the PM
cylinder is Po and temperature T. Find the air pressure as a dP ~--gdh
RT
function of the distance rfrom the rotation axis, in isorhennal
condition. or IPoP,dP ~_Mglhdh
RTo
Solution: We consider a differential layer of Ini.=_Mgh dm,
or
thickness dr, at a distance r from axis. Po RT
(P + dP)A - PA '" dmroo2 _n Fig.1E.101
or P -roe -Mgh.'RT
or dPA '" (pA dr) roo2
In a similar manner if we eliminate P with the help of
or dP "'p{,irdr ... (1) ideal gas equation, we get
From ideal gas equation, we have dP = dpRT
M
PM
p~- ...
(2) The eqn. becomes
RT
RT
dp- ~-pgdh
From eqns. (I) and (2), we get
M

www.puucho.com
Study Anurag Mishra with www.puucho.com

TEMPERATURE, HEAT AND THE EQUATION OF STATE, HEAT TRANSFER


-------- 59
r dp = - Mg f'dh
Pop RTD
reservoirs, A be the area of the material in thermal contact
with cach reservoir, dx be the length of material connecting
or the reservoirs, with the x-axis directed from the warmer to
the cooler reservoir.
Example 102 _ ~--.....---------~--r- _
,,': ,,' L":
An ideal gas of molecular weight M is contained in a tall '.
,': Insulation.'
,'"
,1 :

vertical cylindrical vessel whose base area is A and height h. :::


The temperature of the gas is T, and assumed to be constant.
Assuming uniform gravitational field, find the height at Hot
T" dx
:
, .
'

:
Tc
Cold
which the centre of gra\'ity of the ga.~is located.
reservOir
. :
, : : reservoir

: -,.,'-' - - - - - - --
.. ..
,,1--------- -,;'"
Solution: We consider a layer of
...
differential thickness dh and at height h 1 . , Material

o .
from the base of cylinder. Mass of this layer ~dh ,
is
dm '=p dV =p(A dh) ...(ll
T
h Direction of heat transfer

density at a height h is given by L----Jl Fig. 1.39


~Mgh RT
p =Po e ... (2) Fig.1E.102 . Experiments indicate that the heat transferred per
So ego. (1) becomes second, q = dQ called the heat flow (expressed in J/s ;;;W),
dm == poAe-MghRT dh
d, .
between the reservoirs is directly proportional to the
The height of centre of gravity is gIven by
temperature gradient ~ and the area of contact A.
I; (dmg)h
hCG, = dQ ,>,:A
fo"'dmg d,
dQ dT

I "o Pog Ae-MghRThdh


dx
-oc-
d,
=
I'o Po gAe - Mgh'RTdh
dQ dT
q=-=-KA- ...(1)
d, dx
=-
RT Since heat transfer is by collisions, the transfer through
Mg a cross-section of the rod depends on the number of
interacting electrons and molecules and hence on the rod's
HEAT TRANSFER cross-sectional area. The rate of transfer also depends on the
In heat transfer we study the rate of energy transfer detailed molecular or crystalline structure of the substance
between bodies due to temperature difference between in a way that is difficult to compute and is usually measured
them. experimentally. This dependence is expressed by a constant
There are three basic modes of heat transfer: of proportionality k, called the thermal conducrivity of the
substance.
(1) Conduction, (2) Radiation and (3) Convection.
The proportionality constant K is called the thermal
(1) Conduction
conductivity. Thermal conductivity is expressed in W/mK.
In conduction heat transfer takes place by direct It depends on the material. The negative sign has been
molecular communication without any appreciable affixed to make Q positive. The heat flow is from the warmer
displacement of molecule. Good conductors of heat are also to the cooler reservoirs, so that as x increases, the
good conductors of electricity. Conduction of electricity is temperature decreases. The rate of change of temperature T
attributed to free electron drift; so heat conduction is due to
mobility of free or valence electrons. Conduction in liquids is with respect to x in the direction of heat flow, :' is known
due to elastic impacts while in gases it is due to molecular as the temperature gradient.
diffusion of kinetic energy.
Heat transfer occurs through the material from the hot
reservoir to the cold reservoir. Let dT be the differential
difference in temperature ben'leen warmer and cold

www.puucho.com
Study Anurag Mishra with www.puucho.com

THERMODYNAMICS

I" / l
Concept: So far we have assumed one-dimensional heat heat
transfer under steady stare conditions. In the case of steady T1 now T(x) T(x+dx) T2
state the temperature at each point of tile system remains
constant in the course of time, and it is a function only of ~) \. \ J.----~I =(
space coordinates.
To find heat conducted through a uniform slab we may
.ce. ~,---11
A x+dx~

integrate the equation (1) after separation of variables. ,


q fX2dx =-KA fTc dT (a)q=kA dT
Xl TH d,
q(X2 - Xl) = -KA(Tc -TH)
T
If L is the thickness of slab, X2 - XI :: L
KA(TH -Tel T,
q=---- ..,(2)
L
There is an analogy between the flow of fluid, heat and T,
electricity. The flow is directly proportional to driving
porential and inversely proportional to flow resistance. In o ,
current flow potential difference is driving potential and (bJ Temperature as a runction of
electrical resistance of material is the flow resistance. position along the rod.
Similarly in heat flow, temperature difference is driving Fig. 1.40
potential and there is a thermal resistance to resist the flow
of heat. Concept:
From Ohm's law, " WHY IS CONDUCTIVE HEAT FLUX
. Potential difference V1 - V2 Proportional to the Temperature Gradient?
Current I = - ~--
, Electrical resistance Rei Energy is earned across a surface at coordinate x (Fig.
For heat conduction, 1.41) primarily by electrons moving througl1 (he metal. The

Temperature difference electrons travel an average distance ;. benveen collisions.


Heat flow rate, q = -~------- Particles crossing the surface to the right have energy
Thermal resistance
corresponding to the temperature rex -)..) at the sire of their
TIf -Tc TIf -Te last collision. Similarly, particles crossing to the left have
=---=---
UKA RIll temperature T(x - ;.). Equal number of particles pass in each
direction, so the net energy transfer to tfle right is
R _ TIf -Te _ L
Therefore, III - q KA proportional to the temperature difference
H ocjT(x -)..) - rex + )..)~ From the definition of derivative.
The quantity L/KA is called the thermal resistance for , x+~.
heat conduction. The unit of thermal resistance is K/W. The
reciprocal of thermal resistance is called thermal
conductance.
Heat Transfer Along a Rod
When the ends of a uniform rod of length I are
maintained at different temperatures TI and T2 (Fig. 1.40),
there is a temperature gradient along the rod. We shall
restrict our attention co examples where the temperature T(x-~)
changes at a constant rate along the rod. Then the
Fig. 1.41
temperature gradient is uniform, and :
dT T2 -TI = lim T(x + j.) - rex - A).
dT
-=---
dx 1 dx l......n 2>.
Energy flows along the rod from the hotter end co the So H,,2>'ldTldxj,
cooler end at a rate proportional to the temperature there is a direction associated with both temperature
gradient. The heat flow is greater when the system is funher gradient and heat flux: they are both vectors, and they have
from equilibrium. opposite directions.

www.puucho.com
Study Anurag Mishra with www.puucho.com

TEMPERATURE, HEAT AND THE EQUATlDN DF STATE, HEAT TRA;.;;NS_FE_R _


61

Concept: Conductio/! through a metallic rod with Here ~Iand dTI are the temperature gradients at the
$peeified end temperatures is an e.:mmple of steady state heat ~L dxlJ
flow: rhe temperarure varies with position but is constant in location of cross sections A and H respectively. If is the
time. As a reia/lt, heat flows lhrough each -,liceof the rod Qt
the same rate. medium steady state is not achieved dQz must be less then
d,
heat dQ) some heM is absorbed by the section A.B. The nne at
flow out d,
which the section AB absorbs heat at this instant is given as

~l;l~b,orbe~bYAIl
= d~l _ d~2

"k4[~l<:1,]
If the mass of section AB is m and C is the specific heat of
the material of section AB, the rate at which temperature of
Fig. 1.42 .
section AB'nses - dTI ".
d, AB

We choose a coordinate system with the x-axis parallel to


rhe direction of heat flow Qnd focus on a small piece of an
object, bounded by sUlfaces at x and x+ dx (Fig. 1.42). If
me ~t
"k4[~1-:1,]
more heat flows inward through the surface at x than leaves Materials in series: Many composite structures are
through the surface at x + dx. made of two or more layers of different materials, e.g., a
Then the internal energy between the swfaees would typical cold storage walls have a layer of bricks, a layer of
increase and the temperature there \vould rise. But if the thick insulation and plaster on both sides. When there is
temperarure c110nges in time, the heat flow cannot remain temperature difference on inside and outside wall, they act
steady. Similarly, heat outflow greater than inflow would as hot and cold heat reservoirs. Such a layered system is a
decrease the temperature and disrupt steady flow. series arrangement of layers. The heat flow rakes place
In a steady state, heat flow through any cross. sectional in succession through these materials. In steady state, heat
area normal to the temperature gradient is constant. flow through each material is same. The R values for the t'lm
materials are
When the system does not have a uniform cross-secrional
area the heat flow remains constant, but the temperature
gradient varies Witll position. ...:
,or' •••••• ,,"
,, ,,
•••
. ,
Ins~latiqn
..
.,. ,.~~ ••••• _.. _...••• " •••...
•••• :
" '.. ,', ,
Consider two cross-section A and H in let at an instant
,
othe rate of flow of heat through the cross"section A is dQ1 TH : d, T,
d, Hot : Cold :
reselV(llr: R, reservOir:
-and that through the cross.section B is dQ2 • Then these flow ,
d, ,,
rates ure given as
.... .: .....;;.....- ._ ....,.
.'
dQ1 = kA dTI o ,,'
J.uhction

dt dxA

and dQ2 = kA dTI Direction of heat transfer


eft dx IJ
Fig. 1.44

Low
temperature
eod

Fig. 1.43

www.puucho.com
Study Anurag Mishra with www.puucho.com

THERMODYNAMICS
62
Let T be the temperature of the junction between [\vo window across the same temperature difference. The heat
layers. The heat flow through first layer is flow through material 1 is
dQ) = Til -Tc Til -Tc
( dt 1 Li/KIA Rl
l
dQ) "TH -T ... (I)
( dr 1 R1 The heat flow through material 2 is
and thal through the second material is dQ) " T" -Tc
( dt 2 L2/K2A2
dQ) "T -Tc ... (2)
( dt R2 TH -Tc
2

Heat flows are equal in steady state; we have


R,
The total heat transferred from the hot to cold reservoir
TH -T "___
T-Tc is the sum of the individual heat flows:
___ ... (3)

which on solving for T, yields


R1 R2
(~;L""(~;),+(~;),
Til -Tc TH -Tc
".. -+~-~
T " _R~,_T
H~+_R~lT~c~ ... ( 4) R1 R2
R1 + R2
On substituting expression for T in eqo. (1) or (2), we "(TH -Tc)[~+~]
obtain
RI R2
dQ TH -Tc TH -Tc "If Rcq is the effective thermal resistance for the twO
-"--~" materials,
dt R] + R2 Roq

where Req = R1 + R2 represents equivalent resistance of


a single material that will offer same resistance as offered by
composite layer. we have,
Similarly, if more layers are added in series 1 1 1
--"-+-
between the two reservoirs, the effective thermal Req R} R2
resistance of the composite system is sum of the R
values of the individual layers. Example
- -
Rtotal =R1 +H2 +R3 +...
Materials in parallel: When heat is transferred A wall of dimensions 2.00 m by 3.50 m has a single-pane
through n ••.o or more materials across same heat reservoirs, window of dimensions 0.75 m by 1.20 m. If the inside
the materials are said to be in parallel, see Fig. 1.45. temperature is 20ce and the oucside temperature is -1Dec,
Consider a wall with a window; the temperature difference effective thennal resistance of the opaque wall and window
inside and outside is same for the window and the wall. The are 2.10 m2 KIW and 0.21 m2 KtWrespectively. Determine
the heat flow through the entire wall.
........ _.---------_
.......~ - ---

,
"
" ..,
.
,
'.
'
,,',
'
.
, Insulation ,"
' .,,
.. ~
• Solution: The wall and the window are in parallel
' arrangement; so the heat flow is the sum of the heat flow
~------_ - through the walL
,",:..----"R-, -.••• -..•
, " The temperature difference,
, TH-Tc=30K
TH Insulation
: " Area of window,
HoI ;- --- • .•........... _-----
" R AWlndow = (0.75)(1.20)
6i6i
re,~~
..

... = 0.90 tnl

Heat flow through window pane,


Direction of heat transfer dQ) "1'" -Tc
(
dt wmdow R window

""I'll _.'.~"=
"'_'''''_''. ~.,_ •••• _,!F"i,g. 1.45

t"Deously through wall and

www.puucho.com
Study Anurag Mishra with www.puucho.com

TEMPERATURE, HEAT AND THE EQUATION OF STATE, HEAT TRANSFER


--- 63

~ (0.90).2".- Alternatively, equivalent thermal resistance foc


0.21 materials in series,
= 1.3>< 102 W I 1 1
--=-+-
The area of the wall = (2.00)(3.50)- (0.75)(1.20) R~'l Rj R2
2
=6.10m
2 2 2
Heat flow through wall, K.4rrR K,rrR K2.3rrR
----~---+
[dQ]
d! wall
TH -T,
Rwall 0'
L
K=K,+3K2
L L

- (6.10)(30) ~ 87 W 4
(2.10) Example 105 _
The total heat flow is the sum of the heat flow through
the wall and that through the \..•..
indow. A steam pipe with a radius R1 is surrounded by all insulating
dQ = 1.3 x 102 + 87 jacket with an outer radius of Ro' If the temperature of the
dt inner (TJ) and outer (To ) surfaces are fixed ([/ > To ),jind the
= 2.2>( 102 W constant heat flow through the jacket. Apply the heat
conduction equation to steady state radial heat flow
. Example 104 > corresponding to cylindrical symmetry .

Solution: Consider a cylindrical shell of thickness dr


Fig. 1£.104 shows two concentric cylinders with different and radius r. The radial
thermal conductivities, K 1 and K 2. The two plane faces of the coordinate r is the
system are maintained at different temperatures. There is no perpendicular distance
L .,
heat loss from tile curved surface and the system is in steady
$late. What is the effectIVe thermal conductivity of the system?
from the
cylindrical
axis
surface.
to
The
a
" ,
, ..
00

hem flow is radial,


"
,, ,, o,
"
Solution: The inner
'.
: therefore, the area
solid cylinder and GUler
, . T,
through which the heat
cylindrical shell form a parallel ,,, ''' flows is 2rr.rL. The energy
combination of heat transfer. K, ,, '' Fig.1E.105
, ' is to be conserved, so heat
Cross-sectional area of inner '. flow per second through each of such successive conccmric
cylinder = rrR 2
shells must be same.
Cross-sectional area of outer The heat transferred per second,
Fig.1E.104
oeylinder = rc(2R)2 _rrR2 =3rrR2
q = dQ = -KA dT = -K(2rr.rL) dT ... (1)
Total heat flowing per second is dt dr dr

~';~(~';Le, Le, + [ ~';


Rearranging the variables nod noting that q is same for
each shell of which the cylinder is made of, we have

dQ
-
K}rrR2(Tj
~------
-T.l) 2 K (3rrR2)(T,
+~~--~~~2 -T )
... (1)
f"d'r
R]
~_2m.K
q
r'dTTl
~ L L
If the equivalent thermal conductivity for a cylinder of In [Ro) = _ 2nLK (T2 -Tj)
("adius 2R and length L is K, R, q
dQ K(4rrR2)(T, -T2) or
dQ 2trKL (T1 - T2)
q=-~______ ...(2)
... (2)
dt L dt In(RoiRd
Equating equations (1) and (2), we have At a distance r from axis the Icmperature gradient is
4K=K, +3K2 given by eqn. (I)
(Kj+3K2) dT = __ q_
K=----
4 dr 2rcrLK

www.puucho.com
Study Anurag Mishra with www.puucho.com

THERMODYNAMICS
R R (T -T2)
2 t 1
Integrate this equation to obtain +-----
(R2-R1)r
nr) ""-(-q-)lnr + constant
2dk
Example
On substituting temperature values at the boundaries,
T = T1 at r = R1 and T = T2 at r = R2, and value of heat flow Four identical rods AB, CD, CF and DE are joined as shown in
per second, we obtain the temperature distribution Fig. 1E.107. The length, cross-sectional area and thermal
T(r)=T + (T2 -T))ln(rjR1) conductivity a/each rod are L, A, K respectively. The ends A, E
1
InCRolRd and F are maintained at temperatures Tj, T2 and T3
respectively. Assuming no heat loss ta surroundings,
determine the temperature at B.

Consider two concentric spheres of radius R, at temperature


Solution: Let the temperature at B be T. Total
T1 and radius R2 at temperature T2• A material of (hennal thermal resistance from F to Band E to B is ~ respectively.
2KA
conductivity K is filled in the space between shells. Determine
the steady state radial heat flow Qnd the temperature Heat per second flowing to B from F,
distribution in the material.

Solution: Consider a spherical shell of thickness dr


and radius r. The heat flow is radial, therefore, it flows
through area 4nr 2. The conducting material can be Heat per second flowing to B from E
considered to consists of large number of such shells, like T2 -T
=--
onion peels. The heat crossing each shell is same in the 3L
steady state. 2KA
The heat transferred per second is Heat travelling from B to A per second
dQ , dT T -T1
q ~ - = -K(4nr )- =--
dt dr LlKA
Rearranging the variables and noting that q is same for In steady state, from conservation of energy, net heal
each spherical shell, we have arriving at B must be equal to net heat flowing OUtfrom B-
fRz dr = _ 47tK T2 dT f ... (I) thus we have
Rl r2 q 1i.
11)
--- 4;d{
=--(T -Tt)
2
( R1 R2 q
dQ 4;d{(T, - T,)
or ... (2)
q = dt = (-.!- - -.!-) c B o
R1 R2
At a distance r from axis the temperature gradient is T,
A
given byeqn. (1)
dT q ... (3)
Flg.1E.107
-;I; = - 4n:r2K KA(T,-T) KA(T,-T) KA(T-T,)
-----+ - ---~
Integrating this equation, we have ~L ~L L
Ter) = --q-(-!)
4;d{ ,
+ constant
2 2
which on solving for T we get
On substituting the boundary values, T = T1 at r = R1 T == Zf3 + Zf2 + 31'1
and T = T2 at r = R2, we can calculate the value of constant. 7
On substituting constant and q, we get

www.puucho.com
Study Anurag Mishra with www.puucho.com

TEMPERATURE, HEAT AND THE EQUATION Of STATE, HEATTRAl6fER


------ 65

Example 108 _ I Example 109 _

A cylindrical rod of 50 em length and having lcm2 cross- The temperature drop through each layer of a two layer
sectional area is used as a conducting material between an ice furnace wall is shown in Fig. lE.109. Assume that the
bath at DOCand a vacuum chamber at 27 "'c as shown in external temperature TJ ond T3 are maintained constant and
figure. The end of rod which is inside the VQCllUm chamber T1 > T3• If the thickness of the layers x I and x 2 are the same.
which of the following statements ore correct.
behaves like a black body and L~ at umperature 17 °C in
:;teady state. Find the thermal conductivity of the material of
11., 11.2

tEL
rod and rate at which ice is melting in the ice bath. Given that T,
latent heat of fllsion of ice is 3.35 x 10-5 J I kg.
T,
? )1.,)1.2 )I.

. .
------------- H Flg.1E.109

(a) k1 > k2
(b) kl < k2
(c) kJ =" k2 but heat flow through material (1) is larger than
Ice Bath Vacuum chamber
at2rC through (2) .
atO'C Fig. 1 E.1 08
(d) k1 = k2 but heat flow through material (1) is less than
that through (2) .
Solution: It is given that the system is in steady state.
So heat absorbed by the end of the rod in vacuum chamber Solution: Heat transferred through each cross-section
by radiation is conducted [0 the ice bath through the rod. of slabs in series is same.
Hence dT
q = -KA - = constant
kAeTB -TA) _ A(T' T') <Ix
I -a ",-Jj
As area of cross-section is constant.
dT 1
or k(l7'C - DOC) = 5.67 x 1O~8[(300)'-(290)') Slope of Tvsx",,-x-
0.5 dx K
k "" 5.67 x 10-8[(300/ - (290)4) x 0.5 Slope of T vs x curve is lesser in magnitude in slab 1,
or
17 therefore k1 > k2.
""1.713W I moe
The rate of heat obtained by the ice bath is
dQ "" kA(TA -Ta) Six identical conducting rods are joined as shown in figure.
dt I Points A and D are maintained at temperatures of 20DOC and
1.713 x Ix 10-4 x 17 20°C respectively. The temperature of junction B will be:
= (a) l20'C (b) lOO'C
0.5
(c) 140'C (d) Bo'e
""5.82x 1O-3J I s
This heat is used to melt the ice in ice bath. If m mass of A D
ice is being melted per second, then we have 200'C 20'C

dQ ""TnL Fig. 1E.110 (a)


dt
or 5.82x 10-3 ""TnX 3.35)( 10-5 Solution:
8
or m ""1.74x 10- kg I sec. ___ ~~ • • I T~ T,

R ~ R 20'C 200'C R R R ZO'C

Fig. 1E.110 (b)

(200- 20)
q"" 3R

www.puucho.com
Study Anurag Mishra with www.puucho.com

66 THERMODYIIAMICS

200-T} 50'C
q=---
R
180 T2 -20
=-= 100'C
3R R
Thus T1 = 140°C; T2 = ao°c
3K

O'C
A cylindrical rod with one end in Q steam chamber and other Fig.1E.113
end in ice cause melting of 0.1 gm of ice/sec. If the rod is
replaced with another rod of half the length and double the Solution:
mdilLS of first and thernlal cOIlductivity of second rod is 1/4 IOO-T T-50 (T-O)
that of first, find the rate of ice melting in gIn/sec. L =-L-+-L-

3KA 2KA KA
Solution:
_(dm) Lf -----
q~
_ KA(lOO-O) ... (1)
T:= 200 0C
3
dt 1 L
[A = rt-r2] Example
(dm)
dt
L
2 f
= K(4A)(lOO-O)
4{L/2)
... (2)
An extenor wall of a house consists of 10.2 cm brick
(K=0.7Wm-1K-1) and 3.8 cm gypsum plaster
ego. (2) divided by ego. (1) gives
(K = 0.48 W m-1 K-1 ). What thickness of loosely packed rock
wool insulation (K = 0.065 W m -I K-1 ) should be added to
(_dm)
dez
= 2(_dm)
dtl
= O.2gm/sec.
reduce the heat transfer through the wall by 80% ?

I1T
Solution: ----=q ...(1)
L1 L2
--+-~
A and B are two points on a uniform metal ring whose centre KIA K2A
is C. J"he angle ACB = e. A and B are maintained at OVO fJ.T = 2.q ...(2)
different constant temperatures. When e = 1800, the rate of L1 L2 L)
total heat flow from A to B is 1.2 W. When = 90° , this rate --+--+--
KIA K2A K)A
willbe-
eqn. (2) divided byeq. (1) gives
2. ~ L) = S.8Scm

-O-~3. 2.
Flg.1E.112
Two identical rods are joined at their middle points. The ends
are maintained at constant temperatures as indicated. The
temperature of the junction is :
Solution: 75'C

5O'C+100'C

25'C
Flg.1E.115

Solution: 180-T+_75_-_T +_5_0_-_T+_2_5_-_T=0


L,iKA L/KA L,'KA UKA
Three rods of same dimensions have thermal conductivities
3K, 2K and K. They are arranged as shown, with their ends at T=82.S C C

100cC, SOCC and oce respectively. Find the temperawre ot


their junction. Neglect losses to surrounding.

www.puucho.com
Study Anurag Mishra with www.puucho.com

TEMPERATURE, HEAT AND THE EQUATION OF STATE, HEAT TRANSFER 67

Example 116
~ T

A rod of length L with sides fully illsulaced is made of a


T,,~\.
..
malerial whose thermal conductivity K varies with
,

.
A2 ", _ '

tcmpcrawrc as K = ~ where u is constant. Tile ends of rod is


T (b) (,)
kept at temperature TJ and 1'2(1'2 > 1'1)' Find the rate of !leat Fig.1E.117
flow per unit area of rod.
Area is decreasing
Solution:
In Fig. IE.II? (b), 1:1 is decreasing; area is increasing

T'~~~IT, Example 118 ~


-dx-
Fig.1E.116
A composite body is made up of two cylindrical pipes (joined
Heat transfer across each differential element is same end to end) having equal area of cross-section (A). Its ends are
q= _(<:l' f"IA dTdx maintained at 100°C and DOC as shown in Fig. 1E.118.
[A~sume the lateral surface to be thennally insulatedl

)' o q
dx =_o.4)T,T) dT
l'
1100'c O'C 100'C O'C

q=Tln(~: 1 ~
1.2dI3'I.dI3"1 x
K K

I- d!3 "1-2d13.1
Exam8'e 117
~ Case.1 Case-II
Fig. 1E.118 (a)
The ends of a long homogeneous bar are maintained at
different temperature and there is no loss of heat from the (a) Plot the temperature gradient along the rod in steady
sides of the bar due to conduction or radiation. The graph of state for the cases indicated below
temperature against distance of the bar when it has attained (b) Find the heat flow per second in tenns of K, d and A in
steady state is shown here. The grapl1 shows: both cases.

Solution:
T O'C 100'C T O'C

I I I I
,.
2K 1K K 2K

_2 d
3
• 1• d_
3
.,
d 2d

d;$l(lnce from hot end (b)


Flg.1E.118
1'1
Fig.1E.117 (a)

(a) the temperature gradient is not constant. 100-T (T -0)


(U3) = Cd 3)
(b) the bar has uniform cross-sectional area.
2IV\ KA
(c) the cross-sectional area of the bar may increase as the
100-T (T-O)
distance from the hot end increases.
Cd 3) (U 3)
(d) the cross-sectional area of the bar may decrease as the
KA 2IV\
distance from the hot end increases.
T = 50°C
Solution: I:: I is increasing along length 150KA
q=-d-
dT 1
~'X-
dx A

www.puucho.com
Study Anurag Mishra with www.puucho.com

68 THERMODYNAMICS

'T T Solution: Let thermal conductivity be expressed as

" dT ",_S-
"
dT 1
kA
.[Qr) 1=150 "'CIJ==== j'"
(ji"7:i\'F ~dx-
FJg.1E.120
x=2d13 x=d x"'d/3 )(=2dJ3)(

-15 -15
K=Qx+b
d d at x=O K=Ko

-,-
-150
-,-
-150 x=L : K=4Ko
3K,
I') (.) K =Ko +--x
L
T T Thermal resistance of rod
100'C
,
ISIOpell < ISIopel2 100"C ISlopel, > ISIopel2
1 R = f: (Ko+-
dx
3K, x)A
50'C L
, ,
, , =_Lo [In!K,+3K'x)]t
SO'C
x=2d x=d x=d/3 x=d 3K '\ L 0
3
L
=--ln4
3K,
(9)

Fig.1E.118
Thermal resistance of rod from x = 0 to X

R, ...!:.-[In( K, + 3K, x)]'


=
Example 119
~ 3Ko \. L 0

Heat flows radially outward through a cylindrical insulator of


=_L IJI+3X)
3Ko '\ L
outside radius R2 surrounding a steam pipe a/radius RJ, The
temperature of the inner surface of the insulator is T1• that of Thermal resistance of rod from x = x to x = L
the outer surface is Tz .At what radial distancejrom the centre
of the pipe is the temperature just halfway between TJ and R, = ...!:.-[IJ K, + 3K, x)]t
3Ko '\ L x
T2 ?

Solution' T -T
,
• In(R2/R1)
=
(T-T,)
In(R2iR1)
- 3~o '{l+;X,'L)
2nKL 2rrKL Let temperature at x be T
100-T (100-0)
Here T=T1+Tz ---=----
2 R, R
Thus we get R = )R,R, lOO-T 100
T, --~~-~ = ----
Flg.1E.119 .....!:...-,.f 1+ 3X) ~ln4
3K •..•\ L 3K
o o
,
Example
.:.. .~
-- or T=100,.fL+3X)
In4 '\ L
A slender rod of length L and uniform area of cross-section A
is maintained at a temperature DOCat one end and 100°C at Example
the other. The thermal conductivity of the matenal of the rod
varies linearly from K at colder end to 4K at the other, If no The space between two concentric spheres o/radius RJ and R2
heat is being lost from the lateral surface of the rod, find the is filled with a uniform heat conducting material. It's inner
steady state temperature in the rod as a function of the and outer surfaces are maintained at temperature T1 and 12,
distance' x' from the colder end of the rod. Find the distance from the centre where the temperature is
TJ+T2
2

www.puucho.com
Study Anurag Mishra with www.puucho.com

TEMPERATURE, HEAT AND THE EQUATION OF STATE, HEAT TRANSFER 69


Ii -'[' 1'-1'2 face to face like if flow of heClt tilkes place parallel to the
Solution:
(R -R,) (R, -R) interface.
---
4rrKR1R 4rr.KR2R
T= 7i +1'2 "'- ,,""" >,
, "
,
as ,,
2
,,
On solving for r \ve get
h hi K, K, ,
,
,
H = 2R1R2 ....,,

"
HI + Hz " ,,

Example 122 _
w ~ .'---0-'---
Flg.1E.123
A spherical shell of radius R is filled with water, Temperature
of atmosphere is (-ape, where 0 is a positive constant. The Then total resistance will be the Req of the two parallel
shell is exposed to atmosphere and all water comes down to resistance of each slab.
O°C and then it srartsfreezingfrom outer surface towards the L h h
centre of the shell. A5suming shell to be highly conducting, R = KA' R1 = KAwl' Rz =--
KBIVI
calculate cime for whole mass of water at aoc to freeze.
Thennal conductivity of ice is K and latent heat of fusion is h
R =----
L. Density of water is p. Neglect expansion during fusion.

Solution: Thermal resistance of


1
eq

1
-=-+-
Rfq
K

R1
'. x2xwl

Rz
1
hollow sphere of inner radius r and outer
radius R
R _(R-r)
lh - 4rrKRr _KA+KB
K II ____ ',f heat supp I"ymg ta k ing 1
pace
If a shell of radius r and thickness dr - 2
Fig.1E.122
freezes in time dt perpendicular to the interface then Rj and Rz will be into
Here (dm)Lj = (q)dt the series R.q = R1 + Rz
, 21
------R
p.,.(4,,'d,)L =[~Jdt
f
K,qxwxh - .q
I
4rtKRr R1=--
KAwh

or ~JR
KT 0
r(R -r)dr =t
Rz=--
1
Kgwh
pLR'
oc t=-- 21 I I
6KT ---=--+--
K,qwh KAwh Kgwh
Example 123!--," 2 1 1
-=-+-
K.q KA Kg
A composite body consists of two rectangular plates of the
2 KA+Kg
same dimensions but different thennal conductivities K A and -=~~-~
Kg" This body is used to transfer heat between two objects K,q KAKg
maintained at different temperatures. The composite body 2KAKn
caTl be placed such that flow of heat takes place either parallel ------K fq
KA+Kg
to the inteiface or perpendicular to it. Calculate the effective
_ 2KAKn
thermal conductivities Kn and K.l of the composite body for K~ -~--
the parallel and perpendicular orientations. Which KA+Kn
orientation will have more thermal conductivity ?
K11 >K.l
Solution: Suppose we have two slabs of dimensions
height -.. h, length ~ I, width -Jo wand if we combine it

www.puucho.com
Study Anurag Mishra with www.puucho.com

~O THERMODYNAMICS

So rate of heat retiucing by vessel (1) will conducted by


Example. ':-2~y-- . .. 3 dT
ro d In umt time = -- nR -
4 dt
Two identical thermally insulated vessels, each containing n
mole of an ideal monatomic gas, are interconnected by a rod and. in unit time from (t = 0) heat conducted by rod
a/length 1and cross-sectional area A. Material o/the rod has [beca~s.e initial temperature difference (T1 - T2)}
.:.ermal conducrivily K and its lateral surface is thermally will be equal to the rate of heat reducing by vessels (1)
insulated. If, at initial moment (t = 0), temperature of ga.~ in so at'any time t if temperature difference is T then after unit
twO vessels is TJ and T2 «
TJ), neglecting thermal capacity of . 3. dT KAT
t1me-nR-=--
the rod, calculate difference between temperature of gas in 4 dt L
two vessels as afunction a/time. Here it is imagined that rate of heat supplying will
Solution: Suppose if initially at t "" 0 according (0 remain same.
problem temperature of vessels TJ > Tz then temperature After the unit time because after unit time temperature
difference will be T1 - Tz = To (Initial temperature difference difference between vessels not much more changed. So it is
at t = 0) taken same.
-3 dT KA
(j) -nR-=-dt
(%J 4 T L
K. L,A
f'
n, T,
I n. T2
fT
lj-T2Y=
dT
0
-4KA
3nRL
dr

---4'0:(
Fig.1E.124
= T = (T1 - T2)e 311RL
And in starting heat (internal) energy of two vessels
U=f/2nRT Example y--
125
UI "" ~ nRTI and Uz = ~ nRT2 at this moment if (at
2 2 A highly conducting solid cylinder of radius a and length I is
t =0) temperature of one vessel (1) Decreased by dT, due to surrounded by a co-axial layer of a material having thennal
dQ heat supplying in dt time- then in same dt time dT 2 conducrh'ity K and negligible heat capacity. Temperature of
temperature of vessel (2) will rise because it will receive the surrounding space (out side the layer) is To, which is higher
dQ heat from vessel (1) by conduction through rod. than temperature of the cylinder. If heat capacity per unit
volume of cylinder material is s and outer radius of the layer
So in dt time temperature difference will be reduced by
(dT) is b, calculate rime required to increase temperature of the
cylinder from T1 to T2. Assume end faces to be thennally
dT =dT, +dT2 insulated.
dQ1 = dQ2
and dUt =dU2 Solution: Given condition To > T2 > T,
3 3 (To = outer temperature)
- nRdT1 = - nRdT2
2 2 For cylinder suppose if at any instant temperature of the
dT inner surface is T and this temperature T maintain for time
dT1 =dT2 =-
2 period dt so we can apply the steady-state condition formula
of conduction between outer temp, To and inner surface
dTl = dT2
temperature T (at any instant).
. -3
And If heat dQ = dU1 = - nRdT1 (- ve sign because
2 For small time dt I = -KA dT ,. A = 2itrl
d,
temperature decreasing) reduced by vessel (1) in dt time.
Here area A is not constant it is changing radially. Take
Then heat reduced in unit time = _~ nR dTt dr thickness element at distance r from centre. r is not
2 dt
changing in dr width in steady state. I will be same for every
And dTI = dTI2 , then heat reduced in unit time =
3 dT 1 cross.section.
-nRx-x- (ate=0) dT
2 dt 2 1= +KI2rr.r-
d,

www.puucho.com
Study Anurag Mishra with www.puucho.com

TEMPERATURE, HEAT AND THE EQUATION OF STATE, HEAT TRANSFER 71

mssp "" "


:ra-ls

d,
a's
,=~iog~(bla)log~ [To-T
~~, J
/ '/ .•. ,,"'
2K To -T2

'-- . ./ Example 126


__

, A vertical brick duct (tube) is filled with cast iron. The lower
end of the duet is maintained at a temperature T1 which is
greater than the melting point Tm of cast iron and the upper
end at a temperature T2 which is less than the temperature of
the melting point of cast iron. It is given that the conductivity
of liquid cast iron is equal to k times the conductivity of solid
.•• cast iron. Determine the fraction of the duet filled Wir/I molten
• metal.

Solution: Given TI >Tm >T2 both the ends are at


temperature T) > T2 respectively so if upto LI height the cast
Fig.1E.125 iron melts and rest of L2 part has solid cast iron, And if
conductivity of heat of L2 part is K 2 then for L1 part will be
dr is the drop in temperature in dr width. Here sign is (KK 2) according to given condition at steady state condition
+ve because if dT reducing then dr is also reducing with dT the rate of heat flow from TI to T2
so (+ve), if in case only dr reducing but dr is not decrement L
in radial path (or radius) then sign (-ve) •
f' br
d, =f'To
2.lK dT
1

IOge(~)= 2~IK (To-T) L, L,


Liquid Cast Iron Solid Casllron
I = 2rriK CIa - T)
Fig.1E.126
log,[~) KeqA(T1 -T2)
Q= L ,L=LI+L2
It is the rate of heat supplying to the inner most cylinder
which is at temperature T for dt time and system is in the K ~qA(Tl - T2)
Q=-~---
steady state condition for dt time. Then cotal heat supplied L,
for dt time KK]A(T1 -Tm)
=------
J = dQ = 2,,1K (To -T) L,
dt !oge(b'a)
KIA(Tm -T2)
dQ = 2rr/K (To - T)dt
=~-~-~
L,
log,Cb'a)
L
R=R]+R2,
dQ = mS,pdT KeqA
21tlK ('['0- 'f)d t
ms, dT =---- L) L
p loge(b a) =--+-- 2
KKIA KIA
efT 2nlk dt
L LI (L-LI)
(To -T) ms.,p In(b.'a) --=--+---
K,'qA KK]A KIA
time taken to rise temperature from TI to T2
fT2 dT Jt 27r./K d
L I
--=-----+--
K.qA
I

KKIA
L L
KIA
L
KIA
1j (To -T) - 0 ms
sp
loge(b,'a) I,

[LJ(L L, LJ
t
Iog [To-T,)
--- = ------,
2nlK
K =lKKI -K'; +K
, To - T 2 ms-,p iogc(b-a) cq I

www.puucho.com
Study Anurag Mishra with www.puucho.com

72 --------:\,~,~,~"'
... THERMODYNAMICS

ACTl - T2) = ACT} - T2) extracted per second by conduction then rate of heat

(K~J [:;1 -~: + :J . K. A(O)


conduction APiCC'L- = ,~
dt y
dy

=
KK1A(T} -Tm) f oyyd =f' 0A.LK P,~
y A8 dt ice

L, , ... 2
y KiCC'St 2Kice8t
-=--,y~
Ll(Tl-T2)=KKl(Tl-Tm{~+!:...-.0.-) 2 PiCC'L PkeL
KK1 K} K1
So in time t the y height of water will convert into ice
LI(T1 -T2) =CTt -Tm)[L1 +KL-KLd and when ice will formed then it's volume will be greater
LITI -L1T2 =L}Tt -TmL1 +KLT1 -KLTm than the water
- KLtT1 -KLtTm So mass is constant of ice and water.
TmL1 -L1T2 +KL}Tj -KLtTm =KLT} -KLTm Mice = M waler
L}[(T m - T2) + Key} - Tm)J = KLeTl - Tm) PwxAxYw=PicexAxy
L} = Key} -Tm) Y••.= PiCC'
y, YiCC'> YI
L [(Tm -T2)+K(T1 -Tm)] P.
Key} -Tm) !J.y =y-Yw
= increment in the column height after converting into ice
[K(Tt-Tm)+(Tm-T2)]
from water
Example !J.y=y _PiceY
P.
Water is filled in a non-conducting cylindrical vessel of
uniform cross-sectional area. Height of water column is ho =Y(l-:':)
and temperature is DOC. If the vessel is exposed CO an
atmosphere having constant temperature of -a°e DOC)at « = 2K 8t(1_Pice)
ke

t = 0, calculate coral height h of the column at time t. Assume PiceL P••.


thennal conductivity afice to be equal to K. Density atwater So at time t the total height
is P ••.•and that of ice is P let' Latent heat of fusion of ice is L. =ho+ 2K St(l_Pke)
ice
PiceL P ••.

A lagged stick of cross-section area 1 cm2 and length 1 m is


initially at a temperature of DOC. It is then kept between 2
reservoirs of temperature IDDOC and DOC. Specific heat
Flg.1E.127 capacity is 10 Jjkg_oC and linear mass density is 2 kglm.
Find:

1:;::=.
--'.=::::;.1----1
-'-,
Solution: Suppose at any time t after t = 0, the height
of the ice column isy and time taken to make ice ofy column
from water suppose t then t =? 100'C O'C

At instant t dy layer of water convert into ice into the dt


Flg.1E.128 (a)
time then rate of forming ice?
rf [(A x dy x Pice) x L] energy required to make ice of dy (a) temperature gradient along the rod in steady state.

and it will require ( APice x t XL)


thick layer in dt time then rate of forming ice (Apice x dy jdt)
heat has to be extracted
(b) total heat absorbed by the rod to reach steady state.
x (I-x)

from it (water) to convert into ice. ( Ii, 'hi jl)


• dx

t XL)
So by conduction it will reduce this heat to the O'C
100'C T
environment at -SoC temperature. So ( APice x heat
Flg.1E.128 (b)

www.puucho.com
Study Anurag Mishra with www.puucho.com

TEMPERATURE, HEAT AND THE EQUATION OF STATE, HEAT TRAN.;,;S_FE_R


•••• __ 73

Solution: Suppose the total length of the rod i 1 and of heat supply:ill be( ms (~~)and it will be equal to the rate
-take the element if linear
(dx) from the hot end (lODoG) and
mass density is 2kgjm, then mass of the dx element (2d.x"). of heat supply by conduction
lHere element dx is taken very small like that if in steady tiT KA(To-T)
state temperature at a distance x from 100°C end is T
ms-=
dt L
Ca) Then temperature will not vary in dx length it will [T is the temperature of disc at any time. t, To is the
-remain constant T. So in steady state
To = 400K]
KA(JOO-T) KA(T-O)
3Stl dT f'KAd
x (I-x) 1300 (To - T) == 0 msL t

T = 1000- x//) so in steady state temperature T varies 3;0

with distance x from hot end (laQoC) to cool end eO°C) by KAt == _ log (To - T)
[ ]
msL 300
the relation T = 100(1- x/l) so temperature gradient
dT 100 . KAt
-=-- - = -[loge 400 - 350) -Iog( 400 - 300)1
<Ix I m.~L
_ dY = lOO"'Cjm because I = total length = 1 meter KAt ( SO )
<Ix IllsL = -log 100
(b) If mass density 2kg/m then mass of dx length
will be (2dx) and if 10 J heat required to rise temp. 1°C for I
KAt
I1lsL =-loge
(1)"2
kg then to rise temperature l°C for (2dx)kg required heat
InsL 1 2
will be 20dx J;CC and if lemperamre has to be rised Tee then t == KA og< '
required heat = 20dxT
t = 0.4" 600x 0.4 x 10 2
dH = 20dx x 100(1-x/l) lOx 0.04 ge
because in steady state temperature of dx element is T
t = 240 x 0.693 = 166.32sec
then required heat to rise temperature from OOCto TOC of
this element will be dH = 200Odx( 1 - T}t will be the heat Example 130
~
capacity of (dx) element in the steady state condition so net A thin walled metal tank of swfaee area 5m2 is filled with
heat capacity of whole stick I = 1m water tank and contains an immersion heater dissipating 1

Io H dH = I' 2000(1- xjl)dx


0 = 200~ x r x']'
-2'/ 0
kW. The tank is covered with 4 em thick layer of insulation
whose thennal conducrivity is 0.2Wjm/K. Tile outer face of
the in.mlation is 25"C. Find the temperature of the tank in
the steady $tate.
=2000(1-~)==1000J
Solution: In the steady state condition the power
E'<arnple 129
~ 'which the heater will generate (or dissipating) will
dissipating ou[ side thorough the insulation so
A cylindrical block of length 0.4 m an areu of cross-section I=KA(OI-82)
0.04 m2 is placed coaxially on a thin meral disc of mass 0.4 L
kg and of the same cross-section. The upper face of the If inside temperature is 0 then
cylinder is maintained at a constant temperature of 400K and
1000= 0.2x 5x (8-25)
the initial temperature of the disc is 300 K. If the thermal
4'10-2
conductivity of the material of the cylinder is 10 watt/m.K
and the specific heat of the material of the disc in 600 J/kg-K. 0= 65"C
how long will it take for the temperature of the disc to
increase to 350 K ? Assume. for purposes of calculation, the Exarnpl",
thermal conductivity of the disc to be very high and the system
to be thermally insulated except for the upper face of the A pan filled with hot food coolsfrom 50.1 "C to 49.9 "C in 5
cylinder. sec. How much time will it take to cool from 40.1 "C to
39.9 "C if room temperature is 30"C ?
Solution: If at in any time dt the temperature of disk
is increased by dT. Then amount of heat supply \'lill be in dt Solution: (50.1;49.9) = k[50.1; 49.9 30]
time (ms dT) so if (ms dT) heat supplied in dt time then rate

www.puucho.com
Study Anurag Mishra with www.puucho.com

THERMODYNAMICS

0.2 ~ k(SO _ 30) => -.'l2... ~ k Solution:


5 5x20
T,
(40.1~39.9) ~k[40.1~39.9 -30]

O.2=k(40_30) ~ t=~ "


t lOxk
0.2><5><20 10 "
t~----= sec T,
lOx 0.2

Example Flg.1E.133 (b)

A cylindrical rod with one end in a stream chamber and other 1=11+12
end in ice cause melting of 0.1 gm of ice/sec. If the rod i£ KA(T1 -T2)
replaced with another rod of half the length and double the 11 =--21--
radius of first and rhennal conductivity of second rod is 1/4 KA(TI -T2)
that of first, find tlle rare of ice melting in gm/sec. 12=--21--

Solution: g = }(ACo} - °2) , !(ACO} - 82) = 0.1 x 80 w=1 = KA(T1 -T2)


...(1)
t L L I
K' A'eSI -02) =7 given 1=w
L' If system are like this
given, K'=KI4,A'=4A.L'=~ 1'=1'1+1'2
l' _KA(TI-T2)
K' A'COI -82) = 2><[!(ACOI -02)] 1- 31
L' L
[' ,-_ KA(T I -T
1 2)

And if .£(ACO!- °2) heat supply per sec, melts 0.1 gm ice
L
l'~i(KA(TII-T2)J ... (2)
per sec, then 2 x [KA(8~ - O2)] will melt 0.2 gm ice per sec.
4
from eqn. (1) and (2) l' = - w
3

In the square frame of side 1 of metallic rods, the comers A


and C are maintained at TJ and T2 respectively. The rate of
A hot liquid contained in a container of negligible heat
heat flow from A to C is OJ. If A and D are instead maintained
capacity loses temperature at rate 3 1(Imin, just before it
T1 and T2 respectively, find the total rate of heat flow.
begins to solidify. The temperature remains constant for 30
min. Find the ratio of specific heat capaciry of liquid to specific
Br-------,C
latent heat offusion is in K-J (given that rate of losing heat is
constant).

Solution: Given that rate of losing temperature is


dQ = ms de , dO = 3 K/min, so rate of losing heat is
dt de de
A.L
__ ~----JD dQ=mxsx3=3ms
dt
Fig.1E.133 (a) Here s in (
gm-OK
cal J so dQ
dt
= 3 ms it was the rate of flow

of heat just before solidify so at the time of solidification the


rate of flow of heat will remain the same so in 30 min total
liquid solidified therefore total amount of heat which lost by

www.puucho.com
Study Anurag Mishra with www.puucho.com

TEMPERATURE, HEAT AND THE EQUATION OF STATE, HEAT TRANSFER 75

liquid of mass Tn in 30 min is (mLI) (Lf -+


---------
latent heat of dQ .
where. -- IS heat flow per second in joule/second
fusion) dt
So 30x3xmxs=mLj
(J/s) or watt (W).
, I The constant e is called the emissivity of the surface, is a
Lf
" 90 dimensionless number between 0 and 1. Emissivity is the
effectiveness of a surface as a radiator. A perfect radiator has
emissivity e = 1. Such a radiator is called a black body.
Example V--
135
The constant a is known as the Stefan's constant; its
numerical value is
One end of copper rod of uniform cross-section and of length
1.5 meters is in contact with melting ice and the other end 0= 5.670x 10-8 W/(m2 .K4)
with boiling water. At what paim along its length should a The negative sign indicates that the system is emitting
temperature of 200°C be maintained, so that in steady state, radiations; heat transfer from a system is negative.
the mass of ice melting is equal to that of steam produced in
When a system absorbs radiation, we write the heat
the same interval of time ? Assume that the whole system is
flow to a system as
insulated from the surroundings.
dQ ,
- =+ aAaT
Solution: If m mass of ice melt then m mass of water dt
will conven into the s:eam.
where a is a pure number between 0 and 1 and it shows
_ KA(200 - 0)
m (L)f i~ ------ ... (1) the relative effectiveness of the surface in absorbing the
x radiation from its surroundings. A perfect absorber has Q :: 1.
KA(200 -100)
rn(Lvap )wm~r = ------ ... (2) When radiation falls on an opaque body, part of it is
(1- x) reflected and the rest is absorbed. Light coloured bodies
Dividing ego. 0) by (2) reflect most of the visible radiation incident on them
(Lj)il'c 2(/-x) whereas dark bodies absorb most of it. The radiation
absorbed by the body increases the kinetic energy of the
constituent atoms \vhich oscillate about their equilibrium
80 2(I-x) position. The average translational kinetic energy is the
"---
540 . x measure of temperature of the body; the absorbed energy
2 I-x causes the temperature to rise. But the electrons of the
-"-- atoms are accelerated by the oscillations; consequently they
27 x
emit electromagnetic radiations. According to Maxwell's
2 l.S~x
Given, l=lSm .. -"-- theory of electromagnetic radiations an accelerated charge
27 x is a source of electromagnetic radiations. The emission of
2,= = 1.5>< 27-27.'1' these radiations reduces the kinetic energy of oscillation and
tends to reduce the temperature. When the rate of
29.'1' = 40.5 => x = 40.5 = 1.396m
29 absorption equals that of the emission, the temperature is
constant and we say that the body is in thermal equilibrium
=> x = 139.6cm
with the surroundings.
So 200"C temperature should be maintained at a
The thermal radiations emitted below about 600"(; are
distance of 139.6";::"139cm from the melting ice and
concentrated in wavelengths much longer than those of the
150 -139.6 = 10,4cm from the water container.
visible light. As a body is heated, the quantity of thermal
(2) Radiation radiation emitted increases, and the energy radiated has
It is defined as the transfer of energy through shorter and shorter wavelengths. At about 600- 700CCthere
electromagnetic radiation caused solely by a temperature is enough energy in the visible spectrum that the body glows
difference. In contrast to conduction and convection, heat and becomes dull red, and at higher temperatures it
transfer by thermal radiation does not require a material becomes bright red or even white.
medium. Every object emits or absorbs energy by radiation. The process by which heat is transferred directly from
It has been experimentally found that the rate at which a one body to another without affecting the intervening
system emits radiation is proportional to the fourth power of medium, is called radiation. It is by mdiatioo that the heat
its absolute temperature, surface area. The heat flow via from the sun reaches the earth.
radiation is written as It is universally assumed that <111 bodies at all times are
dQ = _ eAaT 4 emitting energy. This energy is called radialU energy or
dt thermal radiation and is in the form of electromagnetic

www.puucho.com
Study Anurag Mishra with www.puucho.com

------"<!'_!l.1.~..
~.~,-- THERMODYNAMICS
16
waves. These waves travel with the velocity of light and are radiant energy emitted per unit time per unit surface area of
transmitted through vacuum or through a medium like air. the body per unit wavelength range. It is denoted by E,..
When they fall on a body. Which is not transparent to them,
they are absorbed.
The thermal radiation emitted by a body is a mixture of
waves of different wavelengths. At ordinary and moderately Spectral Absorptive Power
high temperatures, mostly the longer waves (infrared) are If an amount dQ of radiant energy between wavelengths
emitted, but at very high temperatures, shorter waves are Aand A + di. be falling on a body at temperature T, of which
also emitted. a fraction aAdQ is absorbed, then in the limit dl. ~ 0 ai. will
Generally in all solids, liquids and gases • thermal be the spectral absorptive power of the body at temperature
radiation is constantly emitted, as well as being absorbed or T for the wavelength f•.
transmitted throughout the entire volume of matter. That is, Prevosts Theory of Exchanges
radiation is a volumetric phenomenon. However for opaque
(Non. transparent) solids such as metals, wood and rocks, Prevost theory of exchange states that every object emits
radiation is considered to be a surface phenomenon. radiant energy in all directions at a rate depending only on
the nature of its surface and its temperature, and that it is
In the electromagnetic spectrum, generally the region of absorbs radiant energy from all surrounding bodies at a rate
thermal radiations extends can not be detected by the eye or depending on the nature of its surface and the temperature
the photographic plate, but they can be detected by the of its surrounding.
Bolometer, which depends on the change electrical
resistance with temperature. It the rate at which heat is lost, is equal to the rate of
gain of heat, then the temperature of the body remains
Properties of Thermal Radiations constant.
Thermal radiation is electromagnetic of radiation. If the rate of loss of heat is less than the rate of gain of
1. Thermal radiation travels through empty space heat, then the temperature of the body increases.
with the velocity of light. If the rate of loss of heat is more than the rate of gain of
2. Thermal radiation, like light, travels in a straight heat, then the temperature of the body decreases.
line.
Concept:
3. Like light intensity, the intensity of heat radiations
decreases in inverse square proportion with Radiation in an Isothermal Enclosure (Furnance):
distance J o:::~. Consider a nonconducting isothermal enclosure, and a
r' body is placed inside it,
4. It can be reflected as well as refracted like light The body placed in the enclosure would eventually
rays. acquire the temperature of the enclosure.
5. It exhibits the phenomena of interference, Radiation inside the enclosure is homogeneous and
diffraction and polarisation. isotropic.
6. It exerts a small, but finite pressure on the surface The radiation inside a uniform temperature enclosure is
on which it is incident. independent of the nature of the wall of the enclosure and the
nature of the bodies present in it, and depend only on
Spectral Energy Density temperature of the enclosure.
It is defined as the energy per unit volume per unit Radiation inside enclosure is same as black body
range of wavelength (A to J,. + d).) for a particular wave radiation.
length. Ie is denoted by u~.
Black Body Radiation
J•
From the above definitions, it is clear that u = u).d). A black body is a perfect emitter and absorber of
o radiation. At a particular temperature and wavelength, no
Total Emissive Power surface can emit more energy than a black body. A black.
The total emissive power of a body is the radiant energy body absorbs all heat radiation incident on it irrespective of
emitted per unit time per unit surface area of the body for all wavelength and direction. It neither reflects nor transmits
wavelengths taken together. It is denoted by E. any of the incident radiation, and therefore appears black,
whatever be the colour of incident radiation. A black body
Spectral Emissive Power emitts radiation energy uniformly in all directions. Black
The spectral emissive power of a body, at a given body is a diffused emitter i.e., emits independent of
temperature and for a given wavelength, is defined as the direction.

www.puucho.com
Study Anurag Mishra with www.puucho.com

--~-----
TEMPERATURE, HEAlAND THE EQUATION Of STATE, HEAT TRANSfER

No substance possesses strictly the properties of a black Eid}. == dQ


77

...(ii)
body. Lamp black and the platinum black are the close to a From equations {i) and (ii), we gets,
black body. The absorb 96-97% of the incident radiation.
e,.d,. == Q,_E,.dt.
Feny's black body is a close approximation black body. It ,
-!o.. = E.
is a hollow copper sphere blackened inside \vith a small hole QJ I.

in the surface. When the radiations enter the hole. They


suffer a number of retlections at the inner ' ..... alls of the Since,E;_ is constant at a given temperature, it follows
sphere until it is completely that,
absorbed. In order to avoid E
_A == constant (at constant temperature for all
direct retlcction of the radiation incident A,
from the inner surface. <l pointed radiation
substances).
projection is made in front of the
hole. Thussmall hole acts as If emissive power ei. of a hody is greater, Kirchhoff's Law
a black body absorber. It is tells that the corresponding absorptive power ai. must be
only the hole and not the greater. Thus good absorbers of radiation are also
Fig. 1.46 good emitters of radiation.
walls of the sphere, that
acts as a black body radiator. Consider a red glass heated in a furnace and then taken
When this sphere is placed in a bath at fixed out from it will, appears green because at room
temperature, the heat radiations come out of the hole as temperature, it absorbs green light strongly, therefore
shown in figure. appears red while at high temperature, it emits green light
strongly, therefore appears green.
KIRCHOFF'S LAW
If a polished metal ball having a spot of platinum black
The ratio of the emissive power to the absorptive power on it, is heated in a furnace and then examined in a dark
for radiation of a given wavelength is the same for all bodies room, the black Spot is found to glmv brilliantly than the
at the same temperature, and is equal to the emissive power metal surface. because the platinum black spot is a better
of a perfectly black body at that temperature. absorber of heat radiation and when it is heated, it is a
Consider a body be placed in a uniformly heated better emitter also, in accordance with the Kirchhoff's law. It
enclosure at a constant temperature. Let an amount of will also act as a good emittrs.
radiant energy dQ between the \•...
avelengths ,. and I. + d,. be
Stefan's Law a Black Body Radiation
incident per second on unit surface area of the body. Let QA
It states that the toral radiant enetgy E emitted per
be the absorptive power of the body for the \"lavelength I..
second from unit surface area of a black body, is
Then an amount of energy Qi.dQ will be absorbed incident
proportional to the fourth power of, its absolute
energy {(l-aiJdQ} will be reflected or transmitted.
temperature T i.e.
If ei. he the emissive power of the body for wavelength I.
at the temperature of the enclosure. Then an amount of
energy e,.dA in the wavelength range and 1.+ di. will be
emitted per second by unit surface area of the body by virtue Where 0" is called Stefan's constant. Its value is
of its temperature. 5.67x 10-5 erg sec-1 cm-2k-4in e.G.S. system and
The total energy sent out by the unit area of the body 5.67 x 10- watt m-2k-4 in S.1. system.
8

per second = (l-a,)dQ + CAdi. If a perfect black body at temperature T is placed in an


If the body is in temperature equilibrium with the enclosure at temperature To, the radiant energy lost per unit
enclosure and the presence of the body does not affect the time, per unit area by the black body is given by
quantity or quality of radiation stream in the enclosure. E == 0"(T4 -T04)
Therefore, Energy emitted or reflected = Energy When a hot object is placed in surroundings at a cooler
incident. temperature, heat transfer takes place between hot object
(l-ai)dQ + eAdi, == dQ and surroundings. Eventually the object and surroundings
or ci.dl. = eidQ ... (i) reach a common temperature and are said to be in thermal
equilibrium. In thermal equilibrium the object must be
But for a perfectly black body,
radiating an amount of energy equal to the energy it is
ui. = 1 (for all wavelengths) absorbing. Thus at thermal equilibrium,
Therefore if E,. is the emissive power of the black body,
we shall have,

www.puucho.com
Study Anurag Mishra with www.puucho.com

78 ... '" • THERMODYNAMICS

Heat flow radiated + heat flow absorbed = 0 W On substituting this in eqn. (l), we get

Hence
-eAcrT
e =a.
4
+ aAcrT 4
= OW
dQ eAcr[To:"cr(1 4~] - To:,~]
dt
= +
~~ect

which shows that a good absorber of radiation is =eAa(4To~jecr I:J.T) ... (2)
also a good emitter.
• If the temperature of the surroundings is higher,
If an object is at a temperature Tobj«l different from
tJ. T > 0 K. the total heat flow is positive and
surroundings T...-toUO<ling.<' then radiative heat flow rate is represents heat flow to the object. If the temperature
-eAoT"~CC1' and absorptive heat flow rate is +eAcrT~l)undi!ll(S' of (he surroundings is low, tJ.T < 0 K the total heat
The (Oral heat flow via radiative heat transfer to the object flow is negative; it represents heat flow from object to
from its surroundings then is surroundings .
dQ •. The eqn. (2) shows that for small temperature
at 4 4
= eAcr(Tsurroundings - T object) ... (1) difference, the heat transfer per second, via radiation,
by object is proportional to the temperature
If m is the mass of the body and c its specific heat, the
difference !!T.
rate of loss of heat at temperature T is dQ dT
dQ dT rA> -=me-
-=me- ... (2) dt dt
dt dt combining it with eqn. (2), we get
From eqns. (1) and (2), we have dT = eAa !!1' 3, !!T
dT 4 4 dt me obJecr
me - = eAo(TsulTOundings - Tobjet.'t)
dt dT
dT eAa 4 4
dt = K (T objecr - T.urrounding.)
0' dt =~ (Tsurroundings - Tobject )
This law is called Newton's law of cooling .
Derivation of Newton"s Law of cooling from •. If a body cools by radiation from T} to T2 in time t, "C

Stefan Boltzmann Law then taking


dT (Tt - T2)
An object at temperature Toojm is in a region where the -=----
temperature is slightly greater. dt t

Tsurroundings = Tobject + 6.T and T=T =(T1 +T2)


av 2
where 6.T is small compared with either Tobjcrt or
Newton's law of cooling, 1.e., equation becomes
Tsurroundings.The rate of heat flow is proportional to
temperature difference between the body and its [CTI ~T')]=K[CTI ;T,) -To]
surroundings.
From Stefan's Boltzman's law we have the rate of heat For a body which is not a black body,
flow due to radiation, E = aey4
dQ 4 4 Where' e' is the emissivity of the surface. Its value
dt =eAa[{Tobje<:t +6.T) -Tobject]
depends upon the nature of the surface where 0 $ e ~ 1.

=eAcr[To1,~(I+~]'-T,i"cr] TObJecr
... (1)
Emissivity is a measure of how closely a sulface,
approximates a black body for which e = 1.
If the body which is not a perfect black body is placed in
an enclosure with temperature To and the temperature of
Now we expand the expression (1 + ~]4. Using the the body is Y(T-To), then
Tobje<:t E = cn:T(T4 - T~)
binomial theorem,
If A is the surface area of the body, then, rate of loss of
(l + X)4 = 1 + 4x + 6x2 + 4x3 + XL
heat(~~)=O"t:(T4 -T 4).
0
If x is small, we can retain just two terms.
{l + x4)~1 +4x If m be the mass of the body and 5 be its specific heat,
then we can write.
Thus we have
dQ dT 4 4
' - = -ntS- = mA(T -To)

( 1+--
6.T
Tobjecr
] ~1+4--
6.T
To~ecr
dt dt

www.puucho.com
Study Anurag Mishra with www.puucho.com

TEMPERATURE, HEAT AND THE EQUATION OF STATE, HEAT TRANSFER 79


_elf == acA(T4 -T~) (v) The energy (E max) emitted corresponding to the
dt ms wavelength of maximum emission (Am) increases with the
Since, III = pV fifth power of the ahsolute temperature of the black body i.e.

So, _dT =
dl p.~ V
m:(A)CT 4 -T01)
EmaxocT5

(vi) The area under the curve represent the total


i.e. rate of cooling r- dT)
, lit
is directly proportional to' the
energy per unit area per unit time radiated at a particular
temperature over the complete wavelength range at that
temperature i.e.
"aria of surface area to volume and to the emissivity of the
,urface of the body. •
E=fE~di.
Spectral distribution of energy in black body o
'adiation :
According to Stefan's Law, since Emax 'lC T4therefore
In the figure, a graph is plotted between spectral radiam area under the curve is also proportional to fourth power of
mensity E ~ and wavelength Aof the emitted radiation at a absolute temperature.
"ixcd temperature.
Temperature of the Sun
E,.
If R is the radius of the sun and T its temperature, then
',1646 K the energy emitted by the sun per sec through radiation in
accordance with Stefan's law will be given by
P = eAcrT4 = 4rrR 2crT4
~
\1159 K [Sun is a perfectly black

.../~""~-">. .
~ body as it emits radiations of all
\1095 K
wavelengths and so for it e = 1.]
~

..
In reaching earth this f
~
\~W4K
energy will spread over a .. ,
Earth

,. sphere of radius r (= average


distance between sun and
•• Sun
.
_ ~~ '
••
..
Fig. 1.47 earth); so the intensity of solar
radiation at the surface of earth Fig. 1.48
The following conclusion arise from these curves ; (called solar constant S) will be
CD The energy is not uniformly distributed in the given by:
"adiation spectrum of a black body. P 4itR ZcrT4
S"--"---
OJ) At a given temperature, the intensity of radiations 4rrr2 4rrr2
ncreases with increase of wavelength and becomes
naximum at a particular wavelength and then aries of heat i. e.,
"adiations decreases.
(iii) Wein's Displacement Law: It states that the
But as r = 1.5 x lOll km, R = 7 x 105 km,
)coduct of the wavelength corresponding to maximum
~nergy, I'm and the absolute temperature T is constant i.e. 5=2 cal =l.4kW
2
cm min m2
I.mT = b = constant
Where 'b' is a constant named as Wein's displacement and
:onstant and has a value 0.2896 cmK or 0.2896x 1O-2mK.
According to this law, A decreases with increase with
ncrease in temperature.
An increase in temperature causes a decrease in Am'
vhere I. max. is the wavelength for which the energy emitted This result is in good agreement with the experimental
s maximum. value of temperature of sun, i. e., 6000 K. The difference in
(iv) An increase in temperature causes an increase in the two values is attributed to the fact that sun is not a
~nergy emissions for all wavelengths. perfectly black body.

www.puucho.com
Study Anurag Mishra with www.puucho.com

80 THERMODYNAMICS

(3) Convection da
-- =K(a-ao)
The heat transport mechanism by convection refers to dr
the heat exchange between a solid surface and the fluid
flowing along the same. e. g _,in a hot water tank the water in
or J'o dr -..!-K f"- I:lJ.
da
(0-00)
immediate contact with the heating element expands and
becomes less dense than the surrounding fluid. Buoyant or t
K
IOge[_o.~-_ao"]
""..!..
O 0 2 - 0
forces then cause the hO( water to rise in the tank. Even
though the water is at rest, after heat supply, the hotter fluid
So here 7 =..!- 10 [60-10]
in the bottom will go up and the colder fluid will come K ge 40-10
down, i.e .• a convective (un"eot starts due to density difference.
The air in contact with a flame expands and becomes less and also 7 = ..!-Io [40
K ge
-10]
0-10
dense, buoyant forces make it rise and convection currents
call be established. In forced convection the' fluid is
log [SO] = log [~]
forced to flow along the solid surface by means of fans Of 30 a -10
pumps. In natural convection the fluid motion is caused 5 30
due to density difference produced by temperature gradient. or -=---
3 (a-l0)
The rate of heat transfer by convection is given by
sa-so = 90
q "" M(ts -t~) or 0"" 28°e
where 11""average heat transfer coefficient. Its unit is
W/m2K. Example
A "" heat transfer surface area
Consider a sphere, a cube and a thin circular plate of same
ts "" solid surface temperature
mass and material, heated to same temperature. Which one of
t "" "" temperature of the fluid stream. these will have the fastest rate of cooling?

Solution: According to Stefan's law,


dT "" cAa (T4 _ T04)
A body cools in 7 minute from 60"C to 40"C. What will be its dt mc
temperature after (he next 7 minute? The temperature of
For all the bodies under consideration, m, c, e, T and To
surroundings is 1DOC.
are same.
Solution: According to Newton's law of cooling, dT
Hence -:x: area
dr
[a. ~a,] [(a. ~a, )-ao]
=K
For a given volume the sphere has least surface area and
the disc has maximum; so the rate of cooling will be
So that [60; 40] [(60 ~40) -10]
=K
maximum for disc and minimum for sphere.

Example
1
i.e., K=- ... (1)
14 Consider a block of copper of radius 5 em. Its outer surface is
Now. if after cooling from 40'{: for 7 minute the coated black. How much time is required for the block to cool
temperature of the body becomes e, according to Newton's down from 1000 K to 300 K? Density of copper"" 9x 103 kg!
law of cooling, m 3 and specific heat 4 kJlkg.

Solution: We consider the block as a perfectly black


body (t ""1). Therefore rate of heat energy radiated out by
the copper block at any instant
which in the light of Eqn. (1), i. e., K = (1114) gives

[4°7- OJ 114[202+ OJ
=
dQ =aAT4
dt
... (1:

where, T is the temperature of the block at that instant


i.e., 160-4a=20+0 or 0=28"<:. and A is the surface area. Temperature T decreases
Method 2: According to Newton's law of cooling, continuously with time as the block cools down.

www.puucho.com
Study Anurag Mishra with www.puucho.com

TEMPERATURE, HEAT AND THE EQUATION OF STATE, HEA••T-"TRA=NSF=ER _ 81

If m is the mass of the block and c is its specific heat, ... (2)
then heat given out, for unit change of temperature, is given
by Where T,\ = temperature of end A of the rod, I is the
dQ ... (2) length of the rod and k its thermal conductivity. In the steady
-=rnc
dT state £1 = E2. Equating equation (1) and (2) we have
From eqns. (I) and (2) we can find the relation between k(TA -Te) =0(1B4 -T04)
change of temperature dT in time dt, that is, I
4 4
d1'=_nAT, ... ()3 k = ol(Te - T0 )
or ... (3)
dt me (TA -Tel
Minus sign shows that temperature decreases as time
Given 0 = 5.67 x 1O-8Wm
K-4. 1= O.lm, -2
increases.
TIJ = 702<>C = 975K, To = 27°C = 300K and T A = 827°C =
Hence, time required for the block to cool down from TI 1100 K. Using these values in equation (3), we get
to T2 is given by
k = 36.6Js-1m-1K-4

Exampl"" 140
V--
or
There are two concentric spherical shells A and B of surface
area 20cm2 and 80cm2• Surfaces of both the shells behave
like black bodies. It is given thar the thermal conductivity of
material of B is very low and that of A is very high. Initially
the temperature of A is 400K and that of B is 300K. Find the
Further, if r is the radius of the sphere, we have
rate of change of temperature of A and B. Given that the heat
m (1 ~(3)P
-,~-~-,-
rp capacities of A and Bare 50 J/ QC and 9OJ/ <>Crespectively.

A 4nr2 3 Solution: Since the thermal conductivity of material


of A is very high so it will not absorb any heat, thus the rate
where p is density of copper. Then, we get
of heat loss by A is given as
t, 'PC
90
[Y,'-Yi)
r ,y ,
3 3
dQ
-
d,
,
= AAoTA

3 3 3 3 = 20x 10-4 x 5.67x 10-8(400)4


= (5 x 10-2)(9 x 10 )(4 x 10 ) ( 10 - 3 )

9 x 5.67 x 10-8 33.103.106 = 2.9J I sec


We have
= 127 x 1035 = 35 hrs, 16 min and 40 s
dQ
- =(ms)A ¥,jT
Examp I"" 139!>- dt dt A
d1"1 1 dQ
or
One end A of a metallic rod of length 10 em is inserted in a dtlA = (ms)A dl
furnace wllOse temperature is 827 <>c. The curved surface of
the rod is insulated. The room temperature is 27'C. When the or dTI ,Jc <2.9
dtA 50
steady is attained, the temperature of the other end B of the
rod is 702 C. Find the thermal conductivity of the metal.
Q
For shell A{ms) A = 50J / sec
Stefan's constant = 5.67><. 10-8W m~2K-4. Now for B it is given that its thermal conductivity is very
poor so it absorbs all the heat incident on it. Thus rate ofloss
Solution: Heat is lost only from the end B of the rod is
by radiation. IfTe is the absolute temperature of end B, then of heat from B can be written as
dQ 4 4
the energy radiated per unit area per time from end B from - =ARoTIJ -AAnTA
dt
Stefan's law is
... (1) or dQ = 80 X 10-4 X 5.67 x 10-8 x (300)4 - 2.9
dt
Where To is the room temperature constant. Also,
= 3.67 -2.9 = 0.77 J/s
energy received at end B by conduction through the rod per
unit area unit time is

www.puucho.com
Study Anurag Mishra with www.puucho.com

82 THERMll:O~D:;YN:::AM:::::ICS~

If dTI ~_1_dQ
Example 142~
dt 8 (ms)a dt
1
=-xO.77 A body cools from aO"e to 70 e in 10 minutes. Find the time
Q

80 required further for it to cool from 70 e to 6ifC. Assume the


Q

For shell B (msJa = BOJ/°C] = O.00963°C/s temperature of the surrounding to be 3ifC.

Example 141 _ Solution: For the transition from SOQCto 70"C


T)o =Tb -T. =80-30=50
(••••
A calorimeter of water equivalent 100gm cools in air in 18 and for 70°C to 60QC•
minutes from 60°C to 4fY'C. When a block of metal of mass (•...
no = 40°C
60gm is heaud to 6a'C and placed inside the calorimeter.
As T = liToe-a
••••
A5sume heat loss only by radiation und. Newton's law of
cooling to be valid. Find the specific heat of.metal if now the Note that temperature difference also decreases
system cools from 60°C to 4fY'C in 20 minutes. exponentially.
40 = 50e-JOe
Solution: Here we apply average form of Newton's
Law of cooling if a body cools from temperature T2 to T, in 30 = 40e-a
time t, then we have
T2 -T1 =...!::-(T2 +T, -T )
J(bln(~)
s
ms
t 2
Where me is the heat capacity of the body and k is the
cr~ln(~)
constant that depends on surrounding temperature and
surface area of body exposed to surrounding. Now
t
In(~)
= lOx __ 3_
calorimeter alone takes IS minutes to cool down from 60° to
40°C thus we have
60-40)~-"-(60+40 T,)
In(~)
( 18 100 2
Example 143!---""""
For calorimeter ms = 100 x 1 = 100
or 20=k(SO_T) A calorimeter contains 50 g of water at The sere.
18 ' temperature Q
falls to 4S C in 10 minutes. When the
10 k calorimeter contains 100 g of water at 50"C, It takes 18 min
or 9"= 100 (SO-Ts) ... (1) for the temperature to fall to 45"e . Calculate the • water
equivalent' of the calorimeter.
Now when metal block is placed in it. takes 20 minutes
to cool down from 60°C to 40QC,thus we have Solution: Assume the surrounding temperature to be
60-40= k (60+40_T) To "C, and that the water equivalent of the calorimeter is 'w'
20 100+60xs 2 • g. In the first case, effectively their is (50 + w) g of water,
k while in the second case, there is (lOO + w) g of water. We
or 1 =----(50-T.) ...(2) have:
100+ 60s
Wk
Dividing equation (1) by (2) we get (45 - To) = (50 - To) x e(so<-w},
10 100+ 60s 18k
9 100 (45 - To) = (50 - To) x eOoo+w),
or 1000 = 900 + 540s 10 18
100 (50+w) (lOO+w)
or s=-=0.lS5cal/gmOC
540 100+w=90+l.Sw
0' 10 =D.8w
w = 12.5 g

www.puucho.com
Study Anurag Mishra with www.puucho.com

TEMPERATURE. HEAT AND THE EQUATION OF STATE, HEAT TRANSFER 83

Example 144 _ ('[' - 25 ) x 20 = IS x IS = 225


T = 36.25"C
A meral ball of mass 1 kg is /teared by means of a 20 W hearer Example 146 _
in a room ar 2crC. The temperarure ofrhe ball becomes sready
at SO-'C (i) Find the rare of lOS$of hear lo the surrounding,
A calorimeter of negligible heat capacity contains 100 cc of
when tile ball is at 50°C. (iO Assuming NewlOn's law of
warer at 40°C. The water cools to 35°C in 5 min. The water is
cooling to be valid, calculate the rate of loss of heat' to
rhe
nm\' repltlCed by K . oil of equal volume at 40"C. Find the time
surrounding, when the temperalllre of the ball is 30"C (iii)
taken for the temperature to become 35"C. (Given densities of
AmHlling that rhe remperarure of rile ball rises uniformly 3
water and K-oil are respectively 1000 and 800 kg.m- ; and
from 20°C lo 30°C in 5 min., find rhe torallass of heat ro the
rheir specific heats are respectively: 4200 and 2100 J/kg-OC)
surrounding in rhis period, (iv) Find the specific heat capacity
of the metal. Solution: From the law of cooling, we have
Solution: (i) The temperature of the ball becomes I I
k,

steady at 50°C. This dearly implies that at 50"C, the rate of (35-To) =(40-Tok q"V,

heat radiated by the ball becomes equal to the rate of heat Since the initial and the final temperature differences
received by the baH from the hei-lter. are the same, \ve must have
= 20 W
PI.Sl.t2 =P2.S2.tl
(ill (ddt
Q)
50
=k(SO-20J t =800:<2100x5=2min
2 1000)(4200
=30k=20W,
Here 20~Cbeing the surrounding temperature; T-To: •
,nd [dQ'1 =k(30-20J=lOk
T'
(,\T)o :
dt /30

Q
d ') =6,67W 0.37(.1.T)0 :••••.•••.
[
dt 30
{O.37)2C'.T)0 ; ...•••••••••..••....• -
The mean temperature of the ball in this interval = 25cC ..o.~ T 2; TI~~(ii
(Assuming uniform rise), Fig.1E.146
dQ) =5k=lOw
[ As the propenies of the exponential functions demand, in
dr a,-u 3
equal intervals of time the fall in the 'temperature
Heat radiated in 5 min. difference' has the same ratio.
=5~60:<10J=lOOOJ
3 Example 147 _
:. Heat given by the heater in 5 minutes = 6000 J
:. The metal takes up 5000 joules to get heated from 20 A copper calorimerer of negligible thermal capacity is filled
with a liquid. The mass of the liquid equals 2S0gm. A heating
to 30 degrees celsius.
element of negligible rhennal capacity is immersed in the
.. 5000 = /1li. 6T liquid. It isfound thar the temperature of the calorimeter and
= 1 x s x 10 its contenrs ri~es from 25°C to 30cC in 5 minutes when a
s = 500 J/kg.K currenr of 20.5 ampere is passed through ir at potential
difference of 5 volts. The liquid is thrown off and the hearer is
Example 145
~ again switched on. It is now found that the temperature of rhe
calorimeter alone is constantly maintained at 32°C when the
Assume rhe temperature of the surrounding to be 25°C. A current through the heater is 7A at the potential difference 6
body takes 5 minures lo cool from 45cC to 40°C. Find the volts. Calculate rhe specific heat capacity of rhe liquid. The
temperature after further 5 minutes. temperature of rhe surroundings is 25°C.

Solution: From the Newton's Law of cooling, we Solution: Liquid is thrown and calotimeter alone
have: maintain a temperature 32°C when current 7A at 6 volts
(40-25) =(45_25).e-(Sk) passes through it so at this time heat generated per sec will
and (T_25)=45_25.e-(Sk)

www.puucho.com
Study Anurag Mishra with www.puucho.com

THERMODYNAMICS
be radiated outward because heat capacity of the
Example
calorimeter is negligible. So at 32°C heat radiated per sec.
p = V x I = 7 x 6 = 42J/s surrounding temperature is
A calorimeter contains 100 cm3 of a liquid of density
25°C temperature difference from surrounding 7ee 3
0.BBg/cm in which are immersed a thermometer and a
at 7°C temperature difference it radiate -42J per sec small heating coil. The effective water equivalem of
at 1"C temperature difference it will radiate 6 J per sec calorimeter, themometer and heater may be taken to be 13
and liquid rise it's temperature from 25"C to 30"<: then gm. Current of 2A is passed through the coil. The potential
average temperature difference from environment 2.5°C difference across the coil is 6.3 V and the ultimate steady state
temperature is 55°C. The current is increCL~edso that the
at 2.5"C temperature difference it will radiate 15J per sec
temperature rises slightly above 55°C, and then it is switched
So in 5 min = 300 sec off. The calorimeter and the content are found to cool at the
2S0xsx 5+ 15x 300:20.5x SX300 rate of 3.6°Clmin.
1.255 + 4500 = 30750 (a) Find the specific heat of the liquid.
1.255 = 26250; , (b) The room temperature during the experiment was 10°C. If
,=
21000Jjkg-'C the room temperature rises to 26°C, find the current required
to keep the liquid at 55°C. You may assume that Newton's
ExalTlple law is obeyed and the resistance of the heater remains
constant.
A solid copper sphere cools at the rate of 2 .SOC per minute,
when its temperatu,e is 127°C. Find the rate at which Solution: (a) If steady state temperature 55°C then
another solid copper sphere of twice the radius lose its at this moment heat energy produced will be radiated at the
temperature at 32;P(;, if in both the cases, the room same rate as it is producing according to given problem at
temperature is maintained at 27°C. 55°C when switch is offed it cools at the rate of 0.06°C/sec.
50
Solution: de = aeA (T4 _ T04) Heat generated == Radiated heat
d, 1m 6.3 x 2 '" (U8B x 100)( sx 0.06 + 13)( 4.2x 0.06)

(~~l
---
de) = -Al x _m2 x (400)4 -(300)4)_
Here s in J/gm-OC
(5 == specific heat of liquid)
and specific heat of water
( d, 2
A2 ml [(600)4 -(300)4] 4.2Jjgm-'C
12.6 = 5.2& + 3.276, 9.324 = 5.2&,
47tR2
= ----x. px4'31txBR3 x[(4)4 _(3)4]
_ ,=--
9.324Jj
5.28
gm-
'C
4x 41tR2 px 4/31tR3 [(6)4 _(3)4]
s = L765J/gm-OC

(~~l
de) =2X( 256-81)= 2)( (175)
1.765
=~-cal/gm.oC
4.2
( d, 2
1296-81 1215
=U42cal/gm-OC
s =0,42 cal/gm-OC
(de) x 1215 (b) dO
ms- rr4
= aAel'
4
-To)
(~~)2 ;5X = dt1 2
de
T = To + ll.T for small temperature difference (Newton's
= 2.8)(1215 =9.72 C/min 0
law)
175x 2 de ,
ms- = aAeTo )( 4ll.T
de

www.puucho.com
Study Anurag Mishra with www.puucho.com

85
TEMPERATURE, HEAT AND THE EQUATION OF STATE, HEAT TRANSFER

=
,
4<JAeTo t'1T
--------- b La b4
TA =-+-,-
= 4oAe(T - 1'0)1'5 I'm k I.~
8
1115 de = 4aAeri(T - To) T • [ 2.89 x 10-3 + 0.5 x 5.67 x 10-
dt A 75000 x 10-10 17
d8
K =4crAeTi, ms =KeT-To) (2.89>< 10-3)4 ]
dt
dO x (75000 x 10-1°)4
P=K(T-To),P=rns-,
dt
TA = 422.1K
K is dependent of environment temperature
Taken
Pl =K1(T-To),P2 =K2(T-T~) 0"::= 5.67 x lO-8wm-2K4

P1 =!S.. x (1' - To) b = 289x lO.3m-K


P2 K'l 1'-1'0

2, 6.3 • 4aAe ,(To)' ,(T - To)


Jx6.3 4aAe To 1'-1'0
A wire a/length 1.0 m and radius 10-3 m is carrying a heary
~.(283)' ,(45), 58,(299)'.)
I 299 29 45 283
current and is assumed to radiate as a black body. At
equilibrium temperature of wire is 900K while that of the
surroundings is 300K. The resistivity of the material of the
I = l.S2A wire at 300 K is n2 x lO-80-m and its temperature coefficient
of resistance is 7.8 x 1O-3/"C. Find the current in the wire.
Example 150;..?' [a:; 5.68x 1O-8wm-2K4]
End A of a rod AB of length L = a.Sm and of unifonn Solution: Given data,
cross-sectional area is maintained at same constant L=lm, r=10-3m, e=l, T=900K,
temperawre. The heat conductivity of the rod is
2
k=17Jjs-m-oK. The other end B a/this rod is radiating To = 300K, P300K = rc x 10-Bn-m
energy into vacuum and the wavelength with maximum a =7.8x 1O-3/oC, I =?
energy density emitted from this end is 1..0 = 75000.4.. If the
emissivity of the end B is e = 1, determine the temperarure of Heat energy generated per sec in wire = Energy
the end A. Assuming that except the ends, the rod is thennally radiated by wire per sec
insulated. According to given problem v.:ire will behave like black
body so e = 1 and at equilibrium condition temperature of
the wire will be 900 K so,
12R90o =oAe[T4 -T04)
7
b = 2.89 X 10-3 m-K, Am = 75000.1. == 75 X 10- m
) . [MelT' - To'1]"
() Ak
L
() R,,,,,
T,
Flg.1E.150

If e ::= 1 at end B then it will behave like a perfect black


.[MeCT' - To')
e..~qoL
-j"
body at steady-state condition the heat energy conducted by a
the rod is radiated by the end B. So in steady state Here L = length, a = wire cross-section area
kA(TA -Til) crATIl4[e=1l, A = 2rcrL, a = Tl:r2, e=1
L
) .[ox
4
2ttrLx (T4~ T0 )x
PJoo(l + a600) x L
a]"
3
ox 2rc2r
4 4
x (T - T0 ) ]"

= [ r:2 x 10-8 x (1 + 7.8x 10-3 x 600)

www.puucho.com
Study Anurag Mishra with www.puucho.com
86 .. THERMODYNAMICS

I "'"[ 5.68)( 10-8 x 2 X n2 x 10-9 x [(900)4 - (300)4] ,


'2 From eqn. (2) and (3)
Tt2 x 10-8 X (I + 7.8>< 600>< 10-3) 4
]
T,, _[T,4
- ~--~+ Zfh J
9 8 1'2 3
I ~[5,68X 2x 10- x 10 [(9)' -(3)'1]
4
10-3)
(l +7.8>< 600><

'2
T24 ""[ 21/ ;Th J
= 5.68>< 0.2><[6561-81]
[ (5,68) ] I' (Th4 - T14)
.Fromeqn.(l)and(4)-"" 4 4'
I =[1296J1,<2 = 36amp ., 1 (Th -T( )

1'~1[(T: -T1')] ",(5)


4 rr4
h _ T/ )

A black plane surface at Q constant hightempcraeure 'fh, is Putting T14, in eqn. (5) and get I' ""1/3
parallel to another black plane swface at COllstant lower
So II ",,3
temperature TI. BetlVeen the plates is vacuum. In order to
reduce the heat flow due to radiation, a heat shield consisting Example
of two thin black plates, thermally isolatedjrom each other, it
placed between the warm and the cold surfaces and parallel to
The shell of a space station is a blackened sphere in which a
these. After some time stationary conditions are obtained. By
temperature T = SOOK is maintained due to operation of
what factor 11 is the stationary heat flow reduced due to the
appliances of the station. Find the temperature of the shell if
presence of the heat shield? Neglect end effects due to the
finite size of the surfaces. the scation is enveloped by a thin spherical black screen of
nearly the same radius as the radius of the shell.
Solution: Plate 1,4 are at constant temperature Th, Tj
these both are black so e ""I, Th > T, Blackened
sphere
So heat current between them envelop
I"" crA(T1r4 -T,4) ",(1)
space to
Area of both are taking equal let A section

So 1 == crA(Th4 - T,4) to reduce this heat current A heat


shield consisting of two thin black plates, which are
thermally isolated from each other are, FIg.1E.153 (a)
placed between them and when
stationary condition will receive then Solution: The radius of the envelop shell is approx,
T, same as the space station and both are blackened then

~0
at this moment temperature of both T"
the plates will be constant and suppose T1 T2 assume that both will behave like a black body [e == 1]
it is Tt, T2 So, In steady state temperature of envelop
Heat received by 2 = Radiated by will be same as the tempecature of the space
3 + Radiated by 1 2 3 4 station (T == SOOK) [Radius of both the T
Flg.1E.152 same] because in steady state condition the
Heat received by energy which the space station will emit,
2 = crAT24 + crATh4 will be emitted (Radiated) by the envelop Fig.1E.153(b)
shell and envelope shell will not absorb any
And it will be equal to the heat radiated because
condition is for the equilibrium [Because it radiate from heat in steady state condition both will be at the same
both side] temperature and radius is approximately, same then amount
of heat transfer (radiate) will be same for both.
2)( (crATi") "" crAT24 + crATh4,
In steady state outer shell will
Zf]4 = T24 + Th4
Same for the plate (3)
",(2) radiate Q heat inward and space station
will also create Q heat inwards (due to
T
CQtoaa a
operation of appliances) so heat
Zf24 "" Tt4 + T,4 ",(3) received by it in the equilibrium
Now the present heat current will be position will be 2Q and it's temperature Fig.1E.153(el
I' "" crA(Th4 - Tt4)
will be T' and outer shell radiate Q heat
",(4)
at T temperature. $0

..~---:::
.'".

www.puucho.com
Study Anurag Mishra with www.puucho.com
TEMPERATURE, HEAT AND THE EQUATION OF STATE, HEAl TRANSFER
------ In first case = 5 x 60 300 sec,
87

Q = aAT4, 2Q = aAT'4 t ==

To = 293K. TI = 353K, TF = 323K


oAT'4 2Q
[A, same because R same]
crAT4 =0 323 = 293(l-c
_H~300
c )+ 353e
-~.300
c
T' = 214 T = (2)1'4 500
-~.30(J -~ _300

Example 154
~
323 = 293 - 293e c
1 __.300
+ 353e
, c

_ =e C
2
A liquid takes 5 minutes to cool from BO°C to 50°C. How
much time will it take to cool from 60"C to 30CC ? The B ~ = log. 2
log, 2 =- x 300,
temperature of surrounding is 20"G. Use exact method. C C 300
In second case
Solution: dQ =crAe[T4 -T04], T =To +h.T t =?,. To == 293 K ,T1 = 333 K, Tf = 303 K
dl
dQ dT
. "
-- I -
,I
-:=-7115- 303 = 293(l-e c ) + 333e c

dT
de de
4 4
__I , ,
--I - " I

-m,- = aAe(T -To) 303=293-293e c +333e c, IO=4Oe c


dl
1 -I
, B log 2
[(-vel because the temperature decreasing] -=C c, +210g,2=--t, 210&,2=+--'-"
4 C 300
dT 4 4
ms=C-C-=crAe(T -To)
dl t = 600sec = lOmin
dT . 4 4
-C-=aAe[(70+h.T) -To] Example 155
~
dl

= crAeTo [1 + 4: T -1]
4

0
A composite rod made of three rods of equal length and
cross-section as shown in the Fig. 1E.155. The thermal
conductivities of the materials of the rods are K/2 ,5K and K
=aAeTo34h.T
respecrivcly. The end A and end B are at constant
dT temperarures. All heat entering the face A goes out of the end
_C =[4crAeT~](T-To),
dl B there being no loss of heat from the sides of the bar. Find the
4aAeTo3 = B (constant) effective thermol conductivity of the bor.
A B
dT dT B
-C-=B(T-ToL ---dl I I
de (T -To) C 1<12 5K K

Here To is the surrounding temperature and' if Flg.1E.155


temperature of the body falls from (T1 K) to (T2K) in time r,
then time t can be calculated Solution: Rrq. :=R1 +R2 +RJ
L
JTF ~
Ii T-To
=-B/C JI de
0
and R:=-
KA
L~ L L L
-B --=--+--+--
Iog (T,
~-~ -To) =-1 KeqA KjA K2A KJA
TI - To C
,
--, and L(q := 3L
, c 3L L L L
--=--+--+--
KeqA KIA K2A KJA

K:= 3 = 3
- ",
.q (11
K;+K +K
1) (21 ](+5K+K
I)
TF =To +Cli -To)e
,
--I
C

-
, I K := 15K
2 J

TF =To +Tte -Toc eq 16


,
C C
,
--t --I
TF =To(l-e C )+T1c C

www.puucho.com
Study Anurag Mishra with www.puucho.com
88 THERMODYNAMICS

Example
, . ,
Solution:
Thickness is also equal = d
A solid copper cube and sphere, both of same mass and Area of cross-section is also equal = A
emissivity are heated to same initial temperature and kept KA(20-IO) 2KA(lo-e)
under identical conditions. What is the ratio of their initial ----- = --~-~
d d
rate of the fall of temperature ?
KA(e+ S1
KA(-S+IO)
= =
Solution: dO = _ aeA (T4 _ 704) because e is same, m d d
de ms t
This is the condition for steady state only
is same, both of copper sphere is same, and both are heated
Upto same temperature and kept under same condition so
e = soc
4
(T - T0 ) is same in initially. Bur (y4 - T04) will not remain
4

same for both every time.


de 0:: -A Two spheres of same radius R have their densities in the ratio
dt 8: 1 and the ratio of their specific heats are 1 : 4. If by
radiation their rates offall of temperature are same, then find
(~:)CUbe A
cube
~~--=--- the ratio of.their rates of losing heat.

(de)
dt 'phm
Asphere

Solution: E = crAe(T4 -To4) Rate of losing heat


4, =M,
Spherepx-1!R R= (3M)'"
_ , Q = ms68
3 4trp
dQ = ms de it is also a rate of losing heat
Asphere = 41/ 3M) " dt dt

2
L =Acube =?
l4rrp
3
pL =M.
(ddtQ)l
m,s,(d8)dt
l

1.'3

L=(~) ,6L2=6[~)
2'3
(ddtQ)2
= m'S2(de)
dt 2

because rate of fall of temperature are same so


"3
Arub< =6[~) (~~J,
=(~~J,
Q
(dat) ,:(dQ)
(*L
de)
Arub< 6X(~r (6)'3
)"3 - ;;
at 2
=~x~

given VI = V2 if radius are same


m2 '2

( de Jphere
= A,phm = 4.( 3M
l4ltp
PI 8 SI 1
-=-, -=-
P2 1 S2 4
ExamF"e
(ddt
Q) ,(dQ)
l'dt
_ V, x p,
2 - V2)( P2 )( S2
s,
The Fig. lE.157 shows the/ace and interface temperature of a
composite slab containing of four layers of two materialS =hx~='8x..!.=2:1
p2 S2 4
having identical thickness. Under steady state condition, find
the value of temperature e.
Exa,m.",le
20~C 10'C e -S'C -WC
iTwo solid spheres are heated to the same temperature and
allowed to cool under identical condition.<;.Compare; (i1
K2KK2K
initial rates of fall of temperature, and (ij) initial rates of loss
of heat. Assume that all the surfaces have the same emissivity
K = thermal conductivity and ratios of their radii, specific heats and deTlSities are
resp'ectively ] : a, ] : 13, 1 : y.
Flg.1E.157

www.puucho.com
Study Anurag Mishra with www.puucho.com

TEMPERATURE, HEAT AND THE EQUATION OF STATE, HEAT TRANSFEIl 89


,;,;;;=='-----
Solution:
(i) Given, R) : R2 = I : 0, 5) : 52 =1 : p
PI : P2 := 1: y, el = e2 A cylinder wich adiabatic walls contains water. The cylinder is
d8=_oACCT4_To4) closed on top with a metal cover with area A, thickness b,
dt ms thennal conductivity K. Heat capacity of water is C,

(dO)
temperature of surroundings is To. assumed to be constant.
AI

(:~)lm~:~
An electric heater produces heat at a rate H. Find the
So
=(~~ = x :: x ::) , ' temperature of water as junction of time after the heater is
switched on.
dt 2 m:<h

(~~)1
--= 4rtRt x_x
__ P2 4"3rtRi
.-x_ 52
3
dO) 4rtRi Pl 43rtR1 51
( d, ,

Flg.1E.161

Solution: Heat conducted through metallic slab,


dQ = KA(T - To)
d, b
Heat gain by water through heater = H
= u. x yx p = apy Net ra:e at which heat is gained by water,
So (dO) :(dO) =a~-I:I
dt 1 dt 2
dQ =H-KA(T-Tol
d, b
dQ "4 dT KA
(ii) -
d,
= aAe(T ~To) e- =H--(T-T o)
d, b

(~;L~=(!!l.)' 1 f'
To
edT
KA
H-T(T-To)
=f'de
0

g)
d A, R, 0'
( d, ,

d( dtQ)dt :(dQ) =1: 0'


e(A-:K)[+_Ao(Tb-To)}[, =,
J 2

be [H - AoK (T - TOl]
---In b =t
AoK H
The maximum in the energy distribution spectrum of tile sun
is at 4753 A and its temperature is 6050K. What will be the
temperature of the star whose energy distribution shows a
H_AoK(T-To) =He-"bC-r ''oK< )
maximum at 9506.4. b

Solution: TAm = b
T1Amj =T2'_rn:z. AK(T-Tl
o [
o=Hl_ebC -l'''K'J]
b
T = T)i'ml
2
I,m,
bH - (AoK
-- ) r]
= 6050 4753 K
X
T=-- l~e be +To
AoK [
9506
= 3025K

www.puucho.com
Study Anurag Mishra with www.puucho.com

90 THERMO=~DY~NAM~~ICS"

Example
= nR
A(Po + pIg)
[TO+ (TI -T,)e -[:';l]
A cylinder with adiabatic wall and base has a conducting
piston on top of it. The piston has area A, length I, thennal
conductivity K. The temperature and pressure of air above the
Example
piston is To, Po respectively. The cylinder contains n moles of
monoatomic gas. Find the length of the gas column as a Consider a cubical vessel of edge 'a' having a small hole in one
function of time. of its walls. The total thermal resistance of the wall is r. At
time t = 0 it contains air at atmospheric pressure Pll and.
temperature T,r The temperature of the surroundings is
T" (> To). Find the amount of gas in mole in the vessel at

B (.J
Fig.1E.162
PA

pAlg

(0'
time t. Take Cv of air SRJ2.

dt.
Solution: If dQ be the heat given to the gas in time

dQ =nCpdT
If the temperature of the gas is T at time t,
... (I).

dQ T.-T
-=-- ... (2)-
Solution: When the piston is in equilibrium, we dt r
have From eqns. (1) and (2),
PaA + pAlg = PA
nCpdT=(T" ;T)dt ... (3)-
P == Po + pig = constant
1fT is the temperature of gas at any instant t, the rate of
(Cpr)ndT=(T" -T)dt ... (4)-
heat flow through the piston is
From ideal gas equation,
dQ = lV\(T - To) _ -nC dT
dt I Pdt Poa3 = nRT ... (5)-

fT dT lV\ df' Gas can leak out of the vessel; hence the pressure inside
T1 T -To =- onC / t the vessel is equal to atmospheric pressure.
p

I [T-To]
n T, -To
2IV\t
=- nCpl
nT = constant
On differentiating eqn. (5), we have
... (6)

ndT+Tdn =0
T-To _ -(~) ndT=-Tdn
----e
T1 -To Also from eqo. (5),
3
T = Poa ... (7)
T=To+(Tl-To)e-(~) nR
On sub~;(ituting eqns. (6) and (7) in eqn. (4), we have
For a mono-atomic gas C p = ~ R 3 3
2 CprPoa
-~--dn poa
= ( T --- )
dt
nR • nR
T=To +(TI-To)e-(~)

If x is the length of gas column at this instant, from ideal f"""--[---p-oa-'-]


dn =-
If' -a-3
-C- -TP-
p o 0 dt
gas equation, we have nR T --~
• nR
P(h) = nRT
nRT I fn dn t
X=--
R 3
[TllnR - Poa ] = - C prPoa3
PA Il()

_I_ln nRT" - Poa


,
RT" ncRTli - Poa3

www.puucho.com
Study Anurag Mishra with www.puucho.com

TEMPERATURE, HEAT AND THE EQUATlDN DF STATE, HEAT TRANSFER


------- 91

Example 164
~ C! dT! =C2dT2 ••• (3)
From eqns. (2) and (3),
A heated peanut (at time t :: 0 and temperature T = To) is C2
taken out of the oven to cool and placed on a table near an dT=(C!;2 )dT! ... (4)
open window. Write an expression for its temperature as a
function of time. If block one looses heat,
dQ = -C1 dT!
Solution: 1fT is the instantaneous temperature and T,
From eqn. (1),
Ibe the temperature of the surroundings, then from NeV'vton's
-law of cooling, dQ _ edT} _ KAT
dt -- 1 di--I- ... (5)
dT
- "-K(T-T)
dt .• From egns. (4) and (5),
On separating the variables and integrating the _ CjC2 dT =KA T
expression, we have C! + C2 dt I
r
To
_d_T_"f'_Kdt
(T -T.,) 0
dT
T
= _ KA(C! +C2) dt
C!C2
In(1' - T.•) -In (To - T.•) = ~ Kt r
To
dT =
T
KA(C1 +C2)
C1C2
f I

a
dt

T -T
1n---$ =-Kt
In I..- =_ KA(C1 +C2) t
To -1'.,
To C1C2
T-T~ -1\'1
---=, _ KArel ~C~)I
To -1'. T = Toe CIC~

Kr
T=T. +(To-T.)e-

_Example 165.---
Two vessels filled with liquids at temperatures T1 and T2 are
Two blocks with heat capacities eland c 2 are connected by a joined by a conducting rod of length L, area of cross-section A
rod of length t, cross-sectional area A and heat conductivity K. and thermal conductivity K. The masses and specific heats of
Initial temperature difference between the two blocks is To. liquids are ml' m2 and C I' C 2 respectively. The vessels and
Assuming the entire system to be isolated from surroundings, rod are thermally isolated from the surroundings. What is the
heat capacity of the rod to be negligible, determine the time required for the temperature difference to be halved?
temperature difference between the blocks as a function of
Solution: Let T! and

-
time.
T2 be the instantaneous m"C,.T, dO
Solution: Let T be the temperature difference temperatures of the vessels at
ibetween two blocks at time t. time t, with T1 > T2. As heat is
conducted from one vessel to Fig.1E.166
the other, the temperatures of
I c, : KA, I : C, I the two vessels tend to equalize. The decrease
temperature difference reduces the heat flow rate.
in

Heat lost by vessell,


Fig.1E.165
dQ =m1C1(-dT1) ." (I)
Heat transferred per second, where (-dT! ) indicates a decrease in T1•
dQ KAT Heat gained by vessel 2
-=-- ",(1)
dt 1
dQ = m2C2 dT2 '" (2)
Due to transfcr of heat, temperature of one block is
where dT2 is increase in temperature of vessel 2.
Ilowered while it increases for the other. Hence change in
otemperature difference, Heat transfer per second is given by
+ dT2
dT =dTj ... (2) dQ =KArT, -T,) ". (3)
dt L
Heat lost by one block is equal to the heat gained by the
oOther. From egns. 0), (2) and (3),

www.puucho.com
Study Anurag Mishra with www.puucho.com

92 ! I. THERMODYNAMICS

dTl _KA(T1 -T2) Solution: For the


-ml C 1 -d-r - ~-L~ ... (4)
time t ~ el' body x cools by
T"",,;' 300 K

m, C '-d-,
dT2 _KA(Tt-T2)
- ~-L~ ... (5)
radiation. According
Ne~on's law of cooling,
to IT. I
= 400 K. ,11 = 0
::350 K,atl::t1
dT
'.' -- ~K(T -TA)
dTt =-~(TI-T2) ... (6) d,
dt m1C1L
. (\ ~=-rlKdt
dTZ KA TQ T-TA 0
-=--{T j -T2) ... (7)
de m2CZL
On subtracting eqn. (7) from (6), we have
1T -T )_K
In ~-~ A
( To - TA - - t1 Flg.1E.167

!!.(T1 -T2}
de
=- KA[_1_
L miG!
+~] mlC2
(Tl -T2)
Kr = _In[(350 - 300)] = In 2 ... (1)0

or J(T1-T2)linaJ
(T\-T2);nilial
d(T1-T2)
T1 -T2
=_KA
L
[_I_+_I_]r
miGl mZC2
dt
0
, (400 - 300)
For the time t > t 1 the heat transfer takes place by
radiation as well as conduction. Rate of heat transfer in
(Tl - T2) denotes temperature difference at any instant t. conduction,

log~ (T} -T2)finaI


(Tl -T2)initial
=_KA
L
[_1_+_1_],
mIGI m2C2
dQ
d,
KA(T-TA)
L ... (2)

If C is heat capacity of the body,


KA( 1
- T '"lq +
1
C
1
"':2 2 { dQ =C(-dT) (3)
Hence (T1 - T)2 final = (T1 - T)2 initial e
dQ=c(_dT) (4,
Let time required for (T1 - T2) to beco'~e halved be T1,I2' dt dt
From eqns. (2) and (4),
J_";("'JIC1+"':21eJTii2) = (T} -T2)1,12 =.!
(T1 - T2)ini!ial 2 _dT) =KA(T-TA)
(
de conduction LC
Taking logarithms on both sides,
From Newton's law of cooling
_KA(_1_ _1_)T",
Lme
IOg,(~)
mlC}
+ =
2 (
_dT) =K(T-TAJ
Z 2
de radiation
_ Llo&t2 _Llo8t2CtC2mlm2
T 1'2
or
. KA(~ --------
1 ) - -------
KA(m1C1+m2Cl). (- ~~)net =(- ~~)radiation +(- ~~)condUctiOn

( --dT)
mlGt +mZC2
KA )
=K(T-TA)+-(T-T ...(S}
A
dt LC
Exomple 167
On separating the variables, we have
:A solid body x of heat capacity C is kept in an atmosphere
whose temperature is T 1\ = 300 K. At time t = 0 the fT'~=_(K+KA)f3"
T-T
T\ A LC t\
d, ... (6)

temperature of x is To = 400 K. It cools according to Newton's


where T2 denotes temperature of body x at t = 3t]. On
law of cooling. At time t 1 its temperature isfound to be 350 K.
solving eqn. (6), we have
i\t this rime (t 1) the body x is connected to a large body y at
atmospheric temperature T 1\ through a conducting rod oJ. In(T,T --TT A) =_ [K + KAJ
LC
(2',)
length L, cross-sectional area A and thermal conductivity K. I A

The heat capacity of y is so large that any variation in its


In(T2 -TI\)=-2Ktl _ 2KAtl
temperature may be neglected. The cross-sectional area A of. T1 -TA LC
the connecting rod is small compared to the surface area of x.
Find the temperature of x at time t = 3t 1. On substituting 2Kt1 = 10 2 and solving for T2, we get

T, = [300 + 12.5 e- ~' ] kelvin.

www.puucho.com
Study Anurag Mishra with www.puucho.com

TEMPERATURE, HEAT AND THE EQUATlDN DF STAT!, HEAT TRANSFER

Example 168
~
---------- Example
93

- -
A conduecing cylindrical shell of inner radi11.SR1 maintained A hollow shpere of inner radius R} = 0.25 m alld outer radius
at a temperature TJ and ollter radius R2 maintained at a R2 = 0.50 m is placed inside a heat reservoir of temperature
temperature T2 respectively. 71lermal conductivity shell vanes To = lOOO"C. The hollow sphere contains water at initial
witll di.~tance from the axis as K : ~ . Find the temperature temperature DOC.The thennal conductivity of the material is
r- 102
K=-W/m-K.
as a function of distance R from the axis of the cylinder. 4rr
Solution: Let heat transferred per second is Q, then
dQ dT
q=-=-K2rr.rl-
dt dr Reservoir
u dT a dT To"'1000.C
or q=2rr./r--=2,,[--
r2dr rdr

u,
:::::::::.:: :r.-:-,
.
1J, ,: \:', Fig. 1E.169 (a)
r r+dr R1 :: :'
,~ '\ (a) Find the time required to raise the temperature of water to
'. "
R2 ~', :,' lOOOC.
" " "
~-t::' (b) Find the time required to convert all the water into steam
at lOOOC.
Flg.1E.168
Assume that heat absorbed by sphere is negligible.
Take specific heat of water c = 4.2 kJ/kg. latent heat of
vapourization Lli = 2268 kJ/kg.
2
(Ri - R1 ) = 2:tlu (T -T ) Solution: Ca) Let T be the instantaneous temperature
2 1 of the water. The rate of heat flow is given by
2 H
Q = 4rrlo:(T2 - T}) dQ
-=-
"T ...{l)
... (I) dt Rrh
(R~_R:2)
where R1h is the equivalent thermal resistance of the
frR1 rdr=2;rluf'
Q Tl
dT hollow sphere. Consider a thin spherical shell of radius rand
thickness dr as shown in the figure. Then
dr
dRth =----
K(4nrz)
(r2 _ R 2)H 1 fR2 dr
T-T1 I
=---~-
4nlu
R1h = 4JtK Rl -;z
2 2 Rz -R1
T -T (r _R )Q j
=
or - ,+----- ... (2) 4rUffi1Rz
4rrla
For R1 = 0.25 m;
10'
Rz = 0.50 m; K = - W/m-K
4rr
R = 0.5-0.25

471:( 140rcO)C0.50) CO.2S)


=2x lO-z K/W

www.puucho.com
Study Anurag Mishra with www.puucho.com

THERMOOYllAMICI i

Example y-
170

A Hon-conducting vessel containing n moles of a diatomic gas


is fitted with a conducting piston. The cross-sectional area,
thickness and thermal conductivity of pis.ton are A I and K
re.~pectively.The right side of the piston is.open to atmosphere
auemperature To. Heat is supplied to the gas by means of an
'electric heater at a constant rate q:
(a) Find the temperature of the gas as a function of time.
Flg.1E.169 (b)
(b) Find the maximum temperature of the gas and
Heat required to raise the temperature of water by dT is (c) What is. the ratio of the maximum volume to the
given by dQ = me dT. Substituting the value of dQ in minimum volume?
equation O),.we get
Solution: (a) Let temperature of the gas in the
medT =1'0-1'
vessel be T at any instant r. The net rate at which heat is
dt R
absorbed by the gas
or meR JOT ~
To - T
= r0
dt dQ
-d-r=q=
KA(T - To)
I ...0;
KA(T - To)
or meRlnl T;-=T]
To I=t =q=--I-
'.'

On substituting numerical values, we get t

ah'"
3
C = 4.2 x 10 J/kg;

m = pV = (10)3 x [~ mD.25)3] = 65.45 kg


-011>0-
R =2>< 1O~2.K/W Flg.1E.170

To = lOOO"q T = 100°C Let heat supplied dQ will change temperature by dT.


t =(65.45)(4.2>< 103)(2x 10-2) Inl 1000
1000- 100
1
The process will be isobaric as piston is open to atmospheric
pressure
7
=576s=9min39s Thus dQ = ne p dT = 11 x - R x dT
2
(b) Heat required to convert 65.45 kg of water into
steam is From eqn. (1) and (2), we have
7nR dT KA(T - To)
Q = mLt, :::; (65AS){2268 X 103) = 1.48 x 108 J 2dt=q- I
The rate of heat flow is dT =_2_ dt
dQ ~T ql - KA(T - To) 7nRI
-=-
dt R On integrating within limit, we get
= 1000-100 =4.5x 104 W dT 21'd
2 x 10-2 I T
T(l ql-KA(T -To) = 7nRi 0 t

:. Time required is _l_x In ql-KA(T-To) =~r


or
-KA ql- KA(To - To) 7nRI
1.48x 108
t=----
4.5>< 104 KA
1n [ l--(T-T ] -2KAt
or o) =--
= 3289 s= 54 min 49 s ql 7nRl

or T = To + ~fl- r(ZKAfH7nR1l] ... (3


KA

www.puucho.com
Study Anurag Mishra with www.puucho.com

TEMPERATURE, HEAT AND TH' EQUATIQN QF STATE, HEAT TRANSFER


------ 95
(b) At the maximum temperature T max the rate of heat Example 172 _
supplied by heater is equal to rate of heat lost by the gas

q=~ erma" -To} We would like to increase the length of a 15 em long copper
rod of cross-sectiOIl 4 mm 2 by 1 mm. The energy absorbed by
or Tmax ='['
0
+cL the rod if it is heated is E1. The energy absorbed by the rod if
KA
it is stretched slowly is £z' Then find EIfEz.[Various
Same result can be obtained by takingt -). en in equation parameters of copper arc: Den"ity =9>< 103 kg/m3,
(3)
Thennal coefficient of linear expansIOn =16><
(c) As the process is isobaric, from ideal gas equation we 10--6 K-1 Young's modulus = 135 x 109Pa. Specific heal
-have = 400 Jlkg-K]
Vo Vmax
-=-- Solution: Temp. is increased by,.",O then
To T max
1':./ = lcu\O
Vmax = Tmax =1+~
Vo To KAYo I'!.O = ,)/
I"
,1
Example 171!--" E, =(pAl)SA8=pAIS-
I"
A rigid box contains onf mole of an ideal gas of molar heat Al
when stretched, stress = y-
capacity at constant volume C. The walLs of the box have total I
surface area A, thickness to and thermal conductivity k.
Initially the gas is at a temperature To and pressure Po' The
£z =-!(yAI)(Al)x Al = Y(l'./)2A
2 I I 21
temperature of the surrounding isToi2 Find the temperature
of gas as a function of time. So, .s. = pAIS!'.l x 21 = 2pSl = SOD
£2 Ix Y{.1./)2A a(I1l}Y
Solution: dQ =c dT
dt dt Example 173 _
de =KA(TJO)
dt to 2 A cylindrical volume of cross section area A and length L
CdT =_KA(r_70) contains a uniJornl mixture of three ga.ses H2,°2 and CO2,
If their ratio by volume be 1 : 1 : 3. Find the rime taken by a
dt to 2
sound to travel this length L if the temperarure varies linearly
from Tj (= SlOG) to T2( == 88"C) across (he !eng(h a/cylindrical
volume.

n =1
ToPe
Solution:
. (~RHH+(3RX3)
C" (miX) = ~--- _
5
21
=-R =2.1R
10
Fig.1E.171 3,8
.(mix =-
2,8
Speed of sound at a point where temperatures is T
T C
f-dT=f-dt
ToT_To
t

0
-KA

to
equals It ~Twhere M is mass, of one mole.

2 Mo +MH +3Mco
Here, M= 2 2 '2

5
Time taken to travel distance dx at distance x from end
at temperature T, = de = --
dxJM as
JyRT

www.puucho.com
Study Anurag Mishra with www.puucho.com

96 THERMODYNAMI.

From (iii) and (iv)

2MS-
de = Ia(w
o -at)
d,
de 1 MR' 4,<g ( 4,<g)
=> dt = 2MS -2-' 3R Wo - 3R t

dO = "gR ("'0 _ ""g,)


dt 3S 3R

A hollow spherical body has inner and outer radii as R1 and


R2 respectively. Thermal conductivity of the material of the
body is k.There is a narrow passage in the body as shown in
the figure At t = 0, temperature of air in the spherical cavity is
Twhile outside temperature is To( < T} Ifno be the number of
moles at time t. Consider air to be largely consisting of
Solving wet gett "'"3.5J.1sec
diatomic gases.

A uniform disc ofmas5 M and radius R rotating with angular


frequency 000 in its own plane is gently placed on another
identical disc which is fixed to a horizontal table. if the
coefficient of friction between the surfaces of the disc be J.I, find
the rate at which temperature of the two disc system increases
with the time. It can be assumed that any loss is K.E. instantly
changes into heat energy. Specific heat capadry of the Fig.1E.115
material of the disc is S. Assume no loss of heat to
surroundings. Solution: Thermal resistance of the spherical
body
Solution: Consider a circle differential strip of
radius x and thickness dx. Mass of the strip dm = 2M x dx
R'
=-1-7~ =_1_[R -R J=r,sa
41tk R, x 4Jtk
2
R2R1
1 y

Frictional torque on this strip is


dQ = (To -T) => dQ = (To -T) d, ...(ii)
dT = !J2Mg x2dx dt r r
R'
Also dQ =I1CpdT ... (iii)
...Torque on the disc is
, nC prdT = (To - ndt ... (iv)
T = f dT = 4;::g f x dx 2

PV =nRT=> [T = :] ... (v)


o
T = 4<MgR ... (i) O=R(ndT+Tdn)
3
ndT = -Tdn .... evi)
T 4,<g
0.""-;;:)0.=- ... (ii)
I 3R From (iv), (v) and (vi)
Now dQ = 2MSdO C prPoV
----dn= ( T -- PoV) de
o
nR nR
dQ = 2MS dO ... (iii)
dt dt where Po is the atmospheric pressure.
dn dt
Bu' dQ = I..'!.
(K. E. ) = I I "'I d", I 11 f

dt dt de LnRTo -PoV = !cprPov


dQ ... (iv)
- =Iaw=la(roo -at)
d,

,, www.puucho.com
Study Anurag Mishra with www.puucho.com

TEMPERATURE, HEAT AND THE EQUATION OF STATE, HEATTRANSfER,' • 97

Integrating \VC get, = 17>( 9 x 1010 :<4=3.25x 1010 N/m2

n ~ ~~~[l
+(; -Iy,~,,] =
47
17 x 36 x 1010 = 13.02x 1010 N/m1
47
_17 x18 1011\-6"1 IOiON!,m 2
_ X -.:;J"
47
Example 176
~
Example 177
~
A composite bar is rigidity auached to the end support. TfIC
temperature of the composite .")'slem is raised by SO"c. Find A container ha.~ 2 moles of nitrogen at 27'C. Tile pressure in
out the stress in ellree portions of tfle bar if the supports are side the container is 2 atmosphere. A,slIming the molecules
rigid. move with roO! mean square speed. Then finu
(a) 111e number of collision~ per second Wllicll lIle mo/eet/lrs
Alumlrllum make with unit area of tile container wall.
~copper ~sleel
....... 101 om' ------- (b) If the container is made thermally in.wlated and moves
with constant speed vo.If it stops suddenly, the proces., reslilu
5 cm2 5 cm2
in the rise of the temperature of the gas by re. Calculate the
25 em SOcm 25 em
speed 1'0'
Fig.1E.176
Solution: The number of molecules n present per unit
[Given: Ys =200 GPu,Us == 12 x lO-ro /"C, volume at pressure P and temperature T
Ya = 90 CPa. aU = 20 x 10 -6joc, n =~ = 2x 1.05x lOS = 5.07)( 1025
Yc = 100 CPa, ac = 16x w-n/"el kT 1.38)( 10-23 l': 300

Solution: Feu = VenAe" (lcllj'cu~T - ~\lc,,) " ~ )3RT ~ /3N "kT


rm.' M \ 1\1
1o,
_ YAlAAjllAli-A16.T-llIAj) = /3 x 6.02 x 1023 x 1.38 x 10-23 )( 300
FAl -~--------
/" \1 28xlO-3
1
_ YSIAsIClsli.Sl.1T-.6.ISt) = 516.75ms-
F" ~~~~~--~~
I" Number of collisions/m 2 =.! x n x l1r .,.m
6
::::> Dis! =[FSt.1 SI J_I_-lsri.sl6.T
ASl YSI =.!x 5.07)( 102s x 516.75= 4.37 x 1027
6
= Ji... -l sl ASt liT ") Ki'
(11 netic energy = -1,nw 0
Y" 2
Feu = FA] = FSI Heat energy gained =Cv.6T =Cv x 1 =C,. and
~lcll + !lIAl + L\/Sl = 0 Cp -C,. =R
C, R
--l~-
C,. C,.
R . Cv =~
y-l=- Le.
C, y -.1
1 ,
C,' = ~mvo
2
R 1 ,
-- =-mvo
r -1 2

which gives Vo = ~
f;;;~
_I 2x8.31 -385ms-1
- V 28x 10-3 x (lA-I) .

www.puucho.com
Study Anurag Mishra with www.puucho.com

98 THERMODYNAMICS

Example Example

A certain amount of air at 300 K is trapped in a glass tube j\ rod is initially at a uniform temperature of DoC throughout.
between its closed end and a 15 em long mercury column as One end is kept at DOC and the other is brought into contact
shown in figure. with a steam bath at 100°C. The surface of the rod is isolated
(a) What will be the length of air column if the tube is so that heat can flow only lengthwise along rod. The
inverted isothennally ? cross-sectional area of the rod is 2 .5cm 2, its length 120 em,
its thermal conductivity is 380 W/m K its density is
(b)To what temperature should the air column be heated so
1 x 109 kg / m3 and it5 specific heat is 520 J/ kg K. Consider a
that it regains its original volume?
shon cylindrical element of the road 1 cm in length. If
Atmospheric pressure = 75cm of Hg Neglect surface tension. temperature gradient at the cooler end of this element is
140 °Cjm and average temperature of the element is rising at
O. 250°C/s. What is the temperature gradient at ,he other
40cm
end of the element?

Solution: Denoting the two ends of the element as 1


Q liT 6.T 0
15cm and 2, H2 -HI =- = mc-, where - =0.250 Cis.
t t t
kA ~!:J _ kA dTI =- m16.C). The mass m is pAAx, so
Fig.1E.178 ~2 dx 1 "l t
Solution: en at initial condition pressure on the
kA dXl
QI + PC""("T).
k t
2
gas is
Pl = (Po - pg15)where Po is atmosphere pressure kA dTI
dx,
= 140°C/m +

Volume of the gas Po = pg75 VI = Uo x A (1.00 x 104 kg,!m 3)(520J/kg. K)(lOO x 10-2 m)(0.250 0(, 5)
where A area of the tube 380Wim.K
When tube is inverted, then
=-174°C/m
P1 = (Po + pgIS)
V2 =? Example
PI VI = P2V2 (process is isothermal)
(a) The shell of a space station is a blackened sphere in which
(Po -pgIS) x 40A = (Po +pglS)V2
a temperature T = 500 K is maintained due to the operation
V2 =Ax I of appliances of the station. Find the temperature of the shell
(Po -pgIS) x 40A = (Po +pglS)Al if the station is enveloped by a thin spherical black screen of
1 = pg60x 40 = 80 em nearly the same radius as the radius of the shelL The
00pg 3 conduction of heat between the shell and the screen is
negligible.
(ij) If it regain it's actual volume then it has heated upto
(b) A gas consisting of rigid diatomic molewle.s of degree of
temperature (n K from 300 K Process is isobaric, then 'freedom f =- 5 was initially at standard the pre.ssure
VocT Po =- 1.013 x lOs Pa and To =- 273K. Then the gas. was
80 compressed adiabatically '1 = 5 times. Find the mean
T1 = 300K, VI =-xA
3 rotational kinetic energy of a molecule in the final state.
VI TI (Given K (Boltzmann Constant) = 1.38 x 10-23 JjK molecule)
-:-
and 5°.4 =1.9)
V2 T2
T2 = (T)K, V2 = 40x A Solution: (a) Here station is a black body which is
80 x A surrounded by another black body. Heat transfer takes place
3 300 by radiation only.
:--
40x A T 1fT = temperature of the surface ofthe space station and
2 300 T' = temperature of the screen
-:--
3 T Rate of loss of energy by space station [Q screen
=sAfT4 _T,4]
T = 450 ~ it should be heated 300 K to 450 K

www.puucho.com
Study Anurag Mishra with www.puucho.com

ruRE, HEAT AND THE EQUATION OF STATE, HEAT

The screen lose energy into the outer space at the rate
=s.AT,4
:::) For thermal equilibrium,
~V'-' =iH-'
T =T."r-l
sA(T4 _T,4)=sAT,4
T4 =2['4 2 2
As y=l+ ,Y-l=j
4 j
T,4 = T
2 T=To.,,:I,1 =TO.,,2'S

T'=2- Since there are two rotatio~al degrees of freedom,


2~i4
KERo{ =[l/2KT]=K.To.,,215
T' = 500 = 420K :::) KERor",(1.38xlO-23)x273xS2s
1.19
"'7.2x l021J
.
(b) We have T Vy-1 = constant in adiabatic process .

www.puucho.com
Study Anurag Mishra with www.puucho.com

THERMODYNAMICS

Subjective Problems

[For coefficient of thermal expanSlOTZ, specific heat and thermometer is placed in thermal contact with the
thermal conducrivir)' refer to tables 1.1, 1.2, 1.3 boiling substance once again, its pressure is 146.65
respectively.] torr. Find the ideal-gas temperamre of the boiling
substance.
[A] TEMPERATURE OF EXPANSION [Ans. 400.5 K]
4. If a rod of original length LI has its temperature
1. A gas thermometer registers an absolute pressure changed from TI to T2, determine a formula for its new
corresponding to 325 mm of mercury when in contact length L:! in terms of Tl> T2 and a. Assume (a)
with water at the triple point. What pressure does it u = constant, (b) a = Cl(T) is some function of
fead when in contact with water at the normal boiling temperature, and (c) (1 = ao + bT where no and bare
point? constants.
[Ans. 444 mm of Jig]
2. An experimenter using a gas thermometer found the
[Ans. (a)LI(I+u,,-\T)(b)LI[l+ f1j
T
2 urr)dT]
pressure at the triple point afwater (O.Oll'C) to be 4.80 (e) Ld1 + (IOU'2 - TI) + ~ (7'/ -TI2)J]
x 104 Pa and the pressure at the normal boiling point 2
(loo"e) to be 6.S0x 104 Pa. 5. A steel tube has an outside diameter of 3.000 em at
(a) Assuming that the pressure varies linearly with room temperature (20°C). A brass tube has an inside
temperature, use these two data points to find the diameter of 2.997 em at the same temperature. To
Celsius temperature at which the gas pressure what temperature must the ends of the tubes be
would be zero (that is, find the Celsius heated if the steel tube is to be inserted into the brass
temperamre of absolute zero). tube?
[Ans. 395°C]
(b) Does the gas in this thermometer obey PI = T1 6. Show that if the length is permitted to change by an
P2 T2
amount tll. when its temperature changes by liT, the
precisely? If that equation were precisely obeyed, stress is equal to
and the pressure at 100°C were 6.50 x 104 Pa,
what pressure would the experimenter have F = Y (tll. _ u.1lT)
A L '
[Ans.
measured at 0.01 "C?
(a) -282"C, (b) no, 4.76 x104 Pa]
"
where F is the tension on the rod, Lo is the original
3. A constant-volume gas thermometer with a length of the rod, A its cross-section area, a its
triple-point pressure P3 = 500 torr is used to measure coefficient of linear expansion, and Y its Young's
the boiling point of some substance. When the modulus.
thermometer is placed in thermal contact with the 7. Standard atmospheric pressure is defined as the
boiling substance, its pressure is 734 torr. Some of the pressure required to support a mercury column exactly
gas in the thermometer is then allowed to escape so 760 mm high, when the temperature is DOC. (a)
that its triple-point pressure is 200 torr. When it is Usually, a barometer is read at some different
again placed in thermal contact with the boiling temperature. Explain why a correction to the mercury
substance, its pressure is 293.4 torr. Again, some of the column height reading must be made. (b) If the
gas is removed from the thermometer so that its barometer reading is h and the Celsius temperature is
triple-point pressure is 100 torr. When the r, show that the barometric pressure corresponds to

www.puucho.com
Study Anurag Mishra with www.puucho.com

TEMPERATURE, HEAT ANO THE EQUATION OF STATE, HEAT TRANSFER 101

the corrected column height /1' == lI(l - Ilr), where p is tempertltllfe, what are the tension in the wite and the
the volume coefficient of expansion of mercury and x-coordinate of the junction between the wires?
the expansion of the glass barometer tube is neglected. (Y""el = 200x 109 N/m2, us1eel == 12x 10 Ii mre,
b
8. A mereul}' c;; CLcopper =17xl0 m/oC)
thermometer is [Ans. 62.8 N; 2.0002 MJ
constructed as A T 13. A steel guitar string with a diameter of 1.00 mm is
shown in Fig. IA.B.
The capillary tube
"1 stretched beh'leen supports 80.0 cm apart. The
temperature is O.O°C.(a) Find the mass per unit length
has a diameter of of this string. (Use 7.86 x 103 kg/m3 as the mass
0,00400 em, and the
bulb has a diameter
( density.) (b) The fundamental frequency of transvcrse
oscillations of the string is 200 Hz. What is the tension
of 0.250 em. in the string? (c) If the temperature is raised to 30.0cC,
Neglecting the T,+!l.T
find the resulting values of the tension and the
expansion of the Flg.1A.8
fundamental frequency. (Assume that both the Young's
glass, find the modulus and the average coefficient of linear
change in height of the mercury column that occurs
expansion have constant values between O.O°C and
\'.'ith a temperature change of 3D,Oec.
30.0°C.)
[Ans. 3.55 em]
[Ans. (a) 6.17 x 10-3 kg/m, (b) 632 N, (c) 580 N, 192 Hz]
9. A liquid with a coefficient of volume expansion jJ just
fills a spherical shell of volume Vi at a temperature ofT, 14. A long, thin metal bar l>

(see Fig. lA.B). The shell is made of a material that has of length I is clamped
an average coefficient of linear expansion u.. The liquid rigidly at its ends at
is free to expand into an open capillary of area A temperature to' When
projecting from the top of the sphere. (a) If the the temperature t is
temperature increases by sr, show that the liquid rises increased, the
expanding bar will Fig.1A.14
in the capillary by the amount L\h given by the
equation j,h = (VdAHI1-3u)b.T. (b) For a typical bow out, as shm'ln
system. such as a mercury thermomcter, why is it a below. If the bowing is not too large, a fair first
good approximation to neglect the expansion of the approximation to the shape of the bar is h'lO equal
straight segments in the form of a wide V What is the
shell?
arch 8 of the bow as a function of t? (This is the
10. A metal rod that is 30.0 cm long expands by 0.0650 cm
distance between the corner of the V and the straight
when its temperature is raised from O°C to 100°e. A
line that represents the form of the bar at to')
rod of a different metal and of the same length
[Ans.o=n'2)/2u{r-lu)]
expands by 0.0350 cm for the same rise in
temperature. A third rod, also 30.0 cm long, is made 15. At an absolute temperature To, a cube has sides of
up of pieces of each of the above metals placed length Lo and has density Po. The cube is made of a
end-to-end and expands 0.0580 em between oec and material with coefficient of volume expansion Il (a)
100°e. Find the length of each portion of the Show that if the temperature increases to To + /!"T, the
density of the cube becomes, approximately;
composite bar.
[Aos. 23 em. 7 cmJ p ::Po(l-P/!"T)
11. The pendulum in a grandfather dock is made of brass [Hint:Use the expression (I + xt ::1 + nx. valid for
and keeps perfect time at 17~C. How much time is I).'1« 1.J
gained or lost in a year if the dock is kept at 25°C? Explain why this approximate result is only valid if /!"T
(Assume the frequency dependence on length for a is much less than liP, and explain why you would
simple pendulum applies.) expect this to be the case in most situations.
[Ans. 40 minJ 16. The rate of a pendulum dock depends on the period of
12. A steel wire and a copper wire, each of diameter 2.000 the pendulum, which depends in turn on the length of
mm, are joined end to end. At 40.0"C, each has an the pendulum. But if the temperature changes, the
unstretched length of 2.000 m; they are connected length of the pendulum also changes and the clock
betv....
een two fixed supports 4.000 m apart on a rate changes. (a) Consider a typical clock pendulum,
tabletop, so that the steel wi.re extends from consisting of a heavy brass bob hung at the end of a
x = -2.000 m to x = 0, the coppet wire extends from long, thin steel rod. Such a pendulum is a good
x = a to x == 2.000 m and the tension is negligible. The approximation to a simple pendulum. Suppose the
temperature is then lowered to 20.0eC. At this lower clock runs correctly at some temperature to with a

www.puucho.com
Study Anurag Mishra with www.puucho.com

~02 THERMOOYNAMICS
pendulum period to. Show that when the temperature change in 1r~a MIA = 2a. What approximation do
differs from toby an amount dr, the proportional error you have tv m",e in deriving this result?
in the rate of the clock is approximately 23. If a fluid is contained in a long narrow vessel so that it
In 1 can expand in essentially one direction only, show that
- =- a !J.t.
'0 2 the effective coefficient of linear expansion a is
where u is the linear coefficient of expansion of steel. approximately equal to the coefficient of volume
(b) Show that the clock will be in error by about 0,5 s expansion f3.
per day for each one-degree variation of temperature 24. fuo rulers made of different materials have the same
above or below to- (e) What would the corresponding length I at temperature To. At temperature t\, the ruler
error be if the pendulum were made of invar instead of B is longer than ruler A by 6.1.Express the coefficient
steel? of expansion a B of the material of which ruler B is
[Ans. (e) 0.0432 s] made in terms of the coefficient a A of the material of
17. A bar consists of two sections of different materials, ruler A.
having coefficients of expansion a} and 0:2' The length
of section 1 is a fraction /1 and the length of section 2 [Ans. all :--
,,' + uA]

is a fraction /2 of the total length of the bar. Show that '"r


25. The relationship Lf = LiO + a 6.T ) is an
the bar, taken as a whole, has a coefficient of approximation that works when the a\'erage
expansion a '" u1 11 + 0:2 f2' coefficient of expansion is small. If a is large, one must
18. Prove that if an object under pressure is raised in integrate the relationship dLl dT = a L to determine the
temperature but not allowed to expand, the increase final length. Assuming that the average coefficient of
in pressure is linear expansion is constant as L varies, determine a
~p=BP~T, general expression for the final length.
where the bulk modulus B and the average coefficient [Ans. L,=L;.e<1toT]
of volume expansion p are both assumed positive and 26. A barrel of diameter 134.122 em at 20°C is to be
constam.
enclosed by an iron band. The circular band has an
19. A cubic block of volume V is made of a material whose inside diameter of 134.110 em at 20°C. It is 7.4 cm
linear coefficient of thermal expansion is a. If the wide and 0.65 em thick, (a) To what temperature must
temperature of the block is increased by .1.t, show that the band be heated so that it will fit over the barrel?
the increase M in the surface area is (b) What will be the tension in the band when it cools
:1,'3
M = 12aV lit. to 20°C?
20. At QOC, the length of an aluminium rod is 1.00 m and [Ans. (a) 27'C, (b) 4.3 x 103 ]\;1
that of a copper rod is 1.38 m. (a) What is the 27. An aluminum bar has the precisely desired length
difference between the lengths of the two rods when when at 15°C. How much stress is required to keep it
the temperature is 100°C? (b) Show that the at this length if the temperature increases to 35°C?
difference in length between any two rods is [Ans. 3.5 x 107 N/m2 ]
independent of temperature if their lengths are chosen
28. A 23.4 kg solid aluminium cylindrical wheel of radius
so that 12/'1 = aJla2'
[Ans. (a) 61= 0.380 m (no change)] 0.41 m is rotating about its axle in frictionless bearings
21. A uniform rectangular plate with angular velocity w = 32.8 radls. If its temperature

~Di
I- l--=-----l
~t.1 is now raised from 20.0°C to 7S.0°C, what is the
of length 1 and width w has
fractional change in cO?
coefficient of linear expansion
[Ans. -2.8 x 10-3 (0.2801b)J
a. Show that, if we neglect w '
ve!)' small quantities, the ~r 1 29. A student measures the length of a brass rod with a
_._.'
change in area of the plate ~Aw steel tape at 20.DoC. The reading is 95.00 cm. What
due to a temperature change will the tape indicate for the length of the rod wht'n
Flg.1A.21
.1.T is M = 2a1w liT. See Fig. the rod and the tape are at (a) _Is.O°C, and (b)
lA.21. S5.0°C?
22. A rectangular plate is made of a material having a [Ans. (a) 94.97 em, (b) 95.03 em]
linear coefficient of expansion a. At temperature To, 30. An iron cube floats in a bowl of liquid mercu!)' at DOC.
the sides of the plate have lengths a and b, so its area is (a) If the temperature is raised to 25°C. will the cube
A = abo Show that, when the temperature is changed float higher or lower in the mercu!)'? (b) By what per
cent will the fraction of volume submerged change?
to a new value t, the plate experiences a proportional
[Ans. (a) float higher, (b) 18%]

www.puucho.com
Study Anurag Mishra with www.puucho.com

31. A dock with a brass pendulum has a period of 1.000 at


-------
TEMPERATURE, HEAT AND THE EQUATION OF STATE, HEAT TRANSFER

suspension are made alternately of aluminium and


103

2a.D°c. If the temperature increases to 3D.DoC, (a) by steel. The length of the pendulum, from the pivot to
how much does the period change, and (b) how much the midline of the heavy weight, is 1.100 m. (a) What
time does the dock gain or lose in one week? should be the length h of the suspension bars if the
[Ans. 9.49 ,,10-5 s, (b) 5.74 s los!] period of the pendulum is to be independent of
32. Use the equation of state for an ideal gas and the temperature? (b) Generalize the pendulum illustrated
definition of the average coefficient of volume below. Let the pivot-to-midline distance be I, let the
r
expansion, in the form = (1,IV)dVldT to show that linear coefficients of expansion of the two metals be
the average coefficient of volume expansion for an at and a2' and suppose that there are 2n vertical
ideal gas at constant pressure is given by p = 1/1' where stls?ension rods on each side of the main pendulum
T is the absolute temperature. rod. Find the proper value of h /1.
33. A hollow aluminium cylinder 20.0 em deep has an
internal capacity of 2.000 L at 20.aoC. It is completely
38. A rectangular copper ,
plate of length x and f,.
filled with turpentine and then warmed to SO.DOC. (a) widthy is clamped snugly y~

.
How much turpentine overflows? (b) If the cylinder is at tWO opposite corners, "
then cooled back to 20.0"C, how far below the surface as shO\vn in Fig. lA.38. ~'
of the cylinder's rim does the turpentine's surface The temperature of the
recede? Fig. 1A.38
plate (but not of the
[Ans. (a) 99.4 cml, (b) 0.943 em] damps) is raised by M.
34. Derive the equation p = 3(.(, for the relation between (a) Find the angle y. (b) Find the shear stress crin the
the volume and linear coefficients of thermal plate. (c) If the plate is three times as long as it is wide,
expansion for an isotropic solid. what temperature difference will give y = 89.9~? What
35. A steel tape is placed around the earth at the equator is the corresponding value of cr?
'when the temperature is DoC. What will be the XMt
[Ans. (a) y = cor-l -- •
clearance between the tape and the ground (assumed y
to be uniform) if the temperature of the tape rises to
b
()a=GB=G (
,
"2-Y J =Gran y
XMI -I
30"C? Neglect the expansion of the earth.
[Ans. 2.1 km] (c) l!.t = 34.6"C, (J == 8.0 X 107 Pal
36. (a) A car has a 60 L steel gas tank filled to the top with 39. A solid disc of radius R and mass M is spinning in a
gasoline when the temperature is 1O~C. The frictionless environment with angular velocity (~ at
coefficient of Volume expansion of gasoline is temperature T1. The temperature of the disc is then
~ = 0.900 x 10-3 K-I. Taking the expansion of the
changed to T2. Express the angular velocity w2,
steel tank into account, how much gasoline spills rotational kinetic energy E 2 and angular momentum
out of the tank when the car is parked in the sun L2 in terms of their values at the temperature T1 and
and its temperature rises to 25 C? C
the linear expansion coefficient a of the disc.
(b) Show that change in the density of an isotropic <l1 MR;ru? MR12~
[Ans.(a)m2= 2,K2= 2,L~=--1
material due to an increase in temperature !1T is (l+uAt) 4(l+u!l.t). 2
given by t!r.p = -pp t!r.T.
40. On a very hot day, when the temperature is 40°C, what
[Aos. (a) 0.78 L]
is the proportional correction I h' - h I/h that must be
37. The grid-iron
applied to the observed column height h of a mercury
pendulum
in Fig.
designed
illustrated
lA.37 is
to
1 Pivot barometer? (b) Weather services usually quote
barometric pressures with a precision of about 1 pan
in 103. Determine whether such a measurement
compensate for the requires further correction for the expansion of the
error. The heavy ~ glass barometer tube, at least on very hot days. If it
weight, containing "":
does, is the volume coefficient p or the linear
most of the mass of the
pendulum,
suspended
midline.
at
It
is
its
is
!
Heavy weight
coefficient a the appropriate quantity to lise in
correcting?
(Hint: Write the fannula for the twice-corrected value h"
of (he mercury column height in terms of h, t and the
connected to the long, Flg.1A.37 appropriate expansion coefficients.]
relatively light, steel
[Ans. (a) I h'-hl/h=0.727%; (b) ugJa>s is the relevant
pendulum rod by means of the suspension shown in parameter since the cross.section:!l area is irrelevant.
the drawing. The vertical rods making up the 1l"=h(l+BHgl)(l-Ugla<,t). Since Pgl,,-,,=2.6xlO-5K-1,

www.puucho.com
Study Anurag Mishra with www.puucho.com

104 THERMOOYNAMIC5
fJ ",rl/3:: Q,9 ><10-5. Even for t =40"{:, (J.~la<J "'3.6><10-4, 3. A 200-g aluminium calorimeter contains 500 g of
which is not enough to affect the reading to 1 part in 103] water at 20ne. A 1OO-gpiece of ice cooled to _20°C is
41. A temperature controller, designed to work in a steam placed in the calorimeter. (a) Find the final
environment, involves a bimetallic Strip constructed of temperature of the system, assuming no heat loss.
brass and steel, connected at their ends by rivets. Each (Assume that [he specific heat of ice is 2.0 kJ/kg K.)
(b) A second 200 g piece of ice at -20'C is added. How

Ft~~~
10.0 em
Brass
much ice remains in the system after it reaches
equilibrium? (c) Would you give a different answer for
(b) if both pieces of ice were added at the same time?
[Ans. (a) 3.01", (b) 200 g, (c) no]
4. For the winter festival, a sculptor creates a 20-kg
statue of a skier made of ice at O"C.To show off the
1...... statue's stability, the sculptor hires someone to
repeatedly slide the statue down a plane B m long and
inclined at 30°. Unfortunately, the sculptor forgot
(-) (b) about the thermal energy produced by friction. If the
coefficient of sliding friction between the ice and the
Flg.1A.41 plane is 0.05, how much ice melts due to the friction
of the metals is 2.0 mm thick. At 20oe, the strip is 10.0 on the first run? (Assume that all the mechanical
em long and straight. Find the radius of curvature of energy lost goes into melting the ice.)
the assembly at lOO°C. See Fig. lA,41. [Ans. 0.20 g]
[Ans. 3.6 m] 5. (a) Show that if the specific heat varies as a function
of temperature, e(T ). the heat needed to raise the
[B] CALORIMETRY temperature of a substance from TI to T2 is given
by
1. A piece of ice is dropped from a height H. (a) Find the
Q= fT
2 meCT) dT
minimum value of H such that the ice melts when it
makes an inelastic collision with the ground. Assume (b) Suppose c(T) =
'.
coO
+ aT ) for some substance,
that all the mechanical energy lost goes into melting where a = 2.Dx 1O-3°C-land T is the celsius
the ice. (b) Is it reasonable to neglect the variation in
temperature. Determine the heat required to raise
the acceleration of gravity in doing this problem? (c)
the temperature from TI to T2. (c) Whal is the
Comment on the reasonableness of neglecting air
resistance. What effect would air resistance have on mean value of, over the range TI to T2 for pan
your answer? (b), expressed in tenns of co' the specific heat at
[Ans. (a) 34 km, (b) yes, (c) the snowflakes reach terminal O"C?
velocity and hit Wilhout melting] [Ans. (b) mco[(T2-TI)+Q(Tl-Tl2)/2/,
2. A 200.g aluminium calorimeter contains 500 g of 1
water at 20ce. An aluminiUm shot of mass 300 g is
'2
(e) cm"an = co[I + a(T2 + Tl)] ]

heated to 100"C and is then placed in the calorimeter. 6. A cooking vessel on a T('C)
(a) Using the value of the specific heat of aluminium slow burner contains 10.0 3.00
given in table find the final temperature of the system, kg of water and an
2.00 •••••••••• _--
assuming that no heat is lost to the surroundings. (b) unknown mass of ice in
The error due to heat transfer between the system and equilibrium at D"Cat time 1.00
its surroundings can be minimized if the initial t = 0. The temperature of 0.00
temperature of the water and calorimeter is chosen to the mixture is measured
be..!. .Mw below room temperature, where .6.tw is the at various times, and the 20.0 40.0 60.0 t(mln)
2 result is planed in Fig. Fig. 18.6
temperature change of the calorimeter and water 18.6. During the first
during the measurement. Then the final temperature 50.0 min, the mixture remains at O"e. From 50.0 min
is ..!:. .6.tw above room temperature. What should be the to 60.0 min, the temperature increases to 2.00"e.
2 Neglecting the heat capacity of the vessel, determine
initial temperature of the water and container if the the initial mass of the ice.
room temperature is 20"e? [Ans. 1.43 kg]
[Ans. (a) 28's"C, (b) 15's"C)

www.puucho.com
Study Anurag Mishra with www.puucho.com

TEMPERATURE, HEAT AND THE EQUATION Of STATE, HEAT TRANSFER 105

7. A copper calorimeter can with a mass of 0.446 kg and 0.012 kg of steam at lOO°Care added to 0.200 kg
contains 0.0950 kg of ice. The system is initially at of water that hns n temperature of 40.0°C.
O.O~C.(a) If 0.0350 kg of steam at lOD.DOC and 1.00 [Aos. (a) 100'C.(b) no ict',00419kgliquid1\'<I\('f. 0,0264 kg
arm pressure is added to the can, what is the final steam, (e) oce, 0.154 kg ice, 00408 kg liquid warer, no
temperature of the calorimeter can and its contents? steam]
(b) At the final temperature, how ffi,lnY kilogmms are 13. A f!mvcalorimeter is an apparatus used to mensure the
there of ice, hmv many of liquid water, and how many specific hent of a liquid. The technique of flow
of steam? calorimetry involves measuring the tempernture
(Ans. (a) 86.rC, (b) no ice, 0.130 kg liquid water, no diffe:"ence between the input and output points of a
.~te;lm] flowing stream of the liquid while energy is added by
8. In a container of negligible mass, 0.0400 kg of steam heal at a known rate. In one particular experiment, a
at 100°C and atmospheric pressure is added to 0.200 liquid with a density of 0.780 g/cm 3 flows through the
kg of water at 50.0°C. (a) If no heat is lost to the calorimetcr at thc rate of 4.00 cm 3/S• At steady state, a
surroundings, what is the final temperature of the temperature difference of 4.80"C is established
system? (b) At the final temperature, how many betvveen the input and Output points when energy is
kilograms are there of steam and how many of liquid supplied by heat at the rate of 30.0 J/s. What is the
water? specific heat of the liquid?
[Ans. (a) 100ce, (b) 0.0214 kg steam, 0.219 kg liquid [Aos. 2 kJ/kg°C]
water] 14. An engineer is
9. At very low temperatures, the specific heat capacity of developing an
crystalline solids varies rapidly with temperature, electric water heater In
according to the relmion c = aT3, where a is a to provide a
constant. Find the heat flow Q required to warm such a continuous supply of
crystal, of mass m, from T,.to 1'/. hot water. One trial
1 • • design is shown in
[Ans. Q == - a(TJ - 1', ) 11l] Voltmeter
4 Fig. 1B.14. Water is
10. One way to make iced coffee is to brew the coffee and flowing at the rate of Flg.1B.14

then pour it over ice. Cookbooks always recommend 0.500 kg/min, the
that the coffee brewed for iced coffee he stronger than inlet thermometer registers 18.0°C, the voltmeter
the tvvotablespoons of coffee per cup of water thai is reads 120 V, and the ammeter reads 15.0 A
best for hot coffee. Suppose the freshly brewed coffee (corresponding to a power input of (120 V)(15.0
is at 88"C and the ice is at the melting point. If all the A) = 1800 W). (a) When a steady state is finaily
ice is melted when the temperature of the mixture is reached, what is the reading of the outlet
5eC, how many tablespoons of coffee should be used thermometer? (b) Why is it unnecessary to take into
per cup of hot water? (Neglect the additional account the heat capacity me of the <lpp<lfatusitself?
strengthening of the recipe required because taste [Ans. 69.6'C]
buds are less sensitive at low temperatures.) [C] GAS LAWS AND KINETIC THEORY OF GASES
[Ans. 4 tablespoonsof coffee]
11. (a) An ice cube with a mass of 0.075 kg is taken from 1, The lenglh of the column of merclll)' in a thermometer
a freezer, where the cube's temperature was -10.0"C, is 4.0 cm \vhcn the thermometer is immersed in ice
and dropped into a glass of water ilt o.one. If no heilt is wateT and 24.0 em ~vhenthe thermometer is immersed
gained or lost from outside, how much water will in boiling water. (a) What should be the kngth at
freeze on to the cube? (b) Is it possible to add so much room temperature, 22.0 C? (b) If the merl"lu)' column
C

ice to the glass that all of the water freezes? Explain. is 25.4 em long when the thermometer is immersed in
[Ans. (a) 4.7 x 10-3 kg, (b) yes] a chemical solution, what is the temperature of the
12, In a container of negligible mass, 0.150 kg of ice at aGe solution?
[Ans. (a) 804 em, (b) 3700 K]
and 0.0950 kg of steam at 100°C are added to 0.200 kg
of water that has a temperature of SO.O°c. (a) If no 2. For each polyatomic gas CO"' 502' H2S compute the
heat is lost to the surroundings and the pressure in the valUe of the molar heat capacity at constant volume,
container is a constant l.00 atm, what is the final C on the assumption that there is no vibrational
L'

temperature of the system? (b) At the final energy. Compare with the measured values in the
temperature, how many kilograms are there of ice, table, and compute the fraction of the total heat
how many of liquid water, and how many of steam? capacity that is due to vibration for each of the three
(c) Repeat parts (a) and (b) if 0.350 kg of ice at O°C gases.

www.puucho.com
Study Anurag Mishra with www.puucho.com

106 THERMODYNAMICS

[Note: CO~ is linear, 502 and H2S are not. Recall should be three times as large as the answer for part
that a linear polyatomic molecule has two rotational (g). Where does this discrepancy arise?
degrees of freedom, and a nonlinear molecule has [Ans. (a) 6.21 x 10-21 J. (b) 2.34><105m2/s2, (c) 484 mis,
three.] (d) 2.57 x 10-23 kg mls. (e) 1.24 x 10-19 N,

VL::
[Ans. co2: 20.79 llmol K, 0.270, S02: 24.94 J/mol K,
(0 1.24 xlO-17 Pa, (g) 8.15
0.205, H2S: 29.94 J/mo] K,0.039]
(h) 2.45 x 1022]
3. A standard cylinder of oxygen used in a hospital has
the following characteristics at room temperature 8. Fig. lC.S shows a plot of volume
(300 K): gauge pressure = 2000 psi (13,800 !<Pa), versus temperature for a process .
volume = 16 L (0,016 m 3), How long will the cylinder that takes an ideal gas from point •••..•••• ~~B
last if the flow rate. measured at atmospheric pressure, A to point B. What happens to the
is constant at 2.4 l.jmin? pressure of the gas?
{Ans. 15 h] [Ans. Pressure increases] T
Flg.1C.8
4. The pressure in a helium gas cylinder is initially 3S
atmospheres. After many balloons have been blown 9. Fig. le.9 shows a plot of pressure
up, the pressure has decreased to 5 atm. What fraction versus temperature for a process
of the original gas remains in the cylinder? Pressures that takes an ideal gas from point A
are gauge pressures. to point B. What happens to the
[Ans. 116] volume of the gas?
5. The pressure gauge on a tank resisters the gauge [Ans. VB < l' A]
T
pressure, which is the difference between the interior Flg.1C.9
and exterior pressures. When the tank is full of oxygen
(02)' it contains 12.0 kg of the gas at a gauge pressure 10. The escape velocity on Jupiter is 60 kmls and the
of 40.0 atm. Determine the mass of oxygen that has surface temperature is typically -150cC. Calculate the
been withdrawn from the tank when the pressure nns speeds for (a) Hz, (b) 0z, and (c) COz at this
reading is 25.0 atm. Assume that the temperature of temperature. (d) If the rms speed of a gas is greater
the tank remains constant. than about 15 to 20% of the escape velocity of a
[Ans. 4.39 kg] planet, virtually all of the molecules of that gas will
6. At 25.0 m below the surface of the sea escape the atmosphere of the planet. Based on this
(density = 1.025 kglm 3), where the temperature is criterion, are HZ,02 and COz likely to be found in
5.00cC, a diver exhales an air bubble having a volume Mars's atmosphere?
of 1.00 cm 3. If the surface temperature of the sea is [Ans. (a) 1.24 krnls, (b) 0.31 kmis, (c) 0.26 kmls, (d) all
20.0cC. what is the volume of the bubble right before it three gases are likely to be present]
breaks the surface? 11. A hot-air balloon achieves its buoyant lift by heating
[Ans. 3.67 cm3] the air inside the balloon which makes it less dense
7. Oxygen (02) has a molar mass of 32.0 glmo!' What is than the air outside. Suppose the volume of a halloon
the average translational kinetic energy of an oxygen is 1800 m 3 and the required lift is 2700 N (rough
molecule at a temperature of 300 K? (b) What is the estimate of the weight of the equipment and
average value of the square of its speed? (c) What is passenger). Calculate the temperature of the air inside
the root-mean-square speed? (d) What is the the balloon which will produce the required lift.
momentum of an oxygen molecule travelling at this Assume that the outside air temperature is OCCand
speed? (e) Suppose an oxygen molecule travelling at that air is an ideal gas under these conditions. What
this speed bounces back and forth between opposite factors limit the maximum altitude attainable by this
sides of a cubical vessel 0.10 m on a side. What is the method for a given load? (Neglect variables like
average force it exerts on one of the walls of the wind.)
container? (Assume that the molecule's velocity is [Ans. 37'C]
perpendicular to the two sides that it strikes.) (0 What 12. An automobile tyre is filled to a gauge pressure of 200
is the average force per unit area? (g) How many kPa when its temperature is 20ce. (Gauge pressure is
oxygen molecules travelling at this speed are the difference between the actual pressure and
necessary to produce an average pressure of 1 atm? atmospheric pressure.) After the car has been driven at
(h) Compute the number of oxygen molecules that are high speeds, the tyre temperature increases to 50cC.
actually contained in a vessel of this size at 300 K and (a) Assuming that the volume of the tyre does not
atmospheric pressure. (i) Your answer for part (h) change, and that air behaves as an ideal gas, find the
gauge pressure of the air in the tyre. (b) Calculate the

www.puucho.com
Study Anurag Mishra with www.puucho.com

gauge pressure if the volume of the tyre expands by


--------
TEMPERATURE, HEAT AND THE EQUATION OF STATE, HEAT TRANSFER

inextensible string, and they can slide


107

10%. through their tubes without friction, A,


[Ans. (a) 220 kPa, (b) 200 kPa] (a) What is the pressure of the gas
13. A vertical cylinder of cross-sectional between the pistons? Siring
area A is fitted with a tight-fitting, (b) Find .o.lI/6.T, the height It through
frictionless piston of mass In (Fig. which the pistons rise, per unit
lC.13). (a) If T! moles of an ideal gas temperature increase.
are in the cylinder at a temperature of / m
[Ans. (a) _(P_,_"'_+_,_og_l.
1; what is the height h at which the '-' ",
piston is in equilibrium under its own
weight? G"
f
h
oR
(bl-----where
(Po"'lA + mg)
M == A2 - All
A,

[Ans. h _ nRT
(mp + PnA)
]
1 Fig.1C.16

17. A thermistor is a solid.state device whose resistance


Fig.1C.13 varies greatly with temperature. Its temperature
dependence is given approximately by R == RoelJ T
14. A cvlinder that has a 40.0 em radius and is 50,0 em
where R is in ohms (0), T is in kelvins, and Ro and B
dec'p is filled with air at 20.DoC and 1.00 arm (Fig.
IC.14a). A 20.0-kg piston is now 100•.•.•ered into the are constants that can be determined by measuring R
cylinder, compressing the air trapped inside (Fig. at calibration points such as the ice point and the
lC.14b). Finally, a 75.0-kg man stands on the piston, steam point.
funher compressing the air, which remains at 20°C (a) If R = 7360 n at the ice point and 153 0 at the
(Fig. lC.I4<:). (a) How far down (.o.h) does the piston steam point, find Ro and B.
move when the man steps on to it? (b) To what (b) What is the resistance of the thermistor at
c t = 98.6°F?

r
50,0 em
r
h,
(c) What is the rate of change of the resistance with
temperature (dR,IdT ) at the ice point and the steam
point?
(d) At which temperature is the thermistor most

L (.J
l (bl «J
sensitive?
[Ans. (a) Ro '" 189 mU, B '" 3.94 x 103 K, (c) -389 nlK at
icepoint,- 4.37n/K at steampoint, (d) moresensitiveat ice
point]
Flg.1C.14
18. (a) For two particles with speeds VI and 1)2' show that
Vnn• ~ Vllv.> regardless of the numerical values of PI
temperature should the gas be heated to raise the
and tl2. Then show that vrms > VIIV, if l'\ 7' 1'2' (b)
piston and the man back to hi?
Suppose that for a collection of N particles you knO\....
'.
[Ans. (a) 4 ><10-3m (b) 27"Cl
Another particle, with speed u, is added to the
15. (a) Show that a collision between two spherical collection of particles. If the new rms and average
molecules, each of radius r, is equivalent to a collision speeds are denoted as v' and ,./ v. ' show that
between a point particle and a sphere of radius 2r, and
, N1'2m, +u2
thus the centre of one molecule cannot penetrate a 1'lTIl'=, N+1
volume equal to that of a sphere of radius 2r 'when two
molecules collide. Nvav.+U
and
(b) Show that the total unavailable volume per mole is N + 1
b == 16nr3N A/3, where N.•.is Avogadro's number, (c) Lse the expressions in part (b) to show that
[Hint: 1nsummingthe total excludedvolume,multiplyby I''r.,,,, > t"",. ' regardless of the numerical value of u. (d)
112 to avoid counting each pair of moleculestwice.] Explain why your results for (a) and (e) together show
note that the unavailable volume is four times the that t'rm, > Pm.. for any collection of particles if the
actual molecular volume. particles do not all have the same speed.
16. The area of the upper piston illustrated in Fig. IC.16 is 19. A simple classroom demonstration apparatus can be
AI' and that of the lower piston is A2. Both pistons are used to make a crude determination of the mechanical
exposed to the outside air, whose pressure is Po' The equivalent of heat J. A tube of length 1 is capped at
pistons, of combined mass /11, are connected by a light, both ends. The tube contains about a handful of lead
shot. The system is allowed to come to room

www.puucho.com
Study Anurag Mishra with www.puucho.com

108 THERMODYNAMICS

temperature to' Then the tube is repeatedly inverted given by N = N oe-x'!.\! where 1M is the mean free path.
so that the shot fall from end to end. A count is kept of This is called the survival equation.
the number of inversions n. Finally, the temperature 26. Using multiple laser beams, physicists haw been able
increase L\t = t - to of the shot is measured. to cool and trap sodium atoms in a small region. In one
(a) Express the value of J in terms of I, n, &t and the experiment, the temperature of the atoms was reduced
necessary constants. to 0.240 mK. (a) Determine the nns speed of the
(b) In a typical experiment, the tube is 1.5 m long and sodium atoms at this temperature. The atol1)s can be
is inverted 25 times. If the temperature increase is trapped for about 1.00 s. The trap has a linear
2SC, find the value of J. Why is this experiment likely dimension of roughly 1.00 cm. (b) Approximately how
to yield too-large values of J? long would it take an atom to wander our of the [rap
[Ans. J = gnl!(6t e\ (b) J = 4.8 )(103 J/kcal; owing to the region if there were no trapping action?
loss of heat. 6Q ""c M will be too small and J therefore too [Ans. (a) 5.10 mis, (b) 20 m s]
large] 27. Because the molecules of a gas occupy space, less than
20. A cylinder contains a mixture of nitrogen gas (N 2) and the entire volume V of a container is available to a
hydrogen gas (H2). At a temperatureTt the nitrogen is particular molecule. Correction for this fact requires
completely dissociated but the hydrogen does not modification of the ideal.gas law. (a) If the molecules
dissociate at all, and the pressure is Pl' If the are not particles, the simplest remaining possibility is
temperature is doubled to T2 = zrl, the pressure is that they are hard spheres of radius a. Consider a pair
tripled due to complete dissociation of hydrogen. If the of particular molecules jand k that are about (0 collide
mass of hydrogen is mH' find the mass of nitrogen mN' with one another, and show that molecule k, by its
[Ans. ms =7mH]
presence, excludes molecule jfrom a spherical volume
21. A vertical closed cylinder of cross-sectional area A is
divided into twn equal parts by a heavy insulating 32 rra3. (b) If a container contains N molecules, show
3
movable piston of mass m P' The top part contains that the toral volume unavailable to the molecules is
nitrogen at a temperature T1 and pressure P2 and the
b 16 rtNa3. (c) Show that rhe equation of state,
=:
bottom part is filled with oxygen at a temperature zrl• 3
The cylinder is turned up-side-down. To keep the corrected for the unavailable volume, is
piston in the middle, the oxygen must be cooled to
T2 = Tt /3, with the temperature of the nitrogen
p(V-b) = NkT, i.e., {v-
1 6
3
rtNa3)=NkT
remaining at TJ• Find the initial pressure of oxygen Pi' This equation is called the Clausius equation of state.
[Ans.12mg/5A] 28. Consider the particles in a gas centrifuge, a device that
22. Three insulated vessels of equal volumes V are separates particles of different mass by whirling them
connected by thin tubes that can transfer gas but do in a circular path of radius r at angular speed ('l (a)
not transfer heat. Initially all vessels are filled \"Iith the Discuss how a gas centrifuge can be used to separate
same type of gas at a temperature To and pressure Po, particles of different mass. (b) Show that the densiry
Then the temperature in the first vessel is doubled and of the particles as a function of r is
the temperature in the second vessel is tripled. The m,.2,,,2 i2kB T
n ( r ) = l10e
temperature in the third vessel remains unchanged.
29. (a) Show that the fraction of particles below an
Find the final pressure P' in the system in terms of the altitude h in the atmosphere is f = l_e(-m,gh kBn
initial pressure Po,
(b) Use this result to show that half the particles are
[Ans. (lB/ll)Po]
below the altitude h' = kBT In(2)/mg.
23. Show that the number of collisions a molecule makes
30. A certain molecule has f degrees of freedom. Show
per second, called the collision frequency, f, is given by
that a gas consisting of such molecules has the
f = V/IM, and thus f = 4,/2 rtr2fj N/V. following properties: (1) its total internal energy is
24. Suppose that a gas contains two types of molecules in fnRT /2:, (2) its molar specific heat at constant volume
concentrations fit and n2• Their radii are r1 and r2 is fR/2:, (3) its molar specific heat at constant pressure
derive the following relation for the mean free path for is (j +2)Rj2; (4) the ratio y =Cp/C" =(f +2) f
type 1 molecules: 31. Show that the mean free path for the molecules of an
1 ideal gas at temperature T and pressure P is
IMl = 2 2
4rrri n1 + rr(rt + (2) n2 1= kBT
25. At some instant, suppose we have No identical ../2rrd2p
molecules. Show that the number N of molecules that where d is the molecular diameter.
travel a distance x or more before the next collision is

www.puucho.com
Study Anurag Mishra with www.puucho.com

TlMPERATURE, HEAT AND THE EQUATION OF STATI, HEAT",TRA=NS


••F",ER
••• _ 109

32. Consider 2.00 mole of an ideal diatomic gas. Find the 39. Air consists mainly of N2 and 02' mixed in the ratio
total heat capacity at constant volume and at constant 4: 1. The atomic mass ofN is 14 u, and that of 0 is 16
pressure (a) if the molecules rotate but do not vibrate, u. Find the mean molecular mass of air.
and (b) if the molecules both rotate and vibrate. [Ans. (111) = 28.8 u]
[Aos. (a) 9.95 caIJK, 13.9callK, (b) 13.9(;al!K, 17.9 callK) 40. At 1 atm pressure and 100'C, a given mass of steam
33. In a crude model (Fig. le.33) of a occupies 1672 times the volume occupied by an equal
rotating diatomic molecule Df mass of water. What is the approximate ratio of the
chlorinc (CI ), the two Cl aLOms are mean distance between the water molecules in steam
2.00 x 10-1 J m <Jpart and rotate to the diameter of the water molecules?
about their centre of mass with [Ans. 11.9]
angular speed (t) = 200 x 1012 rad/s. 41. Suppose that you make a plot on log-log paper ofI"nns
What is the rotational kinetic Flg.1C.33 versus T for an ideal gas. (Alternatively, you could plot
energy of one molecule 0[CI2, which has a molar mass In l'rn" versus tn T on ordinary graph paper.) Show
of 70.0 g/mol? that the plot is a straight line having slope 1/2.
[Aos. 2.33 x 10-21 J] 42. Estimate the molar heat capacity for a gas whose
34. The compressibility K of a substance is defined as the molecules have (a) seven degrees of freedom, (b)
fractional change in volume of that substance for a twelve degrees of freedom.
given change in pressure: [Ans. (a) C'" = 2.91 "104 J/kmol K, (b) e'" = 4.95 x 104
1 dV J/kmol K]
K=---
V dP 43. Oxygen gas is contained in a cylinder whose volume is
(a) Explain why the negative sign in this expression 0.0235 m). The total translational kinetic energy of
ensures that K is always positive. (b) Show that if an the molecules in the gas is 5.03 x 10) J, and the rms
ideal gas is compressed isothermally, its speed of the molecules is 1.97 x 10) mls. (a) Find the
compressibility is given by K} = l/P. (c) Show that if mass of the oxygen. (b) What is the pressure inside the
an ideal gas is compressed adiabatically, its cylinder? (c) What is the total rotalional kinetic energy
compressibility is given by K 2 = l/yP. (d) Determine of the molecules of the gas?
values for 1(1 and "2 for a monatomic ideal gas at a [Ans. (a) m = 259" 10-3 kg, (b) P = 1.43" 105 Pa,
pressure of 2.00 atm. (c) K, = 3.35 ,,103 J]
[Ans. (d) 0.5 arm-l, 0.3 atm-1]
44. A space vehicle returning from the Moon enters the
35. (a) Show that the speed of sound in an ideal gas is
atmosphere at a speed of about 40,000 kmlh.
RT
t' = /''f Molecules (assume nitrogen) striking the nose of the
\ M vehicle with this speed correspond to what
36. Fifteen identical panicles have various speeds: one has temperature? (Because of this high temperature. the
a speed of 2.00 m/s; two have a speed of 3.00 m/s; nose of a space vehicle must be made of special
three have a speed of 5.00 m/s; four have a speed of materials; indeed, part of it does vaporize, and this is
7.00 m/s; three have a speed of 9.00 mls and two seen as a bright blaze upon re-entry.)
have a speed of 12.0 mls. Find (a) the average speed, [Ans. 1.4 ,,105 K]
(b) the rms speed, and (c) the most probable speed of 45. In an ultrahigh vacuum system, the pressure is
these panicles. measured to be 1.00 x 10-10 torr '(where 1 torr = 133
[Ans. (a) 6.80 mls. (b) 7.41 mls. (e) 7 mlsJ
Pal. Assume that the gas molecules have a molecular
37. A gas is at DoC. If we wish to double the rms speed of
diameter of 3.00 x 10-10 m and that the temperature
the gas's molecules, by how much must we raise its
temperature? is 300K. Find (a) the number of molecules in a volume
[Ans. 819°C] of 1.00 m3• (b) the mean free path of the molecules,
38. A container has a mixture of two gases: 711 moles of gas and (c) the collision frequency, assuming an average
I. which has a molar specific heat C 1; and 02 moles of speed of 500 mls.
[Ans. (a) 3.21 "1012 molecules, (b) 778 km,
gas 2. which has a molar specific heatC2• (a) Find the
(e) 6.42 x 10--4 S-I]
molar specific heat of the mixture. (b) What is the
molar specific heat if the mixture has m gases in the 46. A liquid is enclosed in a metal cylinder that is provided
amounts TIl' 02' ") •... , TIm and molar specific heats with a piston of the same metal. The system is
Cj, C2, C), ... , Cm' respectively? originally at a pressure of 1.00 atm (1.013 x 105 Pa)
and at a temperature of 30.DOC. The piston is forced
down until the pressure on the liquid is increased by

www.puucho.com
Study Anurag Mishra with www.puucho.com

110 THERMODYNAMICS

50.0 atm, and then clamped in this position. Find the the balloon maintains the helium pressure at u-
new temperature at which the pressure of the liquid is constant factor of 1.05 times greater than the outside
again 1.00 atm. Assume that the cylinder is sufficiently pressure.
strong so that its volume is not ahered by changes in [Hint: Assume that the pressure change with altitude is
pressure, but only by changes in temperature. P =P(le-0']
COIT')ressibility of liquid, k::= 8.50 x 10-10 Pa-i. [Ans. 3.6 m]
Coefficient of volume expansion of liquid,
P::=4.80x 10--4 K-l [0] HEAT TRANSFER
Coefficient of volume expansion of metal, 1. A double-glazed window is one with
P=3.90x 10-5 K-1) Glass
two panes of glass separated by an
[Ans. 20.2'] air space, Fig. 10.1. (a) Show that
47. A large tank of water has the rate of heat flow by conduction is
a hose connected to it, given by T, AJ, T,
as shown in Fig. leA?
The tank is sealed at the 4.00m
T p ~Q
.6t I,
A(T,-T )
=------t
12 IJ
top and has compressed -+---+-
air between the water
surface and the top.
When the water height h
1 h
: ~
.:~ 10m
---"- -
.. K} . K2 KJ
where K" K 2 and K 3 are the thermal
conductivities for glass, air and glass, Fig. 10.1
Flg.1C.47 respectively. (b) Generalize this
has the value 3.50 ro,
the absolute pressure p of the compressed air is expression for any number of materials placed next tO
4.20 x 105 Pa. Assume that the air above the water one another.
[Ans. l!.Q'" A(T2-T,)]
expands at constant temperature, and take the
atmospheric pressure to be 1.00 x 10 s Pa. (a) What is
M r~-
K,
the speed with which water flows out of the hose when 2. A 2-cm-thick copper sheet is pressed against a sheet oP
h = 3.50 m? (b) As water flows out of the tank, h aluminium. What should be the thickness of the
decreases. Calculate the speed of flow for h = 3.00 m aluminium sheet so that the temperature of the
and for h = 2.00 m. (c) At what value of h does the copper-aluminium interface is (T} + T2 )/2. where T}
flow stop? and T2 are the temperatures at the copper-air anell
[Ans. (a) 26.2 mis, (b) 16.1 mis, 5.44 mis, (c) 1.74 m]
aluminium-air interfaces?
48. In the troposphere, the pan of the atmosphere that [Ans. 1.2 em]
extends from the surface to an altitude of about 11 km, 3. An aluminium rod 0.500 m in length and 'A'ith a.
the temperature is not uniform but decreases with cross-sectional area 2.50 em 2 is inserted into a.
increasing elevation. (a) Show that if the temperature thennally insulated vessel containing liquid helium at
variation is approximated by the linear relation 4.20 K. The rod is initially at 300 K. (a) If one half of.
T =To - ay, the rod is insened into the helium, how many Iitres at-
where To is the temperature at the earth's surface and helium boil off by the time the inserted half cools [0
T is the temperature at height y, the pressure p at 4.20 K? (Assume that the upper half does not yet
height y is given by cool.) (b) If the upper end of the rod is maintained at
In (.J'...)
= Mg In (To - a ) •
y 300 K, what is the approximate boil-off rate of liquid'
helium after the lower half has reached 4.20 K?
Po Ra To
(Aluminium has a thennal conductivity of 31.0
where Po is the pressure at the earth's surface and M is
J/s-cm-K at 4.2 K; ignore its temperature variation.
the molar mass for air. The coefficient a is called the
Aluminium has a specific heat of 0.210 caVg °C and!
lapse rate of temperature.
density of 2.70 g/cmJ• The density of liquid helium is
49. A helium balloon has volume Vo and temperature To at 0.125 g/cm3.)
sea level where the pressure is Po and the air density is [Ans. (a)16.8 L, (b) 0.351 Lis]
Po. The balloon is allowed to float up in the air to 4. A blackened, solid copper sphere of radius 4.0 cm
altitudey where the temperature is T (a) Show that
j• hangs in a vacuum in an enclosure whose walls have a
the volume occupied by the balloon is then temperature of 2DOC.If the sphere is initially at DOC
V = Vo (Ti/To) e+C)' where c = po&/Po find the rate at which its temperature changes,
I
= 1.25x 10-] m- . (b) Show that the buoyant force assuming that heat is transferred by radiation only.
[Ans. 0.22 Kls]
does not depend on altitude y. Assume that the skin of

www.puucho.com
Study Anurag Mishra with www.puucho.com

T1MPERATURE, HEAT AND THE EQUATION Of STATE, HEAT TRANSfER


------ 111
5. A boy has pmented a cooking timer. The timer consists 10. Suppose that both ends of Cross-section
of a 28-cm copper rod having a S.D-em diameter. Just the rod in Fig. ID.12 are J AreaA
as the lower end is placed in boiling water, an ice cube kept at a temperature of Heal
is placed on the top of the rod. When the ice melts T"C, and that the initial T, ~I T,
completely, the cooking time is up. A special ice cube temperature distribution
tray makes cubes of various sizes to correspond to the along the rod is given by
boiling time required. What is the cooking time when a T := (1OO"C) sin rrxj L Fig. 10.12
30-g ice cube at _S.O°C is used? where x is measured from
[Ans. 360 sJ the left end of the rod. Let the rod be of copper, with
6. A solid cylindrical copper rod O.200m long has one end length L:= 0.100 m and cross-section area 1.00 cm2.
maintained at a temperature of 20.00 K. The other end (a) Show the initial temperature distribution in a
is blackened and exposed to thermal radiation from diagram. (b) What is the final temperature
the surrounding walls at SOGK. The sides of the rod distribution after a very long time has elapsed?
are insulated, so no energy is lost or gained except at (c) Sketch curves that you think would represent the
the ends of the rod. When equilibrium is reached, what temperature distribution at intermediate times. (d)
is the temperature of the blackened end? What is the initial temperature gradient at the end of
[Hi.nt:$ince copper is a very good conductor of heat at low the rod? (e) What is the initial heat current from the
temperature, with k == 1670 W/rr: Kat 20 K,the temperature ends of the rod into the bodies making contact with
of the blackened end is only slighdy greater than 20.00 K]
the ends? (0 What is the initial heat current at the
[Ans. 20.42 K]
centre of the rod? Explain. \!Vhatis the heat current at
7. On the average, the temperature of the earth's crust
this point at any later time? (g) What is the value of
increases 1.0"C for every 30 m of depth. The average
the thermal diffusivity klrc for copper, and in what
thermal conductivity of the earth's crust is 0.74
unit is it expressed? (Here k is thermal conductivity,
J/m-s-K. What is the heat loss of the earth per second
p:= 8.9x 103 kg/m3 is the density and c is the specific
due to conduction from the core? How does this heat
loss compare with the average power received from heat capacity.) (h) What is the initial time rate of
the sun? (The solar constant is about 1.35 kW/m2.) change of temperature at the centre of the rod? (i)
[Ans. 4.9 ><1016w;. PI",,/P.~orbed == 5xl0-7 (Note that [he How much time would be required for the centre of
the rod to reach its final temperature if the
effective area for absorption of energy from the sun is 7tR 2
whereas the loss is through area 47tR2.)] temperature cominued to decrease at this rate? (This
time is called the relaxation time of the rod.) Ul From
8. (a) When the air temperature is below O"C,the water
the graphs in part (c), would you expect the
at the surface of a lake freezes to form an icc sheet.
magnitude of the rate of change of temperature at the
\!Vhy doesn't freezing occur throughout the entire
volume of the lake? midpoint to remain constant, increase or decrease as a
function of time? (k) What is the initial rate of change
(b) Show that the thickness of the ice sheet formed on
of temperature at a point in the rod 2.5 em from its left
the surface of a lake is proportional to the square root
end?
of the time if the heat of fusion of the water freezing [Ans. (b) Uniform o'"e, (d) 3.14 x 103 C"/m, (e) 121 W.
on the underside of the ice sheet is conducted through
the sheet. en 0, (g) 1.1 xlO-4 m2/s. (h) -l1"e, 0) 9.1 s,

(c) Assuming that the upper surface of the ice sheet is OJ decrease, (k) -7.7 eO/s]
at -10"C and that the bottom surface it at O"C, 11. A carpenter builds a solid wood door with dimensions
calculme the time it will take to form an ice sheet 25 2.00 m x 0.95 m x 5.0 cm. Its thennal conductivity is
em thick. k := 0.120 Wlm K. The air films on the inner and outer
(d) If the lake in part (c) is uniformly 40 m deep, how surfaces of the door have the same combined thermal
long would it take to freeze all the water in the lake? Is resistance as an additional 1.8 cm thickness of solid
this likely to occur? wood. The inside air temperature is 20.0"C and the
[Ans. (e) 6.0 x lOs s (about 170 h), outside air temperature is -8.0"C. (a) What is the rate
of heat flow through the door? (b) By what faclOr is
(d) 1.5 x 1010 s (about 500 y); no]
the heat flow increased if a window 0.50 m on a side is
9.. The diameter of a rod is given by d := do(I + ax), where inserted in the door? The glass is 0.45 em thick and the
a is a constant and x is the distance from one end. If glass has a thermal conductivity of 0.80 Wlm K. The
the thermal conductivity of the material is k, what is air films on the two sides of the glass have a total
the thermal resistance of the rod if its length is L? thermal resistance that is the same as an additional
4L
[Ans. -----] 12.0 cm of glass.
nkd 2(1 + 9L)
[Ans. (a) 94 W. (b) 1.3]

www.puucho.com
Study Anurag Mishra with www.puucho.com

~12 THERMOOYHAMICS

12. A wood ceiling with thermal resistance R1, is covered 18. (a) A spherical shell has inner and outer radii a and b,
with a la¥cr of insulation with thermal resistance Rz' respectively, and the temperatures at the inner and
Prove that the effective thermal resistance of the outer surfaces are T2 and "fl' The thermal conductivity
combination is R = R1 + Rz. of the material of which the shell is made is k. Derive
an equation for the total heat current through the
13. The passenger section of ajet airliner has the shape of
shell. (b) Derive an equation for the temperature
a cylindrical tube with a length of 35.0 m and an inner
variation within the shell in part (a); that is, calculate
radius of 2.50 m. Its walls are lined with an insulating
T as a function of r, the distance from the centre of the
material 6.00 em in thickness and having a thermal
conductivity of 4.00 x 10-5 caVs em 0c. A heater must shell. (c) A hollow cylinder has length L, inner radius a
and outer radius b, and the temperatures at the inner
maintain the interior temperature at 25.0°C while the
and outer surfaces are T2 and TI. (The cylinder could
outside temperature is at _35.0°C. What power must
represent an insulated hot-water pipe, for example.)
be supplied to the heater if this temperature difference
The thermal conductivity of the material of which the
is to be maintained?
cylinder is made is k. Derive an equation for the total
(AbS. 9.32 kW]
heat current through the walls of the cylinder. (d) for
14. (a) Estimate, using the solar constant, the rate at the cylinder of part (c), derive an equation for the
which the whole earth receives energy from the sun. temperature variation inside the cylinder walls. (e) For
(b) Assume the earth radiates an equal amount back the spherical shell of part (a), and the hollow cylinder
into space (that is, the earth is in equilibrium). Then, of part (c), show that the equation for the total heat
assuming the earth is a perfect emitter (e = 1.0). current in each case reduces to equation for linear heat
estimate its average surface temperature. flow when the shell or cylinder is very thin.
[Ans. (a) 1.7 ><1017W, (b) 278 K (5~C)]
[Ans. (a) H '" 4rRabATj(b -a)
15. A leaf of area 40 cm2 and mass 4.5 x 10-4 kg directly (b) nr) '" [Tlb(r - aJ+ T;aa(b - r)]. (c) H '" 2;tKL Sf
faces the sun on a clear day. The leaf has an emissivity reb-a) In(ba)
of 0.85 and a specific heat of 0.80 kcailkg K. (a) (d) T(r) -'" [TI In (r a) .•.T ~ln (b r)l]
Estimate the rate of rise of the leaf's temperature. (b) In (b a)
Calculate the temperature the leaf would reach if it 19. A body initially at a temperature TI cauls by convection
lost all its heat by radiation (the surroundings are at and radiation in a room where the temperature is To.
20°C). (c) In what other ways can the heat be The body oheys Newton's law of cooling, which can be
dissipated by the leaf? written as
[Ans. (a) 2.3<:"/5. (b) 84~C, (c) convection, conduction,
dQldt : M (T - To).
evaporation.]
16. Liquid helium is stored at its boiling point (4.2 K) in a where A is the area of the body and h is a constant
called the surface coefficient of heat transfer. Show
spherical can that is separated by a vacuum space from
a surrounding shield that is maintained at the that the temperature T at any time t is gh;en by
hAt
temperature of liquid nitrogen (77 K). If the can is 30 T = To + (T, - To) e- ""
em in diameter and is blackened on the outside so that where Tn is the mass of the body and c is its specific
it acts as a black body, how much helium boils away heat.
per hour? 20. Liquid helium is stored in containers fitted with 7 em
[Ans. 97 glh] thick "superinsulation" consisting of a large number of
17. Two copper containers, each holding water. are layers of very thin aluminized Mylar sheets. The rate
connected by a copper rod of cross-sectional area A. of evaporation of liquid in a 200-L container is about
Initially, one container is at T4; the second is 0.7 L per day. Assume that the container is spherical
maintained at T,. (a) Show that the temperature t, of and that the external temperature is 20°C. The specific
the first container changes over time t according to gravity of liquid helium is 0.125 and the latent hear of
T, =Tlle-r,RC vaporization is 21 kJ/kg. Estimate the thermal
where Til is the initial temperature of the first conductivity of superinsulation.
[Ans. 3.1 x 10-{; W/m K]
container, R is the thermal resistance of the rod, and C
is the total heat capacity of the container plus the 21. A pond of water at O~Cis covered with a layer of ice
water. (b) Evaluate R, C and the "time constant" RC. 4.00 em thick. If the air temperature stays constant at
(e) Show that the total amount of hea~ Q conducted in -lO.QoC, how long does it take the ice's thickness to
time t is increase to 8.00 em?
[Hint: To solvethis problem, use the equation dQ '" kA AT
de x
and note that the incremental energy dQ extracr:edfrom the

www.puucho.com
Study Anurag Mishra with www.puucho.com

TEMPERATURE, HEAT AND THE EQUATION OF STATE, HEAT TRANSFER 113

water through the thickness x of ice is the amount required Iron or s:ccl 0.11 450
to freeze a thickness dx of ice 1. e., dQ '" LpA d.\", where p is Lead 0.031 130
the density of the ice, A is the area, and L is the latent heat of Marble 0.21 860
fusion.]
Mercury 0.033 140
[Ans. 10.2 h ]
Silver 0.056 230
Table 1.1: Coefficients of Expansion, at 20°C
Wood 0.4 1700
Coefficient of Coefficient of
Material Linear Expansion, Volume Expan- Water
(l (CO 1'1 sion, I\(C" 1 r Ice (_5°C) 0.50 2100

Solids Liquid (l5°C) 1.00 4186

Aluminium 25" 10-6 75 x 10-6 Steam Ol O~C) 0,48 2010

19 >dO f> 56,,10-6 Human body (average) 0.83 3470


Brass
17 :<10-6 50 x 10-6 Protein 0.4 1700
Copper
Iron or steel 12 x 10 (, 35 x 10 I>
Table 1.3: Thermal Conductivities
Le,d 29 >d0'& 87 x 10-0
Thermal Conductivity, k
Glass (Pyrex) 3)( 10,6 9:<10-6 Substance
kcal/(s-m-CO) J/(s-m-CO)
Glass (Ordinary) 9 10-6
x 27 x 10-6
Silver 10 x 10 2 420
Quartz 0.4,,10-6 1 x to-6
9.2 x 10-2 380
Concrete and brick ",12><10-6 '" 36 x 10-6 Copper

Marble 1.4 ~ 3.5 x 10-6 4_10><10-6 Aluminium 5.0 ~ 10-2 200

Liquids Steel 1.l~10.2 40


Gasoline 950" 10-6
Ice 5 ~ 10-4 2
~Iercury 180~1O~
20 ~ 10-4 0.84
Ethyl alcohol 1100 ~ 10-6 Glass

500~10b Brick 20 x 10-4 0.84


Glycerin
b 20 ~ 10-';
Water 21Ox10- Concre,e 0.84

Gmes Water 1.4 x 10-4 0.56


Air (and most other 3400 x 10--<>
Human tissue 0.5 x 10-4 0.2
gases at atmospheric
pressure) Wood 0.3 ~ 10-4 0.1

Table 1.2: Specific Heats (at 1 atm constant pressure Fiberglass 0.t2x10-4 0.048
and 200e unless otherwise stated) Cork 0.1 x 10-4 0.042

Specific Heat, C Wool 0,1 ~ 10-4 0.040


Substance
kcalf kg_CO Goose down 0.06 ~ 10.4 0.025
J/kg-CO
(- cal/gm.CO)
Polyurethane 0.06 x 10-4 0.024
Aluminium 0.22 900
Air 0.055 x 10-4 0.023
Alcohol (ethyl) 0.58 2400

Copper 0.093 390

Glass 0,20 840

.,... www.puucho.com
Study Anurag Mishra with www.puucho.com

114 ',' "'. -... . ,'. THIRMODYNAMICS

P r' 0 b I e m

ePevel Only'One Alternative is Correct

4. One end of a conducting rod is maintained at


1. lWo sheets of thickness d and 2d and TA T, temperature 50°C and at the other end, ice is melting
same area are touching each other on at oec. The rate of melting of ice is doubled if :
their face. Temperature TA, TB, Tc (a) the temperature is made 200 C and the area of
C

shown are in geometric progression cross-section of the rod is doubled


with common ratio r = 2. Then ratio (b) the temperature is made 100°C and length of rod
of thermal conductivity of thinner c.";,,.-,-._~._,
• 2.
is made four times
and thicker sheet are: (c) area of cross-section of rod is halved and length is
doubled
(aJ 1 (b) 2 (d) the temperature is made 100°C and the area of
(e) 3 (d) 4 cross-section of rod and length both are doubled
2. The temperature drop through 5. A uniform solid brass sphere of radius ao and mass m is
T set spinning with angular speed Wo about a diameter
each layer of a two layer
furnace wall is shown in Fig. k, at temperature To. If its temperature be increased to T
Assume that the external without disturbing the sphere, its new angular will
(0)

temperature T, and T3 are T, be assuming that its new radius is a :


maintained constant and (a) w=:wo
T
(b) 00=:-(1)0
T, > T3. If the thickness of the T, To
layers Xl and x2 are the same,
which of the following (c) w=:(:ofw o (d)oo=(T;oTo)ooo
statements are correct?
Ca) k, > k2 6. The plots of intensity versus wavelength for three
(b) k, < k2 black bodies at temperatures TI•T2 and TJ respectively
(e) k] = k2 but heat flow through material (1) is are as shown. Their temperatures are such that:
I
larger than through (2)
Cd) kt = k2 but heat flow through material (1) is less
than through (2)
3. Two metallic spheres P and Q of the same surface
finish are taken. Weight of P is twice that of Q. Both the
spheres are heated to the same temperature and are
left in a room to cool by radiation. The ratio of the rate
of cooling of P to that of Q is : -j------,
(aJ ],./2 (b) h'l
(a) TI >T2 >TJ (b) Tt >TJ >12
(c) 1:(2)1IJ (d) (2)IIJ:1
(c) T2 >TJ >TI (d) TJ > T2 > TI

www.puucho.com
Study Anurag Mishra with www.puucho.com

-~-
TEMPERATURE, HEAT AND THE EQUATION OF STATE, HEAT TRANSFER

7. Suppose you pour a hoc cup of coffee <lnd the phone volume. They are he<'ltedto the same temperature and
115

rings so you can't drink it. To keep it hot as long as then left to cool. After some time,
possible: (1) sphere will have the highest temperature.
(a) add the cool milk and sugar immediately (2) pyramid wiH have the highest temperature.
(b) don't add the cool milk until you're ready to drink (3) cube will have the lowest temper<lture.
;t (4) sphere will have the lowest temperature.
Cc) stir it once gently without the milk, but don't add
(5) pyramid will have the lowest temperature.
the sugar
(d) don't add the milk and use a black mug if possible Correct option will be :
8. A blClckbody emits radiation at the rate P when its (a) 2 and 3 (b) 3 and 1
temperature is T. At this temperature the wavelength (c) 1 and 5 (d) 2 and 4
at which the radiation has maximum intensity is i.o' If 14. An open pot of water is boiling on a gas stove when
at another temperature T the power radiated is p' and some.one raises the flame. The result will be :
wavelength at maximum intensity is ;'012, then: (a) a substantial increase in the temperature of the
water

0
(a) p'r' = 32PT (b) p'r' = 16PT
ee) P'T' = SPT (d) p'r' = 4PT (b) a tiny decrease in the rate of evaporation
9. Consider two identical iron (c) an increase in the rate of boiling
(d) an appreciable increase in both the rate of balling
spheres, one \'ihich lie on a
and in the temperature of the water
thermally insulating plate, ~
while the other hangs from A B 15. Metal pots are often made shiny on the outside
an insulatory thread. Equal especially on the top and side, and that makes sense
amount of heat is supplied thermally because:
to the two spheres : (a) this conducts heat better
(a) temperature of A will be greater than B (b) this radiates less energy out from the pot
(b) temperature of B \vill be greater than A (c) this lowers the loss to conduction
(c) their temperature will be equal (d) rhis appreciably decreases convection losses
(d) can't be predicted 16. If water at O°C, kept in a container with an open top, is
10. In thermos bottle, the space between the nvo glass placed in a large evacuated chamber:
wall is evacuated (via a tube that is then sealed). This Ca}all the warer will vaporize
is done to : (b) all the water will freeze
(a) decrease conduction (c) part of the water will vaporize clOdthe rest will
(b) decrease the specific heat capacity freeze
(c) decrease radiation (d) ice, water and water vapour will be formed and
reach equilibrium at the triple point
Cd) decrease weight
11. Everyone whole has ever walked barefoot on a beach 17. Two venical glass tubes filled with a liquid are
in summer has noticed how fast the dry sand gets hot connected by a capillary tube as shown in the Fig. The
in the morning. That's because sand has a : tube on the left is put in an ice bath at O°C while the
tube on the right is kept at 30°C in a water bath. The
(a) light colour
difference in the levels of the liquid in the two tubes is
(b) fairly low specific heat 4 em. While the height of the liquid column at DOC is
(c) high thermal conductivity 120 em. The coefficient of volume expansion of liquid
(d) great deal of convection is (Ignore expansion of glass tube) :
12. The power radiated by a black body is P, and it
radiates maximum energy around the wavelength '.0'
If the temperature of the black body is now changed so
- ot>- ••• ----- •••••
that it radiates maximum energy around a wavelength
3i'o/4, the power radiated by it will increase by a 4em
08
000
a a
000
Water
000 a
factor of : 000 000
000 dO
(a) 4/3 (b) 16/9 000 a
000 00
64/27
(e) (d) 256/81 12cm 000 000
13. A cube, a pyramid (with all four faces identical) and a 000 a
000 a a
sphere (all of them hollow) are made from the same 000
0000 a 0 30.C
material and have equal mass and bound equal O'C 000 a

www.puucho.com
Study Anurag Mishra with www.puucho.com

"" , THERMODYNAMICS
(a) 22 x lO-4;<>C (b) 1.1 x 10-4/"C 12 respectively. The coefficient of volume expansion of
(c) 11 x 1O- ;<>C
4
(d) 2.2 x lO--4/"C the liquid is equal to :
18. A horizontal tube, open at both ends, contains a
column of liquid. The length of this liquid column does
not change with temperature. Let Y= coefficient of
t,
volume expansion of the liquid and a = coefficient of
linear expansion of the material of the tube:,
(a)y=(( (b)y=2a
"
" t,
(e)y~3u (d)y~O
19. In two experiments with a continuous flow
calorimeter to determine the specific heat capacity of a
liquid, an input power of 60 W produced a rise of 10 K
(a) I) -12 (b) 11 -12
in the liquid. When the power was doubled, the same
temperature rise was achieved by making the rate of I2l1 -11[2 l)tl -12c2
flow of liquid three times faster. The power lost to the (c) It + l2 (d) It + l2
surrounding in each case was: 12tl +llt2 Itt) +l2C2
(a) 20 W (b) 30 W 25. Two holes of unequal diameters dl
(e) 40 W (d) 120 W and d2(dl > d2) are cut in a metal
20. A steel rod of length 1 m is heated from 25°C to 75°C sheet. If the sheet is heated:
keeping its length constant. The longitudinal strain (a) both dl and d2 will decrease
developed in the rod is : (Given: Coefficient of linear (b) both dl and d2 will increase
expansion of steel = 12x lO--6,rC) . (c) d1 will increase, d2 will
(a) 6x 10-6 (b) -6x 10-5 decrease
(c) -6x 10-4 (d) zero (d) dt will decrease, d2 will increase
26. Two identical rectangular strips one of copper and
21. If Hc,HK and HF are heat required to raise the
other of steel are riveted together to form a bimetallic
temperature of one gram of water by one degree in
strip (aropper > as1eel). On heating this strip will :
Celcius, Kelvin and Fahrenheit temperature scales
respectively then : (a) remains straight
(a) HK >Hc >HF (b) HF >Hc >HK (b) bend with copper on convex side
(c) HK =Hc >HF (d) HK =Hc =HF
(c) bend with steel on convex side
(d) get twisted
22. A pendulum clock (fitted with a small heavy bob that
27. Two elastic rods are joined between fixed supports as
is connected with a metal rod) is 5 seconds fast each
shown in Fig.
day at a temperature of 15°C and 10 seconds slow at a
temperature of 30°C. The temperature at which it is
designed to give correct time, is :
(a) 18°C (b) 20°C
(e) 24"C (d) 25"C
23. The volume of the bulb of a mercury thermometer at
O°C is Vo and cross-section of the capillary is Ao' The
coefficient of linear expansion of glass is ag per cc and
the cubical expansion of mercury Ym per CC. If the
mercury just fills the bulb at ooe, what is the length of
mercury column in capillary at TOC : Condition for no change in the lengths of individual
(a) VOT(Ym + Jag) (b) VOT(Ym -Jag) rods with the increase of temperature. (ai' a2 = linear
expansion coefficient,
AoO+2agT) AoO+2agTJ
A),A2 = area ofrods'YI'Y2 =¥oung modulus)
(c) VoT(Ym + 2ag) (d) VOT(Ym -zag)
(a) ~ =.::LI.!. (b) At '" LIUIYI
AoO + 30gT) AoO + JagT) A2 ((2 Y2 A2 L2U2Y2
24. In a vertical U-tube containing a liquid, the two arms
are maintained at different temperatures, t) and t2• (d) ~ = UV'2
The liquid columns in the two arms have heights 11 and A2 UIYI

www.puucho.com
Study Anurag Mishra with www.puucho.com

TEMPERATURE, HEAT AND THE EDUATION Of STATE, HEAT TRANSfER

28. Find the amount of heat supplied to decrease the


volume of an ice waler mixture by 1 em 3 without any
-~--- 33. Temperature of a body 0 is slightly more than the
117

temperature of the surrounding 00' its rate of cooling


(R) versus temperature of body (0) is plotted, its shape
change in temperature. (Pi"" = O.9Pwmer> Lice = 80
would be :
caVgm)

,,'
k.'"'k.
(a) 360 cal. (b) 500 cal.
ee) 720 cal. Cd) None of these
29. Consider the following statements:
(i) the coefficient of linear expansion has dimension
K-J

k, L,
eE) the coefficient of volume expansion has dimension R R
K-J
Ca) Both (i) and (ii) are correct
(b) en
is correct but (ij) is wrong (e) (d)
ee) (ii) is correct but (i) is wrong
Cd) both CD and (iD are wrong
30. Three identical rods of same material are joined to
34. A black body radiates maximum energy around the
form an equilateral triangle. The temperature of end A
and B is maintained constant as .../3 T and T. The rq.rio wavelength "0'
If the temperature of the black body is
changed so that it radiates maximum energy around
of Te I TB will be (Assuming no loss of heat from
the wavelength 2/'0' the ratio of final to initial power
surfaces)
radiated by it will be :
(.1)1+,/'3 (b)l-J3 (a) 1/2 (b) 2
2 2
(e) 1/16 (d) 16
ee) 1+[2 Cd)1-12 35. A brass wire 2 m long at 27°C is held taut with
2 2 negligible tension between two rigid suppom. If the
31. A thread of liquid in a uniform capillary tube is of wire is cooled to a temperature of _33°C, the tension
length L, as measured by a ruler. The temperature of developed in the wire, its diameter being 2 mm, \vill
the tube and thread ofliquid is raised by t:,.T. can on if, be (coefficient of linear expansion brass
y be the coefficient of volume expansion of the liquid = 2.0x lO-SC-1 and Young's modulus of brass
and u is the coefficient of linear expansion of the
=0.91xlOllpa) :
material of the tube then the increase 6L in the length
(a) 3400 N (b) 3.4 kN
of the thread, again measured by the ruler will be :
(e) 0.34 kN (d) 6800 N
(a) .1L = L(y- u)I'l.T (b) M = L(y -2a)I'l.T
(c) M ",L(y-3(1)I'l.T (d) M = LyD.T
36. rms speed of a monoatomic gas is increased by 2 times.
If the process is done adiabatically then the ratio of
32. Consider the twO insulating sheets with thermal in~tial volume to final volume will be :
resistances R1 and R2 as shown in Fig. The
(a) 4 (b) (4)v,
temperature 0 is :
(c) 23/2 (d) 8
(a) °182R1R2
(81 +02)(R] +R2) " 37. A steel tape measures the length of a copper rod as
90.0 cm when both are at lOoC, the calibration
(b) QjRl + 82R2
(R1+R2)
, temperature, for the tape. What would the tape read
for the length of the rod when both are at 30cC, Given
uitCel 10-s per cC and (Leu = 1.7 x 10-s per dc.
= 1.2.><
(01 + 02)R1R2
(el e, (a) 89.00 em (b) 90.21 cm
Rl +R2
(c) 89.90 cm (d) 90.01 cm
0lR2 + °2Rj
(d) 38. Two ends of a conducting rod of varying cross-section
Rl + Rz
are maintained at 200°C and O°C respectively. In
steady state :

www.puucho.com
Study Anurag Mishra with www.puucho.com

118 •• '0' .""'~.,.._-


THERMODYNAMICS
200°C a ":1 20°C
respectivel). The
temperatuI; of
junction B will be : A B C o

O'C (a) 120°C (b) IOO"C


(e) 140°C (d) BO"C
200.C
43. An iron rocket fragmem initially at -100oe emers the
earth's atmosphere almost horizomally and quickly
(a) temperature difference across AB and CD are equal fuses completely in atmospheric friction. Specific heat
(b) temperature difference across AB is greater than of iron is 0.11 kcaVkg, its melting point is 1535'C and
that of across CD the latent heat of fusion is 30 kcaVkg. The minimum
(e) temperature difference across AB is less than chat velocity with which the fragment must have entered
of across CD the atmosphere is :
Cd) temperature difference may be equal ot different (a) 0,45 kmls (b) 1.32 kmls
depending on the thennal conductivity of the rod (c) 2.32 km/s (d) zero
39. A cylinder of a radius R made of material of thermal 44. Water freezes inside a pipe, it expands by about 9%
conductivity K1 is surrounded by a cylindrical shell of due to expansion. What would be the pressure
inner radius R and outer radius 2R made of a material increase inside the pipe? The bulk modulus of ice is
of thermal conductivity K2" The two ends of the 200x 109 N/m2 .
combined system are maintained at two different
(a) 1.80x 108 N/m2 (b) 3.60x 108 N/m2
temperature. There is no loss of heat across the 7
cylindrical surface and the system is in steady state. (c) 9x 10 N/m2 (d) 7.2x 108 N/m2
The effective thermal conductivity of the system is : 45. Three closed vessels A, Band C at the same
(a) (4K1 +3K2)/4 (b) (Kt +3K2)/4 temperature T and contain gases which obey the
(e) (3K1 +4K2)/4 (d)(3K, +K2)/4 Maxwellian distribution of velocities. Vessel A contains
40. Three rods of identical on:ly02. B only N:2and C a mixture of equal quantities
A
cross-sectional area and of02 and N 2' If the average speed of the 02 molecules
made from the same metal in vessel A is VI' that of the N 2 molecules in vessel B is
form the sides of an isosceles V 2. the average speed of the
is:
°
2 molecules in vessel C
triangle ABC right angled at
B. The points A and Bare (a) (b) VI
maintained at temperatures
T and J2 T respectively in the B
C (d) /2kT
steady state. Assuming that 1M
only heat conduction takes place temperature of point 46. 1\'10 metallic spheres 51 and 52 are made of the same
C will be :
material and have got identical surface finish. The
(a)~ (b)_T_ mass of 51 is thrice that of 52' Both the spheres are
n+1 n+1 heated to the same high temperature and placed in the
(e) T (d) ....I- same room having lower temperature and placed in
.J3( n+ I) n-I the same room having lower temperature but are
41. Sun radiates thermal radiation with maximum thermally insulated from each other. The ratio of the
initial rate of cooling of 51 to that 52 is :
intensity at the wavelength I, = 0.5 ~m while its
(a) -
I (b) _
1
surface temperature is 6000 K If the sum cools down
to a temperature where it emits only 81% of its 3 .J3
present power, the maximum intensity will then be
emitted at wavelength ,.. (c) ~ (d) (~)'/2
(a) i..' = 1.5 ~C (b) ,: = 0.5 ~tC 47. A black body is at a temperature of2880 K. The energy
(c) i.. = 0.53 ~C (d) to' = 0.17 ~C of radiation emitted by this object with wavelength
42. Six identical conducting rods are joined as shown in between 499 nm and 500 nm is U I, between 999 nm
Fig. Points A and D are maintained at temperatures and 100 nm is U:2 and between 1499 nm and 1500 nm

• www.puucho.com
Study Anurag Mishra with www.puucho.com

TEMPERATURE, HEAT AND THE EQUATlDN DF STATE, HEAT TRANSFER 119

is U3'
b:= 2.88>< 106nm_K.
The wein
Then:
constant, constant, (al -' "(I" (h) ~
",
(a) UJ =0 (b) U:l::=O (e) ",
(e) Uj >Uz Cd) U2 >Uj (u" +<1,)
48. A mass m of lead short is placed at the bottom of as 52. The ends of a metal bar of const<lllt cross-sectional
vertical cardboard cylinder that is 1.5 m long and area are maintained at temperatures TI and T2 which
dosed at both ends. The cylinder is suddenly inverted are both higher than the temperature of the
so that the short falls 1.5 m. If this process is repeated surroundings. If the bar is unlagged, which one of the
quickly 100 times successively, assuming no heat is following sketches best represents the variation of
dissipated or lost, the temperature of the short will temperature with distance along the bar "1
increase by : (Specific heat of lead = 0,03 cal/gO C)
(a) D (b) S'C
(c) 7.3'C (d) 1l.3'C
(a) :~~ (b) :~~
49. A block of ice at -lOoe is slowly heated and converted
to steam at IOOce. Which of the following curves Distance Distance
represents the phenomena qualitadvely ?

E E
(a) t2:! (b) ~
"- "- (e) :~~ (d) :~~
Heat supplied Heat supplied
Distance Distance

53. A body X at an original temperawre 100°C and


another body at an original temperature O°C are
placed in an evacuated enclosure, the walls of which
ee) g. (d) are maintained at lO"e. Which one of the following
I' statements is consistent with Prevost'S theory ?
Heat supplied Heat supplied (a) X emits but does not absorb heat
Cb) Y absorbs but does not emit heat
5D. The graph, shown in the T (c) The final temperature of the bodies will be the
diagram, represents the variation 'T mean of their initial temperawres (i.e. SO"C)
of temperature (T) of the bodies, Cd) The walls of the enclosure radiate heat to both X
x and y ha\ing same surface and Y
area, with time (t) due to the ~ 54. Which one of the sketches below correctly compares
emission of radiation. Find the the distribution of radiated energy of a 'grey' body
correct relation between the with that of a 'black' body at the same temperature?
emissivity and absorptivity power (A 'grey' body is one that has an emissivity which has
of the two bodies : is less than unity and which is independent of
(a) Ex >Ey and ax <ay wavelength) .
(b) Ex <E~ and ax >ay
(c) Ex >Ey and ax >ay
(a) (b)
(d) Ex <E~ and ax <ay
51. lWo rods, one of aluminium and the other made of
steel, having initial lengths 11 and 12 are connected
together no form a single rod of length 11+ 12, The
coefficients of linear expansion for aluminium and
steel are Qo and a, respectively. If the length of each
r\~;::y
rod increases by [he same amount when their
temperature are raised by tOC, then find the ratio
_1,_
It + 12
(e) (d)

/ '= ,.

www.puucho.com ,
Study Anurag Mishra with www.puucho.com
r.j';2';/O-----
THERMODYNAMICS
55. When the filament of a light bulb is heated slowly its .plucking the string in the middle, the frequency of the
colour changes from red to orange to yellow and fundamental mode of vibration is:
eventually to white. It never appears or blue. This is (Ys1eel = 2)( lOll N/m2, etaleel = 1.21)( 10-sre)
because:
(a) the maximum temperature attainable gives (ar 44Hz (b) 88Hz
maximum intensity of emission in the ultraviolet (c)- 22Hz (d) 11Hz
pan of the spectrum 61. An'immersion heater takes time t1 to raise the
(b) the temperature range in which green and blue .'l'emperature of a mass M of a liquid from a
light predominate is too narrow . temperature TI co its nonnal boiling point T2• In a
(c) the filament is not black at high temperature and . further time t 2' a mass m of the liquid is vaporized. If
so Wien's displacement law does not apply the specific heat capacity of the liquid is c and heat
Cd) the glass bulb cannm transmit green or blue light losses to the atmosphere and to the containing vessel
56. A vessel contains a mixture of one mole of oXygen and are ignored, the specific latent heat of vaporisation is :
two moles of nitrogen at 300 K. The ratio. of the (a) Mc(T2 - Tdr 2 (b) mc(T2 - T1)c 2
average rotational kinetic energy per a 2 molecules to mY] MY)
that per N 2 molecule is : 'I (c) McTIT2 (d) mel
(a) 1 , 1
mtt Mc(T2-T))t2
(b) 1 ,2
(e) 2, 1
62. Calorie is defined as the amount of heat required to
raise temperature of 1 g of water by 1°C and it is
Cd) depends on the moments of inertia of the two
defined under which of the following conditions?
molecules
'. (a) From l4.SoC to IS.SoC at 760 mm of Hg
57. A vessel contains I mole of02 gas (molar mass 32) at
(b) From 98.SoC to 99.SoC at 760 mm of Hg
a temperature T. The pressure of the gas is P. An
(c) From l3.SoC to 14.SoC at 76 mm of Hg
identical vessel containing one mole of He gas (molar
mass 4) at a temperature zr has a pressure of : (d) From 3.SoC to 4.5°C at 76 mm of Hg
63. 2 kg of ice at -20oe is mixed with 5 kg of water at 20°C
(a) !'. (b) P
in an insulating vessel having a negligible heat
8
(e) 2P (d) 8P capacity. Calculate the final mass of warer remaining
in the contain. It is given that the specific heats of
58. The specific latent heat of vaporisation of water at
water and ice are 1 kcaVkgre and 0.5 kcaVkgrC
20 e is appreciable greater than the value at lOO°e.
0

while the latent heat of fusion of ice is 80 kcallkg :


This is because;
(a) 7 kg (b) 6 kg
(a) the specific latent heat at 20 e includes the energy
0

(e) 4 kg (d) 2 kg
necessary to raise the temperature of one kilogram
of water from 20 e to lOooe
0 64. Liquid oxygen at 50 K is heated to 300 K at constant
(b) more work must be done in expanding the water pressure of 1 atm. The rate of heating is constant.
vapour against atmospheric pressure at 200e than Which of the following graph represent the variation
at lOooe of temperature with time?
(c) the molecules in the liquid are more tightly bound Temp. Temp.
V
to one another at 20 e than at lOooe
0

(d) the root mean square speed of the vapour A


molecules is less at 20 e than at lOOoe
0
(b)
59. A rod of length 40 cm has the coefficient of linear Time

expansion 6
Ct) = 6x lO- re. Another rod of
length; has the coefficient of linear expansion
6
((2 = 4)( 10- ;oe. If the difference in length of the two Temp. Temp.
rods always remain same a all temperatures, then
value of I is:
(a) 26 cm (b) 60 cm (e) (d)
(c) 80 em (d) 32 em nm. Time
60. A steel wire of length 1 m and mass 0.1 kg and having
a unifonn cross-sectional area of 10-6m2 is rigidly
fIxed at both the ends. The temperature of the wire is
65. The velocity of sound in an ideal gas at temperature Tt
lowered by 20°e. If the transverse wave are set up by
and T2K are VI and V2 respectively. If the root mean

, www.puucho.com
Study Anurag Mishra with www.puucho.com

'TEMPERATURE, HEAT AND THE EQUATION OF STATE, HEAT T~RA:::N:.:SF.=:ER:...


•••• _ 121

square velocity of the same gas at same temperature pressure. The mass of mixture is 5 g. Find the ratio of
are '1 and '2 then: mass of hydrogen to that of helium in the mixture:
(a) '2 = '1 (V2!Pl) (b) '2 = c] (,Iv] /0\) (a)I,2 Cb)1,3
ee) '2 '1
= (VI/1.'Z) Cd) '2 '1
= (,.Iv] / V2) (e) 2, 3 Cd) 3 , 2
66. One end of a copper rod of length 1.0 m and area of 71. During an experiment an ideal gas obeys an addition
cross-section 10-3 m 2 is immersed in boiling water and equation of state p2V = constant. The initial
other end in ice. If the coefficient of thermal temperature and pressure of gas are T and V
conductivity of copper is 92 caVmsoC and the latent respectively. VVhenit expands to volume 2V, then its
heat of ice is 8 x 104 callkg, then the amount of ice temperature will be:
which will melt in one minute is : (a) T (b) ../2 T •
(a) 9.2x 1O-3kg (b) 8x l03kg Ce) 2T (d) 2n T
3
ee) 6.9x 1O- kg Cd) SAx 10-3 kg 72. A barometer tube, containing mercury, is lowered in a
vessel containing mercury until only 50 cm of the tube
67. The temperature of a body is increased from 27°C lO is above the level of mercury in the vessel. If the
l2rc. The radiation emitted by it increases by a atmospheric pressure is 75 cm of mercury, what is the
factor of : pressure at the top of the tube?
Ca) C256/81) (b) (15/9) (a) 33.3 kPa (b) 66.7kPa
(e) C4/3) (d) (12/27) ec) 3.33MPa (d) 6.67MPa
68. An ideal gas is trapped inside a test mbe of 73. One mole of a gas expands obeying the relation as
cross-sectional area 20x IO-6m2 as shown in the Fig. shown in the PIV diagram. The maximum temperature
The gas occupies a height L1 at the bottom of the tube in this process is equal La:
and is separated from air at atmospheric pressure by a
mercury column of mass 0.002 kg. If the tube is
quickly turned isothermally, upside down so that 1.... ~o.Po)

mercury column encloses the gas from below. The gas


now occupies height L1 in the tube. The ratio L2 is:
.... i--
,,
-'\(2Vo,Pol2)

L, ,,
[Take atmospheric pressure = lOS Nm -2] v-
(b) 3PoVo
R
(d) None of these

74. A vessel with open mouth contains air at 60°C. When


the vessel is heated upto temperature T, one four the
of [he air goes out. The value of I is :
Ca) 80'C (b) 171'C
Ce) 333'C (d) 444'C
(b) 101 75. 28 g of N 2 gas is contained in a flask at a pressure of
(a)102
99 10 arm and at a temperature of 57°C. It is found that
101
(e) 99
100
(d) 100
99
due to leakage in the flask, the pressure is reduced to
half and the temperature reduced to 27°C, The
quantity of N 2 gas that leaked am is:
!
69. An open and wide glass tube is immersed vertically in (a) 11;20 g (b) 20/11 g
mercury in such a way that length 0.05 OJ extends (e) 5/63 g Cd)63/5 g
above mercury level. The open end of the tube is
closed and the tube is raised further by 0.43 m. The 76. If a mixture of 28 g of Nitrogen, 4 g of Hydrogen and 8
g of Helium is contained in a vessel at temperature
length of air column above mercury level in the tube
400 K and pressure 8.3x lOsPa, the density of the
will be: [Take Palin = 76 cm of mercury]
mixture will be:
(a) 0.215 m (b) 0.2 m
(a) 3kg/m' (b) 0.2kg/m'
(e) 0.1 m Cd) 0.4 m
(c) 2 g/litre (d) 1.5 gllitre
70. A vessel of volume 0.02 m 3 contains a mixrure of
hydrogen and helium at 20°C and 2 atmospheric 77. A cylinder comaining a gas at 27°C is divided into two
parts of equal volume each 100 cc and at equal

www.puucho.com
Study Anurag Mishra with www.puucho.com

122 THERMOOYNAMICS
pressure by a piston of cross sectional area 110.85 83. A block is hanged by means of
em 2. The gas in one pan is raised in temperature to two identical wires having
100°C while the other maintained at original cross-section area A (l mm 2)
temperature. The piston and wall are perfect as shown in the Fig. If
insulators. How far will the piston move during the temperature is lowered by
change in temperature?
.1T(lO"C), find the mass (in
(a) 1 em (b) 2 em
kg) to be added to hanging mass such that junction
(e) 0.5 em (d) 1.5 em remains at initial position. Given that coefficient of
78. An ideal gas of molar mass M is contained in a vertical linear expansion a = 2 x 10-5 foc and Young's modulus
tube of height H. closed at both ends. The tube is y = 5><1011 Nfm2 for the wire.
accelerating vertically upwards with acceleration g.
4 kg
(a) (b) a kg
Then, the ratio of pressure at the bottom and the mid
point of the tube will be: '.' Cel 12 kg Cd) 16 kg
(a) expo [2J\.1gHIRT] (b) expo [-2MgH/RT] 84. We would like to increase the length of a 15 cm long
Ce) expo [MgHIRT] Cd) [MgHIRT] copper rod of cross-section 4 mm 2 by 1 mm. The
79. 1\'10 monoatomic ideal gas at temperature T1 and T2 energy absorbed by the rod if it is heated is EI. The
are mixed. There is no loss of energy. If the masses of energy absorbed by the rod if it is stretched slowly is
molecules of the two gases are ml and nl2 and number E2• Then EdE2 is :
of their molecules are 111 and 112 respectively. The [Various parameters of Copper are: Density = 9 ><103
temperature of the mixture will be: kgf m 3, Thermal coefficient of linear expansion
(a) Tl + T2 (b) IL + T2 =16><lO-6K-l, Young's modulus =135x 109pa,
111+ 112 111 112 Specific heat = 400Jjkg-K]
(c) T
112 l + 111T2 (d) 111Tl + 112T2 Ca) 30.0. Cb) Sao.
111+112 111+112 (c) 480. Cd) sao.
80. Assuming that the maximum temperature reached in 85. Three rods of the same A
an atomic explosion is 5 x 106 K the corresponding cross-section and made of 60.
wavelength for maximum energy radiation will be : the same material form the
(al S.8A (b) 5aA sides of a triangle ABC as
Ce) sao.A Cd) no.saA shown. The points A and B C
are maintained at B
81. A bowl of water, initially at 10°C is brought to boiling temperatures T and J2r respectively in the steady
after being 9 minutes at the stove top. If it remains on state. Assuming that only heat conduction takes plane,
the stove top, how long will it take for all the water to the temperature at point C is :
evaporate?
[Take: specific heat capacity of water = 4.2kJkg-1 K-l
Ca) [2,12 + FJ]T Cb) [ 3,12 ]T
2+FJ 2+FJ
and latent heat of evaporation = 21MJkg-1]
(a) 50 minute Cb) 100 minute Cel [Js]T Cd) [.r:]T
(c) 150 minute Cd) 200 minute
82. A parallel plate capacitor of capacitance 200j.1F is 86. Block A is a 50-g aluminum block originally at 90°C.
charged by a battery of emf lOOV. The battery is now Block B is a 100-g aluminium block originally at 45°C.
disconnected and temperature of the plates is equal to The blocks are placed in two separate 1.0 liter
atmospheric temperature. The plates are now containers of water that were originally at 20°C. When
connected by a thin wire of negligible heat capacity. the systems reach thermal equilibrium, which
Assume 50% of their stored energy increases their aluminum block will have the higher final
temperature till the capacitor gets completely temperature?
discharged and energy equally distributes over the Ca) Block A
plates. If thermal capacity of each plate is 0.5JK-1 and (b) Block B
coefficient of linear expansion is 2x lO-soC-t, (c) The blocks will have the same final temperature
percentage increase in the volume of the plates is : (d) The answer depends on the specific heat of water
Ca) 0.001% (b) 0.002% 87. The root-mean,square speed of molecules in still air at
(c) 0.003% (d) 0.004% room temperature is closest to :
(a) walking speed (2 m!s).
(b) the speed of a fast car (30mjs).

www.puucho.com
Study Anurag Mishra with www.puucho.com

TEMPERATURE, HEAT AND THE EQUATION OF STATE, HEAT TRANSFER

(c) the speed of a supersonic airplane (SOOm!s).


---- L1 attached to a rod of material 2 and of length Lz as
123

Cd) escape speed from Earth (Ll >( 104 m/s). shown. If a1 and u 2 are their respective coefficients of
linear expansion, then equivalent coefficient of linear
88. A resistance n carries a current 1. The rate of heat loss
expansion for the composite rod is :
to the surroundings is I.(T - To) where is a constant, T
(a) (1.ILz + (1.2LI (b) a1L2 + azL2
is the temperature of the resistance, and '1'0 is the
L L
temperature of the atmosphere. If the coefficient of
linear expansion is fl, the strain in the resistance is : (c) (11LI + (1zLz (d) (1.1a2(L~+Lz)
(a) proportional to the length of the resistance wire L (alLI + a2Lz)
u •
(b) equal to -::-l-R 93. Two identical rooms in a house are connected by an
••1 u 2
open doonvay. The temperatures in the two rooms are
(e) equals to --I R maintained at different values. Which room contains
2 I, more air?
Cd) equal to u.i./R (a) the room with higher temperature
89. A cavity waH consists of two layers of brick separated (b) the room with lower tempenlture
by a layer of air. All three layer have the same (c) the room with higher pressure
thickness and the thermal conductivity of brick is (d) same, as both have the same pressure & volume
much greater than that of air. The inside surface of the 94. .1\\10 identical glass spheres filled with air are
wall is warmer than the outside and all conditions are connected by a thin horizontal glass rube. The glClss
steady. Which of the following graph depicts correctly tube contains a pellet of mercury at its mid-point. Air
how the rate of flow of heat P varies with the distance in one sphere is at O°C and the other is at 20CC. If
d from the inside surface to the wall "? temperature of both the vessels are increased by 10°C,
then neglecting the expansions of the bulbs and the
tube:
(a) the mercury pellet gets displaced to\vards the
(a) (b)
sphere at lower temperature
(b) the mercury pellet gets displaced towards the
sphere at higher temperature
(c) the mercury pellet does not get displaced at all.
(d) the temperature rise causes the pellet to expand
without any displacement
(e) (d) 95. At O°C a body emits:
(a) no radiation
(b) electromagnetic radiation of single wavelength
(c) electromagnetic radiation of all wavelengths that
are emitted by it at room temperature
90. In two experiments with a continuous flow (d) electromagnetic radiation of fewer wavelengths
calorimeter to determine the specific heat capacity of a
than are emitted by it at room temperature
liquid, an input pov .•..
er of 60 W produced a rise of 10K
in the liquid. When the pO\ver was doubled, the same 96. A hot black body emits the energy at the rate of
16Jm -2S-1 and its most intense radiation corresponds
temperature rise was achieved by m<lking the rate of
flow of liquid three times faster. The power lost to the to 20,000A.. When the temperature of this body is
surroundings in each case was: further increased and its most intense radiation
(a)20W (b)30W corresponds to 10,000A., then the energy radiated in
(e) 40 W (d) 120 W Jm -2S-1 will be :
91. A thennos bottle containing coffee is shaken (a) 4 (b) 1
\;gorously. Consider the coffee as a system. (e) 64 (d) 256
(a) its temperature decreases 97. The temperature of a body falls from 52°C to 36°C in
(b) its internal energy changes 10 minutes when placed in a surrounding of constant
(c) work has been done by the system temperature 20°C. What will be the temperature of the
(d) heat has been added to the system body after another 10 min. (Use Newton's law of
A composite bar of length cooling):
92.
2 (a) 28°C (b) 20'C
L ;= LI + Lz is made up from a rod
of material 1 and of length L1 (e) 32'C (d) 24'C

www.puucho.com
Study Anurag Mishra with www.puucho.com

124 THERMOOYNAMICS
98. Two identical rectangular rods of metal are welded
end to end in series between temperatures of D"Gand
100°C and IOJ of heat is conducted (in a steady state L 2L
process) through the rods in 2.0 min. How long would
it take for 10J to be conducted through the rods if they
are welded together in parallel across the same (d),
dTldx
temperatures?
(a) 25 sec (b) 30 sec
(e) 40 sec Cd) 45 sec
99. The container A is constantly maintained at lOO"Cand 101. The specific heat of the same substance is expressed in
insulated container B of the figure initially contains ice two units; C1 caVgm"C & C z cailgm"E Then which of
at DOC. Different rods are used to connect them. For a the following relation is true?
rod made of copper, it takes 30 (a) C1 >C2
minutes for the ice to melt and for . (b) C, =C,
a rod of steel of same ~ (c) C) <Cz
cross-section taken in different ~ '(d) C)&CZ cannot be compared
experiment it takes 60 minutes for
102. A thermostated chamber at a height h abo\'e eanh's
ice to melt. When these rods are simultaneously
surface maintained at 30°C has a clock fitted with
connected in parallel, the ice melts in :
uncompensated pendulum. The maker of the clock for
(a) 15 minutes (b) 20 minutes
chamber mistakenly designed it to maintain correct
(c) 45 minutes (d) 90 minutes time at 2D°C. It is found that when the chamber is
100. Two rods of same length and cross-sectional area are brought to eanh's surface the dock in it clicked correct
joined in series. Thermal conductivity of the rods are time. Rt is the radius of Eanh. The linear coefficient of
in ratio of 2 : 1. the material of pendulum is :
a.•.1
O.
2K IK , , (a) !'... (b) _h_
Rt SRt
The ends are maintained at temperatures e 1\ and BBas (c) SRt (d) &..
shown with 0 A > SB and sides are thermally insulated. h h
Which of the following graph represents temperature 103. A long horizontal gas filled tube of length 2L dosed at
both ends is rotated about a venical axis through its
gradient (:) against x in steady state:
centre normal to its length with an angular velocity HI.
dT/dx
If Po is the pressure at centre, T is temperature in
kelvin scale, R the gas constant per unit mass, the
pressure P at the end B will be :

(a) 2L
([ •
()

dT/dx
L 2L A
f B

P. (jiLz [w'" )
(a) P =-"-- (b) P = Poe 2RT
T
(b) (c) p = PowL
-w'
(d) P=Poe Rr
RT
L 2L 104. Density of hydrogen at NTP = 9 x 10-5 gm/em 3,
velocity of sound in Hl at DOC = 12SDmjs,
dT/dx Atmospheric pressure Po = lOs N/ml ,J = 4.2
joules/cal and g = 9.Bm/ sZ. C p and C" are the specific

(e) -
,
,
,
,
,
,
,
,
heats of hydrogen at constant pressure and constant
volume respectively. Then which of the following
, , statement is incorrect?
L 2L

www.puucho.com
Study Anurag Mishra with www.puucho.com

TEMPERATURE, HEAT AND THE EQUATION OF STATE, HEAT TRANSFER 125


4
--""'"==--~---
109. The following graphs shows two isotherms for a fixed
(a) Cp '" 1.41)< 10 c<ll/kgK
mass of an ideal gas. The ratio of r.m.s. speed of the
(b) C" = 2386eal/kgK
molecules at temperatures T\ and T2 is :
(e) C p = 1.41 X 104 JjkgK
Cd) C" = 1 x 104 JjkgK
105. The coefficient of linear expansion of a rod of length P(1O'P': T\tjd'NJl">::~::;::
1m. at 27QC, is varying \vith temperature as u. = 2/T
unit (300K::; T::; 600 K) ,. .
2 - ~. --: _. -~ --:--

where T is the temperature of rod in kelvin. The o


1 •.•.•• : ••• :.•.••
. . V(m3)
increment in the length of rod if its temperature 2 3 4
increases from zrc to 327°C is : (a) 2.J2 (b) .J2
(a)O.3m (b)3m (e) 2 (d) 4
(e) 0.4 m (d) 4 ill
110. What is fraction of molecule below an alritude h in
106.1\vo different isotherms represeming the relationship atmosphere ? Assume uniform gravitational field,
bet\veen pressure P and volume V at a same isothennal conditions, mass of a molecule In,
temperature of the same ideal gas are shown for Boltzmann constant k, temperature T :
masses Inl and In;! of the gas respectively in the. Fig. (a) f = e(mgHtJ (b) f = e~{mgHTl
given, then: (c) f = l_e-(mghkTl (d) f = l_e(mghkTl

111. The liquid of a liquid-thermometer should have the


following properties :
(a) Large value of specific heat & low valUe of
coefficient of the small expansion
(b) Small value of specific heat & large value of
coefficient of the small expansion
(a) 1111 > I7ll (c) Large value of boiling point and low value of
(b) 111] = In:.! freezing point
(e) m] <In:! Cd) Low value of boiling point and large value of
Cd) All of the above are possible freezing point
107. A ring shaped tube contains two P
112. The spectra of radiation emitted by two distant stars
ideal gases with equal masses and are shown below.
molar masses M 1 = 32 and
SlarA
M 1 = 28. The gaseous are
separated by one fixed partition P
and another movable stopper 5
which can move freely without
friction inside the ring. The angle
a is :
(a) 182' (b) 170'
(e) 192' (d) 180'
108. PVversus T graph of equal masses of Hz, He and COz is
shown in Fig. Choose the correct alremative :
o 500 1000 2000 3000
3
Wavelength (nm)
PV
The ratio of the surface temperature of star A to that of
star B, TA:TB, is approximately:
(a) 2:1 (b)4:1
T (e) 1:2 (d) 1:1
(a) 3 corresponds to H:!, 2 ro He and 1 to CO:! 113. The temperature of end A of a rod is maintained at
(b) 1 corresponds to He, 2 to H2 and 3 to CO2 O°C. The temperature of end B is changing slowly such
(c) 1 corresponds to He, 3 to H2 and 2 to CO2 that the rod may be considered in steady state at all
Cd) 1 corresponds to CO2, 2 to Hz and 3 to He time "lOd is given by Tn = ut; where u is positive

www.puucho.com
Study Anurag Mishra with www.puucho.com

126 ---;TH:;;E~RM::':ODYNAM<-llm~ICS'l!loo

constant and t is time. Temperature of point C, at a (c) T\ > T2


distance x from end A, at any time is : (d) Nothing can be said
L 117.Density of hydrogen at NTP=9xl0-s gm/cm3,
•• x •

velocity of sound in H2 at ODC = 12S0m/s,
A. I 08
TA:;:: O.C Ta = at Atmospheric pressure Po = 105 Nj m 2 , J = 4.2
2 '. joules/cal and g = 9.8mjs2. Cp and C~. are the
(a)
ax' (b) a xt '.specific heats of hydrogen at constant pressure and
L 2L
2
.constant volume respectively. Then which of the
(c) ax t (d) a(L - x), following statement is incorrect ?
2L L (a) Cp = 1.41 X 104 cal/kgK
114. A mass of material exists in its solid form at its ruehing (b) Cv 0= 2386caljkgK
temperature 10°C. The following processes then occur (c) C p = 1.41 X 104 J/kgK
to the material:
(d) Cv 0= 1 X 104 J/kgK
Process 1 : An amount of thermal energy Q is added to
the material and 3/4 of the material melts 118.A thermostated chamber at a height h above earth's
Process 2 : An identical additional amount of thermal . surface maintained at 30"C has a clock fitted with
energy Q is added to the material and the material is uncompensated pendulum. The maker of the clock for
now a liquid at 50°C. chamber mistakenly designed it to maintain correct
What is the ratio of the latent heat of fusion to the time at 20"C. It is found that when the chamber is
specific heat of the liquid for this material ? brought to earth's surface the clock in it clicked correct
time. R~is the Radius of Eanh. The linear coefficient of
(a) BOO (b) 60 0

the material of pendulum is:


(e) 4oo (d) 20 0

115. lWo solid objects of the same mass are supplied with
(a) .!!- (b) ~
R. SRe
heat at the same rate IJ.QIIlt . The temperature of the
first object with latent heat L1 and specific heat (c) SR~ (d) Re
capacity c 1 changes according to graph 1 on the h h
diagram. The temperature of the second object with 119. The coefficient of linear expansion of a rod of length
latent heat L2 and specific heat capacity c2 changes 1m, at 27 C. is varying with temperature as a = 2/T
D

according to graph 2 on the diagram. unit (300K:5 T:5 600K)


where T is the temperature of rod in kelvin. The
increment in the length of rod if its temperature
2 increases from 27°C to 327°C is :
(a) 0.3m (b) 3m
(c) OAm (d) 4m
120. Three rods of identical cross-sectional area and made
from the same material form the sides of an equilateral
triangle. The point A and B are maintained at T and Zf
TIme respectively. In steady state temperature of point C is
Te• (Assuming only heat conduction takes place), The
Based on what is shown on the graph. the latent heats
value of Te is
L1 and L2 and the specific heat capacities c. and c2 in
solid state obey which of the following relationships: c
(a)L1 <L2;Cl <c2 (b)L.<L2;Cl>C2
(c) L2 < L1 ; c1 > c2 (d) L2 < L1; c2 > c1
116.1Wo spheres, one solid and another holloware made
up of same matetial and are of same radius. Both the
spheres are heated to same temperature To. Now, both
spheres are left to cool in an atmosphere having A 8
temperature 'fA(TA «To). Mter time t, the
temperature of sphere becomes T1 and that of hollow (a) T/2 (b) T
sphere becomes T2. Then: 2
.:!T
(e) - T (d)
(a) TI < T2 3 2
(b) TI =T2

www.puucho.com
Study Anurag Mishra with www.puucho.com

TEMPERATURE, HEAT AND THE EQUATION OF STATE, HEAT TRANSFER 121


121. In a room where the temperature is 30°C a body cools (e) nH (d) 2nH
from 61°C to 59°C in 4 minutes. The time taken by the 125. A faulty barometer contains some air which occupies
body to cool from 5 PC to 49°C will be : 10 cc. If it indicates 740 mm when a correct barometer
(a) 4 min (b) 6 min indicates 750 mm the volume the air would occupy
(el 5 min Cd) 8 min under standard pressure is:
122. The power emitted per unit area of a black body is R (a) Icc (b) 0.13cc
watt/m 2. At what wavelength will the power radiated (c) 1.3cc (d) 7.5cc
by the black body be maximum. If the Stefan's 126. A sphere of radius o.049m and mass O.S96kg floats in
constant is 0" and Wien's constant is b, then: a liquid bath at DOC. The density of liquid is
1.25x 103 kg/m3• If the temperature of the bath is
(al ),"(~Y' (b)A"(%)" raised to 40 OC, the sphere just begins to sink. The
volume coefficient of liquid is :
(e) ),"(b:Y' (dJA"b(*Y' (a) 8.3x 1O-4°C-l
(c) 4.1Sx 10--4°c -1
(b) 1.3x lO--4°C-1
(d) 2.8x 10-4 cC-1
123. A hollow sphere of mass M (in kg) and radius R (in m)
is rotating with an angular frequency w (in rad/sec). It 127. A sphere, a cube and a thin circular plate all made of
suddenly stops rotating and 75% of the K.E. is same material and having the same mass are initially
convened into heat energy. If s joule/kg-kelvin is the heated to a temperature of 200 C. Which of these
C

specific heat of the material of the sphere, the rise of objects will cool fastest and which will cool slowest
temperature of the sphere is: when left in air room temperature?
(a) Sphere, cube
R" 3R"
(a) --"- (b) --"- (b) Cube, thin circular plate
4~ 20s
(c) Thin circular plate, sphere
R' , 2R' ,
(e) ~ Cd) --"'- (d) All will coo] at the same rate
4Js 3s
128. The pressure and density of a diatomic gas (y = 7/5)
124. The volume of an air bubble is doubled as it rises from change adiabatically from (P, d) to (P', d'). If did' = 32,
the bottom of a lake to its surface. If the atmospheric the value of PIP' should be :
pressure is Hm of mercury and the density of mercury
(a) 1/128 (b) 32
is n times that of lake water the depth of lake is:
(c) 128 (d) none of these
(a) Hln (b) nHI2

www.puucho.com
Study Anurag Mishra with www.puucho.com

1 THERMODYNAMICS

AN9WERS

1. (a) 2. (a) 3. (c) 4. Cd) 5. Cc) 6. (b) 7. Cd) 8. Ca)


9. Cb) 10. Ca) 11. (b) 12. Cd) 13. Cc) 14. (c) 15. Cb) 16. Cc)
17. (c) 18. Cb) 19. (b) 20;" (c) 21. Cc) 22. Cb) 23. (b) 24. (a)
25. (b) 26. Cb) 27. (d) 28" (c) 29: (a) 30. (a) 31. (b) 32. Cd)
33. (b) 34. (c) 35. (c) 36'" (d) 37." Cd) 38. (c) 39. (b) 40. (a)
41. (c) 42. (c) 43. (b) 44. Ca) 45. Cb) 46. (d) 47. (d) 48. (d)
49. (a) 50. (c) 51. (c) 52. (c) 53. (d) 54. (b) 55. (b) 56. (a)
57. (c) 58. (c) 59. (b) '. 60/ Cd) 61. (a) 62. Ca) 63. (b) 64. (c)
65. (a) 66. (c) 67. (a) 68:' Cb) 69. (c) 70. Ca) 71. (b) 72- (a)
73. (c) 74. (b) 75. (d) 76:- Cc) 77. Ca) 78. (c) 79. (d) 80. Ca)
81. (a) 82. (c) 83. (c) 84, (b) 85," Ca) 86. (a) 87. (c) 88. (a)
89. Cd) 90. (b) 91. Cb) 92 (0) 93 .. (b) 94. (b) 95. (c) 96. (d)
97. (,) 98. (b) 99. Cb) 100. (d) 101. (a) 102. Cb) 103. (b) 104. (,)
105. (b) 106. Cc) 107. Cc) 108. Ca) 109. (c) 110. Cc) 111. (b) 112. (a)
.
113. (a) 114. (a) 115. (a) 116. (c) 117; (a) 118. (b) 119. Cb) 120. (d)
121. (bl 122. (d) 123. Ca) 124. (c) 125~ rh) 126. ra) 127. Cc) 128. (c1

www.puucho.com
Study Anurag Mishra with www.puucho.com

TEMPERATURE, HEAT AND THE EQUATION Of lTATE, HEAT TRA'liN!lI:t!fE:!!R.:.'


•••• _ 129

S '0 I uti 0 n

== Level-l: Only One Alternative is Correct

5. [c] From conservation of angular momentum, 34. [eJ A.mT = constant


100)0 = btl where 10 and [ arc the moments of
inertia at temperature To and T respectively. Tf =.!.
2 2 2 2 Ti 2
Now 10 and Hence

r
=-mao I =-rna.
5 5 (Power) P =: ecrAT4
w = (1]0 }.llO = ( ':: {(Io where a is the radius of
Pf =:(Tj)4 =~

the sphere at T. Pi Ti 16

31. [b] Since a ruler is used, the scale used does not 35. [e] Thermal stress thac is produced in an elastic wire
expand with the tube. If the radius of the is =: Y . a . e unit area where Y is Young's modulus
capillary be r, the increase dr due to thennal , a coefficient of linear expansion and e change
expansion is given by dr = mdT for a of temperature. Thus, tension dcueloped in the
temperature rise of dT. Since area of wire, is developed in the wire, is
cross-section is A = rrr2, we see that T =: Thermal stress x area of cross-section
ciA / A = 2dr / r or ciA = A(2a)dT. Thus if the
temperature is increased from T to T + dT, the = Ya. (tJ.T)[1tr2]N
cross-sectional area changes from A to = 0.91 x 1011 x 2x 10-5 x [27 -(-33)]
A(1 + 2o..dT). The volume expansion of the liquid
[1tX 1 x 1O-6]N
gives v' = V + dV = V(l + '(dT), where y is the
coefficient of the volume expansion of the liquid. =0.91x2x6x3.14xlON
This causes change in length of thread of
= 34.4 x ION = 0.34kN
L' =: L + tiL. The mass of liquid is constant, hence
L'A' =: V' =: V + dV =: V (l + ydT) =: LA(l + ydT) 41. [c] By Wien's displacement law, A.mT =: constant
but A' =: A(l + 2ydT) When sun cools to the temperature T where it

Hence, L' =: 1 + ydT


L'( ~~-
1+ 2ydT
1 emits only 81% of its present radiating power,
then

aT' =(~Jo(6000)'
=: L[1 + (y - 2a)dT - 2ay(dT)2] 100
the last term is negligible. ((19 (6000)
T = ...•
Hence, L' =: L[I + (y - 2a)dT] = 0.95 x 6000 = 5700K
or I1L =:L' -L =:L(y-2a)dT Hence ift.m is the required wavelength A.m(5700)
=:0.5 x 6000
6000 10
;"m =: -- X 0.5 =: - = 0.53 ~Lm
5700 19

-:, www.puucho.com
Study Anurag Mishra with www.puucho.com

---';:_====::;Ics
43. [b) The kinetic energy of the fragment is all changed 83. [e] Change in tension = !'J.T
to heat. Expressing mass m in kg everywhere, we
have 2!'J.Tsin37°=!'J.mg
" !'J.T = ilmg
!2 mv2 "" [m(30) + m(0.11)(1535°C + 100°C)]4184 2sin37°

v2 = (8368)[30+ 180] = 1.76x 106 ill = !'J.TL


AY
Hence v = (Jl.76) x 103 = 1.32 km!s
But 61 = LailO
48. Id] Specific heat of lead = 0.03 kcalikgOC.
6mg L = LailS
Gravitational potential energy is convened into 2sin37° AY
thermal energy which is absorbed by the lead
shot. rfTo is the rise in temperature of the shot, =:> .6m=ailOxAxYx2sin37°jg
then (100) [mgh] = (m x T)J . 5; = 2x 10-5 x lOx 10-6 x sx 1011 x 2x 3 -12
T=100xgxh 5x 10
J -s
84. [b) Temperature is increased by 68, then
100x 9.8 x 1.5
=------.e
3
'1=1""8
0.03 x 4.18x 10
= 11.3°C
..
" ,- 68 = ill
10
- _ Y-M.A
60. Idl TenSion = Y . stram .A = --- E, = (pAI)St>e= pAIS-"I
I, 10
T = (2 x 1011). (2.41 x 10-4). (l0~6) = 48.2
when stretched, Stress = Y TM
I,

V fundamental •• 1.: {T =1.)48.2 = 10.98 Hz E2 =~(y~l)( ~I)x AI= Y(6~2A


2lV;; 2 0.1
= 11Hz So, ~ = pA1S.61 x 21 _ 2pS1 = 500
E2 a1 x Y(6l)2 A a(M) Y
80. [al By Wien's displace!TIent law,
AmT = 0.289
86. lal

'm = 0.289 = 5.8,1.


5 x 106
SOx Sal x (90-T) = mw x S••.(T -20)
100 x Sal x (45 - r) = mwSw(T' - 20)
(1)
(2)
81. la] P = Power delivered by the stove top (9Q-T) =T-2Q=~>1
2(45-T') T' -20. 7Q-T'
PCI =mc(lOO-lO) (m= mass of water, t} =9
min] T >T'
Also pt 2 = Am [A= late~t heat of evaporation] 87, Ie] v=)3RT = 3x (25:'3) x 300 '='507.95m/s
t2 , M 29x 10-3
-=-- =0-
t1 c(90)" .. ;,' - .1
88. [bl [2R = ).{T -TO); Strain = - = aCT -To)
2.1 x 106) 1 -- I
t2=9x x-=50sec
( --~
4.2x 103 90
89. [dl Under steady state, dQ =constant at all position
dt
82. leI 2x S x ilT ""~(~CV2); where S = o.SJK-1 P "" constant

(for each plate) 90. Ibl In this case some power is lost to surroundings.
!'J.T =O.5OC Let power lost to surrounding is Q

Now, -"V = y(!'J.T)= 3a(!'J.T)


V
In fIrst case; 60 -Q = ( ~~)s (10)

Percentage change = 3a(!'J.T) x 100% = 0.003%

www.puucho.com
Study Anurag Mishra with www.puucho.com

TEMPERATURE, HEAT AND THE EQUATlDN DF STATE, HEAT TRANSFER

In second case, 120-Q = 3[(


--_ ..._---------
~7)S(lO)] amount
1 . 2min
of wat becomes - tlme:::- -- = 30 sec
131

4 4
120-Q = 3 99. .
[b] Q :ttw 6T
h erel. = h eat flQwrate=-=- 100
60-Q R R
Q = 30W R1 =
(IOOJ
Q )( 30
91. [b] No heat is transfered & work is being done on
the system.

92. [e] a~q(LI+ Lz)6T = L1ujh.T + LZU.2liT


Also, Q =( ~2°)x 30 =
L1ul +L20.2
ueq -
L1 +L2
Now, Q = (-.!.-
R1 Rz
+ ""!"')t reqd'

93. [b] PV = nRT


tn:'qd = Q = 20min
nIT) = canst.;

Room with higher temperature


T1 > T2; n1 < n2

as lesser no. of
C~O)' ;0 +(I~OJ' ;0
moles. 102. [b] Variation of acceleration due to gravity with
96. [d] By Stefan's law altitude

E xT 4

By Wien's displace
gh =g(l-~:)
I ,;.g ~ 2hg
Toc-
Am R,
£1 T14 I..~
Variation of 1 with temperature = iii
£2 = T24 = }.4""
. .
[jnear expanslVlty =--
~I
I(M)
97. [a] Applying Newton's law of cooling
Th = 21t
I"l::
"i"="6i = 21t
1L(,;.g )"
Vg' 1-g-
( ,;.g)
= T 1 + 2g
52-36=":!!"[S2+36 20] V
10 min me 2
T, =2n~I:61 =T(I+ ~)
a 36-8 = -b [36+ 8 -20]
lOmin me 2 The clock shows correct time ifTh =T6
solving, we get e = 28"C 61 ,;.g
-=-
98. [b] 21 2g
R
. ..61,;.g 2h h
Lmear expanslvlty = - =-- =-- =--
la/ 109 10R~ 5R~
Net resistance = 2R
103. [b] Let a be the area of cross.section. Consider an
oj"==R=R==~Eo.c elementary between two parallel planes at
distances x and x + dx from the centre o.
Net resistance = 2R Volume = a. dx
Net resistance = B. mass=a.dxp
2
Centripetal force = mw2 . x
.
as resistance h as b ecome -I tImes
.
4 This also represents the difference in pressure
dP.
heat current increases 4 times
adP = mwz.x
time taken to transfer same

www.puucho.com
Study Anurag Mishra with www.puucho.com

--"'T~Hf!lRMOll:::DYNAM=l::::ICS'<l

dP"'Ul2.xdx,PjRT PV
108. [a] -:tanEl:nR
T
dP ~-x.dx
- ",'
P RT slope a no. of moles
::mgh
llO. [e] Ph = Poe kT Po = 171",&
A
---t.dJ=::J.
~------cO=--- f 171 : mass tHl:z: of column of areaA
1m~ : mass till height IJ of column of area A
2
Integrating [logt P]~o '" ;; [X2 J: Ph:-- --
A
(m~-mh)&

m,'
p
log ( -1"'--
1 ",'x'
for x",L,
(m.., - 17Ih)g _ mrg
--------e
- kT
t Po) 2RT A A
mgh

P "'Poe
[~~] 1_ mh : e -kT-
m.
mgh
104. [a] Density of H2 "'P =9x 10-5 gm/cm3 mh :~=l-e--kT
m~ n..,
""9x 10-2 kg/m3
lli. [b] Small value of specific heat and large value of
Velocity of sound v ""~ coefficient of the small expansion.
ll2. [a] A1.{ATA : A!.4f~TB (Weins law)
2 2 2
y '" v p = 1250 x 9 x 10- '" 1.406 TA _ )'MB ~ 1000 -2'1
P lOs TB - )'1.{A - 500 - .
PV",nRT",,17IRT
M ll3. [a] -.---.-
dT
dx
TB -TA
L
at
L

Cp-C1,
~!!-
M
=.!'.-
pT
~ 10'
9xlO-2x273
~ 4070J/kgK T - T (dT\ ..._ (at)x
C-A+dxr- L
Cp-Cv
C., : 10024:
",OA06C l• o::4070JjkgK
4
1 x 10 JjkgK = 2386cal/kgK
ll4. [a] The first energy results in Q :(~m )L. The

contribution of energy melts the remaining


Cp : 1.4094 x 104 '= l.4lx 104 J/kgK
material and then increases the temperature.
= 3372 cal/kgK
That is, Q:(~m)L+me(400). Equating the
105. [b] dL ~ aLdT ~ 2LdT

L
T
T
energy, ~ 17IL =( ~m)L + me (40°)
=> fdL.2fdT
LoL ToT !L:c(40 0
) =:> L/c=80°
2
=:> In(LLo)=ln(T/To)2:ln4 or L:3Lo=3m

61.:3Lo :3m
115. (a]

~
L,
2

106. [e] PV:mRT 1(


E
" L,
M
Vam '" '"
Time
L2 > L) ['.'Change in state takes more time for 2]

www.puucho.com
Study Anurag Mishra with www.puucho.com

TEMPERATURE, HEAT AND THE EQUATION OF STATE, HEAT TRANSFER 133

C] < '2 [-,'Fina! temperature of 1 is more for solid Cp = 14094>< 104


state]
""1.41 x 104 Jjkg K
Alternative :-
= 3372 callkg K
According to the calorimetric relation
118. [b] Variation of acceleration due to gravity with
1l.8=mcAT
M M
=:> ll.T = ((MJJ...l,..t
mer
t..t
altitude

Slope of T t's t graph will be 'Y.. !, gh=+-:')


!>g = 2hg
R,
2 Variation of 1 with temperature"" D1
' .. ill
Lmear expanslVlty ""--
1(69),

Th =2nJ g-!>g1 =2n


V'1I(1_6
i g )12
g
=T(l+!>g)
2g
Time

Te "" 2Jt {I + 6.1 ""or! 1 + 6.1)


For 1st solid slope is more initially. So it's c will \ g 'l 21
be lesser i. e.,
The clock shows correct time if Th "" To
61 = !>g
For the curve parallel to time axis 21 2g
.1.8 II L2
-=rn-=m-
M t1 [2
' ..
L mear expanslV1ty ""-
61
=-!>g
101 109
=--
lORe
=- h
SRe
2h

r,
L1 =- <1 '2L
119. [b] dL=aLdT=-dT
Lt <L2
" T
I. dL r dT
116. [c] As the mass of solid sphere will be more than
dT 1
1-=21-
~ L To T
hollow sphere. - x - hence T} > T'2'
dt m
In(~) = 1n(T/To)2 "" In4
117. [a] Density of
or L",,3LO""3m
H2 = P = 9 x 10-5 gm/em 3 = 9 x 10-2 kg/ m3
M=3Lo""3m
JYP
Velocity of sound I' =
tr- 120. [a] Let heat resistance of each section be Ro the
hence heat current from A to C will be equal to
from C to B.
y = v2p = 12502 X 9)( 10~2 = 1.406 ZI'-T, =Tr-T
P lOS
R R
PV = nRT == !!!.. RT 'JT "" ZI',
M :IT
PR lOS
T~-
, 2
Cp-Cv =_=_= =4070 JjkgK
M pT 9x 10-2 x 273
Cp -c v = 0.406C v = 4070J/kg K
~ =
4
k(30J} t 3 6'
121. [b]
Cv =10024=lx 104 Jjkg K = 2386eal/kg K
~ ""k(20) ~"4""2~t = mm

www.puucho.com
Study Anurag Mishra with www.puucho.com

122. [d] R = aT4 IxlO=76xx


b beal'.
A=-=--- x = D.13c.c
T (R)L'4
126 [sl When it just sinks we have the mass of floating
2wx body equal to the mass of lithium displaced .
123. [sl .!.KMR 75 =ms.1.8
2 100
~ rr(O,049)3dl = 0.596
kr' , 3
AS =~__w_
8 , d1 = 0.596)( 3 = 1.21 ')< 103 kg/ms
41tx (0.049)3

124. [eJ Assuming temperature to be constant do = dJ [l + yt] where y is the cubical coefficient

. ....- .
of liquid.
(P+ h~g)v=px 2V 1.25 x 103 = 1.21 x 103(1 + 4Oy)

Where P is the atmospheric pressure, h the depth 1 + 40y = 1.25


of lake and p the density of mercury 1.21

p = hpg or Y =&3x 1O-4K-1


n 127. [el The temperature of these objects being identical
But P=Hpg the cooling rate will depend upon surface area
Hrg = hpg The circular plate has the largest area and hence
n will cool faster than cube and sphere. Sphere
having less surface area than a cube will cool at
h =nH
the lower rate.
125. [b] The air has depressed mercury column
(750-740) by lOmm. Hence volume of the 128. [el Jl d1)'/'
£ = P' (1)'
d " for adiabatic change
trapped air is IOc. c under a pressure of lOmm.
Required volume under a pressure of 76cm of
Hg, by Boyle's law !.- = (.'!.)7!S = (32//5 = (2SfJ5 = (27) = 128
P' d'

www.puucho.com
Study Anurag Mishra with www.puucho.com

TEMPERATURE, HEAT AND THE EQUATION OF STATE, HEAT TRANSFER


~---- 135

Problem

More than One Alternative is/are Correct


Pevel
1. A vessel is panl}' filled with liquid. When the vessel is 4. A container holds 1026 molecules/m 3, each of mass
cooled to a lower temperature, the space in the vessel, 3 x 10-27 kg. Assume that 1/6 of the molecules move
unoccupied by the liquid remains constant. Then the with velocity 2000 m/s directly tmvards one wall of
volume of the liquid (Vi)' volume of the vessel (V,. ), the container while the remaining 5/6 of the
the coefficients of cubical expansion of the material of molecules move either away from the wall or in
the vessel (Yv) and of the liquid (h) are related as : perpendicular direction, and all collisions of the
(a) "lL >1,- (b)YL<Y, molecules with the wall are elastic:
(e) '("lo'L = VJVL Cd) "(.,hl = VI/V" . (a) number of molecules hiuing 1 III 2 of the wall every
2. Heat is supplied to il certain homogeneous sample of ' second is 3.33x 1028
matter at a uniform rate. Its temperature is plotted (b) number of molecules hitting 1 m 2 of the v....
all every
against time as shown in the Fig. Which of the second is 2x 1029
following conclusions can be drawn: (c) pressure exerted on the wall by molecules is
24>< 105P<l
(d) pressure exerted on the wall by molecules is

1
E
~
5.
4x lOsPa
The temperature drop through a two layer furnace
wall is 900CC. Each layer is of equal area of cross
nme__ section. Which of the following actions will result in
lowering the temperature 0 of the interface?
(a) Its specific heat capacity is greater in the solid
Inner Outer
state than in the liquid state layer layer
(b) Its specific heat capacity is greater in the liquid
state that in the solid state 1000.C 100.C
(c) Its latent heat of vaporization is greater than its
latent heat of fusion
(d) Its latent heat of vaporization is smaller than its o
latent heat of fusion (a) By increasing the thermal conductivity of outer
3. When the temperature of a copper coin is raised by layer
soce, its diameter increases by 0.2% : (b) By increasing thermal conductivity of inner layer
(a) percentage rise in the area of a face is 0.4% (c) By increasing thickness of outer layer
(b) percentage rise in the thickness is 0.4% (d) By increasing thickness of inner layer
(c) percentage rise in the volume is 0.6%
(d) coefficient of linear expansion of copper is
6. Let ii, t'nn, and t' p respectively denote the mean speed,
0.25x 10-4/"e root mean square speed, and most probable speed of
the molecules in an ideal monoatomic gas at absolute
temperature T. The mass of a molecule is m. Then:

www.puucho.com
Study Anurag Mishra with www.puucho.com

136 THERMODYNAMICS
(a) no molecules can have a speed greater than 3
(a) the new temperature is - times the old
.J2 t'nns 2
(b) no molecules can have speed less than v p ./'2
/ • temperature
(e) vp <v,vrms 2
(b) the new temperature is - times the old
3
Cd) the average kinetic energy of a molecule is ~ mv~ temperature
4 (c) the power radiated by the body changes by a
7. 1\vo identical vessel contain helium and hydrogen at
factor of 16
same temperature, then: 81
(a) average kinetic energy per mole of hydrogen = (d) the power radiated by the body changed by a
average kinetic energy per mole of helium . 81
f actor IS-
(b) average translation kinetic energy per mole of 16
hydrogen = average translational kinetic energy
11. A gas kept in a container of finite conductivity is
per mole of helium
suddenly compressed. The process :
(e) average kinetic energy per mole of hydrogen = ~ (a) must be very nearly adiabatic
5
average kinetic energy per mole of helium , (b) must be very nearly isothermal
(c) may be very nearly adiabatic
Cd) average kinetic energy per mole of hydrogen = ~ (d) may be very nearly isothermal
3
average kinetic energy per mole of helium 12. What is/are the same for 02 and NH3 in gaseous
8. Two bodies A and B have thermal emissivities of 0.01 state.
and 0.81 respectively. The outer surface area of the (a) ratio of specific heats
two bodies are the same. The two bodies emir total (b) average velocity
radiant power at the same rate. The wavelength AB (c) maximum no. of vibrational degree of freedom
corresponding to maximum spectral radiancy in the (d) none of these
radiation from B shifted from. the wavelength 13. Which of the following statements is/are not an
corresponding to maximum spectral radiancy in the assumption of the kinetic theory for an ideal gas?
radiation from A, by 1.00 ~m. If the temperature of A (a) The duration of a collision is negligible as
is 5802K : compared to the time between successive
(a) the temperature of B is 1934 K collisions
(b) JoB = 1.5~m (b) The molecules have negligible attraction for each
(c) the temperature of B is 11604 K other
Cd) the temperature of B is 2901 K (c) The molecules have negligible momentum change
9. From the following statements concerning ideal gas at on collision with the container walls
any given temperature T, select the correct statement (d) There is no total kinetic energy change of the
(s) , molecules on colliding with each other or with the
(a) The coefficient of volume expansion at constant walls of the container
pressure is the same for all ideal gases 14. Select the correct statement about ideal gas:
(b) The average translational kinetic energy per (a) Molecules of a gas are in incessant random motion
molecule of oxygen gas is 3 kT, k being Boltzmann colliding against one another and with the walls of
constant the container
(c) The mean-free path of molecules increases with (b) The gas is not isotropic and the constant 0/3) in
decrease in the pressure equation P = (1/3)p v';'s is result of this property
(d) In a gaseous mixture, the average translational
(c) The time during which a collision lasts is
kinetic energy of the molecules of each component
negligible compared to the time of free path
is different between collisions
10. The maximum spectral radiancy of a black body (d) There is no force of interaction between molecules
corresponds to a wavelength ofJ .. lf the temperature is among themselves or between molecules and the
changed so that the maximum spectral radiancy now wall except during collision
3
corresponds to a wavelength of - A. It follows that:
2

www.puucho.com
Study Anurag Mishra with www.puucho.com

I TEMPERATURE, HEAT AND THE EQUATION OF STAlE, HEA::.T.;:TRA=NS"F;:ER:... _ 137


15. Select the correct statement(s) : 20. Consider the shown case of a _ O.C
(a) f.m.s. speed of 8 gm oxygen gas in container at
27°C is approximately 484 mls
(b) r.m,s. speed of 8 gm oxygen in container at 2TC is
~:~i:~;~:~~a~uet~~;:;:t~~:
Mt@irJ/Mt«fii
:::::::::::::X5'!~!~.~.?::x::r
(-00C). Thickness (x) of ice
approximately 968 mls layer is small in comparison to ~~~~~:~:,::::::::::,::
(e) For number of molecules greater than one, r,m,s. depth oflake. Rate of increase
speed is greater than average speed in x will be greater:
Cd) A gas behaves more closely as an ideal gas at low (a) if environmental temperature increases
pressures at high temperatures. (b) for larger thickness of ice layer
16. Hydrogen gas and oxygen gas have volume 1 em 3 (c) if environmemal temperature decreases
each at N.T.P.: (d) for smaller thickness of ice layer
(a) Number of molecules is same in both the gases 21. The ends of a long
(b) The r.m.s. velocity of molecules of both the gases is homogeneous bar are
the same maintained at different
(e) The internal energy of each gases is the same temperatures and there is no
Cd) The average velocity of molecules of each gas is loss of heat from the sides of
the same the bar due to conduction or
17. A gas is enclosed in a vessel at a constant temperature radiation. The graph of Distance from hot end
at a pressure of 5 atmosphere and volume 4 litre. Due temperature against distance
to a leakage in the vessel, after some time, the of the bar when it has attained steady state is shown
pressure is reduced to 4 atmosphere. As a result, the: . here. The graph shows:
(a) volume of the gas decreased by 20% , (a) the temperature gradient is not constant
(b) average K.E. of gas molecule decrease by 20% . (b) the bar has uniform cross-sectional area
(c) 20% of the gas escaped due to the leakage (c) the cross-sectional area of the bar increases as the
(d) 25% of the gas escaped due to the leakage distance from the hot end increases
18. 1\vo metal rods X and Y having equal cross-sectional (d) the cross-sectional area of the bar decreases as the
areas are joined end to end to form a composite bar, distance from the hot end increases
one end of which is heated. After some time has
elapsed, the temperature gradient along each rod is
found to be uniform, but greater in X than rhat in Y.
Which of the following can be inferred:
Ca) both the rods are well lagged
(b) the heat current is the same in both the rods
22. Four identical rods which have
thermally

Points
insulated

B,C, D and E
lateral
surfaces are joined at point A.

connected to large reselVoirs. If


heat flows into the junction from
are XC E D
(c) both the rods are of equal lengths point B at race of 1 Wand from
(d) X is a better conductor of heat than Y point C at 3 W inside, flows out from D at 5 W, which
19. A metal cylinder of mass 0.5 kg is heated electrically relation(s) is/are correct for temperature of these
by a 12 W heater in a room at 15°C. The cylinder points?
temperature rises uniformly to 25°C in 5 min and (a) TA <TE (b) TIJ =Tc
finally becomes constant at 45"C. Assuming that the (c) Tc > '1'0 (d) 18 = TE
rate of heat loss is proportional to the excess 23. Two spherical black-bodies A and B, having radii rA
temperature over the surroundings: and rn, where 1"0 = 2rA emit radiations with peak
(a) the rate of loss of heat of the cylinder to intensities at wavelengths 400 nm and 800 om
surrounding at 20°C is 2W respectively. If their temperature are TA and Ta
(b) the rate of loss of heat of the cylinder to respectively in Kelvin scale, their emissive powers are
surrounding at 45"C is 12W E A and E a and energies emitted per second are P A and
(c) the rate of loss of heat of the cylinder to Fa then:
surrounding at 20°C is 5 W (a) TA/Ta = 2 (b) PA/P8 = 4
(d) the rate of loss of heat of the cylinder to (c) EAlEs = 8 (d)E A/E8 =4
surrounding at 45"C is 30W

www.puucho.com
Study Anurag Mishra with www.puucho.com

8 !HE_a
dB
24. In Newton's law of cooling, dt = -k(B - Bo), the
(a) ~~ is the same for points on the rod.
constant k is proportional to :
(a) A, surface area of the body (b) I will decrease as we move from higher to lower
(b) S, specific heat of the body temperature
ee) ..!.. m being mass of the body (e) 1=1<. dB
m dl
Cd) e, emissivity of the body Cd) All the above options are incorrect
25. The two ends of a uniform lagged rod of thermal 26. The net rate of heat loss by a hot body depends upon:
conductivity k are maintained at different hut constant (a) temperature of body
temperatures. The temperature gradient at any point (b) temperature of surroundings
on the rod
is dO (equal to the difference in ee) material of body
dl (d) nature of the surface
temperature per unit length). The heat flow per unit
time per unit cross.section of the rod is I.

ANQWERQ

1. (a,d) 2. (a,c) 3. (a,e,d) 4, (a,c,d) 5. (a,d) 6. (c,d)

7. (b, d) 8. (a,b) 9. (a,c) 10. (h, c) 11. (c, d) 12. (a, d)

13. (e) 14. (a, b, d) 15, (a, c, d) 16. (b) 17. (a,C,d) 18. (a,b)

19. (a,b) 20. (e,d) 21. (a,d) 22. (a,c,d) 23. (a,b) 24. (a,c)

25. (a,c) 26. (a,b,d)

----_ ..
www.puucho.com
Study Anurag Mishra with www.puucho.com

TEMPERATURE, HEAT AND THE EQUATIDN Df STATE, HEAT TRANSfER 139

Solution

= Level.2: More than One Alternatives is/are Correct

19. [a, b]
22. [a, c, d]
Heat flow
At A
B~W C
3W
Rate of heat loss, [dQ) =kCS-So) x+l+3=5
xWASW
d( lo,s x =1 Heat in flows from E
At 45"C, T£ > TA E 0
Rate of heat loss = Rate of heat supplied =12W Tc >TA >TD
k(4S-1S) = 12 TB -1~\=T£ -TA
=> k=2/SW/'C .. TB =TE
Q 23. [a, b]
[dde) =k(20-1S)=2W
loss TA = Afl = 800 = 2
20. [c, d) TB I-A 400
Q = KA(~T) = KA[O - (ell; de
-=--
KAe
EA=crTA=16
4

x x dt x
£8 aTs4
21. [a, d]
h angmg.
. S0 s Iope IS
. PA = crT': x 4n:rl
In grap h -dT IS
. not constant It.. IS C 4 2 =4
dx PB cr1,4
B x rrrB

changing in equation (1) in steady state its constant 26. [a, b, d]


and k is also constant and ~ is increasing Heat loss = [Heat loss] radiation + [Heat loss] convection
=eAcry4 +k[T-To]n
continuously thee A is decreasing.
where e = surface property ;
T = body temperature
To = surrounding temperature

www.puucho.com
Study Anurag Mishra with www.puucho.com

THERMODYNAMICS I

Problem ~I

Pevel CID -_ __ ..._ ..•... .•.••....-._---------------


"f) •...r:ni
p" ,"~~
••• _VU_I' ••.•~

Two bodies of equal masses are heated at a uniform rate One kg of a substance (initially solid at DOC)is supplied
under identical conditions. Their changes in temperature heat at a constant rate of 2 kcaVrnin. Coefficient of thermal
are shown graphically. Select the answer following:' expansion of the solid (a) == 4,.. 10-6 j0c. Its temperature
T varies with time as shown.
(II)
110 -------- •••••• --
100 ---.--- ••• - •• ---
(I) Temperature
60 , (in .e)
,,
40 ....
,

o 246789 t (sec)
o 5 10 25 35
Time (in min)
1. What is the melting point of body (I) ?
(a) 40 (b) 60 1. Which of the following is not correct?
(e) 70 Cd) so (a) Melting point of the substance is 20°C
2. What is the melting point of body (II) ? (b) Melting point of the substance is 110°C
(a) 40 Cb)60 (c) Latent heat of vapourization for the substance is
(e) 70 Cd) SO greater than the latent heat of fusion
3. What is the ratio of their latent (LI/LIl) ? (d) Specific heat capacity of liquid is more than that of
Ca) 3/4 ' (b) 1/2 the solid
Ce) 1/4 (d) 1/8 2. Specific heat capacity of the liquid is :
4. What is the ratio of their specific heats for solid states? (a) 3 caVgmOC (b) ..!. caVgmCC
3
(a)2:1 (b)3:1
(e)I:1 (d)I:4 (c) 2 caVgmOC (d) .!. caVgmOC
5. What is the ratio of their specific heats for liquid state 2
Sl/Sn ? 3. Percentage change in volume of the solid in the first
(a) 2 : I (b) 3 : I 150 sec. is :
(e) 1: 1 (d) I : 4 Ca) 0.012% (b) 0.006%
(c) 0.024% (d) 0,004%

www.puucho.com
Study Anurag Mishra with www.puucho.com

---_ ..~---
TEMPERATURE, HEAT AND THE EQUATION OF STATE, HEAT TRANSFE"R- •••• -- 141

Solids and liquids both expand on heating. The density of A thin copper rod of uniform cross section A square meters
substance decreases on expanding according to the and of length L meters has a spherical metal sphere of
relation: P2 '" Pl where, PI = density at T1,P2 = radius r m at its one end symmetrically attached to the
1 +y(T2 -Tl) copper rod. The thermal conductivity of copper is K and the
density at T2."f = coefficient of volume expansion of emissivity of the spherical surface of the sphere is f;. The
substances when a solid is submerged in a liquid, liquid free end of the copper rod is maintained at the temperature
exerts an upward force on solid which is equal [0 the T kelvin by supplying thermal energy from a P watt source.
weight of liquid displaced by submerged part of solid. Steady state conditions are allowed [Q be established while
Solid will float or sink depends on relative densities of solid the rod is properly insulated against heat loss from the
and liquid.
sides. Surroundings are at O°C. Stefan's constant = aWl
m2 K4:

1. After the steady state conditions ate reached, the


temperature of the spherical end of the rod, T, is :
(a) T, oT _ P(~ c) (b) T, 0 DOC

A cubical block of solid floats in a liquid with half of its (el T PL


0 (d)T 0 T-I'I:.
volume submerged in liquid as shown in Fig. (at 'KA .' KA
temperature T). 2. If the metal sphere attached at the end of the copper
a s ~ coefficient of linear expansion of solid rod is made of brass, whose thermal conductivity is
YL -t coefficient of volume expansion of liquid K b < K, then which of the following statements is
p s --)0 density of solid at temperature T true?
P L -+ density of liquid at temperature T (a) The temperature of the sphere will, under steady
state conditions, continue to be Ta
1. The relation between densities of solid and liquid at (b) The power that will be radiated oue from the
temperature T is : sphere will still be P,
(a) Ps = 2pL (c) It will take smaller time for steady state conditions
(b) Ps = (V2) PL to be reached
(c) Ps =PL Cd) The rate of thermal energy transmitted across the
(d) Ps = lV4) PL copper rod, under steady state, will be teduced
2, If temperature of system increases, then fraction of 3. The net power that will be radiated out, P" from the
solid submerged in liquid: sphere after steady state conditions are reached is ;
(a) increases (bl P,
PA
(b) decreases
(e) remains the same ec) P, = 0
(d) inadequate information
3. Imagine fraction submerged does not change on
increasing temperature the relation benveen Yr and
as IS ; Assume that the thermal conductivity of copper is twice
(a) '(r = 3us that of aluminium and four times that of brass. Three metal
(b) YI == 2as rods, made of copper, aluminium and brass respectively are
(c) h == 4«s each 15 em long and 2 em diameter. These rods are placed
(d) h == (312) Us end to end, with aluminium between the other two. The
free ends of the copper an brass rods are maintained at
100~C and O°C respectively. The system is allowed to reach
the steady state condition. Assume there is no loss of heat
anywhere.

www.puucho.com
Study Anurag Mishra with www.puucho.com

THEIMODYNAMICS
1. When steady state condition is reached everywhere (c) The density of gas is constant ifl<!I1 =.B...
which of the following statement is true? dhl Mg
(a) No heat is transmitted across the
(d) The density of gas is uniform if I~II= Mg
copper-aluminium or aluminium-brass junctions Idhl R
(b) More heat is transmitted across the
copper-aluminium junction than across the
aluminium-brass junction
(c) More heat is transmitted across the A 500 g teapot and an insulated thermos are in a 20°C
aluminium-brass junction than the room. The teapot is filled with 1000 g of the boiling water.
copper-aluminium junction 12 tea bags are then placed into the teapot. The brewed tea
(d) Equal amount of heat is transmitted at the is allowed to cool to BO°C,then 250 g of the tea is poured
copper-aluminium and aluminium junction will be from the teapot into the thermos. The teapot is then kept on
2. Under steady state condition, the equilibrium an insulated warmer that transfers 500 caVmin to the tea.
temperature of the copper aluminium junction.\.0Ii he: Assume that the specific heat of brewed tea is the same as
(a) 86°C . (b) l8.8°e that of pure water, and that the tea bags have a very small:
(e) S7°e (d) 73°C . mass compared to that of the water, and a negligible effect
3. Under steady state condition, the equilibrium on the temperature. The specific heat of teapot is 0.17 Jig K
temperature of the aluminium-brass junction will be: and that of water is 4.18 Jig K. The entire procedure is
(a) S7°e (b) 3soe aone under atmospheric pressure. There are 4.18 J in one
(e) l8.8°e (d) 28.Soe calorie.
1. After the tea is added to the thermos, the temperature
(" ", " ~-"'VI r, (;;;;:\ of the liquid quickly falls from Boac to 75°C as i
ww_tJ •••~
reaches thermal equilibrium with the thermos flas¥
A very tall vertical cylinder is filled with a gas of molar mass What is the heat capacity of the thermos?
M under isothermal conditions temperature T. The .density, (a) 9.SJjK (b) l4JjK
and pressure of the gas at the base of the container is Po (e) 9SJjK (d) 878JjK
and Po, respectively. 2. An alternative method for keeping the tea hot wouk
be to place the teapot on a 10 pound block that ha~
1. Select the correct statement(s):
been heated in an oven to 300"C. A block of which a
(a) Pressure decreases with height
the following substances would best be able to keer
(b) The rate of decrease of pressure with height is a
the tea hot?
constant
(a) copper (specific heat = 0.39J/g K)
(c) dP = _pg where p is density of the gas at a height h (b) granite (specific heat = 0.79J/g K)
dh .
(d) p~pRT (c) iron (specific heat = OASJ/g K)
M Cd) pewter (specific heat = 0.17 J/g K)
2. Select the correct statement(s) if gravity is assumed to 3. If, after some of the tea has been transferred to th{
be constant throughout the container: thermos (as described in the passage), the teapot witt
(a) Both pressure and density decreases exponentially its contents (at a temperature of 80"C) was placed or
with height the insulated warmer for 5 minutes, what would b{
_Mgh the temperature at the end of this 5 minute perio(
(b) The variation of pressure is P = Poe RT (Assume that no significant heat transfer occurs witl
_!:.f~~ the surroundings):
(e) The variation of density p = poe RT (a) 8(l7°e (b) 82SoC
(d) The molecular density decreases as one moves (e) 83.2°e (d) 9S.2"C
upwards
3. Select the correct statement(s) : (k " ,"~." ."/•• rCo)\
(a) The density of gas cannot be uniform throughout .••.. - "" w - t:J ••• '!!!!)
the cylinder A calorimeter of mass m contains an equal mass of water in
(b) The density of gas cannot be uniform throughout it. The temperature of the water and calorimeter is lz. A
the cylinder under isothermal conditions block of ice of mass m and temperature t 3 < OCC is gently
dropped into the calorimeter. Let Ct, Cz and C3 be the
specific heats of calorimeter, water and ice respectively and
L be the latent heat of ice.

www.puucho.com
Study Anurag Mishra with www.puucho.com

-----------
TiMPERATURE, HEAT AND THE EQUATION OF STATE, HEAT TRANSFER

1. The whole mixture in the calorimeter becomes ice if: (a) 472.6gm (b) 483.3gm
143

(a) clr2 +Czt:<: +L+C]t] >0 (e) 493.6gm (d) SOOgm


(b) CItz +Czt2 +L+C3t3 <0 3. Amount of the steam left in the system, is equal to :
(e) CIrZ +Czt1 -L -C]t3 >0 (a) 16.7 gm
Cd) CItz +Cztz -L -C3t] <0 (b) 12.0 gm
2. The whole mixture in the calorimeter becomes water if (c) 8.4 gm
(a) (C1+C2)tZ-C3t3+L>O (d) Ogm, as there is no steam left
(b) eel +C2)tz +C3t] +L >0
(e) eel +Cz)cz -C3t] -L >0
~p n \~\~,,/_~I ~ ~O
Cd) eel +CZ)t2 +C3t3 -L > a ..•.._'OiU-tJ •...~
3. Water equivalent of calorimeter is : Consider a gas at temperature T occupying a volume V
(a) mel (b) mel consisting of a mixture of two gases having N a & N b atoms
C, of masses ma& mb respectively.
Cd) none of these
1. Give an expression for the total pressure exerted by
the gas:
(a) eNa +Nb)kT
V
In a container of negligible lv!at capacity, 200 gm ice at DOG (N" +Nb)RT
(e) ----- Cd) none of these
and 100 gm steam at 100°C are added to 200 gm afwater V
that has temperature 55°C. Assume no heat is lost to the 2. Suppose now that N a = N b and that the different
surroundings and the pressure in the container is constant atoms combine at constant volume to form molecules
1.0 atm. of mass m" + mb' Once the temperature returns ro its
(Lf = 80 caVgm, Lv = 540 caVgm, Sw = 1 caVgm °e) original value, what would be the ratio of the pressure
after combination to the pressure before?
1. What is the final temperature of the system?
(a) 1 (b) 1/3
(a) 48'e (b) n'e
(c) 1/2 Cd) none of these
(e) 94'e (d)100'e
2. At the final temperature, mass of the total water
present in the system, is :

www.puucho.com
Study Anurag Mishra with www.puucho.com

144 THERMO AMICI.

MATCHING TYPE PROBLEMS


3. A cylindrical isotropic solid of coefficient of thermal
expansion u and density p (at STP) floats partially in a
1. A copper rod (initially at room temperature 20"C) of liquid of coefficient of volume expansion y and density
non-uniform cross-section is placed between a steam (at STP) :
chamber at lOO"C and ice-water chamber at DOC.
Column I Column II I
100.C O'C (a) Volume of cylinder inside the liquid (p) Y""0
B remains constant
Steam Ice water
Chamber Chamber
(b) Volume of cylinder outside the (q) y"" 2u
liquid remains constant
A C
(e) Height of cylinder outside the (r) d
y",,3a-
liquid remains constant P

(d) Height of cylinder inside the liquid


remains constant
Column I Column II I
(a) maximum at 4. The surface of a house hold radiator has an emissivity
Initially rate of heat flow ( ~~) will be (p)
section A
of 0.55 and an area. of 1.5 m 2. Its equilibrium
(b) maximum at temperature is 50~Cand the surroundings are at 22°C.
At steady state rate of heat flow (~;) (q)

will be
section B (0 = 5.67 x lO-sW/m 2K4)
,
(e) At steady state temperature gradient (r)

I(:)1 will b,
maximum at
section C
\ Column I
(a) At what fate is radiation emitted
by the radiator
\
"
(p) ISS
Column It

(h) A, what rate i, radiation (q) 509


(d) At steady state rate of change of (5) minimum at
temperature
. (dT) .
dt at a certam POint . .
section B
absorbed by the radiator

«) What is the net rate of radiation (,) 354


from the radiator
will be

(tl same for all 5. A piece of metal of density PI floats on mercury of


section density P 2' The coefficients of expansion of the metal
and mercury are 11 and 12, respectively. The
2. Match column I and II : temperatures of both mercury and metal are increased
\ Column I \ \ Column II I by IlT. Then match the following:
\
(a) Wien's displacement
explains
(p) Why days are hot and nights
cold in desens

(b) Planck's law explains (q) Why a blackened platinum wire,


<a)
Column I
IfYz>Yl ""
(p)
Column II
no effect on submergence

fraction of the volume of metal sub-


when gradually heated, appears (b) 12 =11 (q)
merged in mercury
first duJ] red and then blue

«) Kirchhoffs l,w (,) The distribution of energy in


«) Ifyz <y, (,) the solid sinks
explains black body spectrum at shorter (d) (Yz-Yl)ilT (s) the solid lifts up
as well as longer wavelengths

(d) Newton's second law (s) Why some stars are hot ter than
explains others

www.puucho.com
Study Anurag Mishra with www.puucho.com

TEMPERATURE, HEAT AND THE EQUATIDN DF STATE, HEAT TRANSFER

6. In a container of negligible mass' /11' grams of steam at


------ 8. Entries in column-I consists of diagrams of thermal
145

lOoDe is added to 100 gm of water that has conductors. The type of conductor & direction of heat
temperature 20"C. If no heat is lost to the flows are listed below. Entries in column-II consists of
surroundings at equilibrium, match the items given in the magnitude of rate of heat flow belonging to any of
Column I with that in Column II. the entries in column-I. If temperature difference in all
the cases is CTt - T2), then match column.
Column I Column II
Column I Column II
(al Mass of steam in the mixture, if (p) 114.8
(a) 3R
m '" 20gm (in gm) •
(b) Mass of water in the mixture, if (ql 76.4
m '" 20gm (in gm)

(e) If m = 20gm. final temperature of (e) 5.2


the mixture (in °C)
Thick cylindrical shell, flow along
(d) If m = 10gm, final temperature of (,) 100 axis
the mixture (in °C)
(bl (q) rtkoR-(J -T)
3In 2 I 2
7. In a vessel a liquid A is mixed with a block of ice B. The
doned line shows the temperature of A and dark line
T,
of B.
\ Column I \ \ Column II

(a) T (p) The equilibrium temperature


below freezing point of water Thick spherical shell, radial flow

(b)
Ct
T (q) At the equilibrium, the liquid
partly freezes.
(e)

tRU.;J~_.~~
2R

T,

(e)
Ct
T (e) The ice partly melrs
(d)
Thick cylindrical shell, radial flow

T(IDuuJR._!~J
3R •
(,) 4nkoR
In 2
(J -T )
1 2

Lt
:_x
Solid cylindrical flow along axis,
variable k as k '" ko(l + x I (3R))

9. Suppose that both


(d) T (,) The ice fully melt ends of the rod are
Cross-seclion
Area A

L,
kept at a temperature
of PC, and that the
initial temperature
distribution along the
rod is given by
Heat

_L_
T '" (lOO°C) sin rrx/L, where x is measured from the left
[tl The equilibrium temperature end of the rod. Let the rod be of copper, with length
is freezing point of water
and cross-section area A. Column -I represents graph
of certain physical quantities as we move from left to
right end of rod. Column-II represents those physical
quanities

www.puucho.com
Study Anurag Mishra with www.puucho.com
146 ... ~.-..• ---_.
, ,~,~, --'"'T.TH"IAMo.lliOIlDYNAMllm:lICSm
Column I Column II 11. Column I shows certain thermodynamic systems and
(a) (p) Initial temperature gradient column II represents thermodynamic properties.

(b)
'Pc
o

,
x'='L
,

(q) Initial temperature.


(a)
Column I

(p)
Column II
Internal
energy
the gas is
increasing
of

, An ideal gas is fUled in a cylindrical


vessel of height h which is enclosed
x=L
,' by a massless thermally insulating
(e) , (r) Final temperature
piston. Mercury is filled above the
piston as sho\'.ll. Now gas is slowly
distribution along rod.
supplied heat. Mercury does not
Graph coincides
with x-axis
spill.
x=L • (b) (q) Pressure of
the gas is
increasing.
(5) Final rate of heat transfer
along rod.
10. Column I shows some heat conductors of various
shapes. Column II shows certain parameters related
A cylindrical vessel is enclosed by a
with heat transfer. Match the column r with column U.
light piston. The piston is connected
Note that comparison is to be made in direction of heat to ceiling by an ideal spring as
flow and in steady state. In all the Fig. the surfaces shmvtl in Fig. Spring is initially
shown in cloned are isothennal surfaces. relaxed and then heat is supplied
slowly to the ideal gas in the vcssel.
Column I Column II The system is kept in open
(a) atmosphere.
~(P Modulus of temperature
gradient is increasing as (e) (r)
R~) Temperatur
we move from higher
e of the gas
temperature to lower
Tapenng rod is i,
temperature
well inswa!ed decreasing.

(b) (q) The rate of heat transfer


across the dotted
surfaces shown. is A thermally insulated cylindrical
constam vessel is enclosed by a light
thermally insulated piston. Some
sand is kept on lOp of piston as
shown in Fig. The system is kept in
(e) (r) Modulus of temperature open atmosphere. Now sand grains
gradient is decreasing as are rcmoved slowly one by one.
we move from higher
tempcrature to lower
temperature

(s) Heat flows perpendi-


cular to the cross-secnon
shown by the doned line

www.puucho.com
Study Anurag Mishra with www.puucho.com

TEMPERATURE, HEAT AND THE EQUATlDN DF STATE, HEAT TRANSFER 147

(d) (s) -----------


Work done by gas
is positive.
4. Statement-I: When water is heated by a burner in
metallic container its level first decrease then
increases.
Statement-2: Thermal conductivity of metal is very

~lll!!lmill large compared to water.


5. Statement-I: It is necessary to wait for thermal
equilibrium to be established before recording the
A good conducting cylindrical \'essel
temperatures of any system/systems in thermal
is enclosed by a light thermally
insulated piston. Some sand is kept interaction.
on top of piston as shovm in figure. Statement-2: Heat or thermal energy is exchanged
The system is kept in open between systems only when they are in thermal
armosphere. Now sand grains are equilibrium.
added slowly one by one. 6. Statement-I: 1\vo solid cylindrical rods of identical
(t) Molar heat size and different thermal conductivity K 1 and K 2 are
capacity of the connected in series. Then the equivalent thennal
gas is positive conductivity of two rods system is less than that value
of thermal conductivity of either rod.
Statement~2: For nvo cylindrical rods of identical
size and different thermal conductivity K1 and K'l
connected in senes, the equivalent thermal
conductivity K is given by
ASSERTION AND REASON 2 1 1
-~-+-
K K1 K2
Direction: In the questions that follow two statements 7. Statement-I: Absolute zero temperature is not the
are given. Statement-2 is purported to be the explanation for temperature of zero energy.
statement-I. Study both the statements carefully and then Statement-2: Only the translational kinetic energy
select your answers, according to the codes given below: of the molecules is represented by temperature.
(a) If Statement-l is true, Statement-2 is true; 8. Statement-I: In natural convection, the fluid motion
Statement-2 is the correct explanation for is caused due to density difference produced by
Statement-I. temperature gradient.
(b) If Statement-l is true, Statement-2 is true,
Statement-2: In forced convection, the fluid is
Statement.2 is not a correct explanation for
forced ro flow along the solid surface by means of fans
Statement-I.
or pumps.
Cc) If Statement-l is true; Statement-2 is false.
Cd) If Statement-l is false; Statement-2 is true. 9. Statement-I: Greater is the coefficient of thermal
conductivity of a material smaller is the thermal
1. Statement-I: As the temperature of the black body
resistance of a rod of that material.
increases, the wavelength at which the spectral
intensity (E ,) is maximum decreases. Statement-2: Thermal resistence is the ratio of
temperature difference between the ends of the
Statement-2 : The wavelength at which the spectral
conductor and rate of flow of heat.
intensity will be maximum for a black body is
proportional to the fourth power of its absolute 10. Statement-I: The thermal resistance of a multiple
layer is equal to the sum of the thermal resistances of
temperature.
the individual laminas.
2. Statement-I: The bulb of one thermometer is
spherical while that of the other is cylindrical. Both Statement-2: Heat transferred is directly
have equal amount of mercury. The response of the proportional to the temperature gradient in each layer.
cylindrical bulb thermometer will be quicker. 11. Statement-I: Red dye molecules are placed in a
Statement-2 : Heat conduction in a body is directly cavity maintained at constant temperature higher than
proponional to cross-sectional area. that of molecules. Inside cavity radiations of dye
molecules appear red.
3. Statement-I: Water is considered unsuitable for use
in thermometers. Statement-2: The colour of cavity radiation depends
only on the temperature and not on the chemical
Statement-2: This is due to small specific heat and
nature of system.
high thermal conductivity.

www.puucho.com
Study Anurag Mishra with www.puucho.com

12. Statement-I: When you touch two bodies, the body 13. Statement-I: It is convenient to define two specific
which is felt wanner must be of higher temperafure'.'\.,"l,:;' ;I~ heats C and Cv in case of a gas. However it is not
than the other one. general{y necessary to define two specific heats in case
Statement.2: The heat flows from higher of a solid or liquid.
temperature to lower temperature. S~~~n;t~~t-:2 ..
:,For a given temperature rise, the
expahSiOItot a solid or liquid is negligible as compared
_ . _ ..•. to.that.of a gas.
o-~._..•

.......• ..;:.'-':
,

..... ... " '

. , , .
"
," .,
, .,'

"
,.
.', .'.

www.puucho.com
Study Anurag Mishra with www.puucho.com

TEMPERATURE, HEAT AND THE EQUATlQN OF STATE, HEAT TRANSFER


._-~------
. '. I'~":J " .
149

. -,' ANSWERS

=: Level.]: Comprehension Based probh!.rn~ "

•. (b) 1. (d) 2. (b) 3. (a)


Passage-1 : 1. Ca) 2. (b) 3. (a) Passage-2:
5. (a)

1. (0) 2. (d) 3. (a) Passage-4: 1. Ca) 2. (b) 3. (a)


Passage.3:
1. (e) 2. (b) 3. (d)
Passage-5: 1. (d) 2. (a) 3. (a) Passage.«5:
2. (b) 3. (e) Passage.a: 1. (b) 2. Cd) 3. (b)
Passage-7: 1. ee)

,. (d) 2. (b) 3. (a) Passaoe-10: 1. (a) 2. (e)


PassaQe-9:

=:; Matching Type Problems'

1. (a)-p, r; (b)-t; (c}-g, r; Cd)-t 2. (a).q; (bl.s; (e)-r; (d)-p

3. (a)-p; (b).r; (e)-s; (d)-q •. (a).q; (b)-r; (e)-p

5. (a).q; (b).p; (e).s;(d).q 6. (a)-r; (b)-p; (e)-s; (d)-p

7. (a)-r, t; (b)-f, t; (e)-s; (d)-p, q 8. (a).r; (b)-p; (e)-s; (d).q

9. (a)-q; (b)-p; (c)-r, S 10. (a)-p, q, s; (b)-g, r, s; (c)-g, c, s

11. (a)-p, s, t; (b)-p, g, s, t; (c)-r, 5; (d)-q

== Assertion and Reason

5. lb) 6. (d) 7. (a) 8. (b)


1. (a) 2. (a) 3. (e)
•• (a)
9. (b) 10. (e) 11. (d) 12. (d) 13. (b)

www.puucho.com
Study Anurag Mishra with www.puucho.com

Solution

Level-3: Comprehension Based Problems 2500x 4.18 = (750 x 4.18 + 500x 0.17).1.e
~ ae = 3.24 final temperature = 83.24°C
Passage-5
Passage-8
1. Cd] Under steady state conditions, the temperatures
1 [b] Heat lost by water and container
at all sections in the system remain constant and
maintain a constant temperature gradient for a mCt(r2 -O)+mC2(t2 -O)+mL <mC3(O-t3)
given material, The temperature gradient in
Ctt2+C2t2+C3t3+L<0
copper, aluminium and brass will not be same,
however the rate of heat conducted across all 2. [d] Heat lost = mCt (t2 - 0) + mC2(t2 -0)
sections whether in copper or aluminium or Heat gained = mC3(0-t3) + mL
brass will be the same.
.. Ctt2 +C2t2 >C3t3 +L
2. [a] Since heat transmitted per second in the steady
.. Clt2 +C2t2 +C3t3 -L >0
kA(T,-T,) an d th e d'ImenSlOns
, Q 0= --~~-
state IS , 0f
L ' 3. [b] mCt =MC2
copper, aluminium and brass rods are identical,
we must have .. M = mCI
C,
K1(lOO-Tca)=K2(Tca -Tab)=K3(Tab -0)
Passage-9
Where KI• K 2 and KJ are thermal conductivities Suppose final temperature = 100
of copper, aluminium and brass respectively and
then, Heat absorbed by ice = meL x SaT)
Tea and Tab are the steady state temperatures of
copper -aluminum and aluminium-brass = 200x 180cal
junctions Heat absorbed by water = mSaT
Now Kl = 2K2 = 4K3 (Given) = 200x 45cal
Total heat absorbed = 200x 225cal
Hence, 4(100-Tca) = 2(Tca -Tab) = Tab
Suppose 'm' mass of steam condenser then heat
Solving for Tab and Tca we obtain returned = mLv
1200 °C =540)( m = 200x 225
14Tca =1200~Tca =--=85.7=86
14 Suppose final temperature = T
then Heat absorbed by ice = mice(Lf + S. T.)
AlsoT'b =~T =2x86=57°C
3 ~ 3 = 200(80 + T) cal
Passage-7
Hear absorbed by water = mWaler. SeT + 55)
1. [a] 250gmx 4.18x(80-75) = (ms)(75-20)
= 200(T - 55) cal
:,(ms) o 95JjK
Heat released by steam = msteam{Lv + S(l 00 - T)}
2. [b] Granite (Maximum specific heat) = 100(540+ 100+T)
3. [c] as Rate =500caVmin :.5min = 2500cal = 100(640-T)cal
Heat released by steam =Heat absorbed by ice + water

www.puucho.com
Study Anurag Mishra with www.puucho.com

THERMODYNAMICS
=> 100 x (640 - T) = 200(80 + T) + 200(T - 55) (c) Rate at which net radiation is emitted by the body
:::::> 640-T=2(80+T)+2(T~SS) = Qn.J/s =Q, -Qa
"(509-354)W" 155W
T = 590 > 100°C:::> All steam has not condensed
5 5. (a) Heat MI added to the gas along the straight line
and hence final temperature it lOQ°C. path AC.
Suppose mass x of steam has condensed. (0 Work t.. WAC done b~ the gas along the straight line
men,
Heat absorbed
Heat released by steam:: 540x
by ice = 200(80+ 100)
path AC = area (I pdV J under the path.

Heat absorbed by water = 200(100 - 55) 1 3


Heat absorbed by steam =Heat absorbed by water and t..WAC ""PAVA + "2PAVA = "2PAVA'
ice. (ij) Increase of internal energy ~U AC is t..U AC
=> 54Qx = 200 x 225 3
250 =-nR(Tc -TA)
X"- 2
3 3 9
=-(PcVc -PAVA) =-PAVA
Water present in final system = 400 + x = 483.33gm 2 2
Steam present = lOO-x = 16.67gm Hence heat added, MI AC = t..W AC + ~UAC
Passage-10 3 9
=-PAVA +-p,\VA =6PAVA
nRT 2 2
1. [a] P = ~-, where n =number of moles of all gases 9
V (b) From above, t..UAC =~PAVA
2
N +Nb
r! = Q ,N = Avogadro number (c) Work done t..WABC = t..WAB + t..Wnc
N
=O+Pc(Ve -VA) = 2PAVA
p=(Na+Nb)RT =Na+Nb RT=(Na+Nb)kT (d) Heat ilH added to the gas along ABC,
N V V N V 9
MIABe =MiM +-PAVA +2PAVA
2. [e] After getting combined the no. of moles becomes 2
half in this case
at canst. V and T
P xn
P becomes half

:I Matching Ty~ Problems


4. (a) Rate at which heat is radiated from the body
4
= Q,J/S = ecraTI
= 0.55)C 5.67)C 10-8 x 1.5 x (323}4 J/s
= 509W
(b) Rate at which heat is radiated is absorbed by the
body
= Qafs = ecr:AT14
= 0.55 x 5.67 x 10-8 x 1.5 x (295}4 J/s

= 354W
= Assertion and Reason

12. The correct reason is because under steady state


conditions, when temperarure constant, the rate of
conduction of heat across every lamina is the same.

www.puucho.com
Study Anurag Mishra with www.puucho.com

Thermodynamics is the science of energy related with The properties of a macroscopic system as a whole are
the conversion of energy from one form to another. The called its bulk properties, example is the system's mass,
word energy itself comes from a Greek root meaning "to do volume, density, temperature and pressure. Macroscopic
work." and thermodynamics is concerned with converting systems are also characterized as being either solid, liquid.
stored energy of various forms, such as the energy in fuels, or gas. These are called the phases of matter, and we will
into useful work. be interested in when and how a system changes from one
Thermodynamics is the systematic study of converting phase to another.
heat energy into mechanical motion and work. Hence the Ultimately we would like to understand the macroscopic
name thermo + dynamics. properties of solids, liquids, and gases in terms of the
Thermodynamics deals with macroscopic systems, the microscopic motions, of their atoms and molecules.
prefix macro (the opposite of micro) meaning large. We will Solids, Liquids and Gases
be concerned with systems that are solids, liquids, or gases,
rather than the "particles". The variables of thermodynamics The ice cube you take out of the freezer soon becomes a
will be temperature, pressure, volume, and moles instead of puddle of. liquid water. Then, more slowly, it evaporates to
position, velocity and force our queries will be of : become water vapour in the air. Water is unique in its
properties. It is the only substance whose three
How do temperature and pressure change during a phases-solid, liquid, and gas, are familiar to us from
certain process? everyday experience.
How does a physical system such as an engine do Each of the elements and most compounds can exist as a
mechanical work? How is the work related to the solid, liquid or gas. The change between liquid and solid
temperature and pressure inside the engine? (freezing or melting) or berween liquid and gas (boiling or
A room full of air, a beaker of water, are condensing) is called a phase change that occur at a
examples of macroscopic systems, systems that are well-defined temperature.
large enough to see or touch.

www.puucho.com
Study Anurag Mishra with www.puucho.com

~ THERMODYNAMICS 153

Atoms vibrate around A system is in thermodynamic equilibrium if every


equilibrium positions. state variable has the same value throughout the system. In
equilibrium there is no tendency for it to undergo
spontaneous macroscopic change.
State Variables
In describing a thermodynamic system we use few
variables called thermodynamic variables. e. g.,
Melt thermodynamic variables for an ideal gas are pressure P,
volume V, temperature T, number of molecules N,
(a) A solid consists of panicle-like atoms connected together by
internal energy V, and entropy S.
spring like molecular bonds. Vibrateing around an equilibrium The parameters used to describe a macroscopic system
position. but an atom Is not free to move around inside the solid.
Solids are nearly in compressible.
are known as state variables.
The state variables may dependent on each other. For
Atoms are held close together
by weak molecular bonds, but example that a system's mass density p is defined in terms of
they can slide around each other. the system's mass M and volume Vas
p = M (mass density) ...(1)
V
Condense
If we change the valUeof any of the state variables, then
the state of the system change. If we compress a gas ir means
Boil
to decrease irs volume. Pressure and temperature, may also
cltange as the volume changes.
(b) Like a solid, a liquid is nearly incompressible. The molecules in a A system is said to be in thermal equilibrium if irs
liquid are about as close together as they can gel. Like a g8S, a liquid state variables are constant and not changing.
nows and deforms to fit the shape of its container,
Atoms are far apart and travel ATOMS AND MOLES
freely through space except for The mass of a macroscopic system is directly related to
occasional collisions.
the total :lumber of atoms or molecules in the system,
denoted by N.
The number of atoms or molecules per cubic metre in a
system is called the number density. It characterizes how
dense the atoms are packed together within the system. In
an N-atom system that has volume V,the number density is
N

(c) In gas each molecule moves through space as a free. non.


V
interacting particle il collides with another molecule or with the wall of The 51 units of number density are m -3. The number
the container. A gas is a fluid. A gas is also highly compressible.
There lois of space between the molecules.
density of atoms in a solid (N jV}wlid _ 1029m -3.
The value of NjV in a ulliform system is independent of
Fig. 2.1
the volume V.
THERMODYNAMIC PROCESSES The basic panicle depends on the substance. Helium is n
mono-atomic gas, meaning that the basic particle is the
A thermodynamic system refers to a collection of
helium atom. Thus 6.02x 1023 helium atoms are 1 mole of
panicles which we wish to focus. A system has boundary this
boundary may be real such as that of a container or it mar helium. But oxygen gas is a diatomic gas because the
be an imaginary boundary placed around a particular part of basic panicle is the two-atom diatomic molecule 02' 1 mole
matter. A standard system is that of a gas enclosed in a of oxygen gas contains 6.02 x 1023 molecules of02 and thus
cylinder equipped with a movable piston. The cylinder walls 2x 6.02x 1023 oxygen atoms. Table 2.1 lists the
and the piston form the boundary and the enclosed gas is the mono-atomic and diatomic gases.
system.
A thermodynamic system is said to be in a steady
state if all the state variables remain constant with time at
each point within the system.

www.puucho.com
Study Anurag Mishra with www.puucho.com

154

Tables 2.1 Mono-atomic and diatomic gases There are four basic quasi-static processes III
Mono.atomic Diatomic equilibrium thermodynamics. The processes are
characterized by the thermodynamic variable that is kept
He H, constant.
Ne N, (a) Isothermal process: A process is
isothermal when the temperature of the system is kept
0, constant. The word comes from the Greek isos, equal and
therma, heat.
IDEAL.GAS PROCESSES
(b) Isobaric process: A process is isobaric
A process occurs when the system changes from one
when the pressure of the system is constant. The word
state (one set of values of its physical properties) to another
comes from the Greek baros, weighr.
state. The system returns to its original state when all of its
macroscopic physical properties resume their original (e) Isochoric process: A process is isochoric
values. when the volume of the system is constant.
Heat transfer and work are nvo processes that change (d) Adiabatic process: A process is adiabatic
the state of a thermodynamic equilibrium. A quasi-static when no heat is transferred to or from the system. The word
process is one in which the system changes slowly enough comes from the Greek adiabatos, not passable.
so that each succeeding state through which it passes is The P - V Diagram
essentially in equilibrium. All the reversible processes We represent ideal-gas processes on a graph called a
are quasi-static (occur very slowly). An equilibrium state is a p. V diagram. A graph of pressure verstL~ volume. Each point
static state. During a reversible process the system can on the graph represents a single, unique state of the gas, it
deviate from equilibrium by only an infinitesimal directly specifies the values of P and V. But knowing P and V,
amount. and assuming that n is known for a sealed container, we can
In a reversible process equation of state find the temperature by using the ideal-gas law. Thus each
applies at all times. point actually represents a triplet of values CP, V, T)
Imagine a pan of water at temperature To which is put in specifying the state of the gas.
contact with reservoirs at temperature To + dT, To + 2 dT Quasi-Static Processes
and so on until it reaches its final temperarure. Such a
Note thal the ideal-gas law applies only to gases in
process would require infinite contacts with heat reservoirs,
thennal equilibrium. A system is in thermal equilibrium if its
therefore, we can only approximate a reversible process,
state variables are constant and not changing. Consider an
strictly speaking quasi-static processes do not exist: they are
ideal gas proces.s that changes a gas from state 1 to state 2.
idealizations. Notice that if we make an infinitesimal change
The initial and final states are states of thermal equilibrium,
in the temperature T (in reverse direction, change T by
with steady values of P, V and T. During process, some of
-dT ), we reverse the direction of the process, hence the
these state variables change. The gas is not in thermal
process is called reversible process.
equilibrium while the process of changing from state 1 to
state 2 is continuning.
~
.•••• 'T,
!>
T,....e..
I
To use the ideal-gas law throughout, we will assume
that the process occurs slowly that the system is never far

I
Reservoir at
To + 2dT To + dT
<!!T!!!,-!!!d!!!T

Reservoir at
from equilibrium. In other words P, V and T at any point in
the process are essentially the same as the equilibrium
temperature To temperalure T,
values they would assume if we stopped the process at that
point. A process occurs essentially in thermal equilibrium at
Fig. 2.2
all times, is called a quasi-static process. It is an idealization,
A process that proceeds in only one direction is like a frictionless surface, but one that is a vel)' good
approximation in many.real situations.
irreversible. The equation of state does not apply during an
irreversible process. The thermodynamic variables are not
constant throughout the system, the system is not in
equilibrium. All real processes are irreversible.

www.puucho.com
Study Anurag Mishra with www.puucho.com

IHfRMODYNAMICS

venical line 1 -4 2 on the PV diagram of Fig. 2.4 (b). A


constant-volume cooling, by placing the container on a block
Piston moves
very slowly
of ice, would lower the pressure and be represented as the
vertical line from 2 to 1. Any isochoric process appears on a
p-v diagram as a vertical line. While 1 ---+ 3 ---+ 2 is not
isochoric process because dt, ~ 0.

•••
atmosphere

P,

'..'
•• -
Ao ;,oct'O';o
process appears

D
Quasi-static process _- on a PV diagram
, 3 as a vsrticalline
1-)
P,
Membrane
suddenly bursts 1

/ '------,,----V
V

(;) 1/ Fig. 2.4 A constant-volume (isochoric) process.

Constant Pressure Process


Vacuum
A constant pressure process is called an isobaric
process, where baric is from the same root is 'barometer"
and means "pressure." An isobaric process is one for which
Pj = Pi (M = Oand dP:= 0) ...(2)
Irreversible process Fig. 2.5 (a) shows one method of changing the stelte of a
I') gas while keeping the pressure constant. A cylinder of gas
Fig. 2.3 The slow motion of the piston is a has a tight-fitting, melssless piston that can slide up and
quasi-sIalic process. The bursting
of the membrane is not The mass maintains a constant

.•• ..
pressure In the cylinder

An important characteristic of a quasi-static process is


.hat the path through the PV diagram can be reversed. If you •
,,
~uasi-statically expand a gas by slowly pulling a piston out,
=15shown in Fig. 2.3(a), you can reverse process by slowing
Jushing the piston in. The gas retraces its PV path until it has
"eturned to its initial state. What happens when the
nembrane bursts in Fig. (b). That is a sudden process, not at V,
111quasi static, but irreversible process in Fig. 2.3 (b) cannot
)e represented on a PV diagram. It is often reffered as a free
~xpansion.
::onstant Volume Process
Before After
Gas processes take place in a container of constant, Fig. 2.5 (a)
Jnchanging volume is called an isochoric process, where
so, is prefix meaning "constant" or "equal" while choric is
down but the container is closed so that no atoms enter or
'rom a Greek root meaning "volume." An isochoric process is
escape. The mass on top applies a constant downward force
me for which
Mg on the piston. The atmosphere also presses down on the
Vj :=Vj (.:.\I':=O,dv=O) ... (I} piston. In equilibrium, the upward force PA on the piston by
For example, suppose that you have a gas in the closed, the gelSin the cylinder, where A is the piston's area, exactly
"igid mntainer heating the gas with will raise its pressure balances the downward force Mg plus the air-pressure force
.vithout changing irs volume. This process is shown as the P n;A. Consequently, the gas pressure inside the cylinder is
31nlt

www.puucho.com
Study Anurag Mishra with www.puucho.com

--->lTHEI!'RMOrulillDYNAM!U'2 •• ,;;;cs;'!

P = Pa,mos + Mg = constant ... (3) slowly, then heat energy transfer through the walls of the
A cylinder will keep the gas at the same temperature as the
This pressure is independent of the temperature of the surrounding liquid. This would be an isothermal
gas or the height of the piston, so it stays constant as long as compression. The reverse process, with this piston slowly
M is unchanged. pulled out, would be isothermal expansion.
If the cylinder is warmed, the gas will expand and push We have the P
the piston up. But the pressure, determined by mass M, will relationship
not change. This process is shown on the p. V diagram of
p = nRr An Isothermal process
Fig. 2.5 (b) as the horizontal line 1 ~ 2. We call this an 2
./ appears on a PV
V
isobaric expansion. An isobaric compression occurs if the gas •• diagram as a hyperbola
is cooled, lowering the piston. Any isobaric process appears
constant
=----
V
.:
on a P - V diagram as a horizontal line.
The inverse P,
P
relationship
between P and V
P2 An isobaric process appears on
causes the graph of
.•.
a PV diagram as a horizontal line
an isothermal
process to be a v, v,
v
, • 2 Ibl
hyperbola.
The process
shown as 1 ~ 2 in
F;g. 2.6 (b)
Increasing
v represents the temperature
v, Ibl
v, isothermal
Fig. 2.5 A constant-pressure (isobaric) process compression shown
Isotherms
in Fig. 2.6 (a).
Constant Temperature Process An isothennal
A constant temperature process is called an expansion would
isothermal process. An isothermal process is one for move in the
which Tf = Ti• Because PV = nRT, a constant temperature opposite direction v
along the 1'1
process in a closed system(constant n) is one for which Fig. 2.6 A Constant-temperature
hyperbola. (isothermal) process.
p/V/ =P;Vi .•• (4)
The location of
is an isothermal process.
the hyperbola depends on the value of T. A
One possible isothermal process is illustrated in Fig. 2.6 lower-temperature process is represented by a hyperbola
(a). A piston being pushed down to compress a gas, but the closer to the origin than a higher temperature process. Fig.
gas cylinder is floating in a large container of liquid that is 2.6 (c) shows four hyperbolas representing the temperatures
held at a constant temperature. If the piston is pushed T1 to T4 where T4 > T3 > T2 > r}. These are called
Push isotherms.
•• Work Done in A Thermodynamic Process
• Fig. 2.7 shows a cylinder with a movable piston. By
•• virtue of its pressure the enclosed gas has moved the piston
• through a small displacement dx.

••

:( ........... ~

•••••••••••••

/
Fig. 2.6 (a)

www.puucho.com
Study Anurag Mishra with www.puucho.com

IHERMODYNAMICS

Area A For an expanding gas (V2>V 1)


The area under the PV curve is
positive (integration direction is
dv=Adx to the right) Thus the environment
p does negaliv~ work on the gas,
.....
,,
o

F =PA ..~'.

o
,
0

,
1 ", ..•.. ::
Fig. 2.7
.
f
o
'" "~.~~
o
o o
o o
o
The infinitesimal work dW is o
o o

,
o

dW =; F dx = (PA) dx = P(A dx)


f o

=p dV
> ~
i o
o
v
The product A dx is change in the volume of the gas.
The work done by a gas when its volume changes from Vi to
Vj is
VI
--~--.
Integration
V2

direction

W=f1pdV
v, .' (.J
For a compressed gi.'ls (V2>Vj)
The gas p~hes on lt1e piston The area is negative because the
p integration direction is 10 the right
with force Fgas outward Thus the environment does positive
To keep the piston in place. work. on the gas.
an external force must be
Pressure P
equal and :OPPOSile 10 Fgas

F,.,

v
, •
o Piston area A Integration direction

(.) (bJ
Fig. 2.9 The work done on a gas is lhe
As the piston moves OX. the negative of Ihe area under the curve.
external force does work Fga.dx
on the gas During a quasi-static process any of the thermodynamic
" variables may be considered to be a function of other, and
the functional relation between them can be depicted as a
\ :o
curve on a two-dimensional graph. A common choice is a
\~'
~:
d,"o
~
F
.~ p - V diagram.
Fig. 2.10 (a) is a graph

'.
, 0 p
,
,
0
0
of P versus V for several
o
: : ,
different T values. The PI

:, :: form of the equation


PV = nRT is a hyperbola.
The volume change by dV=Adx For larger values of PI
as the piston moves dx constant the hyperbolas
(bJ are farther from the origin. v, V,
Fig. 2.8 The external force does work The constant temperature
on the gas as the piston moves Fig. 2.10 (<I)
lines are called
isotherms.

www.puucho.com
Study Anurag Mishra with www.puucho.com

158 THE DYNAMICS

Fig. 2.1O(b) shows a graph of P versus T for several W=JllfpdV


different constant V-values. From the ideal gas equation, v,
P P v Since the pressure is constant in an isobaric process, the
integral becomes

~;~'
V3>V2>V1
v, P,P
J" P2 >Pl
W =Pj
v[
!v, dV =Pi (VI - Vi)
v, P,
v, P, = Pi L\V
T, T T
If the gas expands, V, > Vi' so L\ V > 0 and the wor\l'
V done by the gas is positive. When the gas is compressed,
PV = (nRT) P ~(":)T V~(";)T V, < Vi so that L\V < 0, and the work done by the gas is
negative.
(b) (e) ('j
Fig. 2.10
The work done by the gas is represented on the P-v
diagram by the rectangular area under rhe isobaric path OJ)
P=(":)T the diagram. Whether the area is positive or negative,
depends on whether the gas expands or is compressed.
In the PT-plane this relation is linear with slope nRjV. Work Done by a Gas in an Isochoric Process
For smaller V values, the slope of the P versus T line is large. In an isochoric process, volume of the gas remaim
The lines of constant V are called isochores. constant, it is represented by a vertical line on a p. v
Fig. 2.10(c) shows the graph of V versus T when P is held diagram. Since the volume does not change,
constant. From the ideal gas equation, P For an isochoric process

V=(~)T i
,."
••' the area under the PV
curve is zero. No worK

j
; is done.

A graph of V versus T is a straight line with a slope nRjP.


Small P-values yield steeper slopes. Constant P lines are f
P,
called isobars.
The work done by a gas is the area under the P versus V
curve. There are two important points regarding the ~-~----v
V1=V,
definition of work:
Fig. 2.12
1. The work depends on the path between initial and
final states. Note that area under the P versus V culVe dV = 0
depends on the shape of the culVe.
W=!VlpdV=O
2. When a gas expands, the system does work on the v,
environment; the work done is positive. When the Whether the pressure change is positive or negative. tht:
environment does work on the system, the work done is work done by the gas in an isochoric process is zero.
negative.
Work done by a gas in a process in whicl
Work Done by the Gas in an Isobaric Process PV n = constant, where n is a constant
A horizontal path on P-V diagram is an isobaric process. It can be considered to be a general process.
The work done is found from the equation P
Isotherm for
P For an isobaric process
the area Is PAV. The worK I"m""","" T
/-- done on the gas is PIN

i:'
.
.,
f
P -_ •.•••••• ~ ••••• ,
: .. ,,
,,
,,
..,
:

,,
AV:

...' Pf - •••
,,
r
,
_ ,
v v
v, v,
Fig. 2.11

www.puucho.com
Study Anurag Mishra with www.puucho.com

THERMODYNAMICS 159
pv" = p;v;n = PI V/ = constant Work Done by a Gas Taken Around a Cycle
Pyt PjV; Some thermodynamic systems involve taking a gas
P~-~-- around a closed path on a P-V diagram, so that some useful
V" Vn
work is done around the closed path in each repetition of
W~fVI ,, PdV. tycle. A closed curve on a P-V diagram is called a cycle; the
gas returns to initial point on the P-V diagram irrespective of
where the process begins on the cycle.
P Path 1 P

~ ~
,, '' , '
,,, '
'' :
,,
Wz<OJ:
,,
, '
= p;vt (V)-n _ v/-n) ,
,,
'
'' :, '''
, ' , , '
1- n v
v, v, v, v, v
Pfix!
V Vl-n - p.V" V'-"
"xi'
1- n Fig. 2.15

Positive work is done by the gas in expanding from


volume VI to V2, as the gas expands. Negative work is done
by the gas when compressed from volume V2 back to Vl•
Hence the total work WlO1a1 = WI + W2 is represented by the
area enclosed by the cycle curve on the P-V diagram.
If the cycle is completed in a clockwise manner as shown
Work Done by a Gas in an Isothermal Process in Fig. 2.15, WI"'41 > 0, otherwise for anticlockwise
In an isothermal process both the pressure and volume Wto1al < O.
change along the path. From ideal gas equation, Work Depends on the Path
p = nRr Fig. 2.16 shows two different processes that take a gas
p
V from an initial state i to a final state f. Although the initial
The work done by the gas is and final states are the same, the work done during these
W = IV, P dV VI = Pi p The area under the process A
V, Vi PI ,.'-'curve is larger than the area
T Process A ••• under the process B curve.
~fVf nRT dV v ,,
v, V Vi Vf

= nRrfVf dV Fig. 2.14


v, V
PM
e~-=>ecr.P
RT Process B

~ nRTln (VI ) ~ nRTlJ!l)


Vi '\pf ~ nRT 1J 2)
'~ef
V
Fig. 2.16 The work done during these two
ideal gas processes is not the same.
If Vj > Vi' the gas is expanded and the work done is
.positive. n\lo processes is not the same. The area under the curve of
If Vi < V" the gas is compressed and the work done is process A is larger than the area under the curve of process
oIlcgative. B, so IWAI>IWBI. As this example shows, the work done
The work done by the gas is the area under the path on during an ideal-gas process depends on the path followed
through the P.V diagram.
othe P-V diagram and is positive or negative depending on
whether the gas is expanded or compressed.

www.puucho.com
Study Anurag Mishra with www.puucho.com

THWIODYIIAMICS
,
The path dependence p p
P=aV2
of work has an important
implication for multi step
processes such as the one
shown in Fig. 2.17. The
total work done on the gas Work = Area
during the process y
1 -+ 2 -+ 3 must be Vo = 0.10 m3, VI 0.25 m3 =
calculated as Flg.2E.2
Wlto3 = W\to2 + W2to3' In
this case, WJ102 is positive
and W2103 is negative. The Fig. 2.17 The war\( done during the
Y
, 2
w=fv/av dv=a""""3
(V )'! 3
Vi
process1'--lo2--t3 must be
initial and final states are calculated In two steps
the same, but the work is a 3 3
"-(V! -V)
the same because work depends on the path followed 3 '
through the P-V diagram.
"(~J [(0,25)3 - (0.10)3]" 9,75x 10-3 J

Example 3 V-
Fig. 2£.1 shows Q clockwise circular path of a gas on a P-V
diagram. Volumes and pressures are as shown. What is the A thermodynamic system is taken from initial state A to final
total work done by the gas around one cycle? state C via an intermediate state B, as shown in Fig. 2£.4.
P(alm) p Compute the cotal work done by the gas from A to B to C.
P.Pa

----0
Volume radius

5,00

1.00
T
---~-
,,
2.00
'
:
,,
.
8,00
• V (liter)
L
0- p""",, I
radius
.'y
600

300
A

C~B
:,
,,
'

2.00 5.00
Flg.2E.1
Fig.2E.3
Solution: Area of the cycle on the P-V diagram
represents total work done. Since the path is clockwise in Solution: The work performed by the gas is area
P.V the total work is positive. bounded by the curve and V.axis. From A to B total area is
Area = 1t (pressure radius)(volume radius) the area of triangle ABC, and rectangle below CB.
= rt{2.00x 1.013x 105)(3.00x 10-3) wAH • .!c [(5,00x 1O-3)-(2.00x 10-3)][600-300]
2
= 1.91x 103 J + [(5 x 10-3) - (2 x 10-3 )](300)
We have expressed pressure in pascal and volume in
= 1.35 J
cubic metre to get work in joule.
Work done from B to C is negative because the gas is
being compressed, volume is decreasing. Be is isobaric
process. Thus work done is
Pressure and volume of a gQ.';are related by the law P = aV2, Wac =P~V=PB(VC -VB)
where a = 2 N / m2. The gas expands from a volume of 100 to = (300)[(2.00x 10-3) - (5.00x 10-3)]
250 1. How much work is done by the gas during the
expansion? "-0,90J
The negative work shows that work is done on the
Solution: Fig. 2E.2 shows a graph of the expansion. system. Change in volume, ~V, is always Vj - Vi' The net
The work is work done by the system is
WIOta! =WAB +WBC =1.35J+ (-0.90J)

= 0.45 J

www.puucho.com
Study Anurag Mishra with www.puucho.com

~HERMODYNAMICS

Concepts : Determining work done in some, special P


casesWork done when
A P.V graph for a process in which pressures is ~
a linear function a/volume. The net workdone/or the
process 1 -+ 2 area of the graph between volume axis and the ~
given process as shown shaded in the Fig. 2.18 L v
P Fig. 2.20

P,

P,
._...• ____
.2

4
---; 3
,,

,
,,

:5
P,

P, .--:-~r IWAI<IW,I
So the ne[ work done"" +WA + (-WB) ""negative
Work done in a cyclic process represented as a
circle on P-V diagram
v v,
v, v, v, Such a Fig. 2.21 is an ellipse of P
.!. (P2 -
(-)
Fig. 2.18
(b) semi major axis of
2
PI) and
P, .....
0
semi minor axis of.! (V2 VI).
.,,, ,,,
1 -
2 =- x (Vz - VI )(P2 -PI)
WI ....•
2
+ (V1 - VdPI 2 P, -----Q
Thus the work done, ,
1 v
WI ....•2 = - x (P2 + PI )(V2 - VI) W = Area of ellipse. v, v,
2
= jt x (Semi major axis) Fig. 2.21
In the process shown gas expands therefore work done is
positive. x (semi minor axis)
Work done when = _"::(P2 -Pj)(V2 -V1)
4
A process is performed on the system such that
the volumes at initial and final state 1 and 2 are Work-Energy Theorem
the same. In this process, although. Fig. 2.22 shows a cylindrical container fitted with
(V) final - (V) initial = d V = 0 piston. Cylinder contains an ideal gas. Forces acting on
P
piston are contains a gas. An external force Fex1' spring force,
P2 •••••••• --- ;2
,
3

P, System
boundary
V,
v
Fig. 2.19

J
But the net work done PdV in the process is not be zero
the work is given by, Fig. 2.22
Wntr = W1->3 + W3 ....•
2
= - (Area under the curve 1 ~ 3 enclosed with V-axis) force due to atmospheric pressure (Po), force due to gaseous
+ (Area under the curve 3 --l' '2 enclosed with V-axis) pressure (Pg~5 A) etc. act on the piston. In such situations
processes take place slowly. If the piston moves very slowl};
Note that W1-.3 is negative as volume decreases and W3 ....•
2
then by work energy theorem,
is positive as volume increases.
Forces acting on piston
Work done for a Cyclic Process IW =N<
In the cyclic process shown in the Fig. the work done
(W A) in the cycle A is positive as it is clockwise and the work
done (WB) in the cycle B is negative as it is anti-clockwise. Wgas: Work done by the gas
Since the area of cycle A is smaller i.e. Watm; Work done by atmospheric pressure
Wex1 : Work done by external force
W.pri~g: Work done by spring force

www.puucho.com
Study Anurag Mishra with www.puucho.com

~62 THIRMOD'fNAMICS
Wgr : Work done by gravitational force THE FIRST LAW OF THERMODYNAMICS
Note that when the piston moves Durward, expansion of When a thermodynamic system is taken through a
gas takes place so its volume increases, say by 6.V , then the process in which heat Q is added to the system and wotk W is
volume of the air atmosphere will decrease by the same performed by the system, irrespective of the nature of the
amount but with opposite sign, i.e., the change in ,the process, the value of Q - W is same for same initial and final
volume of air is negative.1.V (volume decreases by .1.V). states, which means Q - W depends only on the initial and
final states. This new variable Q - W is the change in
HEAT TRANSFER TO A SYSTEM
internal energy between initial and final states.
Heat CQ)is the thermal energy transferred via particle
t:.U=UJ -Uj=Q-W
collisions, from a region of higher temperature to a region of
lower temperature. A body stores or contains thennal This equation is called first law of thermodynamics.
energy, not heat. Heat is thermal energy in transit and once Heat Q is negative when heat energy leaves the system and
transferred it is no longer called heat. that work is negative when work energy enters the system;
therefore, Q - W represents the net energy entering the
system.
For an infinitesimal changes in the internal energy, heat
transferred and work performed,
(.) (bl
dU =dQ -dW. dQ =dU + dW

the bar on the differential d is to emphasize that they are


not exact differentials of any function but merelv represent
«) (d)
sma:!l amounts of heat and work. .
FIg. 2.23 The first law of thermodynamics is a statement of
Heat and work (Q and W ) are the two ways of bringing conservation of energy for pure thermodynamic system. The
energy into or taking it out of a system. Heat and work can very fact that the internal energyU is a state variable implies
produce identical results. Heat transferred into a system can that energy is conserved. Suppose a system is isolated so
increase its temperature, as when the Sun warms the air in a that it cannot interact with its surroundings, i. e., Q = 0. and
bicycle tyre; similarly when work is done on the air by W = O. It implies that t:.U = 0. or that the internal energy of
pumping it into the tyre, the temperature rises. an isolated system is constant.
The quantities that depend only on the state of the
system are called state variables or state functions or point Concept: An isolated system has no interaction with its
environment.
functions. On the other hand, both work and the amount of
heat transferred depend on the particular path of the The opposite of isolation is thennal contact:
process; we call them path variables. A system that can exchange thermal energy with its
Internal Energy environment is in thermal contact with environment.
In thermodynamics, as in mechanics, an equilibrium
The internal energy is the p state of an isolated system is one that has no tendency to
sum of the individual kinetic and • change.
potential energies associated ;~,
with the mmions and
interactions of all the individual
j~ ... ..
.
Two systems are in thermal equilibrium with each other
if there is no net transfer of thermal energy when they are put
particles comprising the system. .. .. in thermal contact.
...
...
In a particular thermodynamic
The zeroth law of. thermodynamics:
state a system has a definite ... Two systems each in thermal equilibrium with a third are
internal energy. When the v in thermal equilibrium with each other.
system changes its state, it is Flg._2_.2~4
__ A system unable to transfer energy to any other system in
only changes in the internal energy t:.U that are physically thennal contact with it has the lowest possible temperature,
significant. called absolute zero.
When energy is transferred from one system to another as
The value of internal energy is a characteristic of the
state of the system; it is a state variable. a result of the temperature difference between them the
process is called heat transfer.
Heat means the amount of energy that is transferred
through a large number of random events.

www.puucho.com
Study Anurag Mishra with www.puucho.com

IHERMODYNAMICS 163

Heat transfer is a process and does not involve any new .\U =Q
form of energy. Heat transfer may increase the internal For a constant volume process,
kinetic energy of a system, for example. Second, we are using
fiU=nC,.(1'j -Ti)=nC,.fiT
"heat" as a description of the tramJer process. Systems do not
contain heat any more than they contQin work .systems fo.U represents change in a srate variable Qnd therefore it is
contain internal energy that may be changed either by heat independent of path from T,. to Tf . Hence llU must always
transfer or by doing work. be equal to nC"IlT irrespective of the process. We
11le first law of themodynamics expresses conservation can calculate the change in internal energy for any process
of energy during any process. between temperature 1',. and Tf by finding the change in
The increase in a system's energy equals the heat energy for an isochoric process between the same two
transferred to the system plus the work done on the system. temperatures.
Equation suggests that the temperature of a system is
related to the internal energy of the system; t:.U is related to
Specific Thermodynamic Processes
temperature change. Writing the equation in differential
In this section we will analyze the changes in internal form,
energy, the amount of heat transferred, and the work
dU = nCt, dT
performed by a system during some specific processes.
The molar specific hear was defined earlier by the c =.!.dU
t' n dT
equation
For an ideal gas, when temperature is constant, internal
Q = nCfiT
energy is constant, i. e., isotherms of an ideal gas are
where 11 is the number of moles and fiT is temperature constant internal energy curves. These statements are
difference. The heat Q transferred into or out of the system is true for real gases only when they behave as ideal gases, at
dependent on the particular process; therefore, the equation low pressures and far from me vicinity of phase transitions.
Q = nCfiT depends on the path. The magnitude of Q may be During phase changes internal energy can depend on the
different for different paths between the same two pressure and volume per mole of the gas, and the relation
temperatures. But n and fiT remains same. So the molar Q =: nC".11' is not applicable.
specific heat C is path dependent.
Isobaric Process
During an isochoric (constant volume) process the
Work performed during an isobaric process is
amount of heat transferred is
Q == nCL.t:.T
where C is the molar specific heat at constant
I'
W == J:
,
P dV =,P(Vf -Vi) =PllV

volume. For this case first law of thermodynamics can be written


as
During an isobaric (constant pressure) process the
amount of heat transferred is 6U =Q-W
nC"t:.T =' nC p llT -PfiV
Q=nCpt:.T
=: nC p llT - n.RliT
where C p is the molar specific heat at constant
pressure. Ct. =Cp-R

During an isothermal process, fiT = 0 or Cp - C,. = R (ideal gas only)


Q Equation says that the specific heat at constant pressure
C=--=x
TlIlT is greater than the specific heat at constant volume. We can
Specific heat of the system is infinite. explain this result from the definition of specific heats. We
defined molar heat capacity as amount of heat necessary to
During an adiabatic process, llQ = O.Specific heat of the
raise the temperature of one mole of a substance by one
system is zero.
temperature degree. During a constant pressure process,
Isochoric Process some of the heat transferred is used to perform work, as the
There is no change in volume of the gas in an isochoric volume changes, whereas during a constant volume process
process so that dV == 0 and the work done by the system is no work is performed; the entire heat contributes to the
zero. temperature increase (change in internal energy). Thus we
W=JPdV=O see that the extra heat is required in [he constant pressure
process to perform work.
The first law of thermodynamics reduces to

www.puucho.com
-
Study Anurag Mishra with www.puucho.com

Isothermal Process In (yY ) + In (P) = In (constant)


Work done in an isothermal process is given by In (PyY) = In (constant)

W = nRT In (~ ) (ideal gas only) , or pyY = constant ... (5)

" The eqn. (5) is the mathematical relation for the curve
In the isothermal process, in P.V plane for an adiabatic process. Relations describing
t.U=nCvl1T=:O adiabatic behaviour of an ideal gas in terms of Y and T, and
So from fjrst law we find Q =W
in terms of P and T can be derived by combining eqn. (5)
with the ideal gas equation PY = nRT, to obtain the
Q =nRT In(~) following results:
PyY = constant
Adiabatic Process
IVy -} = constant
An adiabatic process is performed in a well insulated
container so that no heat flows across the container pO-1)!YT = constant
boundary. When a process is performed suddenly, there is A family of curves representing quasi-static adiabatic
little time for heat transfer. We can approximate it as an process may be plotted on a P-Y diagram by assigning
adiabatic process. different values to the constant in eqn. (5). The slope of any
As Q = 0, the first law becomes adiabatic curve is determined by taking the differential of
eqn. (5).
flU =-W or W =-/1U
W=-nCv6T ...
(1) Pd{YY) + VYdP = 0
In the differential form, or PyyY-1dY+YY dP=O
dU=-dW=-PdV dP P
Then -=-y-
nC1, AT = -P dV ... (2) dv V
We eliminate T from this equation with the help of the Slope of an isothermal curve is
ideal gas equation PV = nRT.
dP P
d(PV) = d(nRT) -=--
dV V
PdV + VdP =nRdT
It follows that an adiabatic curve has a steeper negative
n dT = P dV + V dP ... (3) slope than for an isothermal curve at the same point.
R
Work done by the gas in an adiabatic process can be
Substituting expression for n dT in ego (2), calculated from the first law of thermodynamics. Since
CuCF dV + V dP) =_P dV dQ =0,
R
Rearranging this resu!t, we have
(Cv+R)dV+ dP=O P, Adiabat P'/I '" Constant
C ... (4)
V " P )
ButCv +R =CP' So eqn. (4) can be written as
P, _.•••••••• -flsotherms
C,)dV +dP =0
( Ct. V P
v, V,
v
We integrate this expression and write the integration Fig. 2.25
constant on the right hand side as natural logarithm of
another cons tan(.
dW ==-dU=-nC" dT
(~:) In(V) + In (P) = In (constant) Wadiabafic = J dW = J -nC" dT

C = - nC., ,~.T= nC f' (Tj - Tf)


The ratio -!!.. , ratio of specific heats, is customarily We have assumed that Ct' is constant, for an ideal gas, U
C"
written as the Greek letter gamma, so that C p/Cv = y. With is proponional to temperature, and therefore, Ct. = du is a
dT
this substitution. we have constant.

www.puucho.com
Study Anurag Mishra with www.puucho.com

THERMOOYHAMICS 165

Using PV '" nRT, 'we obtain Adiabatic Free Expansion


We now consider what
W"di"h"lic =nC,. (PIV1 _P2V2J happens \•... hen a gas is
nR nR
allowed to expand
=~ (PIVi -PIV/) adiabatically without doing
R
any work. Fig. 2.27 shows
c, t\NO vessels connected by a Fig. 2.27
=
Cr -C" tube with a stopcock.
Initially, one vessel is filled with gas while the other is
where we have used R = C p - C I'. Dividing numerator evacuated. The system is thermally insulated; that is, Q ::=0.
and denominator bye and writing '{forC piC,., we obtain
I'
When the stopcock is opened the gas quickly expands to fill
PIV, -Pj Vi the second chamber. The uncontrolled expansion is not
lVadiabatic '" 1
y- quasi-static and cannot be depicted on a PV diagram. Since
the gas does not expand against a piston, it does no work,
Concept: 1. Isothennal (constant-temperature), W ::=O.From the first law we conclude that
iso-chone (constant-volume), and isobaric (Free expansion) ilU ::=0
(constant-pressure) processes require heat transfer however,
heat transfer is relatively slow and is negligible in many In an adiabatic free expansion the internal energy of
processes that occur rapidly. Examples of such processes any gas (ideal or real) does not change.
include pressure oscillations in a sound wave and the In the special case of an ideal gas, there is no change in
expansion phase in the operation of an automobile engine. temperature in an adiabatic free expansion. One can
In an adiabatic process, a system does not exchange heat conclude that the imernal energy of an ideal gas depends only
with its environment: on temperature, not on pressure or volume. Sensitive
experiments show a small temperature change for a real gas
Adiabatic proees.s c:> Q = Q
at high pressures and low temperatures. This indicates that
An adiabat is the path/allowed by the system paim in the the internal energy of a real gas is also a function of pressure
p-v diagram during an adiabatic process. or volume.
2. In an adiabatic expansion, the system does work at the Table 2.2 Summary of Thermodynamic Processes
expense of its internal energy: ~U ::= -W. The adiabat drops to
Process and general result Result for ideal gas
lower isotherms as tile internal energy (and hence
temperature) decrease. Adiabar.s are steeper than isotherms. bochoric

P W =0 (True also for nonisochoric


ilU::= nCv /1T process)
Q=W
Isobaric
T, + dT
W '= PilV
ilU ::=nCl, ilT::=Q - W
P, Q::= nC p /1T
dV Isothermal
w = J P dV W =Q = nRT In(~o)
Q::=ilU+W
v, V
ilU =0
Fig. 2.26: A differential piece of a P-Vdiagram
showing two neighboring isotherms and an Adiabatic
adiabal crossing them.
W :-/1U ::=-nCv I1T PVY::=constant(y::=Cp/C,.)
When the gas is compressed along an adiabat, its pressure Q-O
rises from PI to PI + dP. Work is done on the gas but no heat
transfer occurs. According to the first law of thermodynamics,
the internal energy and temperatu,e of the gas increase. Thus
point E is on a higher isotherm than A Adiabats are steeper
than isotherms.

www.puucho.com
Study Anurag Mishra with www.puucho.com

mJ~6!!6=:;::;:=:
Graphs: p.v, V.T and poT v
1. Isochorlc process:
P
P,
---
]8
-------- --- v
v,

v, •••... /
8

....
,-'
...
:
A,
.
••

.,B
.' . T

Pl __._ •• A
•• ...


Fig. 2.29 (e)

v, v T dV nR
(e) slope=- =-

-----
I') (b) dT Po
Fig. 2.28
3. Isothermal process:
. F F p p
V = constant therefore ..1. = ...l. and Pl < P2
T1 T2 A

dF dV
(a) slope = - = 00 (b) slope=-.0
dV dT
P
~8 f
A
P, ••.... ; ..• ~ 8
v T
P1 .~ i (.) (b)

... ...
," :
.'
Fig. 2.30

1
To = constant; P:x: -
11 T2 T V
FIg. 2.28 (c) To = constant
dF dF
dF nR (a) slope = -; PV = nRT (b) slope =- =00
(c) slope =- =- dV dT
dT V
=> PdV + VelP = a
2. Isobaric process: dF p
p p
;> -=--
dV V
v
A
8 A 8

I
Po m •••••• :

v 8
(.) Ib)
Fig. 2.29 T
Fig. 2.30 (e)

P = constant,
v V
therefore...!. = -l. and V2 > VI dV
T1 T2 ee) slope = - = 00
dT
dF dF
(a) slope=-=O (b) slope::: - = 0
dV dT

www.puucho.com
Study Anurag Mishra with www.puucho.com

THERMODYNAMICS

Adiabatic Graph CONVERSION OF GRAPHS


P Example I.

P, ~ P
P P ,,
B A

P, ---L------
(.J
,
,A
,,
v A/
(OJ
,,
,
T
c
B

V
....
... c
.Ll T

::::~B
V,

'-.:>11
A

V
v, ,, ," ,;, ....

..
,
T
Tt T2
T
('J
Fig. 2.31 Fig. 2.33

StutcPl,Vl,Tl-P2,V2,T2; Suppose?! <P2 The sense of the cyclic process in P-J" diagram is always
(a) PV"r = K(const.) ~ P = K/VY opposite to the.sense of the cyclic process in V.7' diagram.
slope can be determined as Example II.

P'"lVHdV + Via? = 0 ~ elP = _ yP


, dV V

(b) pl-YrY = K:::>p~Y_


T1-1

(c) nw1 =K
Comparison of an Isotherm and Adiabat
Slope of adiabatic curve =- yP
V

Slope of isothermal curve =_f


V
Adiabatic slope = P
(isothermal curve)
In the P-V graph if t\\'o ,,
----~
,
,,
intersecting curves are shown
then both are not adiabatic and : isotherm
both are not isothermal because : adiabat
t\vo isothermal curve can not L---'-----_V
intersect and two adiabatic curve Fig. 2.32
can not intersect also.

www.puucho.com
Study Anurag Mishra with www.puucho.com

168
Example III. Example V.
p p p p
A

1'\,'"
C~" 17"
"
L-----~V T .,,' D
V T
v V

v
C

"A
,c, ,
" ,,
T "
T
Fig. 2.35 Fig. 2.37

Note: On a P-V diagram we have a cui point then poT and V-T Example VI.
diagram we will also have a cut point.

Iv.

'i) '{J'
Example
p , p
A
\A

,:,' T L
v

V
.,,::~: T
V D C

ED"
V
D

.: ••••• A
C
" /.,-
T

A
T Important points
------' Fig. 2.38

Fig. 2.36 (1) Temperature Analysis on P-V diagram: In the


diagram three process shown on the P.V diagram. Here
temperature will increase because we are moving from a
lower isotherm to a higher isotherm.

www.puucho.com
Study Anurag Mishra with www.puucho.com

IHIRMO ICS 169

p . . .. General Conclusions
If the same gas is expanded p
from the same pressure and
volume to the same final volume
,,
then the work-done by the gas in
2 : isotherm
an isothermal process is greater : :
T. than the work-done in an ~
:, :. adiabal
T, ,,
adiabatic process.
T, v
T, If the same sample of gas is V, V2
v expanded from the same initial Fig. 2.43
Fig. 2.39 state to the same final volume by
(2) Work done in adiabatic process 3 ~ 4 = work done three different process then the work done by the gas has
in the adiabatic process 1 -) 2, (In adiabatic process work following inequality
done is equal to the change in internal energy.)
p p
3

di
mono
V1 V2 V
2 Fig. 2.44

Fig. 2.40 In me case of the comprcssion the incquality is revcrse.


Win >Wdi >Wmono
(3) Compare the heat supplied sand
for the process A, Band C. /'

QA =6U+W"
QB = 6U

=>
Qc =ilU-W"
QA >Q" >Qc
Im~~ol[I] 1 I H~O

Molar heat capacity of A is Fig. 2.45


maximum.
(4) The plot a,b, ande represent adiabatic process for The sand particle are remove very slowly and the gas
H2, He and H20 identify them: expand adiabatically. In the case of mono atomic gas sample.
The final pressure will be minimum and the removal of sand
p
will be maximum.
The final temperature 'A'ill be minimum in the case of
monoatomic gas sample.
First Law of Thermodynamics In differential form
, dQ::= dU + dW ; dU::= IlC1,dT ; dW ::=PdV
b dQ = nC"dT + PdV ;
, for a differential time dt
dQ dU dW
_=_+_ ...
(1)
V
Fig. 2.42
dt dt dt
dQ --+ rate at which heat is supplied i.e., Thermal
a = H2O (trO de
b = H2 (di) power supplied.
c = He (mono) -dU --+ rate 0f.mcrease 0 f.Imerna I energy.
de

www.puucho.com
Study Anurag Mishra with www.puucho.com

~0 --~T"'HE;llRMOIln"IiDYNAM==Ill':CSrl
.-;;:==:;;:-::-:===--\----~R~e~la~t?,o=n:-
t Connecting process \
-dW ~ me ch'lamea power output.
dt
Isothermal PV = constant
For a dT change in temperature
dQ dU dW Isochoric P
-=-+- ,.... (2)
- = constant
dT dT dT T

How to Determine Specific Heat of a Process


'." Isobaric V
- = constant
To find the molar specific heat of any process in general, T
let C proceS!J be the molar specific heat for the given process Adiabatic PV Y = constant
taking one mole of gas. The amount of heat dQ supplied to
TV 1- I = constant
increase its temperature by dT is
pO-Y)i1T =constant
dQ = C proc~""dT
dQ dU C. Use Table-J to calculate ~U,Q and W for each path.
dT =Cproces,; dU =Cl,dT; dT =Cv• dW =PdV.
dW PdV Example
--=--
dT dT
from eqn. (2) ~gm; cylinder hm; O.6mole of an ideal gm;. It is taken through
the process A-B-C as shown in Fig. 2E.4. What is the work
PdV
Cprocess =Cv +-- perfonned by the gas, the heat framferred. and the change in
dT internal energy for each of the processes A ~ Band B ~ C.
Problem.Solving Strategy for Thermodynamic Specific heats at comtant volume and constant pressure of the
Cycles gas are 12.5 JLmol!K and 20.8 J/moVK respectively.
A thermodynamic cycle may involve unknown
Solution: Process A ~ B is isobaric, and B ~ C is
parameters. The following general strategy may help in
isothermal. We construct a table and fill up flU = 0 for
determining them:
B~C, because there is no change in internal energy along
I. Sketch the process in the P.V plane. Make a table of . heern 0f'd
a n Isot an leal I •••.
as.
t.V, Q and W for each segment of the cycle.
i' Process .u Q w
A. Fill in the given informations in the appropriate box
of the table. A-->B
B. Apply first law,.1.U =Q - W for each segment. B-->C 0
C. At the end of a complete cycle all the state variables
return to their original values, including the internal energy. Total
= =
For a complete cycle, 6U cycle Q Therefore, Q - W 0, i. e.,
Process A ~ B is
work obtained from a reversible cyclic process is equal to the P, Pa
heat transferred into the system. We reemphasize the fact isobaric, so the work done Isotherms
Is
that irrespective of the process performed bet'Neen t'NO
v,
states with temperatures Ti and Tf, the change in internal
energy is
WJ\B = f v, Pdv 100

6U =nCv(Tf -Ti) = P(Vf - Vi)


If all the boxes are not filled by using I, apply the = (100)(2 -1)
strategy in II below: = 100 J V,m3
V, '" V2 = V3
II. Determine the temperature, pressure and volume at The change in 123
each vertex of the cycle. internal energy from A to
Flg.2E.4
A. If two state variables are known, the equation of B is
state can be used to find the third. tJ.U = nCv~T
B. If one variable is known at a given vertex, the second = (0.06)(12.5)(400 - 200)
can be determined by relating the values at neighbouring
vertex by knowing the process connecting the two vertex, = ISO J
The heat transferred can be computed in two ways.
1. Q = nG pliT
= (0.06)(20.8)(400 - 200)
=250J

www.puucho.com
Study Anurag Mishra with www.puucho.com

THERMODYNAMICS 171

2. From first law, Q == E,U + W Example


== (150 + 100) J
~2S0J A cylinder witll a movable piston encloses one mole of helium.
Process B --) C is isothermal; so the work done is The cylinder is placed in contact with various reservoirs and
insulated when required; the helium peiform.~ the cycle as
wBe ~Jv1--v lpdV
V,=V2
shown in Fig. 2£.5. Compute the internal energy change, hear
transferred and work peifonnedfor each segment of the cycle
Vf = v3dV
=nRT J Vi" V2
-
V
and the toral amount of each of these quantities for the entire
cycle. Assume helium to be an ideal gas, with Ct, = 12.5
j/moVK, C p = 20.81/moVK.
~ nRTln[ ~: J
Solution: First we calculate temperatures
~ (0.06)(8.314)(400) In(~) TA =TB, Te, TD• the volume VB' and the pressure PRo We can
determine temperatures from the ideal gas equation
:: 80.9 J T = PAVA =(8.00)( 104)(2.00)( 10-3)
A nR (1)(8.314)
Since 6U = a for an isothermal process,
QBC == Wile == 80.9 J = 19.2 K

We can now complete our table.


We use the law for adiabatic process PV"' , for the process
B --) C to compute pressure and volume at point B.
PeVJ=PIJVJ ... (1)

For helium, we have


'{ == Cp =(20.8)= 1.66 TA"'Ts"'19.2K (aBC" 0)
C,. 12.5 (.J 10J
However, we have two unknowns, both Pfl and VB' so we
need another equation, which is
PBVB =nRTB ... (2) r
Eliminating Pjj from eqos. (1) and (2), we have
PeVZ =PBV; =PSVB(VB r-1 ~P '" 0

=nRTB(vB)r-1 Po <0
On solving for VB' we have (01 (dJ

VB = (PcVZ )1'(}-1) Flg.2E.5

nRr,
p. Pa

.".jmfi
On substituting numerical values, we have
VB == 4.55 x 10-3 m 3

Again from eqo. (2), Isochor B


nRTiJ <4 Adiabat
PB = -- = 3.51x 10 Pa 3" 104 ----- C
V, D: Isobar :

Now, the pressure, volume and temperature at each L_-'-__ -'-__ .'V.1o-3m3
2.00 5.00
vertex of the cycle are known. Now we can calculate heat,
work, and internal energy: Fig. 2E.5 (e)
Process W Q \ w
PcVc (3)( 104)(5)( 10-3)
A ~B 150 J 250J 100 J Tc=--~
nR (1)(8.314)
B~C 0 80.9 J 80.9 J = 18.0 K
Total 150 J 331 J 181 J

www.puucho.com
Study Anurag Mishra with www.puucho.com

172 THERMODYNAMICS
PVVD (3):: 104)(2)< 10-3) which 57 J of heat is converted to work and the remaining is
To =--
nR (1)(8.314) transferred to the low temperature reservoir.
,7.22 K ',' Ie..
for each segment of cycle. :;!..
Isotherm A --+ B Consider the vaporiuztion oflg o/water at 100'C to steam at
l00"C at one aonosphere pressure. Compute the work done by
WAS=nRTAln(~:) the water system in the vaporization and change in internal
energy:0/ the system.
3 ...
, (1)(8.314)(19.2) In(4.55X 10- J Solution: To change a system of mass m of liquid to
2x 10-3
vapour, heat required is
= 132 J Q =mL,.
Along an isotherm, tJ.UAS = 0, so QAB = W AB = 132 J. The process takes place at constant pressure, and so the
Adiabat B -+ C work done by [he system is the work in an isobaric process.
We know that W =P.1.V
Wac =-liUBC =-nCutJ.T where .1.V = (Vvapour - V1iquid )
, -(1)(12.5)(18.0 - 19.2) This work is positive since V,,,pour > Vliquid
From first law of thermodynamics
,15.0 J
aU'Q - W
and ""Vae =-Wac =-15.0J
= InIll - P(Vvapour - VliqUid)
The work done by the gas in this segment of the cycle
Latent heat of vaporization of water
decreases internal energy; work done is at the expense of
internal energy, as QBC = O. Lv = 22.57 x lOs J/kg
Isobar C -to D Q = (1.00 x 1O-3)(22.57x 105)
WQ) =P6,V= (3.00x 104)[(2.00x 10-3) =2.26x 103 J

- (5.00x 10-3)]=-90.0 J Moles of water in 1 g = ~ = 0,0556 mole


18
Qev = nC paT = (1)(20.8)(7.2- 18.0)=-225 J
V , nRT, (0.0556)(8.315)(373)
Wev 'Qev -Wev = (-225J)-(-90J),-135J vapour P 1.013 x lOS
We may apply I:1U = nC u AT for calculation of change in
=1.70x 10-3 m3
internal energy.
[sochor D -+ A The density of water is lOa x 103 kglm 3 = 1.00 glcm 3

For a constant volume process, W = 0, W VA = a Vliquid = 1.00x 10-6 m 3

tlUVA=QVA =nCvtJ.TVA =(1)(12.5)(19.2-7.2)= 150J Thus the work done by the water system in vaporization
We can summarize our results in the table form is
I Process .U Q \ w I W = P(Vvapour - Vliquid)
A ->8 0 132 J 132 J = (1.013x 105)(1.70)( 10-3 - 1.00x 10-6)
B->C -15 J 0 15 J =172J
C->D 135 J 225 J 90 J The work done by the system is positive since the
D->A 150 J 150 J 0 volume of the system has increased.
Total 0 57 J 57 J From first law,
It is a cyclic process. Therefore, ~U'Q -W
.1.U = (2.26)( 103 - 172) J
AUcyclic =0
and Qcycle = W cycle = 2.09x 103 J
During each complete cycle 132 J + 150 J = 282 J of Note that larger percentage of energy 92% is utilized in
heat is removed from high temperature reservoir, out of changing the internal energy and only 8% went into work
done by the system on surroundings. In vaporization, the

www.puucho.com
Study Anurag Mishra with www.puucho.com

='"T~HnERM=ODl:lYNillAMftlI"l(P1S---

energy is required to weaken the intermolecular forces (c) Find the expression for work done in the cycle in terms of
within the liquid. minimum and maximum volumes VI' V3, TH• TL, Pl, VI'

Example 7 ........-- Solution: (a) (i) Isothermal expansion,


1-» 2: The system is immersed in a bath at temperature
Consider the melting of 1 g of ice at DOC to liquid water at.DOC TH and is allowed to expand quasi-statically doing \\'ork on
at atmospheric pressure. Compute the work done, on ice an external mechanism as heat flows into the system from
system in melting and change in internal energy a/the system. the bath.
p
Solution: Heat required to change the phase of solid, ,,
Q = mLf ,
,, Isothermal
,,
Work done by system at constant pressure, ,,
,
W = P(VIi'luid - Vso:id )
From first law of thermodynamics,
4
~u=Q - W Adiabatic
,"
'-" .lsothermT H

= rnL f - Pevliquid - Vsolid) QL 3.'.-lsothermTL


Latent heat of fusion of water, V

Lf = 3.335x lOs J/Kg Fig.2E.8

Q = (l x 10-3 )(3.335 x 105) The work done by an ideal gas in an isothermal


expansion,
=334J
V,
The density of ice is 920 kg/m 3. WI == nRTH In- ... (I)
V,
3
V 1 x 10- = 1.09><10-6 m3 Since the temperature is constant in an isothermal
solid = 920
process, the internal energy is constant. nUl = 0 J. From
6
VUquid=lx 10- m] first law of thermodynamics,
Thus work done by the system in melting is V,
QH =LlU+W =OJ+WI ==nRTH In - ... (2)
V,
W = P(\t)iquid - Vso1id)
= (1.013>< 105)(1 x 10-6 - 1.09><10--6) It shows that all of the heat transfer QH to the gas is
converted into the work W1•
=-9><10-3)
(ii) Adiabatic expansion, 2 -» 3: The system is
Work done by the system is negative because the system enclosed in a perfect insulator, the quasi-static expansion
decreased in volume. follows until the temperature has decreased to TL•
From first law, Since Q = 0 J, the first law of thermodynamics
~U =Q - W = 334-(-9,10-') Q = 6U + W becomes
OJ=6U2 +W2
-= 334 J
LlU2 =-W2 =nCu(Tr -TH) ... (3)
Nearly all the heat transferred to the system in the
melting goes into changing the internal energy of the Since work done of expansion is positive, the change in
system. The absolute value of work done by the system is internal energy is negative, the internal energy decreases.
only about 0.003% of the heat transfer to melt the ice. Since internal energy is a function of temperature, the
temperature of the gas decreases to Tt.
Example 8:.--'" (iii) Isothermal compression, 3 -» 4: The
system is immersed in a bath at temperature TL and
A Carnot cycle takes an ideal ga.>around a cycle consisting of compressed quasi-statically by an external agent as heat
alternating isothermal and adiabatic processes, as shown in flows out of the system to the bath.
Fig. 2£.8. (a) Calculate work done, heat transfer, internal The process is isothermal, I\U3 = 0 and
energy change in each segment of the cycle. (b) Show that the
cylinder volumes in the stares 1, 2, 3, 4 bear the relation
VI V2
W 3 =nRTL In(~:) ...(4)
-=-
V4 V3

www.puucho.com
Study Anurag Mishra with www.puucho.com

rt7

The work done is negative because V4 < V3 (the temperature and TI = T2 = TH and at the minimum
logarithm of a number less than 1 is negative). The first law temperature T3 = T4 = TL.
of thermodynamics, Q = 6U + W becomes WlOtal = WI + W3

QL =OJ+W] =nRTL In(~:) .;:,.. (5)


V2 V4
=nRTH In- + nRTL 10-
VI V3
Since work done is negative, QL is also negative, Vt V2
=nRTH In- + nRTL In-
implying that heat is transferred from the gas (system) to V2 VI
the bath at temperature Tr.
V V2
(iv) Adiabatic compression, 4 -t 1: Tne system Using eqn. (11), - 3 0_

V4 Vt
is enclosed in a perfect insulator, the quasi-static
compression continues until the temperature of the system V,
Wlotal = -nR(TH - TL) In- ... (12)
has increased to TH' VI
The process is adiabatic, Q = 0; the first law becomes From ideal gas equation,
.'
OJ=.6.U4 +W4 nR=PtV1
.6.U":-W4 =nCv(TH -TLl ... (6) TH
Since W" is negative (compression work), tJ.U4 is For adiabatic process, 2 -)0 3,
positive, indicating that temperature of the gas increases.
For complete cycle, the gas returns to its initial state; the
T
Vz = --l::....
( TH
)1'('-" V3
total change in internal energy of the gas is zero. i. e.,
L\Utotal =L\Ut +L\U2 +L\U3 +L\U4 By substituting these values in eqn. (12), and doing
some algebraic manipulation, we obtain
=OJ+L\U2+OJ+L\U4
001 ... (7) Wtotal = Pt \'t (1- TL ) (_1_ In !i. + In V 3)

The two internal energy changes L\U 2 and !:tV 4 during l TH lY-l TH VI
two adiabatic processes of the cycle are of equal magnitude
and opposite sign, see expressions (3) and (6). The internal
energy is a function of temperature alone. In one adiabatic
process temperature decreased from TH to TL and in the 'Fig. 2£.9 shows a cyclic process perfonned on one mole of an
other temperature increased from TL to TH• ideal gas. A total of1000) of heat is withdrawn from the gas
in a complete cycle. Find the work done by the gas during the
The first law applied to entire cycle is therefore, process B -)0 C. •
QtOlal = Wtotal + L\V tOlal T
QH +QL =W1 +W2 +W3 +W4 +OJ C

Taking sign of heat transferred and work, we get


QH-QL,
Net heat tiansk""d
=Wt+WZ-W3-W4
Total work done by
400 K

300K
[>.
•• ,' A
to the gas the gas "
" V
(b) For the adiabatic process 2 -)0 3,
Flg.2E.9
THV2y-t =TLV3y-t ... (9)

For the adiabatic process 4 -)0 1, Solution: In a cyclic process,


THVly-1 =TLV/-I ... (10) L\U=O
Dividing eqn. (9) by (10), we have Qey<;le = W cycle .•• (l)
V2 V3 whereas Wcyde = WA ...• B + WB ...• e + We....•
A ... (2)
-0-

VI V4 Process C -)0 A is isochoric, hence We ...•A = 0


VI V2 Process A -)0B is isobaric, as its T-V graph is a straight
0' -0- ••• (11)
V4 V3 line passing through origin.
(c) The net work done in adiabatic parts of the cycle is In an isobaric process, work done is
zero. It follows that all the work done in the cycle is W =P(V2 -Vt)
isothermal work, performed at maximum system
=nR(Tz-T1)

www.puucho.com
Study Anurag Mishra with www.puucho.com

- THERMODYNAMICS 175
.,
WA->B =lx 8.31(400-300)
= 831 J ... (3)
= 2(~R) (600- 300)

W A -./l is positive; expansion of gas takes place. = ISOOR
From eqns. 0), (2) and (3), Heat is added to the system.
-1000 = 831 + W/l_.e + a Process B -+ C is isothermaL
Wu ...•c = -1831 J From first law of thermodynamics,
Negative work dO:le implies that compression of the gas QBC =WBC' as I:1U =0
takes place.

Example
.
= nRTe In(~:)
= (2)(R)(600) In(4V2V A)

Two moles of a monatomic ideal gas is taken through a cyclic A


process starting from A as shown in Fig. 2£.10. The volume = (l200R) In (2)
ratios are VB = 2 and Vv = 4. If the temperature TA at point = (1200R)(0.693)
VA VA
A is 27"C, calculate: ~ 83L6R
Heat is added to the system.
v
Process C -+ D is isochoric.
v, ----ODe From first law of thermodynamics,
Vs •••• B QCD = nC,.(TD -Tc)
A '
VA ---~
...•..
i, =n(%R )(TA -T,)
TA Ts
T

Flg.2E.10 =2(%R )(300-600)

(a) the temperature of the gas at point B, = -900R


(b) heat absorbed or released by the gas in each process, Negative sign implies heat is rejected by the system.
(c)the total work done by the gas during the complete cycle. Process D -+ A is isothermal.
Express your answer in terms of the gas' constant R. QDA =WDA
Solution: Gas is monatomic; hence
3
C =-R and C =-R
5
= nRT, In(~; )
u 2 p 2
= (2)(R)300 In ( VA )
Number of moles n=2 4VA
TA =ZrC=300K
(a) Process A -+ B is a straight line passing through =600R In(~)
origin; hence it is an isobaric process.
= -83L6R
VA VB
-=-
T T
Negative sign implies heat is rejected by the system.
A B
(c) In a cyclic process,
Ts =(~:)TA ilU=O

=2x 300K=600K
(b) Process A -+ B is isobaric. Wne1 = QAB + QBC + QCD + QOA
Qp =nCp(TB -rAJ = ISOOR + 83l.6R - 900R - 831.6R
= 600R.

www.puucho.com
Study Anurag Mishra with www.puucho.com

176
The pf;fSSures and temperatures at A, B, etc_, are denoted by
PA, TA, PB, TH, etc., respectively. Given TA = 1000 K,
An ideal gas having initial pressure P, voll1~e, V and PB = (2/3)PA and Pc = (lj3)Pk Calculate (a) the work done
temperature T is allowed to expand adiabatically until its by the gas in the process A -) B, (b) the heat lost by the gas in
volume becomes 5.66 V while its temperature/ails to Tj2 (a) the process B -} C, and (c) temperature TD-
How many degrees of freedom do the gas molecules have? (b-'. Gil'en (2/3)(2'5) = 0.850_
Obtain the work done by the gas during expansion as a
function of initial pressure and volume. Solution: (a) For an adiabatic process,
Tr pl- r = constant
Solution: (a) For an adiabatic process, temperature Trp1-r _Trpr-1
and volume are related as AA -BB

Vr-1

r
TV y-1 -T where y = 5/3 for a monatomic gas.
1 1 - 2 2

1Vy-1 =(~)(5.66V)Y-l
T•• TA (~:

y-l= logt2 =0.6931=0.4


loge 5.66 1.7334 .1000G) ".• 850 K
1.4y =
Work done in an adiabatic process is given by
The relation between y and degrees of freedom is
2
w = nR(TA -TB)
y=1+- y-l
/
1. 8.31 (1000- 850)
2 •
1.4.1+ (5/3) -1
7
/.5 -1869.75 J
(b) Process B -} C is isochoric.
(b) Work done in an adiabatic process is given by
Hence W =a
w = PI VI - P2V2
From first Jaw of thermodynamics,
y-l
Q =.t!.U + W
as PIV} =nRT=PV
=nCvt1T+O
T PV 3
PlVl =nR - =-
2 2 .n-R(Te - 350)
2
Thus W = 1 (PV _ PV) = 1.25PV For an isochoric process,
(lA-I) 2 p
- = Constant
I:::xam~le T

One mole of a monatomic ideal gas is taken through the cycle


-.-
Pc
Te T
PB
B
shown in Fig. 2E.12. Pe
Te=-.T,
T P,
A
_ (I/3)PA T

t:::J c 2
-(2/3)PA'

.-T•
'-------_v
Flg.2E.12
850
2
.-
A -} B adiabatic expansion = 425 K
3
B -} C cooling at conMant volume Hence Q = 1 x - x 8.31(425 - 850)
2
C -} D adiabatic compression
= -5297.625J
D -} A heating at constant volume
Negative sign implies that the system has lost heat.

www.puucho.com
Study Anurag Mishra with www.puucho.com

IHfRMODYNAMICS
ee) Process C +4 D is adiabatic. _5x10"xl0
TA - 500x 8.31 120.33 K

Tn -:=lOx 10" x 10 =240.66K


500 x B.31
Pressure at D. PD is unknown, but pressures at A and C
are known. T = lOx 10" x 20 =481.32 K
c
Process D --+ A is isochoric; hence 500x 8.31
4
PD Tv Tn -:=5x 10 x 20 =240.66 K
-=- 0'
PA TA 500 x 8.31
Therefore, (b) Change in internal energy is path independent;
hence we cannot predict the path taken.
(c) Process A B C consists of isochoric process
-}> -}>

A -}>B and isobaric process B C. -}>

H'
rl,'! =T ~ WA--->s ....•c -:=WA ....•B + Wn ....•c
D cPT
[ ]
-:=0+P(V2 - VI)
C A
-:= 0 + 105(20 -10)
T" =(T,2" )[ (1'3)P P
D A
A
x 1000
]"
-:= 106 J

['21(2)" ]( 3 )" Change in internal energy,


= '3 x 1000 1000 h.U-:=nC,,(Tc-TA)

Tv =500K
=(500)(~ R)( 4~0 _1~0)

E':><<:l'TIPt ••. -:=2.25x106J


,,-------- ... -_ ......,..-.
'

From first law of thermodynamics,


A sample of 2 kg of monatomic helium (assumed ideal) is Q -:=f,U + W
taken through process ABC and another sample of 2 kg of the
-:=(2.25 x 106 + 106) J
same gas is taken through the process ADe. Given molecular
mass of helium = 4: (a) What is the temperature of helium in =3.25x 106 J
each of the states A, B, C and D? (b)ls there any way a/telling Path A D C consists of isobaric process A
-}> -}> -}> D and
afterwards which sample of helium went through the process isochoric process D C. -}>
ABC and which went through the process ADC? (c) How much
is the heat involved in each of the processes ABC and ADC?
p = P(V2 - VI) + 0
:=5x 10"(20-10)
=0.5x 106 J
Change in internal energy aU -:=2.25 x 106 J; there is no
need to calculate h.U once again. Internal energy is path
10 20 independent; therefore,
Flg.2E.13 tJ.UA ....•B ....•C :=f,UA ....•D ....•C

From first law of thermodynamics,


Solution: Ca) Number of moles of helium,
Q=au+w
n -:= mass -:=500 mol
molecular weight '" (2.25x 106) + 0.5x 106

From ideal gas equation. =2.75xI06J

T= PV
nR

www.puucho.com
Study Anurag Mishra with www.puucho.com

THE
Example Wcycle ""WA~B + WB~C + WC~A
""2.0BRT - 1.5RT + 0
Three moles of an ideal gas [c p =~ RJ at pressure P and " O.5SRT
W cycle"" Qcytle "" O.58RT
temperature T is isothermally apanded twice to its initial
W cycle and Qcycle are positive, hence heat is added to
volume. It is then compressed at constant pressure to its
system, while net expansion takes place.
original volume. Finally the gas is compressed at constan~
volume to its original pre.ssure P. (a) Sketch p.v ~nd p.T. Example
diagram/or the process. (b) Calculate the net work done by'
the gas and heat supp'lied to the gas during the comp'/ete cx.cle.
One mole of an ideal monatomic gas is taken around the
Solution: (a) The initial state A is characterized by cyclic process ABCA as shown in Fig. 2E.15; calculate:
state variables: pressure P, volume V, temperatureT. Process (a) the work done by the gas,
A --. B is isothermal expansion.
(b) the heat rejected by the gas in the path e -4 A and the heat
PAVA =PaVs absorbed by the gas in the path B -4 e,
F = PA VA = PA (c) the net heat absorbed by the gas in the path B -}o e,
B
VB 2 Cd) the maximum temperature attained by the gas during the
f'Y,c1"e, _
Process B -+ C is isobaric process.
VB c V Solution: (a) Work done in a cyclic process is area
-"-
T T
B c under P-Y diagram of the cycle. Work done is taken positive
Vc T if the cycle is clockwise.
Tc =-Ta =-a
VB 2 Area of triangle ABe ""..!. (Base»( (Height)
2
Process C -+ A is isochoric process.
1
(b) For a cyclic process l:!.U = 0, as the gas returns to its ==- (3Po -Po
2
»( (2Yo - Yo)
initial state at the end of each complete cycle.
P P ""PoYo
P P
LJ(P'TI gas
(b) Process C A is isobaric,
-}o

For an ideal monatomic


P

PI2 C~B
(PI2, V)

V
(P/2, 2V)

2V
V

Flg.2E.15

From first law of thermodynamics,


P/2
C (Pf2, T/2) 8 (Pf2. T)

TI2 T
T
C p "-R
2

TA == PoVo
s

From ideal gas equation,

nR •
3::_1~~c ,
,
Vo 2Vo
,,

T == 2PoYo
B Fig. 2E.15 (a)
Qcycle == l:!.U + W cycle nR
Clcrcle == W cycle Qp ""nCpCTA -Te)
Isothermal process:
VB
"n(~ R)(POVO _ ZPoVo)
2 nR fiR
W.''\~B ==nRT Jog~v
A S
=--PoYo
""3RT logt 2 ""2.08RT 2
Isobaric process: Negative sign implies that heat is rejected by the system
to surroundings.
WB....•
C ""P(Vc-VB)=nR(Tc -To)
Process A -}o B is isochoric.
",,3R(~-T ) ""-I.5RT
For an ideal monatomic gas, C v "C -R"-R3
p 2
Isochoric process:
From ideal gas equation.
WC_.A "" 0

www.puucho.com
Study Anurag Mishra with www.puucho.com

THERMODYNAMICS
Temperarure versus T
volume curve is a parabola 3PoVl)
as shown in Fig. 2E.1S(b). P
Q" =/le,,(ra -TA) During expansion,
=n~R(3PoVo _PoVo) temperature will increase
2 nR nR and af::er reaching a
maximum will decrease.
= 3PoVo v
For maximum Vo~V 2Vo
Positive sign implies that heat is added to the system. 4 0
temperature,
Fig. 2E.15 (b)
(e) Out of-the three processes dT = 0
A -) B, B -+ C, C -+ A, we know the heat involved in the dV
two processes A -+ Band C -+ A. We also know the oct work
done in the cycle. Since !'J.U = 0 for a cyclic process, R (4
Po - V
o
V+ 5 ) = 0
Qcycle = W cycle
S
QC ....•A + QA-->B + QB ....•C = W(:rde V =- Vo
4
QB ....•C =W..-yde -QC ....•A -QA ....•1l
Po [ 2 2S 2 2S ]
S
= PoVo + - PoVo - 3PoVo
Hence Tmax =R" - V
o
16 Vo +4 Vo
2
1 25 PoVo
= - PoVo =---
2 8 R
Cd) Along process A .......•
B, temperature increases This is the maximum temperature attained during the
from entire cycle.
PoVo 3PoVo
TA =--toT n =--=3TA• Examp~le 16y--
nR nR
Along process C -+ A, temperature decreases from Two moles of an ideal monatomic ga.~is taken tllrough a cycle
T = ZPoVo to TA = poVo. ABCA as shown in the P-T diagram Fig. 2E.16. During the
c
oR nR process AB, pressure and temperature of the gill vary, such
The process B -) C is a general process whose equation that PT = constant. 1fT) = 300 K, calculate:
is nm known. Since it is a straight Hne with negative slope P
and positive intercept with axis, its equation is 2P,
B c
P =-aV + b
where a and b are slope of straight line and P intercept
respectively.
The straight line passes through points B(3Po, Yo) and
P, ••• •••
.
,A

T
C (Po, 2Vo); so we have T1 2T1

3Po =-aVo + b Fig.2E.16


and Po =-2aVo T b
(a) the work done on the gas in the process A -) B, and
On solving these equation, we get
2Po (b) the heat absorbed or released by the gas in each of the
a=- processes.
Vo
Give answers in terms of the gas constant R.
and b = SPo
So the equation of the process is Solution: From ideal gas equation
2Po PV = nRT ..,(I)
P=--V+SPo
Vo On differentiating the gas equation,
pdV + vdp =nRdT .. ,(2)
From ideal gas equation, T = PV for one mole of gas,
R .. ' (3)
pdV + (n~T) dP = nR dT
T = Po [_
R
2
Vo
v sv]
2 +

www.puucho.com
Study Anurag Mishra with www.puucho.com

Process A -) B follows the law


(atm)p
PT = constant 3P,
On differentiating we get
PdT+TdP=O ",(4) 2~o ------_.B
TdP=-PdT
On substituting expression forT dP in eqn. (3), we h<j.ve
.. P,
PdV-nRdT=nRdT
V (lit)
P dv = 2nR dT ",(5) 2
(a) Work done in process A -). B, Fig. 2E.17 (b)
B
WA-oD = fPdV =2nR fT: dT = Area under the curve BC -

=2nR(TB -TA) Area under the curve AB

=2nR(T}-ZT2) = Area of the half circle = nR 2 atm x lit.


= (2)(2)(R) (300 - 600) 2
= -1200R n
= -atmx J'u.
2
(b) For a monatomic gas,
R = (volume)2 -(volume)l = 1(lit)
3 5
C =-R C =-R
" 2 ' p 2
~_>C5P,~ele
liU = nCvliT
The average degrees of freedom per molecu.lesfor a gas is 6.
=(2)(~R)(300-600) rhe gas perfonns 25 J of work when it expands at constant
pressure. Find the heat absorbed by the gas, [Pi = P2 = P (at
= - 900R
Pconstanl )}--------,---
From first law of thermodynamics on process A -) B,
Solution:
QA-oB =WA ...•B +Ii.U
liQ = liW + liU ""P(V2 - VI) + nCvdT
= -1200R - 900R nR
=P(V2 -Vl)+--(T2 -TI)
= -2100R (y -I)
Negative sign implies that heat is rejected by the gas. "Q = P(\' _ v ) + (nRT, - nRT, )
2 I (,-1)

=P(V _V,)+(P2V2-PIVI)
In the P-V diagram shown in Fig. 2E.17. ABC is a semicirde. 2 I (,-1)
;Pindthe workdone in the process ABC. 2 P(V - VI)
- =P(V2 -\'t)+-~-~~[':PI =P2 =PJ

-s "
" (y -I)

.3 "L" ,C
Q.. B : liQ = P(V2 - VI )[1 + _1_]
1 •• ~- :A '(-I

-+-'~2~.'V(lit)
Fig. 2E.17 fa)
= P(V, - v,[-y-]
1,-1
Given Ii W = P(V2 - VI) = 25,
Solution: Workdone in the process ABC is divided
into two parts AB, Be. In A -) B compression and B -) C and y =1 + 2/1 = 4/3.1 =6
expansion Upto initial volume. But pressure is changing 4/3
tlQ = 25 x -'- = 100J
continuously and increase at every stage. 1/3

www.puucho.com
Study Anurag Mishra with www.puucho.com
181
[HERMODYNAMI(S

Examplg. 3P =Pn x(~)x ZTo ...(2)

1 mole of an ideal gas at initial temperature T was cooled From (1) and (2)
isochorically till the gas pressure decreQ.';edn times. Then by
an isobaric process, the gas was restored to the initial
temperature T. Find the net heat absorbed by the gas in the
whole process.

Solution: p_V diagram of monoatomic ideal gas is a straight line


TI =T, PI =P, VI =v v = constant pocT passing through origin. Find the molar heat capacity in the
T P process.
T2 =-.
n
Pz =-, V =v
n
2
P = constant VocT
Solution: Equation of the line is P = KVbeacuse P x V
T3=T,
P
PJ=-,V3=?
V2 T2
V =T '
V
-0_0-

V3
Tin
T
1
n
6Q = 6W + 6U, f
nCdT = PdV + nCvdT,
n 3 3

V3 = nV C=Cv+--
J PdV
(ndT)
Total change in (U) 6.U = 0

J" J V2 K z 2 -VI)
2
so .1.Q=6.W 6W= pdV=K VdV=-(V
P P VI VI 2
c.Q= hoW=-(V3 -V2) =-(nV-V) 1 , 2
n n = - (KV2 -KV\ )
=PV(l-l/n) 2
p
and PlY] =nRTl• n=l, PV=RT' 6W='!-(PZ xVZZ_ \ 2
XV1 )
2 Vz
~QoR1\l-~J 1
V1

Example
.---.
~\...>
=-(PzVz
2
ndT = n(T2 -
-PIVl)

nR
Tl) = -(Tz
R
- Tt)

Pressure versus temperature graph of an ideal gas is shown. 1


=-(nRTz ~nRTI)
Density a/gas at point A is Po' Find the density of gas at B. R

----7"
P 1
=-(P2VZ -P1VI)
R
3P ••
C =C •. +B.x (PzVz -ptV\)
p __.0j),: :
2 (P2V2 -PlVt)
,, oC, +Rj203Rj2+Rj2oZR
,
• T C oZR

-.~ampLe
.-...- ...-""""""'
..=.
Solution:General equation for an ideal gas An empty pressure cooker of volume 10 litres contains air at
PV = nRr atmospheric pressure 105 Pa and temperature of 27°C. It
P'= m RT contains a whistle which has area of 0.1 ern 2 and weight of
M V 100 gm. What should be the temperature of air inside so that
p=_x_xT
m R the whistle is just lifted up ?
V M
p=px(~)xT
P =Pox(~ }To ...(1) Flg.2E.22

www.puucho.com
Study Anurag Mishra with www.puucho.com

THEIMOIllIWIJCS
Solution: When the whistle is just lifted up, then
pressure inside of the cooker Example

P2 = Pi" =(Po +7) One mole of a gas mixture is heated under constant pressure,
and heat required t.Q is plotted against temperature
= 105 + 100x 10-3 x 10 difference acquired. Find the value 011 for mixture.
0.1 X 10-4
= 2x 105 N/mZ
2':O~~
V2 = V = constant, Tz =?
Pi" V =nRT2
T _Pi"V o 100K AT
2 - nR
,-_'.'9••.2E.24
PIV} = 105
n =-- lOx 10-3_
X
Solution: t.Q =nCpt.T
RT} Rx300
"Q
-=nCp
.•. _ 2xlOs x lOx 10-3 xRx 300 "1'
" - 105 x lOx 10-3 x R "Q yR
--=c p =--
T2 "" 600K, or 327°C "Tn (y-1)
2500 yR
Example --=-
100 (y-1)

V.T cu"'e for 2 moles of a gas is straight line as shown in the YR~25(J)
(y-1) K-mol
graph here. Find the pressure of gas at A.

V(lil(~ (~~ + 1)=y


y=(~~Y+1)
T(K)
Flg.2E.23 1=1(1- ~:)
Solution: V"" tan 53°T
4
V=-T
(.: n = 2)
1 = y(25;5&3) = ye::)
3 25
4 16.7 = Y
VA =-TA
3 1.499~1.5=y
PA VA = 2R.TA
Exam..R.'e
PA =2RX(~:)
Ideal diatomic gas is taken through a process t.Q = 2t.U. Find
PA = 2R. x_
3 the molar heat capacity for the process. (where t.Q is the heat
4 supp..lied andt.u is cha[!ge in internal energy)

PA =.:?R Jjlitre Solution: "Q = I>W + I>U


2
Given, ~Q= 2liU
P = 3/2x &31 J/m3
A 10-3 UU =t.W +t.U

PA =1.25x 104 N/m2


~w=t.U
I'~Q= nCt,dT
t.u = nCvdT
nCdT = 2ne ,.dT

www.puucho.com
Study Anurag Mishra with www.puucho.com

IHERMODYNAMICS

C =2x~R(fOrdiatomicgasc,. =~R) aV2 == nRT


T _ uV~
1 - nR
CdR
a(nVo)2
Exampl.e and T2 ==---
nR
2
A gas is undergoing an adiabatic process. At a certain stage A, l'.U == nC 1:( a112V~R-aVo )
the values of volume and temperature =: (Vo,To) and the
magnitude of the slope of V-T curve is m. Find the value of C p R aVo2(n2 -1)
o
and C", Of -1 R
Solution: PVY = K
l'.U == aV~ (n 2 -1)
1Vy-1 =K1 (y-I)
log.T+(y-l)log. V=log.K1
.!.+ ("(-1)
T V
dV =0
dT
(ii) W 0 J pdVo J::' aVdvoa[V:[
(y-I) dV 1
---=-- == 2:[n2Vi - Vi]
V dT T 2
dV __
_= Vx __ 1 a~,2 2
W 0_[71 -1]
dT T ()'-1) 2
(iii) Consider one mole of gas undergoing process.
(~~)(VO.TO)=- ~: x (y~l) P == HV

V, 1 dQ ==Cprocc"dT; dU ==C"dT: dW =PdV


m=-~x--
To (y-l) dQ == dU + dW =:> Cprocm

(y -1) 0 I;:,:, HT~~I by


c:=

PV ==RT
C
"
+ PdV
dT

because y > 1 and (y -1) ---)tvc


=:> uV2 == RT
so C_yR C_R dV
_0 __ R
'-(y-I)' "-(y-1)
dT 2aV

CI' =(m~:'o), =( n~:o) Cp R+


=:> 2a vdV
PdV
_0_
c:= RdT
R
dT 2
=mRTo Cp=U(l+mTo)
C,' Vo '\ Vo R R
C pro('ess == --,
,- +-2

Example How to Determine Equation of Process

An ideal gas whose adiabatic exponent equals y is expanded .E:<Clmple


according to the law P = aV, where a is a constant. The initial
volume of the gas is equal to Vo' As a result of expansion the The amount of heat given to the gas is equal to the decrease in
volume increases n limes find: its internal energy. If y is given then find out the equation of
(i) increase in internal energy process. Given the work done by the gas is 2 times heat given.

(iO work perfonned by the gas


'Q~hW~"Q~"U
(iii) the molar heat capacity of the gas in the process.

... (1) Flg.2E.28


Solution: P == uV
(i) l'.U ==IIC"l'.T ==nC,.(T2 -TJ)
by PV == nRT from eqn. (1)

www.puucho.com
Study Anurag Mishra with www.puucho.com

Solution:
w = -2tJ.U [ 1+ KKR -1]e" InV+ [KKR -1ell ] InP=constant
PV = nRT
PdV = -2nC~dT [ ~ R +lJlnV+lnPOK
Pdv + VelP = nRdT (K-1)-
y -1
PdV =_2nCv(PdVn~VdP)

Pd~1+2~, H2~, )VdP=o


1 Kh-U)
PV ( •. (K-1J = K)

(R+2C,,) fdvy-+2C" fdPp=O


Molar heat capacity of an ideal gas varies as C = C v + aT,
(R + 2C,,)ln V + lev lnP = constant:::K
C =ev +J3V and C ::e" +aP, where a, J3 and a are
R+2Cv InV+lnP=K constants. Find the equations of the process for an ideal gas in
t
2C, temu of. the variables T and v. ~ _
R+2Cv

PV 2c" = K1 Solution: (i) From first law of thermodynamics,


dQ=dV+dW
p)2~+1) =K
t
dQ=CdT=Cl,dT+PdV
C = dQ =c +pdV

PV
[,.,)
2 = K} =C
dT

+RTdV
tt dT

(1)
I' V dT ...
since it varies as, C.= C I' + aT ... (2)
On comparing expressions (1) and (2). we have
In a given process K times the heat given is equal to the
increase in the internal energy K < 1. Find out the equation of. aT=(R:)(:~)
process.
dU = KdQ
(*)dT=~ ... (3)

dW=da-dU On integrating eqn. (3), we have


=(1-K)dQ
~ dO aT
log V - - = constant
Flg.2E.29 R
or V = (another constant) eQTiR
Solution: dw = (l-K)dQ

dW=(';Kyu
or V= Ve-{nT/R) =constant ... (4)
Eqn. (4) is the equation of process.

Pdv =(' ;K}C,dT


(ii) We have C =C + RT dV
II V dT
Comparing it with given molar heat capacity,
PdV =(';K}C,(PdV ~VdP) C=CII +jW
Hence; we have
PV=nRT
=- Pdv + VdP = nRdT
RTdV =pV
V dT
PdV[l-C~K)~ ]+K~lCv .VdP=o dV
V2"=RT
P dT
... (5)

[,_(l-K)C]f
KR"
dV +(K-1)C
V KR"p
f dP = constant Above equation integration yields
- ..!.. =..@. log T + constant
V R

www.puucho.com
Study Anurag Mishra with www.puucho.com

JHERMOOYNAMICS ____ """'185


R ___
(6) dT = Tou e"v dv = TadV
log T + - == another constant
PV
or TelR'PV) =anotherconstanc --_(7) dQ =nd'lC' + :V J
Above equation is the law of the process. The molar heat capacity mathematically is defined as
(---)
III
0 n comparmg
- C =
RT dV
C t' + -- C =.!. dQ
V dT 11 dT
and C =C1,. +aP In the above process
we have R
C proc<:ss = C,' +-
RTdV =aP aV
V dT
dV PV
-=0-
dT RT
Process is P =PoeaV jindCprocw'
Since PV = RT for one mole, hence
dV Solution:
-=0
dT
or V =uT
or v = anT
for n moles dQ = nC"dT +PdV
Calculating the molar heat capacity for general
process: =11"C d-r +---
nRdT dP = PadVand
1 + oV PV == nRT

_.r:=~an:>pl!,,-31\.7 dQ = nd-rlC,_+ _R_J PdV + vdP = nRdT

Q joule of heat is given to n mole of monoatomic gas. The gas


'l 1 + uV

performs a work g on its surrounding. Find the molar heat


I dQ R
PdV(l + uV) = qRdT
2 C ",--=C +--
capacity for the process. process n dT V 1 + aV

~
w=Q/2 One mole of an ideal gas whose adiabatic exponent is y
Flg.2E.31 undergoes a process given by the equation.
a
P=Po+-
V
Solution: g = t.U = ~nR!lT
2 2 Find out the heat capacity of a gas as a function of volume.
Q = 3nRi>T Also find our increase in the internal energy, work done by the
The amount of heat given to 1 mole such that the gas and amount of energy rransjelTed if the volume changes
temperature increase by 1°C is equal to 3R. from VI to V2.

Solution: dQ=nC"dT+PdV
a -a
p = Po + -; dP = -dV
V V'
An ideal gas, it is given that the molar heat capacity C!! at
constant volume. Find out the molar hear capacity of gas as a PdV - .c: dV = nRdT
function a/V. lfthe process peiformed by the gas is T = ToeClV• V
PV = nRT
Solution: dQ = au + dW To and 0. are constant
dQ = nC,.dT + PdV dV= nR dT
T = Toe
uv
(p -~ J
nRT dT
dQ = nel,dT + ---- PdV + Vdp = nRdT
V Ta

.I ~."' ••• -.
www.puucho.com
, . ."..
Study Anurag Mishra with www.puucho.com

l86 THERMODYNAMICS •
nPR
dQ = nCudY+--dT
p-!!. Q = J dQ = J:" (c, + :~ )adV
V
dT = adV

Mixing of Gases
Consider mixture of three gasses "1 mole of monatomic,
"2 mole diatomic'"3 mole triatomic, Heat supplied to raise
temperature of gas by tIo T is given by
Heat supplied Volume = constant
': P""Po +~ 3 5

dU =nCudY ..
V

ilQ = n1 -R6.T + n2 -R6.T
2
6
+n3-MT.
2
n, -+ Mon
"2 -+ Dia
"3 -+ Tri
T
dU = nC PodV
"nR 2
~Q 3 5 6
= "}-R + n2 -R + n3-R
dU= PoC" J V2
dV =PoCv (V2 -VI)
-
tIoT 2 2 2
R VI R
nRT =: PoV + a .: nRdT = PadV £\Q --). is the heat capacity of the gaseous mixture
~T
dW=PdV=(Po+~}V ~Q,'~T
Molar heat capacity at constant volume =
"l+n2+n3
W = J dw = J (Po + ~ )dV Dissociation
Suppose we have n2 mole of diatomic gas and n1 mole
V,
=Po(V2. -V1)+uln- of monoaromic inside a closed insulated box after some time
VI we find that K fraction of diatomic gas molecule dissociate

Example Remaining mole of Diatomic = n2 - Kn2 = n2(1- K)


Remaining mole of Monatomic = ZKn2 + n}
One mole of an ideal gas with heat capacity at constant Heat capacity
pressure C p undergoes a process given by the equation, ~Q 3 SR
-=(n1 +2Kn2)-R+(1-K)n2-
T =To + aV, tIoT 2 2
(i) Find out molar heat capacity as a function
(ii) Find out amount of heat transferred it the volume be
of' v' .
>(nI ~ R )+ n 2 5: which is greater than molar capacity

increases f!..om VI to V2' if the heat capacity is increases in dissociation

Solution: 0) dQ = nC"dT + PdV '; T =To +aV Molar heat capacity = £\Q/£\T
nl+n2+Kn2
dQ =nC dT +PdV ',' dT =u=> dV =..!...; PV=nRT
dT v dT dT dv dT a The molar heat capacity is decreases.

nJ;l(T
= nC" + ••.•
\ V +
o a)"!"a '; P = nR
V (To + aV)
C =..!.. dQ = c + RTo + Ra In the compartment (1) shown in the Fig. 2£.36 a
n dT tt aV a aissociation had not taken place monoatomic gas is present in
RTo the state shown. In the compartment (2). A diatomic gas has
=Cp -R+--+R
aV been enclosed in the state shown. Both the compartment are

C =Cp
RTo
+--
aV
thennally, insulated.
-------
(ii) dQ =CdT

= ( C + RTo)
p aV dT -=a
dT
dV

www.puucho.com
Study Anurag Mishra with www.puucho.com

THERMODYNAMICS

Mixture of Gases
Consider a homogeneous mixture of 'n' inert
mono
T Oi (non-reacting) ideal gases (G1 ,G2 •... Gn). Let their-moles be
Pl' V,. T1 P2,V2.T2
~ll' )..I2')..I3.... Pn respectively.
Molar masses be M},M 2,M3 .... M n.
P1V, P2V2 Molar heat capacities at constant volume be
n1= RT , n2=--
RT, C Y ,C v2 ,C v), .... Cyn.
j

Flg.2E.36 Under the ideal gas approximation, the properties of a


gas are not influenced by the presence of other gases, and
(a) Find out the final pressure, volume and temperature of the
each gas component in the mixture behaves as if it exists
gas mixture. alone at the mixture temperature T m and mixture volume
(b) Find out the CVmix. and C Prnl~_ Vm•

Solution: (a) Let us assume that the final Molar Mass of the Mixture
-temperature of the gas mixture is T . Weight of the mixture
Ma 1ar mass a f mixture = -~-------
Initial internal energy of the mixture total no. of moles
3 5 IlIMl +)..I2M2 +1l3M3+ ....
="1 -RT, +"2 -RTz M mixture "" ~~~~~~~~~--
2 2 ~tl+~t2+P3
Final internal energy of the mixture Internal energy of the mixture
3 5 Internal energy is an extensive property. It is additive in
"'", -RT + n2 -RT
2 2 nature i.e ..
Since the internal energy of the gas mixtures can not Umax, ""UI +U2 +U3+ +Un
-change therefore.
3
nj-RT1 +nz-RTz
5 RT
=-(3n. +5n2)
".m..,.,....RJ" =.'.,.,.RT + f' _,RT + ".3R_T_+ ..... + _f,~"_R_T_
2 2 2 ('(mix -1) (y\-1) (Y2 -1) (1'3-1) (Yn-1)

T=-
2 (nt . ~RTI
2
+ n2 .~RT2)
2
(where T: temperature
)..Irnix. ~ll +
of the mixture).
)..12 + )..13 + .... + )..In
R(3n] +5n2) ('(mix-1) (y\-1) (Y2-1) (13-1) (Yn-1)
The final volume of the gas mixture VI + Vz
Where )Jmix.
p= nRT
VI + V1
SO.
(b) liU =- AU, + .1.uz )JI +)..12 +)J3+ ... )Jn )..11 + )..12
Q1 =(nl +nz)CVtnix.tJ.T (Ymix-1) (y\ -1) (Y2 -1)
=- "IG" mono L1T + T!2C"A_. ul /),T + )..13 +..... + lin
=- rt1CL'mono +n2C"df (Y3-1) (rn-1)
C"mlx --------
n}+n2
with the help of above expression '(nix, can be obtained
Q, is the heat given to 1 mole of the gas mix to increase for a mixture of gases.
its temperature by 1"C at constant volume. Cp and Cv for a mixture
C _ n,C"".ono + n,Cp' ,,' For a mixture
Prm.-
") + "2 + "3 Umix =UI +U2 +U3+ +Un
n} ~ mono, n2 ~ di, "3 ~ tri
(C¥)mi~. _Pl(Cv)l +)..I2(C¥}z +113(Cvh+ .... l-ln(Cv)n
"ICl' +"2C"
j +"3C"
C"mhr. "" 2 3 PI +P2 +~13+ ... +Pn
"I +"2 +"3
As Cp -Cy =R
"" "ICPl + "2Cp2 + "3Cp3
C Pmhr. So, from the above expression,
"1+"2+"3
(CP)mix =R+(C~')mix
'l _
CPmi>.
'ml" --C~-
,~

www.puucho.com
Study Anurag Mishra with www.puucho.com

lij188g:::::lZ==
R + PI (CV)1 + }1z(Cv h + 1l3(CV h+"'lln(CV)n A B
,
Vo no
III +112 +fl3+ ... lln
. '.'

1l2(CV}z +J.L3(Cvh+ .....+lln(CV)n


~+-------------- PoVo 2PoVo
III +J.12+J.I3+ ...•• +j..ln
nR nR
IlI[R + (Cv)d + )..l2[R + (Cv hl + 1l3[R + (Cv hl + Temperature (T)

...•• +lln[R + (CV)n] Fig. 2E.37 (b)


~
III +Ilz +J.l3+ .... +J.ln

Ilt(Cp)l +j..l2(Cph +1J3(Cph+ .....+lln(Cp)n


III +J.l2 +Jl3+ ....• +lln Draw the P.T and V-T diagrams for an isobaric process at.
p:pansion, corresponding to n moles of an ideal gas at a
pressure Po> ftom Vo to 2Vo'
---
Solution. From the equation state PV = nRT
Draw the P-T and V-T diagrarru of an isochoric process of n
moles of an ideal gas from pressure Po volume Vo to pressure A B
2Po indicating the pressures, and temperatures, of the gas, in ., Po u; •
the inieial and the final states. €
Solution. From the equation of state for an ideal
gas PV = nRT for an isochoric process
PoVo 2PoVo
(a) P = T~T i.e., P oc T
nR nR
Temperature (T)
PoV
For p =po and V=VnT=-- o Fig. 2E.38 (a)
"' nR
and at P = 2Po• v=v"
v'
T=2PoVo
nR
We have, V ~(~)r v~r or

The graph of P vs T is a straight line passing through the At V=Vo T1 .= PoVo


origin, when interpolated. nR
and V=2Vo T = 2PoVo
B 2
nR
for the graph of P versus T the variation is a straight line
the temperature vatying from T1 to T2 as shown in figure.

PaVo 2PoVo
nR nR
Temperature (T)
Fig. 2E.37 (a)

PoVo 2PoVo PoVo 2PoVo


(b) Since, temperature increases fr am -- to __
nR ,nR nR nR
Temperature (T)
volume remaining constant, so, the graph of V.T will be a
straight line parallel to T axis as shown in figure. Fig. 2E.38 (b)

For the graph .of V''Versus T the equation V = ( ~ )T is a

straight line the volume varies directly, as the temperature


(Charle's Law).

www.puucho.com
Study Anurag Mishra with www.puucho.com

THERMODYNAMICS 189

Efficiency of a Cyclic Process Illustration: Find the efficiency of the thermodynamic


Efficiency (11) of cycle is defined as the ratio of the net cycle shown in Fig. 2.48 for an ideal diatomic gas.
work performed to the total heat supplied to the system, per
cycle.
work done per cycle
Th us 11 = -----------
total heat supplied per cycle

__
~Lt
Vo 2Vo
Volume (V)
Fig. 2.48


: . Let n be the number of moles of the gas and the
Vo 2Vo
temperature be To in the state A.
Volume (V) Net work done during the cycle is area under curve,
Fig. 2.47 1 1
= W = - x (2Vo - Vo)(2Po -Po) = -PoVo
2 2
Illustration: Fig. 2.47 shows the pressure' versus For the heat given during the process (6QI) A --+ B, we
volume graph corresponding to n moles of an ideal have
monatomic gas for a cyclic process ABCDA. Find the
efficiency of the cycle. ~Ql =~WAB +~UAB

Network done by the gas during the cyclic process is ~W AB = area under the straight line AB
given by area enclosed by cyclic process 1
=-(Po -2Po)(2Vo -Vol
2
W = (2Po ~Po)(2Vo - Vol
= PaVo ~UAB=nCt,~T = {S:)c4To -To)
For the isochoric process A to B the heat supplied is
given by, ~ lSnRTo _ lSPoVo

~Q=8.U+~W 2 2
3 15
= nC"t.T + 0 ~Ql =-PoVo +-PoVo =9PoVo
= n( 3:) :-Po)
Vo(2P~
2 2
The processes B --+ C and C --+ A involve the rejection of
heat by the gas.
3R
=-PoVo . ( ) Work done per cycle
2 Effi Clency 11 = ------------
Total heat supplied per cycle
Temperature in respective states A,B,C,D is given by
1
T
A
= PaVo TD = 2PaVo T = 4PoVo T = 2PoVo
c i.e.,
2:PoVo 1
nR'u oR' nR' D nR 11 =--- =-
9PoVo 18
for the isobaric process B to C the heat absorbed will be
given by
~Q2 =flCp~T

= n( S:) 2Po(2~- Vo) An isolating vertical cylinder fitted with an insulating


frictionless piston contains n moles of an ideal diatomic gas.
The initial volume and temperature of the gas are Vo and To
For the processes C to D and D to A, the heat is rejected respectively, with the piston fixed tightly. The atmospheric
from the system. pressure is Po. if the piston be released, find the equilibrium
. ( ) f h I Net work doen per cycle temperature and volume of the gas, if the heat capacity of the
Erfi C1ency 11 ate eyc e = ----------
Tmal heat given per cycle cylinder and piston is negligibly small. Weight of the piston
and cross sectional area of the cylinder are Wand A
PoVo ...3.-
respectively.
3PoVo
--+
5P"
oVo
13
2

www.puucho.com
Study Anurag Mishra with www.puucho.com

Solution. (a) Since the system is isolated from The process B -+ C is isochoric in volume remains
the surrounding, the work done by the gas will be at the cost constant at 3Vo'
of its own internal energy. Let the final temperature and From equation of state PV = nRT
volume be T and V respectively. Evidently, the final pressure
P oc T
P of the gas will be just enough to hold the piston in
equilibrium against its weight and the atmospheric pressure. Since P changes form Po to 2Po, so accordingly T changes
W from To to 2To'
i.e., P = Po + - ...0)
A Next, the process C -+ D is an isothermal compression,
Change in internal energy of the gas, the temperature remains constant throughout at Zfo state D.
nR8T The process D -+ A is isobaric in nature, the gas getting
-tJ.U=nC tJ.T=--
II (y-l) compressed from a volume 2Vo to Vo-
nR(T -To) SnR(T-To) From equation of state PV = nRT
=
G -1) =
2 VxT
the temperature varies linearly from Zfo to To.
Work done by the gas against a constant external __ 3PoVo
Here, To
pressure Po + W, will be nR
A
EO B C
aW =pav=(po + :}v-Vo)
~
• ---20
:> 2To
------- -- •• ,,D
Using equation of state PV = nRT, we have ~ To
E ,
,
, .:
' ,
'
:
-------A ----:,
,
~
(Po + :)v = nRT ...(ii)
Vo 2Vo 3Vo To 2To
Volume (V) Temperature (T)
since tJ.W +tJ.U = 0 [..."Q : OJ
(.) (b)
=:> (p W}v
0+-
A
- V)0 SnR(T-To)
+----~:O
2
... (iii) Fig. 2.50

Illustration: Fig. 2.50 (c) shows the V.T diagram for a


Solving Eqs. (ii) and (iii), for V and T, we get
cyclic process performed on n moles of an ideal gas.
T = ~o + (Po + :):~ D C

and V = 2Vo + 5nRTo

7 {Po+:)
Illustration: Fig. 2.49 shows the P-V diagram for the
cyclic process ABCDA performed on n moles of an ideal gas. TO 2To 3To 4To

Here we draw the T.V and P-T diagrams for the process. Temperature (T)
Fig. 2.50 (c)
Initially, if To be the temperature of the gas in state A,
then from equation of state (3Po)(Vo) = nRTo Here we draw P-V and P.T diagrams.
Let Po be the initial pressure of the gas,
Po .: nRTo
Vo
Process A -+ B is isochoric the volume remains constant.
Vo 2Vo 3Vo If V is constant, then P IX T
Volume (V) Since temperature increases from To to zro, the pressure
____ '.19.2.49 also increases linearly from Po to 2Po'
The process B -+ C, is isobaric, because the straight line
0' To = 3PoVo
BC passing through the origin, thus, V :x T.
nR
the process A -+ B is isothermal expansion, so the The pressure remains at 2Po from state B to state C. For
temperature of the gas remains constant at state B it is To. the process C -+ D, the volume remains constant,

www.puucho.com
Study Anurag Mishra with www.puucho.com

THERMODYNAMICS 191

pxT and 6.QJJC = oWC4 (:.oUC4 = 0)


As the temperature decreases from 4To to ZTo the Total work done oW = llW AB + oWBC + oWC4
_pressure also gets halved to Po from 2Po' = 3nRTo + 0 - nRTo In 4
The process D -4 A is agains isobaric in nature as the =nRTo(3-1n4)
.line DA, pusses through the origin.
Total heat supplied, oQin = oQAll
'" 3nC T = 2nyRTo

--~Ll g; 2Po pO (y-1)

, ----Z0---~,, ~,, ,
" ---- A
,
' ,
:. Efficiency 'r]' for the process is given by
"W
:, ' ~ Po
: ' .
,---~
, ,D, "=--
oQ in
Vo
Volume (V)
2Vo
To 2T03T04To
Temperature (T) = nRTo(3-1n4)
3nyRTo
=[1_~)(1_1n4J
y 3
("I (.)
Fig. 2.50 (y -I)

Illustration: Let us find the efficiency of the cycle E><ample


shown in Fig. 2.51.
An ideal gas is made to undergo a thennodynamic process
given by V 0::: y2; find the molar heat capacity of the gas for
the above process.

Solution: Given; V x T2
V", kT2 where k is constant.
Differentiating both sides, of the above equation, we
have
dV = 2kTdT
Let n be the number of moles of the gas and To its initial
temperature in state A. AJso, from equation of state V = nRT = kT2
. P
Process A ~ B is isobaric expansion, we have
or p '" nR
Tx.V kT
T~ = VB = 4 dW =PdV
fA VA nR
=-2kTdT
Til = 4To kT
dW AB = nR.t..T = nR(4To - To) = 3nRTo '" 2nRdT
and 1'.QAR = nC ph.T = 30G pTo dW =2R
The process B -} C is isochoric and hence, ndT
The molar heat capacity of the gas for the process will be
"'WBC = 0
C",C,,+W'
and 6-Wac =nC,.!'iT
=nC,,(To-41 o) ",~+2R
y-1
=[2y-lY-1)R
-3nC"To
=
1
Finally, the process C -} A is isothermal compression. Method-2: Compare it with PV" '" const. to get n "'-
2
"'WBC = nRYo In(~;)
= nRTo In( :v) = -nRTo In 4 Determine the average molar heat capacity of an ideal gas
undergoing a process shown in Fig. 2£.41.

www.puucho.com
Study Anurag Mishra with www.puucho.com

19

A Total work done dW = ,..R (T2 - Tt)


1-n
_ P2V2 -PIV1

~B
1-n

Vo 2Vo 3Vo
~ P,V,
I-n
(1- 1 TI
T2
Volume (V)
Flg.2E.41 Since 1V1l-1 = constant and ~ = constant,
pll-1

then
Solution: Let n be the number of moles of the gas,
6W ~ P,V,[1_(V, )""1]
I-n Vt
T =2PoVo
A

and T
B
nR
= 3PoVo ~ P,V,
I-n
[1_(!l)1";]
P2
nR
Now, work done by the gas during the process is given
The above equations can also be expressed in terms of
by area under curve
PI VI' It also evident that,
1
L\W -'" - (Po + 2Po)(3Vo - Vo) = 3PoVo I
2
Work done per mole of the gas per unit rise in
;: =(~~r-l =(::)1-;
temperature.
w,=L\W = 6,W From equation W' =....!!:....- :::: constant,
i.e., a polytropic is
1-n
nliT n(Ta -rAJ one in which the molar work is a constant.
= 3PoVo "" 3R The molar heat capacity of the process will be
n[3~o _ 2P~o] C=C •. +W'
R
=C" +--
Molar heat capacity of the gas for the above process will 1-n
be R =Cp -Ct. =yC" -C" =C,,(y-l)

C=C,,+W""--+3R=
'(-1
,R (31-2)
--
y-1
R C --C, +~---
Cv(y-l)
1-n
Polytropic Process ~c,,[y-n] 1-n
...(i)
A polytropic process is one in which [he equation of the
pressure versus volume curve is of the form PY" = constant; (i) C ~ c, ; for an j'ochoric process;
where n is a constant throughout the process and may adopt since, forTI -+:tco; y-n =1-n
any numerical value.
(ii) C = C p ; for an isobaric process;
for the isochonc process n --+ 1::00;
since, forn =O;C =yev -=Cp
for the isobaric process n = 0 ;
(iii) C -+ co ; for an isothermal process;
for the isothermal process n = I;
since, for n -+ 1, the denominator of eq. (i) = 0
and for the adiabatic process n = y = C P (The adiabatic (iv) C = 0 ; for an adiabatic process;
Cv
since, n = y
index)
The work given by per mole of the gas per unit rise in
temperature, will be
W,::::_R_ An ideal gas is taken through the process P -= k.ff where k is a
I-n' constant. Determine the molar heat capacity of the gas for the
.. "R(T I -T)2 "R(T 2 -T)1 process.
For an adiabatIC process dW = _
l-TI I-n
= where ~ is the number of moles.

www.puucho.com
Study Anurag Mishra with www.puucho.com

Solution. If n be the number of moles of the gas, then dW R


--=-
dW =PdV ndT 2
From gas equation, PV :: nRT The molar heat capacity of the gas for the process will be
or V:: nRff C=C,,+W
k
Differentiating both sides, we have =Y~I+%=(~:n~
.6.V:: nRdT Method-2: Compare the given process with PV" to get
2k.ff n =~1
k.ffdV = ,>RaT R R
Thus, C:: -- ---
2 y-l n-l
PdV= nRdT R
2 =-+R/2
1-1
dW :: nRdT
2 CONDITION FOR THE MOLAR HEAT
The work done bJ the gas per mole per unit change in
CAPACITY IS NEGATIVE
For C = -ve, (y - n) and (1 - n) should have opposite
temperature will be ~ :: B..
ndT 2 signs,
The molar heat capacity of the gas for the process will i.e., n>land1>n
be, Thus, the molar heat capacity C is negative, for
C ::C,. + W' polytropes that lie between an isotherm and an adiabatic
passing through a common point in a PV diagram.
= Y~1 +~=(~:~)~ Now, from dQ = nCdT
dQ and dT have opposite signs i.e., when heat is
Method-2: P = k.ff
extracted from the gas (dQ being negative), then dT is
P=k
rPV positive and when heat is supplied to the gas (dQ being
V-;;R positive), then dT is negative (the temperature drops).
or PV-1 =const. From first law of thermodynamics.
Compare with equilibrium of polytropic process Since .6.Q= dU + dW
n =-1 or, dU = dQ -dW
R R Here, dW > dQ, so in performing work dW (greater than
Thus, C =-----
1-1 n-l dQ), a part of its internal energy is used up decreasing the
R temperature of the gas, although heat is supplied.
=-+R/2 Similarly, when heat is abstracted from the system, more
,-1
work is done on the system, with the effect that, the surplus
work done gets stored in the form of internal energy rising
its temperature.
An ideal gas is made to undergo a process P a: V; find the Concept: 1. Graphs of various polytropic process.
molar heat capacity of the gas for the process. From, the equation of a polytropic, we have

Solution: Given P oc V py" = constant


.. P = kV Where k is constant. Differentiating both sides of the above equation.

nRT = kV P(nV"-ldV) ..•.V"dP = 0


or
V dP -nP
or - = -- (assuming V '1' 0)
Differentiating both sides, we get dV V
nRdT = 2kVdV which gives the slope of the p.V curve.
= 2PdV when have, for an isothennal, slope dP = E.. and for an
dV V
or PdV = nRdT
2 a d'lQb atlc,sope-=-
' I dP nP
dV V

www.puucho.com
Study Anurag Mishra with www.puucho.com

THI
i.e., (slope of adiabatic) = n (slope of isotherm) = y 7. For n = -1 or (l = 1, the equation of curve is P = kV

(,-n)
(slope of isotherm) (:. n = y)
In other words, the adiabatic cUn>e is more steeper (y C =Cy --
1-n
being greater than 1) than an isothermal one, Qoth passing
through a common point. Putting n = -1; we have C 1
--CV(Y+2 )
2. For a general polytropic process, we have
Pv" = k where k i.~constant. 1
=-(yCv +CvJ
k 2
or P=_
1
V" =-(Cp +Cv)
2
CP;Cv",c",Cv
Example
J !
c
Fig. 2E.44 shows a cycle performed on an ideal gas, referred to
" as Otto- cycle. Show that its efficiency is given by

~=l-(~:r-l
where VJ!V2 is called compression ratio.

Solution: The heat exchanges take place at constant


volume.
IQHI=nC,,(T3-T,)
IQLI=nCv(T4 -T1)
p
Adiabatic

Volume (V)
Fig. 2.52 v
Flg.2E.44
For all positive values of the index n (i.e., 0 < n < 00), the
curves are hyperbola, which touch the coordinate axes at Efficiency of a heat engine is given by
infinity. In particular for an isothermal process n = 1, the
~ = 1- J..g;J
equation reduces to P ox; .!, which represents rectangular. IQH I
V
hyperbola and/or other process (where 0 < n < 'J) or n :f'I), =1_[T.-T,] ... (1)
the curves are unequilateral hyperbolae. T3 - T2
3. For the isobar n = 0 and P = constant, C = C p and is Process 1 ~ 2 is adiabatic.
represented by the horizontal line.
4. For the isochor 11 ~ ::!:(X) and V
is represented by the vertical line.
=- constant, C = C y and T,
T)
=(.".!.)'-'
v2
... (2)

5. For values of n such that y > n > 1, the value C (molar.. Process 3 ~ 4 is adiabatic.
heat capacity is negarive[shown in figure).
6. For values of n which are negative, say n = -a, the
equation of the polyrrope can be rewritten as P = kV<l, which
;: =(~:r-I=(~:r-I
pass through the origin of coordinated (since P = 0 and V = 0, As v4 = v) and V3 = V2 [isochors], therefore,
satisiY. the above equation)". _~ _ T2 T3
-=-
T) T4

www.puucho.com
Study Anurag Mishra with www.puucho.com

THERMODYNAMI(S 195

Process 2 -t 3 is isobaric.
T2 V2 1
- =- =-
T3 V3 r(

Process 1 --)0 2 is adiabatic.


'-,
Hence
TI = V2
T2 VI ( )
1
~--
1
T1 =T2-- ,- ,
Fig. 2£.45 shows Diesel cycle on a P-V diagram,
two adiabatic processes and one isobaric and
it consists of
one isochonc 1', 1
~~--
'.
process. Find the effidency
, '(lV,
compressIOn ratIO rk -,
of this cycle,
' ,
expansIOn ratIO rr
in tenns of
v,
= -,
,,.
cut-D)}
r, r:- 1

V2 V3 Substituting the values of Tl, T2 and T4 in [he


, V, expression for efficiency, we obtain
ratto r, =-
V,
1'",:-' _1', _I ]
p
11==1-
[ rr-I
k
r
'k
r1-1

2~ :d"''''O r[T -T
3 3. ~]

~IQd 'l==1-!_1- r/ -1
1 V r r;-l r,-l
Flg.2E.45

Solution: Heat is supplied at constant pressure


Fig. 2E.46 shows an isotherm and two isobars of two gases on
IQHI=nCI'(T3-Tz) a work done versus heat supplied curve. The initial states of
Heat is rejected at constant volume. both gases are the same and the scales for the two axes are
I QL 1 = nG,! (T4 -T1) same. \i\t'hich straight line corresponds to which process? The
graphs of what process coincide with the coordinate axes?
Efficiency of cycle,
w
"~1_l'kl4 lanA, =1-
IQHI tan62=~
2
=I_C"CT4 -Ttl
tan 03 '" 1
Cp(T3-Tz)

=1- (74 -T1)


y(T3 - T2) Q

Process 3 ~ 4 is adiabatic. Flg.2E.46

~(VV,')r-
t
1'1", = rr-1
1 Solution: From first law of thermodynamics,
,
Q ==W + t.U
We can see that
For an isothermal process, Q == W, stope of W-Q graph is
1, hence straight line 3 represents an isothermal process.
Thus, For an isobaric process,
W == Rt.T (for one mole of gas)

www.puucho.com
Study Anurag Mishra with www.puucho.com
THERIIOOYItAMICS

Q=C "T=(f;2)Rl>T
p
corresponds to C p of a monatomic gas and G of a diatomic
L.

gas. Slope of straight line 1 is maximum, i. e., it corresponds


where f is the number of degrees of freedom. Slope ofw to minimum heat capacity, hence it corresponds to a
versus Q line is monatomic gas in an isochoric process. Similarly, slope of
W 2 straight line 3 is less, corresponding to greater heat capacity.
-=-- Hence.it represents an isobaric process involving a diatomic
Q f +2 gas.
For monatomic gas f = 3 and for diatomic gas f = S.
Thus slope of the straight line representing the W versus Q is
2/5 for a monatomic gas and 2/7 for a diatomic gas. Straight
line 1 corresponds to a monatomic gas and straight line 2 A gas is expanded from pressure PI' volume VI to final state
corresponds to a diatomic gas. volume V2 in two ways, isothermally and adiabatically. In
In an isochoric process work done is, zero, the which of the two processes is the final pressure higher and in
corresponding straight line coincides with the horizontal which is the work greater?
axis. p p
Compression
In an adiabatic process heat is not absorbed, the
corresponding straight line coincides with the vertical axis.
,,
Example ,,,
--~-"''-._'=-_.-.
.,
i~i .
. •
,: Isothermal. ,
In Fig. 2£.47 are shown different processes for a monatomic
and diatomic gas on a temperature versus heat supplied v, v, v
diagram. Which processes correspond to these straight lines? (.J (')
The graphs of what processes coincide with the coordinate Fig.2E.48
axes? The initial states of the two gases are the same.
oT Solution: An adiabatic curve is steeper than an
isotherm. Hence for expansion to same final volume, the
final pressure in the case of adiabatic process will be lower.
The area under isotherm is greater than the area under
3 adiabatic curve. The area under P-V diagram represents
work done.
In case of compression, from same initial volume to final
a volume, final pressure attained in case of adiabatic
Fig.2E.47 compression.

Solution: Along horizontal axis, I!T = 0, hence it


represents an adiabatic process.
Along vertical axis Q = 0, hence it represents an Fig. 2£.49 shows two adiabatic curves for two gases, helium
adiabatic process. and carbon dioxide. Which cu",e corresponds to which gas?
Heat absorbed in a process is given by p

Q =nGI!T

I!T =(n~)Q

Hence a straight line on I!T- Q diagram has slope (l/nG).


Molar heat capacity of a monatomic gas in an isochoric
process is, G" = (3/2)R and that of a diatomic gas is,
'--------_v
Gv = (S/2)R. Molar heat capacity of a monatomic gas in an Flg.2E.49
isobaric process is, G p = (S/2)R and that of a diatomic gas is,
C, = (7/2)R. Solution: An adiabatic process is given by
Since heat capacity Gv of a diatomic gas and the heat PVY = constant
capacity Gp of a monatomic gas are same, the slope of tiT_Q
curve (a straight line) is same. Therefore, the straight line 2 IG
where y = G p \J = 1 + (2/ fl. f represents degrees of
freedom. A helium molecule (monatomic) has three degrees

www.puucho.com
Study Anurag Mishra with www.puucho.com

THfRMOOYNAMICS

of freedom and that of a carhon dioxide molecule has six. On the log-log scale the pressure dependence of
Therefore, for helium we have y = 5/3 = 1.67 and for carbon compressibility is shown by a straight line. The lines
dioxide we have y = 8/6 == 1.33. The greater value, of y corresponding to isothermal process and adiabatic processes
represents a steeper P-V curve. The upper curve corresponds involving argon and carbon dioxide are shm ..•..
n in Fig. 2£.52.
to carbon dioxide and the lower curve corresponds to
Example 51
helium. • •.'c' ....' .--
.. '.

Find the /lumber of degrees of freedom of molecules of a gas:


(a) whose molar heat capacity (C) = 29 l/mol-K
Give expression for the compressibility of an ideal gas as a (b) whose molar heat capacity (C) ""29 llmol-K when
function of pressure for two cases, one when the gas is PT = constant.
compressed isothermally and the other when the gas is
compressed adiabatically. Draw a rough graph of Solution: (a) According to eguipartition of energy
compressibility versus pressure in the two cases. Also draw on theorem gas with N degrees of freedom has internal energy
the log-log scale the pressure dependence of compressibility for N
U "" - oRT for 0 moles of gas
argon and carbon dioxide. [s the isothennal line for argon 2
and carbon dioxide same?
C, ~~dU~.!.3-(N nRT)~NR ... (1)
ndT ndT 2 2
Solution: The compressibility is given by the
relation From the relation,
P logP
Cp ""C" +R
N
=-R +R
2

~ ,,,'"'~,,
P'~"
Isothermal
~(N;2)R ... (2)

Adiabatic process
P logP Hence y=C =(N+2)
p ... (3)
C" N
Fig.2E.50
From egn. (2),
I dV R 2
~~--- -~--
VdP Cp N+2
For an iSOlhermal process, 8.314 2
PV = constant --~--
29 N + 2
PdV+VciP=O which on solving for N, yields
I
Hence, (f\)isOlh"rmaJ = p N =5
(b) The given process is
For an adiabatic process, PT = constant (1)
PV~ = constant From ideal gas equation, PV = oRT (2)
'(PVy-1 dV+VYdP""O Eliminating T from eqns. (1) and (2), we have
I p2V = constant or PVL2 = constant
H~nce, (Il) 3diabatic =-p
y This is a polytropic process, with n = 1/2.
For an isothemtal process,J3P = 1 ""constant For a pOI}1rOpicprocess, molar specific heat is given by
R R
For an adiabatic process, I3P =.!.. "" constant C~----
y y-l /1-1
On all versU.'i P diagram isothermal as well as adiabatic
If N = degree of freedom, " __(NN+ 2)
I

processes are represented by hyperbolas; the constant


decides the nearness to origin. The constant is lesser for
adiabatic process, hence it is nearer to origin.

www.puucho.com
Study Anurag Mishra with www.puucho.com

N+ 2
With C = 29J/mol-K, R = 8.314J/mol-K, Y = -- and Example
N
I
n =-, we have
2 Consider an ideal gas with density p = 1.3 mglcm3 at
29 I 1 standard pressure and temperature. If velocity of sound
-- + ---
&314 (-1/2)
= ~---~
[(N;2)_I] propagation in this gas is v = 330 mis, calculate the degrees
of fr.eedom of gas molecules.
'.' ,
On solving for N, we get, N = 3. Solution: Speed of sound is given by

v=ff
COTL5idera mass m = 15gm of nitrogen enclosed in a vessel at where p is density, P is pressure, and y is adiabatic
constant for the gas.
temperature T = 300 K. Find the amount of heat required to
double the root mean square velocity of. its molec!;IJes. From the above equation. we can find y.

Solution: All the diatomic gases (H2• ° 2, ;-J2• etc.)


pu'
y=p ... (1)
exhibit 5 degrees of freedom between 100-1000 K, 3 of
translation and 2 of rotation. Hence, internal energy of n Now, y depends on the degrees of freedom of gas
moles of N 2 at T = 300 K would be molecules. If there are f degrees of freedom, then average

U =~ nRT
inte~al energy of each molecule is f>< ..!. kT. Therefore,
. 2
2
internal energy of a mole of gas.
Translational kinetic energy of each molecule is
I , 3 V=No><l.kT
"2 maurms = '2 kT 2
1
=-RT
where rna is the mass of the molecule. 2
If rms speed increases a times, v'rms = a v mu' then where No is the number of molecules per mole
temperature has to be raised to T' such that (Avagadro's number). The molar specific heat at constant
volume is
2.Qrms
m v.z =~ kY'
C =dU =l.R
2 2
t' dT 2
or Y' = a.2y 1+2
and Cp =C" +R=--R
The internal energy therefore increases to 2
Cp f+2
U. =~ nRT' Therefore. y=-=-- ... (2)
2 C, 1
Change in internal energy, From eqns. (1) and (2),
W =U' -U =~ nR(T'-T) we get
2 1= (Pv,2
--I
)
5 , P
~-nRT(a -I)
2
On substituting numerical values, p = 1.3 kg/m3,
=~mRT(a2_l)
2M v = 330 mis, and P = 1.013 >< lOs N/m 2, we get

where the number of moles n = m ,m is the mass of gas n=S


M Degrees of freedom are 5. It is a diatomic molecule with
and M is its molecular weight. 3 translation and 2 rotational degrees of freedom.
On substituting numerical values, m = 15 g, T = 300 K,
a = 2 and M = 28g/mol for N 2' we get
5 IS
U =-x - x 8.31><300><(4-1)J Determine the specific heat capaczttes, C" and C p' for a
2 28
mixture of7 g afnitrogen and 20g of argon. Assume the gases
= 104 J. to be ideal (take y for N 2 and Ar as 1.40 and 1.67,

----
resp"ectively).

www.puucho.com
Study Anurag Mishra with www.puucho.com

THERMODYNAMICS

Solution: Let masses ml and


be mixed. If M 1 and M 2. are their respective molecular
m2. of the ideal. gases
n1(_R
Y1-1
]+n,(_R Y2-1
J~(nl+n2) R
1rnix-1
weights, then the number of moles of the two gases present
in the mixture are where C,. = ~ . On solving for Yrna we arrive at egn.
m1 '1-1
") =- (4).
M1
'.

and n2=-
m, Example
M,
Further, let 6Q1 and L'!.Q2 be the heats required for unit Consider one mole of an ideal gas whose pressure changes
rise of temperature, at constant volume. for the two gases
with volume as P = aV, where a is constant. If it is expanded
when they are heated separately. In isochoric heating, all the such thar its volume increases 'l times, find the change in its
heat energy goes in raising the temperature ofthe gas; thus
internal energy, work done by the gas, and heat capacify of
the gas.
6Q1 =nje"l
and 6.Q2 == rtZC"2 Solution: Let the initial volume of the gas is Vand it
where, C til and C"2 are molar specific heats at constant
is expanded to volume 'lV. The work done in the process is
given by
volume of the two gases, respectively.
Now, the gases are mixed and again heated at constant W = v J,V
P dV =
Jv,v aVdV
volume till their temperature rises by one degree. Let tJ.Q be 2
the heat supplied to raise the temperature of both =aV ('l2_1)
components individually by one degree, 2

i. e., dQ = .6Q} + dQ2 The pressure of the gas as a function of volume is


P = aVo The initial and final values of pressure would be aV
If C" is the molar specific heat at constant volume of the
and TlaV respectively. Hence, the change in internal energy
mixture, then we have
of the system is
+ n2)C" ... (1)
(nl u = p[V[ -PjV; _ 112aV 2 - aV2
1-1 y-l
,
= aV (Tl2 -1)
In terms of masses of the gases, we have
Y-1
m}M2Cc' + m2M}C"
C" = I 2 •.. (2) Internal energy change corresponds to change in
m]M2 +m2M]
temperature from T; to T[ , such that
n]CPi + n2Cp
Similarly Cp = 2 ••• (3) T _ PiVi _ aV2
n1 + n2
i- -
nR nR
Adiabatic constant Yis given by p[ V Tl2aV 2
and Tf =--~~--f
Y = Cp = n1CPi + n2Cp2 nR nR
C" nlC"l + 7l2C"2 The total heat exchange in the process is given by
_ n]y] (rz -1) + 7I212(Y1 -1) Q =U + W
7I](Y2 -1)+ 11 2(y}-1)
... (4)
~ aV'("' -I) (_1_ +~]
y -1 2
where in the last expression we used the identities
R =!UV2(112 -1) ()' + 1)
C,. =-- 2 (y -1)
y-1
and Cp=yC" The heat capacity C of the gas is, therefore,
substituting numerical values Q
On C~--
7 20 Tf -T,
n1 =- and n2 =- andalsoY1 =1.40'Y2 =1.67,weget
28 40 .!aV2(Tl2 _1)~y+l)
C" = 0.42J/K; ~ _2 ~('1_-_1,-)
C p = 0.65 J/K. uV2('l2 -l)/nR
Alternatively we can proceed from egn. (1) as

www.puucho.com
Study Anurag Mishra with www.puucho.com

nR(y+l)
=----
2 (y - 1) ,1m
Since n = 1, we have
" k
C=!!:.(Y+l)=C +!!:. m,A
2 (y -1) " 2
g"

Exq;m(;!1
.- .-~- -. ~_.

COnsider one mole of an ideal gas whose volume changes with


Fig. 2E.53 (a)
temperature as V = ~ , where a is constant. Find the amount
T Initial Condition:
a/heat required to raise its temperature by ~T, ifirs adiabatic
constant is "(. From equilibrium of piston we get
PiA = -mg + PalmA
Solution: According to ideal gas equation, mg PjA mg
Pi =-+Pa1m ... (1)
PV =nRT A Fig. 2E.53 (b)
Since volume V of the gas varies with temperacure as by PV = nRT,
V == ~, pressure as a function of volume is PV
T PiV=nRTi~Ti=-'-
nR
PV=nR~ Final Condition:
y

or p = nRa
P,A = PalmA + Kxl + mg ~K',
Kxl mg
y' P, =Parm + __ +_
A A P,A mg
The work done in the process is
PjV, =nRTI ~TI
P,V,
=-- Fig. 2E.53 (e)
W =fvf PdV=fv/ nRa dV nR
VI Vi V 2
•. Change in internal energy 6.U = ~ nR(Tj - Tj)
2
=nRa(2- __ ! j=nR(T; -TIl •. Increase in G.P.E.of the system = mgxl
Vi VI
••. Increase in elastic P.E. =..! Kxl
Since Tj < Tf ' W is negative which implies that work is 2
done on the gas. Change in internal energy is given by •. Work done on atmosphere = ParmAxI (P.1.V)
nR(TI - T) Using first law of thermodynamics, we get
u= I
y -1 3 'I
Q = -nR(TI -Tj) + mgxl + -Kxl + PatmAxJ
Thus, heat required to raise the temperature by 2 2
6.T=Tf -Tjis mg

Q:U+W P.~
= nR!J.T _ nRtlT
y-l P••• A

illustration: n moles of monoatomic gas is present Fig. 2E.53 (d)


inside an insulated cylinder. The initial volume of the gas is
V and initially the spring is in a relaxed state. Now by the
Work done by the gas on the surroundings
help of the heater the gas is slowly heated and finally the (i) Atmosphere, (ii) Spring (iii) Against gravity
compression of the spring is Xl find out, (i) initial mg Kx
Pgas=Pum +-;-+-:4
temperature, pressure, (ii) final pressure, volume and
temperature (iii) heat supplied by the heater.
w = f PgasdV

Kx
Adx
A

www.puucho.com
Study Anurag Mishra with www.puucho.com

THERMODYNAMICS

Alternatively we can use work energy theorem to get Example 57 _


--N~a, + Wa11ll + Wgrn'ilY + Wspring == 0

Wgra,;ry =: -mgx} One mole of an ideal gas is present inside a vertical cylinder at
1 a temperature T. The piston is ma$sless. Find out the work
Wspring =-k(x} -x1) =-PoA\"}
2 done by an external agent in increasing the volume of tlie gas
Wa1rn = Po[V! - Vi] = -PON!
by K times by slowly lifting the piston. Temperature of the gas
1 , remains constant.
Thus Wga, = mgxl + POAxJ + - kx1
2
Palm
Illustration: n mole of diatomic gas has been enclosed F
in cylinder (1) and (2). Initially the spring is in a relaxed state
A
_and pressure inside cylinder (1) is Pl, find out initial
-----------~~~
------_ .._--------
..~~.~.~----------
-temperature of cylinder (1) and initial temperature, pressure ._---------------_
---------------~.~
-~~~~~~~~~~~~~~ ..._ ... -
.---.-
of cylinder (2). Now Q joule of heat is given slowly co cylinder -~~~~~~~~~~~~~~
-------------~.~ ...._---- .
(I), finally the spring gets compressed by x and the pressure
-------_
--.-------- ....~~~.~--------
-----_
:.:.:.:.:.:.:.:.:.:.:.:.:-:.:
P gas :.:.:.:.:.:.:-:.:.:-:.:.:-:-:-
_~~-~~~ -••• -.-.-_. c"_-_"_-_~-_-_-_-_-_-
inside cylinder (2) is P2' Find out final volume, temperature
-----------------~-~~
-------_ .._-"""~"~------- ..--.
and pressure in both the cylinder. Find our the value of Q. .-~~~~~~~~~~~~~
~~~~~~~~~~~~~~
-------_ .....~----------
----_.......
------------ -----------
--~..~..----
2

a- - --- --- "m -- K


a
Fig.2E.57

~
-- - -- Solution: (1) Wga< + WF = Wmm

,,
P,A
P ,V
- - PalmA
PatmA-
-- {
P2,A

V,
work done by the gas + external agent = work done on
the atmosphere
A
(2) If gas is doing work and at the same time its
A
temperature is constant. Then it means that it is getting heat
Fig. 2.54
from the surrounding
PI V} Q = Wgas
Initial Condition: T =--
" nR WF =Walm -Wga• ""Pmm(KV-V)-RTlnK
(By force balance on both)
Initial Condition P; V = RT ; Pi = Palm
Palm V ""RT
Final Condition: V2f =V2 -Ax'; WF =RT(K-l)-RTlnK
Vlf == VI + Ax The heat absorbed from the surrounding,
PitA =P2A+Kx Q = RTinK
Plf Vlf = P2t V2j Illustration: A piston can freely move inside a
Tlf =-_.; T2t
nR nR horizontal cylinder closed from both ends. Initially the
We can also find out the final temperature, pressure and piston separates the inside space of the cylinder into two
equal parts each of volume Yo, in which an ideal gas is
volume.
.•. Net on workdone on the atmosphere is zero because contained under the same pressure Po and at the same
compression on one side = Expansion on another temperature.
side What work has to be performed in order to increase
isothermally the volume of one part of gas n times compared
Wga< + Walm + W<pring = 0
to that of the other by'slowly moving the piston?
Thus Wgas= -Wspri,,~
and from first law of thermodynamics we have
Kx'
Q=t.Ut+t.U2+--
2

www.puucho.com
Study Anurag Mishra with www.puucho.com

202
IfQjoules of heat is slowly supplied by a heater to L.H.S.
" :Pl P,
till the final pressure of R.H.S. becomes P2 Find out (i)
P, (Yo +~) ". Pressure, volume and temperature finally (ii) the total heat
,,,
". ""Pc
- 1--,
, supplied by the heater.
, The gas in R.H.5. gets zero heat from the surrounding
TO = constant T(l = constant therefore the process in R.H.S. will be adiabatic
• Fig. 2.55 ••• __ • _. • • ••••• System
: : boundary
Finally F = (Pz - PI)A for (I) chamber
: Po.Vo,To PoVo,To ,:
PoVo =:: PI (Vo + Ax) ,
for (II) chamber
Q
,,
,
.
•,
,,
,
PoVo =P2 eVa -Ax) .._-----
~ __ .._._-'
F ~(PoVo PoVo 1.", FIg. 2.56 (a)
Va-Ax Vo+AxJ'
POV02A1x Povo" = P2 VfY Y = ~ for monoatomic
~ , 3
,,2 A 2X2

[A~,'3)"
" - V - 0 0 •
fR - P ,
W.,cnl =fF'dx=2A2PoVof,,,"1 X dx
Vo 2_A2x2

The cotal no. of moles in R.H.S. = PoVo


Initially x =:: a RT,
Finally x=x1 P2Vf
Final temperature in R.H.S. Tf = --'-
Vo + Ax} = neVa -Ax,) _.. R nR
Vo(n -1) Vir =2Vo -VI
:::> XI =
A(n + 1) . Since the process is carried out very slowly therefore the
The total heat given to the whole system goes to piston will always remains in the state of rest of mechanical
2 equilibrium.
increase the internal energy ofL.H.S. and R.H.S. + Kx i. e.• PIr = P2 (Pressure from both sides equal)
2 P2V
Q ==.1UL+6UR +-Kx
1 , --"..!.Lf
TfL-nR
2
If we consider both sides as one system then the net heat
Calculating Work Done by Integration
given to this system will be equal to the increase in internal
Since the process is reversible therefore the piston energy of L.H.S. and R.H.S.
should be in the state of mechanical equilibrium at all the 3 3
stage of the process. t..UL=n-R(Tf -To); t..UR =n-(TJ -To)
2 L 2 R
Pga,A = mg + PalmA + Kx
If in the question given piston is adiabatic therefore no
mg Kx
Pgas =-+Pa1m +- heat transfer can take place across it.
A A
Case when the piston is diathermal: In the above
f
W = PgasdV = f: v
Pa1mdV + ":: J dV +f ~ .Adx diagram the piston is diathermal. The cross-sectional area of
the piston is A. The spring shown is initially in a relaxed
:=:Pa1mdV+ mgJ:l dx+KJ;l xdx state. The final volume of R.H.S. = V2• Find out the final
pressure, volume and temperature on both sides. Find out
Kx' the total heat given by the heater.
W "" Pa1mAxl + mgxl + --'
2 The temperature of L.H.S. and R.H.S. will increase
If the force balance was not taking place then the kinetic uniformly. Let us assume that the final temperature is T
energy of the system would have continuously increases. L.H.S R.H.S

Illustration: In the diagram shows an insulated


cylinder di••ided into two equal parts by an insulated piston. Q
Equal moles of monoatomic gas has been enclosed inside the K
L.H.S. and R.H.S. chamber. The diagram shows initial states
of the gases in L.H.S. and R.H.S. FIg. 2.56 (b)

., www.puucho.com
Study Anurag Mishra with www.puucho.com

!THERMODYNAM CS 203

Applying ideal gas equations


PiR V2 '" nRT ... (1) =~[3nRT+ ~KL' T)
y-1 nR
Pir (2Va - V:2) = /tRT ... (2)
The piston will always remain in the
state of mechanical equilibrium. The final
. . h . Va - V2 p~"
=( 2nRT + %KL
2
)%
compressIOn In t e spnng x = ---
A
If the force balanced on the piston ~ PfRA
Q -Q
s r
=3nRT+!'KL'+~X2[K(~)']
4 2 2
Kx+PjRA=PftA ... (3) Fig. 2.56 (c)
5 ,
we can calculate the T from the above equations (3) by = 3nRT + - KL
2
,
substituting PI and P,., from ego. (1) and (2).
Illustration: Figure shows two cylinders partition by Examele
movable diathermal spring loaded piston. Left chamber is
diathermal i.e.) allows heat to loose to environment. Heat A large botrIe is fitted at its neck with one-liole stopper
given by heater is Qg and heat rejected by left chamber is Qr' through wliicli passes a precision bore glass tube. A steel ball
If we assume the cross-sectional area to be A. Difference bearing of mass m, radius rjust fits inside the tube. The bottle
Qg - Qr will be equal to the sum of increase in the is filled with the gas for which y(C p Ie is to be calculated.
l' )

mechanical energy and thermodynamical energy. Inside the rube gas leaks through the narrow space between
the ball and the tube, gas forms a gas cushion that keeps the

1 t 1
-U2- -2L-~-----..j
A
ball from rouching the wall. Thus the ball can slide with little
friction. A very slow flow of gas into tlie bottle through the
K side arm compensates the leakage of gas. When the ball is
lowered into the tllbe, it would compress the gas to an
o ,T o ,T
equilibrium pressure Po, volume Vo' A small displacement
-L--L- Q
,

Fig. 2.57 (a)


Qg
from this equilibrium position will cause oscillations of the
ball with frequency v.

Mechanical energy
-
=
2
(L)'
2 x -1 K -
2
Internal energy wiH change only of the
gas present in the R.H.S. KL KLJ2
AL 2"
Pf .-=nRT
, 2
2nRT Fig. 2.57 {bJ
Side arm

AL

Pit A +KL =PfRA


KL
Pf , =Pf , +-
A
Fig.2E.58
2nRT KL
=--+- 2
AL A
(a)5how thaty = 4v mVo, assuming process to be adiabatic.
3 poy4
Pf -LA = nRTf
, 2 '
(b) Determine the frequency of oscillation if the process is
2nRT +~)3LA isothermal.
( AL A 2
TfJ.: nR
(c) What is the force constant of trapped gas?
(d) What is the amplitude of oscillation of the ball?
2
(3nRT + %KL )
Solution: (a) The weight of the ball and the
nR atmospheric force P'lIm x A must be balanced by the upward
force PA exerted by the gas pressure.
l'.UR =nC,,(TfR-T)

www.puucho.com
Study Anurag Mishra with www.puucho.com

lHEIIIODYNAMl
mg Cd) Amplitude of oscillation,
Po =- + P"trn
A mg mgVo
X=-=---
The conditions are adiabatic, hence k VPo A2
PVr = constant ... (1)
Differentiating this gives
PCyVr-1) dV + V7dP = 0
:4. glass U-tube of unifonn cross-sectional area A, open at both
dP=_yPdV ends, contains a volume Vo/liquid of densityp. Thw the total
V length of the liquid column from surface to surface is I = VI A.
=_VPo.(Ad.y) ... (2) rrhe tube is jiggled to set liquid into oscillation.
Vo (a) When the liquid surface on the right lies a distance y from
where dy is small vertical displacement of ball. its equilibrium position, show that the liquid experiences a
Hence restoring force is restoring force of the form F = -kyo Find the period of
2 oscillation.
dE =_ yFoA dy
Vo (b) One end of the tube is now closed so that the length of the
entrapped air column is L. Show that for small oscillations of
Acceleration of the ball is thus 'I
the liquid column oscillation is approximately simple
2
a =_ yPoA d'"" ... (3)
harmonic. Find the time period of oscillations assuming
mVo adiabatic and isothermal conditions.
This is equation of simple harmonic motion with (c) Assuming adiabatic conditions determine y for the trapped
angular frequency of oscillation gas.
r-~~
2
YPoCnr
",=,1-"="-'- )2 Solution: (a) When the

from which
mVo
=2nv ... (4)
liquid column is displaced by y, in
left column, the weight of excess
r
D
liquid in left column is the
4v2mV:
y = 0 ... (5) restoring force y
Por4 F =-(pg2y)A
(b) If the process is isothermal, Since F oc(-y), the oscillation
PV = constant ... (6) is simple harmonic.
PdV+VdP=O ... (7) . Therefore, acceleration of the Fig.2E.59
dP =_PdV liquid column,
V a=-2pgAy
PAdy pIA
=---
V
dF =_
Z
PO(rtr Y2
Vo
dy "'=J~Z:=ff=:~
p. (nr2)2 To = 2rr~ ...(1)
a ""_ 0 dy ... (8)
mVo
•. The period of oscillation depends not on p but only on
p. (nr2)2 . I and is mus independent of me panicular liquid used.
00= 0 =2nv ... (9)
mVo This problem was proposed and solved by Newton.
(b) For an adiabatic process
v= l..-jP 2
o(nr )2 ... (10)
dP=-Y(~)dV
2n' mVo
ee) From eqn. (4), yPoA
=----::;Ly
yPo
=--y
L
Force constant, Total force on the liquid column in right arm is due to
gas pressure and weight of excess liquid.

www.puucho.com
Study Anurag Mishra with www.puucho.com
IHERItIOOYNAMICS 205
poVJ = P(Vo - AxV
Restoring force = -[ y~O+ 2pg] Ay
poVJ
which is proportional to y, hence motion is simple P, - ( ,
harmonic.
Acceleration of liquid column,
VJ1-~:)
(Y~o+4W
Q=_._-----
lAY "Po(l- Ax)-' Vo
pAl .',
= Po ('f1Ax)
+ V
From the expression for Q, o
"("'yP"o-,L""'+c:q,"g:')C7A 2it
"- (as Ax is small quantity)
pAl T Vo
pi Similarly for the left part,
Tadiab.1tic = 27t ... (2)
(ypoiL + :q,g) poVJ ""PLVl =PL(Vo+Ax)T
For an isothermal process, proceed according to.
PdV+VdP=O
dP"-l'.dV
V Fig. 2E.60 (b)

to obmin T
isothemlal -
- 2Jt I
\ (PoiL + 2pg)
pi
.' Net force acting on {he piston,
(c) From eqns. (1) and (2), we can obtain

y=pgL(T(f -1) F "-HI+ Y~)A -Po(I-Y~H


Po Tu~
2
=-2 PoyA x
Vo
Acceleration of {he piston
A closed and lsolated cylinder contains ideal An adiabatic
gCL5.
separator of mass m, cross-sectional area A divides the 2P. yA 2
o n
cylinder into two equal parts, each with volume Vo and Vom
pressure Po in equilibrium. Assuming that the separator can
On comparing the acceleration with a = -(J}2 x,
move without friction, find the oscillation frequency when the
separator is slightly displaced. We get <.tl = {2PeYA 2 = 2l't = 2;-cv
r--T'r-'-~,---, \ Vern T
: po. Vo 2
, v=-.!..-/2POYA
: PR
:- 2l't \ Vem

Fig. 2E.60 (a) Exo,,:,gl.e


Find the molar heat capacity (C ) of an ideal gas in a
Solution: Let the pismn be displaced by a small
polytropic process represented by PV II = constant. Ratio of
distance x towards right.
specific heats is y. Find the value of n for which heat capacity
Volume of the right part ""Vo- Ax is negative.
Volume of the left part"" Vo + IU"
As the cylinder is isolated from surroundings and
Solution: In a polytropic process represented by
separator is adiabatic (n<m~conducting), the process is pv II = constant, work done on one mole of gas is given by
adiabatic. the expression
=P1V\-P2V2 =R(T\-T2)
W --- --- ... (1)
n-l n-l
[right part 1

www.puucho.com
Study Anurag Mishra with www.puucho.com

206 !HIUIODYNAMICS •

Change in internal energy. when temperature changes


-2R)dT =PdV
from T1 to T2, is (
'. y-l
liU ""Cv (T2 -Tt) = R(T2 -Td ... (2) ::-From ideal gas equation,
, -I
PV =RT (one mole of gas)
If C is the molar specific heat,
. Hence RT dV+2RdT =0
Q =C(T, -T,) ... (3) V y-l
From first law of thermodynamics,
Q=h.U+W
... _2_)dT + dV = 0
( y-l T V
C(T -T ) = R(T1 -T2) + R(J'2 -T1)
2 I
n-l y-l dT +(Y-I)dV =0
T 2 V
C=~-~
'(-1 n-l
R(n -,)
logt T +(Y;I) logt V = constant

=-----
(n -1)(, -I)
(Y -1)

Therefore IV 2 = constant
For C to be negative, we must have 1 < n < y.
(b) Equation of the process in terms of pressure
Example and volume of the gas can be obtained by eliminating T with
t~~ help of ideal gas equation PV = nRT.
In a thermodynamic process a gas expands such that the heat Equation of the process is
transferred to the gas (dQ) = decrease in internal energyj
(-dU> PV
(''')
2 = constant
(a) Find the molar heat capacity and the equation of the
process in variables T and V. Th"IS IS a po I'ytroPIC process wit. h n = [,-2-
+ I ) .

(b) Find the work performed by one mole of the gas, when its
volume increases '1 times, if the initial temperature of the gas Work done is W = PIVI - P2 V2
is To. n -I
R(Tl - T2)
Solution: (a) (i) Change in internal energy when = ---- (for one mole of gas)
n -I
temperature changes from T to T + liT, for one mole of gas,
is
r
T+ ••• RTo(I-~)
fj,U= JT C<.dT =----
n -I
(sinceTI =ToandTz =T)
T+AT R
=
J T
--dT
y-l
From equation of the process,

=--
MT T1 = To = (V, )(Y-I}!2 = 11(Y-1),12
,-I T2 T VI

Heat transferred to system for one mole of gas, RTo(l - 11(Y-lJ/2) RTo(l- 11(Y-I)'2)
So, W= -------
Q =C~T = -~U =[__ R_] ~T
(y -I)
(,;1)-1 (';1)
-R
or C =--
,-I
(ii) From first law of thermodynamics, :4 vertical cylinder of volume V has n mole.s of an ideal
dQ=dU+dW monatomic gas. The walls of the cylinder are thermally
insulated, the piston is weightless. When a mass M is placed
But dQ = -dU
on the piston. the piston is displaced by a distance h. What is
Hence 2dQ = dW the final temperature of the gas after the piston has been
or 2CdT=PdV aisplaced. the area of the piston is A and the atmospheric
pressure is Po?

www.puucho.com
Study Anurag Mishra with www.puucho.com

THIRMODYNAMICS 207

Solution: Since the gas is

[_B
M Solution: if 6.Q is the heat
thermally insulated, the entire work done supplied (Q the system, it is spent in
on the gas is equal to change in internal the change in internal energy of the
energy. When the piston is displaced by II, system. Since there is no friction and
the volume becomes V - hA. The nct the vessel is thermally insulated, Fig.2E.64
pressure on the piston is Po + (mg/A). AQ = AU
From ideal gas equation before and after
6.Utotal =6.U~as + f,U.pring ... (1)
loading we have
Flg.2E.63 There are two changes in the internal energy, change in
internal energy of gas &Ugas and that of spring t.Uspring'
PoV '" nRT1 ... (I) 3
t.Ugas ='2R(T2 -Tl) .•. (2)

[Po + (M~ )] [V - hAl = nR1', ... (2)


1 , ,
t.U.pring ='2 k(X2 -Xl) ...(3)
The change in internal energy
where Xl and x2 are compression in spring at
~u= nC"t1T = ~ nR(T2 - 1'1) temperatures Tt and T2 respectively.
2
In equilibrium,
Work done on the system = Mgh
Pressure due to gas = Pressure due to spring
Therefore, from first law of thermodynamics
• P= kx
Mgh =~ nR(T2 -1'1) ... (3) A
2
PA
Subtracting eqo. (1) from eqo. (2), we have x=- ...(4)
k
[PoV + (:g) V -PohA -Mgh -PoV] From ideal gas equation, for one mole,
PV=RT
=nR(T2-T1)
P = RT = RT ... (5)
nR(T, -1',) =[[ ~)v-PohA - Mgh] ... (4)
V Ax
From eqns. (4) and (5), we have
Substituting nR(T2 - 1'1) from ego. (4) in eqo. (3), we , RT
x =-
have k
MgV 2 1 2 2 R
-- -Mgh - PoM = - Mgh Hence,-k(x 2 -xl)=-(T2-T1) ... (6)
A 3 2 2
Hence we obtain From eqns. (1), (2) and (3), we have
3 R
h = MgV L\Utolal =-R(T2-Tt)+-(T2-Tt)
A (PaA + SMg/3) 2 . 2
;2R(T2 -Tt)
Substituting h in eqo. (2) and solving for 1'2' we get
(PoA + Mg)(3PoA - 2Mg)V
Thus heat capacity of the system is
T2 =~~---~~---- C=AQ= AU -2R
(3PoA + SMg )AnR
L\T (T2-TI)

ExamRle
Example
A non-conducting piston divides a thennally insulated vessel
into two parts. The left part of the vessel contains one mole of. A thin heat insulating piston divides a horizontal cylindrical
an ideal monatomic gas and the right part is vacuum. A vessel of length 21inro two equal parts. Each part contains n
spring connects the piston to the right wall of the vessel, the moles of ideal monatomic gas at a temperature T. Two springs
natural length of the spring in the free state is equal to the of spring constant k each are connected to the piston on either
length of the vessel. Neglecting heat capacities of the vessel, side as shown in Fig. 2£.65(a). When heat Q is supplied to the
piston and spring, determine the heat capacity C of the gas in the right part, the piston is displaced to the left by a
system. distance x = 1/2 The left part is in contact with a thermostat
at temperature T all the time. Determine the heat Q' given
awa~ to the thermostat.

www.puucho.com
Study Anurag Mishra with www.puucho.com

IH

I_ 2/ _I
Example.
r:::s.::rc;:] One mole of an ideal gas is comained under a weightless
~ piston of a vertical cylinder at a temperature T. The space over
Fig. 2E.65 (a) the piston opens into the atmosphere. What work has to be
performed to increase isothennally the gas volume under the
Solution: Net heat added to the system is Q -Q'. piston n times by slowly raising the piston? Friction is
From first Jaw of thermodynamics, negligible.
Q=,1U+W
Solution: As the piston is displaced slowly the
Q-Q'=~U+Us ... (1)
change in kinetic energy is zero. From work energy theorem,
where 6.U is the change in internal energy of the system we have
and Us is the elastic potential energy stored in the spring.
The work done by the gas has changed the potential energy
of the spring.
The temperature in the left part has not changed as it is
in contact with a thermostat and piston wall is
non-conducting. Hence the change in the internal energy of "VO dV
the gas is due to the heating of the gas in the right part by
~T.
=RT Jv, -
V
[PV = nRT]

=RTlnn
So, ... (2)
Thus, wex1 =-wann -wga.o;
Gas is monatomic. =-[-PoVo(n -l)+RTlnn]
=RT[(n-I)-lnn]

Fm'es acting 0:" ~h:Rpiston


shown in Fig. 2E.65(b). In equilibrium
Me :'A:~.P"'~
2=kAx killl=2
~xnmple

In Fig. 11E.40 shown, an adiabatic cylinder of volume V has


PLA + 2kM = PRA ... (3) Fig.2E.65(b)
monatomic gas in both the compartments. The piston is
When the piston has moved by a distance x = 1/2, the diathermic. Initially the piston is keptjixed, and the system is
volume of the left part, VL = A(l-1/2) and the volume of the allowed to acquire a state of thermal equilibrium. The initial
right part VR = A(l + 1/2) . From ideal gas equation, pressures and temperatures in respective compartments are
P, = nRT ... (4) PI' P2 andT}, T2• Calculate the final temperature and the final
A(I-1'2) pressure. Now the piston is set free to move slowly such that a
state of mechanical equilibrium is also achieved
P _nR(T+.6T) (5)
R - A(I + 1!2) ... (T1P2 > PIT2). Calculate the final volume of the gas in each
compartment.
Substituting expression for PL and PR in eqn. (3), we
have Solution: (i) Let T be the final temperarure in both
2"R(T + ~T ) 2nRT kl
(he compartments, then from first law of thermodynamics
----=--+- ...
(6) applied to the cylinder,
3Al AI A
which on solving for .6T, yields
2 P,. T,. va
tiT = If + 3kl
2nR
From eqn. (1), Flg.2E.67
Q'=Q -aU -Us
6Q=tiU+6W
=Q -~nR
2
[2T + 3kl']
2nR
_ kl '
4
As (he cylinder wall is adiabatic and fixed,
6Q=6W=O
5 ,
=Q-3nRT--kl so there is no change in internal energy of the system,
2 6U=O

www.puucho.com
Study Anurag Mishra with www.puucho.com
209
THERMODYNAMICS
Solution: We have, for an adiabatic process,
7Vy-1 =-constant
Differentiating with respect to T,
vr-I.1+T.(y_1)vy-2dV =-0
dT
dV v
For the left compartment,
dT
" T(, -1)
P1 _
~_PI P\ = final pressure in the left compartment
But from the graph,
T, T

or P
'
, -_ PI (PIz ++ P2 }Tz
PIT PlYI
IdVI
dT To. Vo
=-tan(;r-O)=-tanO

V
Similarly, p'z can be obtained. o =tan e
To(y -1)
P ,-
' _ Pz(p] + P2)T\
PiT2 + P'}.TI V, --
or y-1=-
(iii) As P2Tj > PIT2, it follows that P'2 > P'l .
To tan e
Therefore the piston moves to the left. Let 6.V be the or y = Vo _ 1
change in volume of any compartment. To tan e
Then on applying the ideal gas equation, we have "lR (Vo -To tan O)R
C ," __ "_~~~--_Totan 0
y-1 ToranO.Vo
Pf(~2- "v)
_____
1'1
=p\v
ZT1
... (I)
=(1-~: tanO)R
P_____
f(V2- + 6.V) =P2V
and C\' =~
Y -}
= RTo tan fl
Vo
and ... (2)
1'[ Zf2
Example
- -.-
On dividing ego. (2) by (1), -'.- .-

V A gaseous mixture enclosed in a vessel of volume V consists of


- +:<.V P T
_2__ =2-!- one gram-mole of a gas A with y(=- C piC v) = 5.'3and another
~ -6.V PtT2 gas B with y =7/5 at a certain temperature T. The
2 gram-molecular weights of the gases A and Bare 4 and 32
:. Volume of left compartment respectively. The gases A and B do not react with each other
and are assumed to be ideal. The gaseous mixture follows the
=~-6.V=V PlT2
2 P2Y1 + PiT2
equation PVl9-13 = constant, in adiabatic processes.
(a) Find the number of gram.moles of the gas B in the gaseous
and volume of right compartment
mixture.
Y
=~+D.V=V P2 I (b) Compute the speed of sound in the gaseous mixture at
2 P2YJ + PITz
T = 300K.
(c) 1fT is raised by 1 Kfrom 300K, find the percentage change
in the speed of sound in the gaseous mixture.
(d) The mixture is compressed adiabatically ro 1/50/its initial
A gas is u.ndergoing an adiabatic process. At a certain stage A,
volume V. Find the change in its adiabatic compressibility in
the values of volume and temperature:; (Vo• To). From the
details given in the graph (Fig. 2E.68), find the value of C p terms of the given quantities.
and Cv. Solution: (a) The y of the mixture is given by
v
'" --~~--~-~-
nl"l1 (12 -1) + TlZ"l2(Y1 -1)
TIl("12 -1) + n2(Y1 -1)

A5 YI =5/3, 12 =7/5,n1 =1,andy=19/13


-t-----T
Fig.2E.68 we find; 112 =2

www.puucho.com
Study Anurag Mishra with www.puucho.com
o THfRMOllYNAMlCS
(b) Speed of sound in an ideal gas is given by

v~(:r
where y and p refer to mixture of two gases,'" "" 19j13for
Example

n moles of an ideal gas is made ro undergo the cycle


~-t 2 -t 3 -t 4 -t 1as shown in Fig. 2£.70. Process ~ -t 4is
the mixture. The density p of the mixture is given by a straight line. The gas temperatures in states 1, 2 and 3 are
ntM) + n2M2 p
Tj, T2 and T) respectively. Also points 3 and 4 lie on the same
p~~----_.- isotherm. Detennine the work done by the gas during the
n1 +n2 RT cycle.
For n, =1, n2 =2, M, =4>dO-3kg/mol,
3 2 ~ •~3
M2 "" 32)( 1O- kglmo! and T = 300 K, we get
p=O.9xlO-5p kg/m3
~
,!. \4"':;"h'~
Therefore, Ii = (19 x 1 )1:2::: 402 mls Fig.2E.70
13 0.9xlO-5
(e) In terms of temperature. the speed of sound Solution: Let P and V be the initial pressure and
can be written as volume respectively, then for process 1 -t 2,
P, P
- =- ('.'volume remains constant)
T2 T,
T,
Thus, we have P, =P- ...(1)
T,
Ii ~ T~.'2
. For.process 2 -t 3
ill! 1 !1T 1 1 1
0' -x 100=--)(100=-x_xl00=_
'Ii 2 T 2 300 6 . Y3=~ => Y3=yT3 ... (2)
T3 T2 T2
i. e.• speed changes by 1/6%.
Cd) Adiabatic bulk modulus of a gas is B = "(P, so For process 3 -t 4 (temperature at points 3 and 4 is the
same),
Adiabatic compressibility is,
1 V4=yT2 ... (3)
K~- T,
yP
.. The work done by the gas = Area of the trapezium 1
Hence, the change in adiabatic compressibility is -t2-+3-+4
IV{ =Kj -Kj =!(J... _..!.-)
Y PI Pi
During adiabatic compression, we have

~vy =Pf(~r
or ~ =(~r But

Thus, we get

l1K ~2.(p,-1)
yPi PI Example 71
=~~(2.
sy
-1)
Y nRT :t\gas takes part in two thermal processes in which it is heated
from the same initial state ro the same final temperature. The
V
=-0.025- processes are shown on the P-Y diagram by straight lines
T 1 -+ 3and 1 -+ 2 Indicate the process in which the amount of
where we put n = 3 and y = 19(13 for the mixture. heat supp'lied to the gas is larger.

www.puucho.com
Study Anurag Mishra with www.puucho.com
211
JHERMOOYNAMICS

P ,,
, " . Therefore the process A --Jo B is
isobaric. Process B -) C is isochoric
P
P, \2 because density is constant. Process
P,
P,
'" C --) A is isothermal because internal
energy is constant. Now P-V diagram is ~A
v as shown in Fig. 2E.72(b).
VI V2 VJ V
Fig.2E.71 (b) Total heat absorbed by the Fig. 2E.72 (b)
system,
Solution: Since the temperature difference is same.
1'.T=O ~ Cl'.V=O)
1
Now W12 =-(PI +P2)(VZ -Vt)
2 As process B --) C is isochoric
1
Wl3 =-(PI +P3)(V3 -V1)(P2V2 =P3V3) QB-~C =t.U =3Uo
2 As process C --Jo A is isothermal,
1
W12 -W13 =- [PI(V2 -V'l) + (P3 -Pz)Vtl <0 VA
2 " QC ...•A = nRT In-
Vc
W12 > WI3
Q C~A =Rx lOUo In Pc
Q'2 > Q13 3R PA
Example 72~ =lOUu In~ (asp,\ =L.ro;Pc =PR =5po)
3 2
Fig. '2E.72 (a) shows the variation of the internal energy U IOUu
Q = ~2Uo + -- In (2.5)
with the density p of one mole of ideal monatomic gas for a 3
thennodynamic cycle A -) B -) C -) A.
u Rectangular by =(l~ In 2.5 - 2 o )U

:~:~.'..'~.'.~~~i" ,
•,
(c) WA ...•R =t.QA ...•R -L'J.UA_~fj
=-SUo-(-3Uo)

= -2110

2po r .Example
Fig. 2E.72 (a)
Fig. 2E. 73 shows two vertical cylinders with thennally
Assume the process A -) B to be a part of a rectangular insulated walls and pistons of same mass fitted on the top.
hyperbola. The cylinders enclose n moles of a monatomic gas. The initial
pressure is P and it is given that spring I is relaxed. An electric
Draw the P-V diagram of the above process.
heater slowly supplies heat to the system. Finally spring II
Find the total amount of heat absorbed by the system/or the
becomes relaxed.
cyclic process.
Find the work done in the process AB. "
k k
Solution: (a) Process A -) B is part of a rectangular
hyperbola; hence
Up = constant
From ideal gas equation, we have
"f" unction 0 r temperature
_P "_ RT an d"Interna 1energy ISa
r M Fig.2E.73

So U = aT, where a is a constant


What is the initial state of spring II? Given that A2 < AI.
PM = constant
Up = aT -
So What is the final pressure?
RT
If the initial volume is V, what is the final volume?
0' P = constant

www.puucho.com
Study Anurag Mishra with www.puucho.com
12
THEIlMODYNAMICS
Solution: (i) Let Po be the atmospheric pressure, then
considering the equilibrium of piston I, '
PAl =mg +PoA,
mg
or p = Po + - ... (1)
A,
Now consider the forces acting on piston II. According to
given condition, spring II is relaxed, finally. Before that we .Example
have to determine the state of spring by comparing
downward and upward force. 0.01 mole of an ideal diatomic gas is enclosed in an adiabatic
0'linder of cross-sectional area A = 104 m2• In the
Net upward force acting on piston II
arrangemenrshown in Fig. 2£.74, a block of mass At = 0.8 kg
A,
= PA2 = Po A2 + mg - [from Eqn. (I)] is placed on a horizontal support, and another block of mass
A, m = 1 kg is suspended from a spring of stiffness co1t~tant
Net downward force acting on piston II k = 16N/m. Initially, the sfring is relaxed and the volume of
the gas is V = 1.4_x 104 m ,
=mg +POA2
M
As A2 < AI' so net upward force is less, than net
downward force.
k
Hence to keep the piston in equilibrium, the spring must m
apply an upward force on it, therefore spring is stretched
initially

and spring force lex = mg (1 - ~: )

where x is the extension in the spring n. Flg.2E.74

(ij) When spring II is relaxed, let P , be the pressure. fa) Find the initial pressure of the gas.
Then from the equilibrium condition of piston of area A2 we (b) If the block m is gently pushed down and released, it
have, with the help of eqn. (1) oscillates hannonically; find its angular frequency of
P'= mg +Po =p_ mg + mg oscillation.
A2 A, A2 (c) When the gas in the cylinder is heated up, the piston starts

=P+mg (AtA A-A2) moving up and the spring gets compressed so


is just lifted up. Determine the heat
that the block M
sup~Ued. Take
I 2 atmospheric pressure Po = lOS Nm2, g = 10 ms .
(iii) When the pressure increases, spring I is
compressed. Let x' be the compression, then from the Solution:
condition of equilibrium of piston of area A" we have mg
Ca) p=po +-
P'AI =kx' + mg +POA1 A
or kx'=P'AI -mg -PoA, =10s + (1)00) = 2x lOS N/m2
10-'
-{ p+ mg ( A~,~~')} A, -mg -(p_ ~)A, Cb) For an adiabatic process,
dP dV
-=-y-
kx'_mg(~: -1) p

dP=_ypdV
V

.. Change in volume of the gas V


~V=X'AI +xA.2 Resultant restoring force on piston,

= mg
k
(~-l)A, (1-~)A,
A2
+ mg
k A2
d'x
m-=-
2
dt
[yPA-+kx
dV
V
]

[A'A
mg -, -At +A2 --'
=- A'Al ] =_[yp~2X +kx]
k 2

www.puucho.com
Study Anurag Mishra with www.puucho.com

THERMODYNAMICS
2 2 Solution: As the piston is heat ,, ,,
d x =_[YPA +.!5..] x conducting, the temperature on both , ,
,,
dt2 mV m ,,
the parts is equal at each instant. ,, ,,
2 The system is isolated, so there is no ,, ,,
"fPA k
w=,-;v+-;
/ heat transfer to or from the system. Displaced
From first law of thermodynamics, posilion
(1):= /(l.4)(2x lOs)(lO-4)~'~~ ,1U+W =0 Fig.2E.75
\ lxl.4xlO-4 1 From the condition of the
=6 rad/s problem,
(e) In order to lift the block M, compression in the Vo + V =l](Vo - V)
spring must be where l' is the displaced volume.
x" Mg " (0.8)(10) "0.5 m
k 16
V"(~-l)Vo~+1
... (I)

Work done by the gas is The work done by an external agent in displacing the
W =mgx+- 1 kx' + PoAx' piston slowly increases the internal energy of both the
2 compartments.
or W = (1)(10)(0.5) +..!. (16)(0.5)2 + (105)(10--4 )(0,5) Fe~tdx = 2nC dT -= (PI -P )dV
li ... (2)
2

2 where PI and P2 are the respective pressures of the two


"12 J compartments. From ideal gas equation, we have
Change in internal energy is PI =~, P2 = nRT ... (3)
Vo-V Vo+V
.1.U=~ nR(T'-T) =2(p'V'-pV)
2 2 On substituting eqn. (3) in (2), we have
t mg kx
Here p =Po+-+- nRT _ nRT ) dV -= 2nC t' dT
A A ( V - V Vo + V
o
"10' + (1)(10) + (16)(0.5)
10--4 10--4 or
c:-
R rV
Jo (V 2 _
n
V2)
dV -
-
f'To T
dT

= 2.8x 105 Nm-2 0

R
V'=V+Ax ke,. =--, wehave
y-1
= (1.4x 10.-4) + (}0-4 )(0.5)
v
(y-l)f V dV=lnI-
= 1.9 x 10-4 m3 o V 2 _ V2 To
0

/':"U = 2 {(Z.B x 105)(1.9 x 10--4)


or InI-=[Y-l][_ln(Vi_V2)]~
2
To 2
-(2x 105)(1.4x 10-4)]
2
"63 J or In.!..= In[ vl ](V-1l
To Vo2 _V2
Using first law of thermodynamics,

or
Q=W+ilU
Q=12+63=7SJ
m T"To[(~:~)'r-l)'
Example ANALYSIS OF THERMODYNAMIC CYCLES
The P- V diagram shows a cycle A, Band C. Process AB is
A heat conducting piston can freely move i1L5ide a closed isothermal expansion, Be is isobaric compression and 0\ is
thennally imulated cylinder with an ideal gas. [n equilibrium adiabatic compression. We are given PA> Vjj and Te. If one
the piston divides the cylinder into t\NO equal parts, the gas mole of monoatomic gas is made to perform the cyclic
temperature being equal to To. The piston is slowly displaced. process. Find out the pressure, volume and temperature at
Find the gas temperature as a junction of the ratio 11 oj the all the points
volumes of the greater and smaller sections. The adiabatic
exponent of the gas is equal to y.

www.puucho.com
Study Anurag Mishra with www.puucho.com

214
THEIlMODYNAMIG
p p

,B
,, B
,,
c,.
v v
Fig. 2.58 Fig. 2.60

(0 PAVA =P,V, (isothermal, T =- canst.) Thermodynamic efficiency


(ii) PH = Pc (isobaric B -l' C) Wnet =-~-~~-~
WAB+WBC+WC\.
~=--
(iii) TCVCr-1 =TAV/-t (adiabatic C -lo A) C2gi\••.•
n C2gi""n

Since we have monoatomic gas therefore y = ~ In a cyclic process from A to A, change in the internal
3 energy is equal to zero.
The additional equation for all the points is PV =: nRT The total work done by the gas on the surroundings is
T = PAVA =< C2given - Qrejened
A
nR Example: Carnot's Cycle
We can calculate the pressure, volume and temperature P A
at all the points.
Work Done in One Cycle
Work done by the gas p
is positive only in the
A
process of expansion
V,
WAB = nRTA 10-
VA
WBe =-PB(VB -Vel
Fig. 2.61
v
WCA = -nC., (TA -Te) B
The total work done B -t C is adiabatic expansion
by the gas in one cycle is and
equal to the area enclosed
v D -)0 A is adiabatic compression
Fig. 2.59
inside the cyclic process C2g;ven ::0 nRT1 In -V,
and this is VA
W=WAB +WBC+WCA. Vc
Q,eje<."led ::o+nRT21n-
When a thermodynamic cycle has clockwise sense the VD
work done by the gas on the surrounding. (0 PAV ••• ::oPBVB
Thermodynamic Efficiency of a Cycle (ii) PaV; ::0 Pc V!
This is defined as the ratio of net work done in one cycle Tt V;-l ::0 T2 Vr1 and T2 Vr1 ::0 T2 VJ-l
to the heat given.
VB V
Qgiven =: nRT A In-
V, (iii) Pc VC =PDVD - =-c
VA VA VD

In the isothermal expansion process heat is given to the (iv) poVJ =p .••VX
gas by the surroundings. In the isobaric compression process Total work done by the gas in one cycle is
heat is rejected by the gas to the surroundings.
W""t =Qg -Qr
W Q
'l=~=l--'
Qg Qg

www.puucho.com
Study Anurag Mishra with www.puucho.com

THERMODYNAMICS
=l_TzlnVc,VD T, Vs = K2
=1--
V, T, V,
T In--- j
2
V, 1 T InK+T 1nK-T 1nK
2 3
~=~--~--~~-
In adiabatic expansion work done = Decrease in T} InK + TzinK
internal energy. _T}+Tz-ZT3
If a sample of gas performs an adiabatic expansion TI + T2
between two given isotherm then work done by the gas will
always same Example

Find out the efficiency of a cycle consisting two isothoric and


fWO adiabatic line. The volume of the ideal gas changes by n

times within the cycle?


An ideal gas is taken through a cycle consisting of alternate , p
isothermal and adiabatic process. The isothennal process are 2
perform at temperature Ttl T2 and T3 Find out the efficiency>

of the cycle if in each isothermal expansion the gas volume


increases in same amount.
p
~. ,4
, V
Vo nVo
Fig.2E.77

"-,
~ : •••••• ~-. T, Solution: Process 1 -) 2 isothoric heating process and
•••• _- T process 3 ~ 4 is isothoric cooling process
2
Qg =nC,,(T2 -1i)
5 -'--'r3
Qr =nC,,(T3 -T4)
v Qg -Qr
Fig.2E.76 "= Q,
_ nCt, [(T2 - T1) -(T3 -T4 )]
Solution: Process 1 --) 2, 3 -;. 4 and 5 --) 6 are the
nC,,(T2 -TI)
isothermal process. In the expansion process since the gas
volume increases in the same proportion therefore, T3 -T4
11=1- --
(1) V2=V4=K T2 -TI
VI V3
(Equation of adiabatic process)
V, T2VJ-l =T3(nVoF-1
(2) Qg] = oRT! In-
V,
and TI V",-I
0
- T4 (n\(0 ),-1
~
V,
(3) Qg~ = nRT2 In- ::::> (T2-T})Vrl =(T3-T4)(nVO)y-1
V,
V T -T
3 4 1
(4) Q, = nRT3In2 ---=--
V, T2 ~Tl ny-I

Qh + Qg2 ~Qr 1
11=1---
~= n y-)
QXl + QK2
T V'(-} - T Vy-l Example 78
(5) ] 2 - 2 3 '
T Vy~l - T vy~}
24-3S'
Find out the efficiency of cycle consisting fWO isobaric and two
TVY-] -TVr-} adiabatic line. The pressure change n times within the cycle
3 6 ~] 1

V2V4 V6 = V3VSV1
and y ~ given.

_._=-Vs
V2 V4
V3 VI V6

www.puucho.com
Study Anurag Mishra with www.puucho.com

16 IH
p PI VI = nRT ...(1)
"po

P, •• ---4
Q Or 3
V
By equation (3)/(2)
V,
P2VI = nR(nT)
P2V2 = nRCn 2T)
PI V2 = nRT3

-=n
V,
... (2)
...(3)
...(4)

Fig.2E.78 VI =-
V, n
Solution: Qg = nC peT2 -T1) Put in (1)
Q, =nCp(TJ -T4) PI' = nRT ~ PIV =- nR(nT)
...!.....1. ... (5)
2
Qg -Q, (T2-T1)-(T -T-4) n
~ = ~-- = -~~-~-~ 3
Qg (T2 -T1) From equation (4) and (5), TJ = nT
T -T Gv (nT -T) -G p(n2T -T) -G~ (n2T -nT) -Gp(nT-T)
'1""1 3 4 "=~----~----------~---
2
T2 -T1 GI, (nT -T) +G p(n T-T)
(nP o )1-1T1 - pi-rTY
2-03 solve it
and (nPo)1-YT1Y = ptTT: ... (1) ExamplE<
(nPo )o-ylfYT - p(l-y)/yy
2-0 3

and (nPo)o-Y)/YT1 =(Po)'-Y/YT4 I\n ideal gas goe.<; through a cycle consisting of isochoric,
... (2)
adiabatic and isothennal lines. The isothennal process is
By equation (1) - (2), perform at minimum temperature. If the absolute
(I-1) temperature varies K times within the cycle then find out its
n y (T2-T1)=T3-T4 efficiency.
73-74 1 P
T2-T1 n(H)/1 2 (KT)
1
~-1---
- nO-1)/1

Example

An ideal gas goes through a cycle consisting of two i.sochoric


and two isobaric line. The absolute temperature of the gas
.,
,3

V, V
rises n times both in isochoric heating and in isobaric V,
extension. If y is given rhenfi.nd out the efficiency of the cycle. Fig.2E.80

P
• 0" Solution: Process 2 3 is adiabatic expansion and
--)0
P .1
Process 3 --)0 1 is isothermal compression.
2 n1 \'.
From process 2 3 ; the internal energy continuously
--)0

T
decrease (i.e., temperature continuously decreases).
P, 4, Point 2 is at highest temperature Qg = nG ~ (KT - T);
,
V I'
Qr = nRT In...1.
V,
V
Process 2 ~ 3 (KT)V/-I = 7V;-1 "::)-2 = (K)\'(y-I)
Solution:Qg, =nCv(nT-T);Qg2 2
=nCp(n Y-T) V,
Q'l =nCv(n2y-nT); Qr2 =nCp(nT-T)

"=~~~~-~-~
+ Q -Q
Q +Q
Qg} g2 r] -Q'2

g2
SI

www.puucho.com
Study Anurag Mishra with www.puucho.com

THE ODYNAMICS
p
I Example 81!.---'

Q4
2
A cycle is made of three process isobaric. adiabatic and
isothermal. Isothermal process has minimum temperature.
Absolu£e temperature changes by K rimes within the cycle.
Find out the efficiency.

Solution:
' '
1 ..
• Q'
"
,3
v
Qg = nC p(KT-T) v, , Vo KVo
Flg.2E.82
v, v2
(K)1-1
I -=--
Q, =IlRTn- V, Vj
V,
1
K
Solution: Qg =nC,,{T2 -'1'\)
Qr =nCp(T] -'1'1)
, '1'1 '1'3
Process 1 ~ 2 is isobaric, VI '= ~ For process 3--'!o 1 ---'
T KT V3 = K(Kr-1 Vo-KVo'
V,
For process 2 -» 3 T20v:r-1 T
--30 (KV )"(-1

Process 2 --'!o 3 is adiabatic, - KT . T - T Ky-I


T3-}'2-3
K1Vr1 =7Vr1
, Q Cp(T3-T\)
=1--r =l-~---
11
V] = (K)7=-i Q.~ C,,(T2 -'1'\)
V,
'1'3-'1'1 =K -1 and '1'2-'1'1 =K"(-1
p '1'1 T1
(K -1)
11=1-)" .
(KY -1)

Example 83y--

A cycle is made of two isobanc and two isothermal line. The


volume changes by K times and the temperature changes by n
times within the cycle. Find out the efficiency.
p

Q,.
4

Q"

Vo KVo
v
Fig.2E.B3

Solution: Qg1 = nC,. (nT - T)


QS2 =nRllrlnK
An ideal gas through a clockwise cycle consisting of adiabatic,
isobaric and isothoric line, the volume of the gas increases by Qrl =nC,,(nT-T)
K times in adiabatic process. Find out the efficiency. Qr2 = nRTlnK
Qr
'1= 1- Q"l + 2
Qg1 + Qg2

www.puucho.com
Study Anurag Mishra with www.puucho.com

I THEIIIIODYIWIKS
THE SECOND LAW OF THERMODYNAMICS ," A perfect heat engine has efficiency of unity. Expression
AND HEAT ENGINES for efficiency shows that, it is possible only when QL = O.A
Fig. 2.62 shows schematic High-temperature perfect heat engine converts all the heat transferred I QH I
diagram of a heat engine. This device reservoir into work.
converts internal energy into The second law of thennodynamics states that a perfect
mechanical energy by absorbing heat heat engine is impossible. The first statement of second law
QH from a high temperature heat of thermodynamics is called the Kelvin-planck
reservoir at temperature TH• statement. It states that it is impossible for any cyclic
performing an amount of work W. process duting one cycle to convert imernal energy into
and rejecting heat QL to low work, without any heat rejection.
temperature reservoir. The high
In btief:
temperature heat reservoir is
referred to as source and the low 1. Efficiency of a real heat engine is always less than
temperature heat reservoir as sink. one.
The ideal heat reservoirs are Low-temperature 2. No heat engine can transfonn heat transfer from hot
assumed to be of infinite heat reservoir reservoir QH completely imo work.
capacity so that heat transfer does Fig. 2.62 3. In a cycle the heat engine returns to its initial state of
not change their temperature. pressure, volume and temperature.
From first law of thermodynamics heat transfer to any Carnet Cycle
engine during one cycle is equal to the change in internal
energy plus the work done. We have already studied the carnot cycle in example 9,
so we briefly summarize the results in the table below.
Q~li.u+W
Table 2.3: Carnol cycle
At the end of one cycle the internal energy has returned
to its initial value. So !J.U = 0.
j'rocess \ .U \ Q \ w
For a cyclic process,
Qcyclc = W cy<:lc
l~ 2
(isotherm)
0
nRTH In (~:) nRTH In(~:)
where Qc,'de is the net heat added to the engine system 2~3 nC"UL -TH) 0 -nC,crL -TH)
and WTIC' is the net work done by the system duting the (adiabat)
cycle.
3~ 4 0
The net heat transferred (Q a system is the difference (isotherm) nRTL In (~:) nRTlln (~:)
between the heat transfer (Q the system from the hot
reservoir (I QH D, and the heat transferred from the heat
engine to the cold reservoir (I QL I).
[Alternative
• •
Q,,, =IQHI-IQd forms] -nRTlln (~) -oRT! In (~)
Magnitude has been shown [Q avoid confusion.
According to our sign convention, heat added (Q the system 4~ 1 nC,.er" -Tt) 0 -nC •.tTL - TH)
(adiabat)
is positive and heat rejected from the system is negative.
without absolute signs, TOTAL 0
nRrrH-TL)ln(~) nRcrH-TL)ln(~~)
Qnt"' =QH + QL
As
From first law, with fJ,U eyde = 0.
w =IQ" I-IQLI
The efficiency '1of a heat engine is defined as the ratio of Wnet =Qnet =nR(TH -TL) 1n-VB
the net work done to heat added to the system. VA
Net work done per cycle Efficiency,
"- Heat input per cycle

=
IQHI-IQLI
IQul
=l_IQLI
IQul

www.puucho.com
Study Anurag Mishra with www.puucho.com

THERMODYNAMICS 219

As V4 < V3, we can write absolute value of QL by


lpplying In (a/b) "" -In (bfa)
IQd nRTL In(V3/V4)
IQul nRTJlln(VzlVJ)
V2 V3
-=-
V} V4 = Til
T,
IQd !i
Thus coefficient of performance of a Carnot refrigerator
IQ" I T"
is
The efficiency of Carnat engine is
1 T,
T, Ilr = . =
Tl'"l~- (TH1TL)-1 Tff -TL
T"
A heat pump and refrigerator are the same device, but
.Refrigerators and Heat Pumps they differ in the desired effect. For a refrigerator the desired
A refrigerator accomplishes heat transfer from a cold to effect is the heat removed from the low temperature
-a hot reservoir. Fig. 2.63 reservoir whereas for the heat pump the desired effect is the
==shows a schematic p heat transferred to the high temperature reservoir. In other
-diagram of a refrigerator. words, we can say that in a refrigerator, low temperature
lHeat QL is extracted from Adiabatic reservoir is system and high temperature reservoir is
II!:he low temperature surroundings; reverse is true for heat pump.
-reservoir and transferred The coefficient of performance Il hp of a heat pump is
-[0 the high temperature
defined differently.
reservoir. The
I Heat transferred to the high
-effectiveness of a
particular refrigeration remperature reservoir I
;;proces5 is characterized L
I Work done on the system I
-by a coefficient of
Fig. 2.63 IQHI
_performance f.l. r'
Heat removed from low temperature reservoir
IQ"I-IQ,I
p = T"
r Magnitude of the net work done on the system =-~-
TI/ - TL
IQ, I
=--- A perfect refrigerator is one that has I W I = a and an
I WnPl I infinire coefficient of performance. The second sratemenr of
Q, second law of thermodynamics is called Clausius
IQ" I-I QtI statement. It states that it is impossible for any cyclic
1 process during one cycle to remove heat from a cold
reservoir to hot reselvoir without any work done on the
152"1 _ 1 system.
IQ,I
ABSOLUTE ZERO AND THE THIRD
For a Carnot refrigerator,
LAW OF THERMODYNAMICS

IQ"I
--=
nRIH In(tz,J
..
Efficiency of a Caroot heat engine is exactly unity when
1~ = a K; such a heat engine cannot be constructed
IQ,I nRTL In
(Vv: J according to Kelvin-Planck statemem of the second law.
Hence absolute zero cannot be attained; this is third law
of thermodynamics.
V2 V3
As shown earlier, -=- For a Caroot engine in a complete cycle,
VI V4
IQIII =!i
and we have to consider magnitude only,
I Q, I T"

www.puucho.com
Study Anurag Mishra with www.puucho.com

TH ICl

We can determine an unknown temperature T of any reversible cycle as a family of Caroot cycles. Thus the
reselVoir by operating a Carom engine between it and efficiency of any reversible engine is equal to the Carnot
another reselVoir at reference temperature Tref, with T = TL efficiency. All the reversible engines have same efficiency
and T,ef = Tn when they operate between the same two temperatures.
The Clausius inequality states
T~~T r
IQ,efl re
L:g<O
The absolute temperature scale, that is independent of cycle T
the working substance, is defined by taking T ref as triple The equality hold for a reversible cycle as we have
point of water. shown for a Caroot cycle. If we take large number of cycles,
T,ef = 273.16 K each of which is infinitesimally separated in temperature,
the sum becomes an integral.
T ~ J.9.L (273.16 K)
pdQ$O
IQ~rI T
General Cyclic Processes Equality holds for a reversible process and inequality for
From Camm cycle we can see that an irreversible process.
Q,
TL
~-nR In(V,) V]
Entropy: A State Variable
For a reversible process, Clausius inequality becomes

QH
Til
=nRln(V,) VI
pdQ = 0
T

On adding these two equations, we have Thus the quantity dQ must be a state variable as its
T
QL + QH = 0 sum for a complete cycle is zero.
TL TH
Entropy is defined as
Fig. 2.64 shows that any arbitrary engine cycle can be
dS = dQ
p p T
Adiabat Isotherm

liS = r di
The change in entropy
reversible path
between any two states is

~ The entropy change of a system as it changes from


~ one state to another is:
Actual engine
process '- .T (i) positive if heat is transferred to the system
T
{'I {bl (dQ > 0 J).
Fig. 2.64 (ii) negative if heat is taken from the system (dQ < 0 J).
(iii) zero for adiabatic process (dQ = 0 J). Adiabatic
broken into a large number of Carom cycles. If we travel processes are also called isentropic processes, implying
along each segment, the interior segments cancel. So for all constant entropy processes.
the Carnot cycles,
(iv) zero in a cyclic process; since the initial and final

~[~:: + ~:}o states are the same, entropy is a state variable .


• Although the equation is written for a reversible path,
but it will be same for any irreversible process
N = number of processes in the family ofCamot cycles. connecting the same initial and final states .
If we take processes into account, • For an isochoric process,

Ti
i:
i-I
Q, =0
dQ =nCv dT
dT
dS =nCv-
N = number of processes in the family ofCamot cycles. T
• For an isochoric process,
Clausius Inequality
No engine can have a greater efficiency than the dQ = nCt. dT
corresponding Carnot engine. This result is called Carnot's dT
dS=nCv-
theorem. Previously we have modelled an arbitrary T

www.puucho.com
Study Anurag Mishra with www.puucho.com

THERMODYNAMICS 221

f dT Tf From ideal gas equation


tS=nC" I i T=rzct.ln- -;
T p
-=-
,>1<

•.. For an isobaric process, T V


Substi[ming this into second term of eqn. (3), we have
dQ=nCpdT

dS = nC
I'
elT
T
!is = IIC" r ~: + J/ d:
nR

f T VI
f."S == nC p f -dTT = nC
I
J
f'
T
In...L
Ti
= nC" In--I- nRIn-
1', Vi

.•. For an isothermal process, We have calculated !is without consideration of the path
Q
followed. Since entropy is a state variable, the entropy
f."S =- change will be the same regardless of the process between
T two stares,
•.. For melting or boiling mass In,
•. For an isothermal process, TJ = Ti
mL
f.,,5=-
T In TJ = In (1) = 0
T;
where T is the absolute temperature during phase
Vf
:hange and L is the latent heat involved during' phase L\S = nR In~
:hange . V;
•.. Freezing or condensation of a mass In, •. For an isochoric process, VJ = Vi
f."S=_rnL V
In....£. =In (1) = 0
T V; .
•.. Warming or cooling a solid or liquid of mass nI, T
T/ 65 = nC" ln~
ilS=mCln- T,
T,
where c is the specific heat . Example ~5_~
••.For heat transfer to a reservoir at temperature TL•
Fig. 2E.85 shows a container I,,'itha membrane partition. The
,IS = + LgJ left hand side, volume 11;,is filled with an jdeal gas; the rig/It
TL
hand side is initially vacuum. Total volume of the container is
••. For heat taken from a reservoir at temperature Til' Vj . The membrane is punctured and free expansion of gas
t.S = - LgJ takes place, which is an irreversible process. Find the entropy
change associated with rhis free expansion of an ideal ga.~.
TH
Entire system is thennally insulated from its surroundings,
Example 84~
Vacuum

An ideal gas is taken from initial state (Vi. Ti) to a final state Membrane
(Vj• Tf ) by an unspecified process; find the entropy change.
Fig.2E.85
Solution: From first law of thermodynamics,
dQ =dU+dW Solution: From first law of thermodynamics,
dQ = nC" dT + PdV Q =tJ.U+W
dQ =nC dT + PdV In free expansion, W = 0
... (1)
T l' T T The temperature of an ideal gas is unchanged during
The entropy change is free expansion; internal energy is function of temperature
only; hence !iU = 0,
~s=Ii, dQ
T ... (2)
The system is isolated; there is no heat transfer to or
from the surroundings; Q = O.
=nC,I! dT +1'T1 ~dV ... (3)
t'T As dQ = 0, it seems that 65 = If, dQT is zero, bur this is
not the case,

www.puucho.com
Study Anurag Mishra with www.puucho.com

mERMODYNAMICS

From the example 84 The first term is negative and second is positive, but.
T V total entropy change is positive. Let's see it in a special case.
~ =rzC" In...L+ rzRln..l..
Tj V; m] =m2, C1 =C2

This expression is applicable to any process, reversible


or irreversible, between same initial and final states. Since
T =(Tl ;T2)
j

temperature of gas is constant (free expansion), T'


V ." IiSto1a1 =mCln-f-
liS = nR In..l.. T)T2
V,
Since final volume Vj is greater than initial volume, the
entropy change is positive, entropy has increased. This is
=mCln[C' ~T,J'l,T,]
one of the examples in which dS ~ 0 but dQ = o. . T +T
Smce I 2 is the arithmetic mean of twc
2
Example
temperatures T1 and T2, ~T]T2 is geometric mean, and
1\ well insulated container has a thin membrane partition. AM> GM
Each of the two chambers has two different liquids, as shown >0
ASIOlal
in Fig. 2£.96. The membrane is punctured and liquids are which shows that total entropy change will be POSitiV
allowed to mix. Find the entropy change associated with this in this irreversible process of mixing.
free mixing of liquids.
Example
m,
tr~]lli T,
e,
Find the entropy change when heat I Q I is transferred from a
hot reservoir at temperature TH to a cold reservoir at

-----
T, >T2 temp'erature Te.
Fig.2E.86
Solution: Process of heat transfer is irreversible.
Solution: We first compute the final temperature of Entropy change of hot reservoir
the mixture. dS =fJ dQ =_L9J
Qnel = 0 i T 1H
mlC) (Tj - T)) + m2C2(Tj - T2) "" 0 Entropy change of cold reservoir,
_ m)C1TI +m2C2T2
Tf - dS=fJ dQ =+L9J
mlCI +m2C2 j T Tc
Change in entropy of liquid 1, The total entropy change of both the reservoirs is

dS, = f: ~Q liSlotal =L9J_L9J


Te TH
TJ dT
=mlCI f T, -T =IQI[TH-Tc]
TcTH
TJ
=mjClln- Since TH > Te, the total entropy change is positive.
T,
Since Tj < T1, this entropy change is negative.
Entropy and the Second Law of Thermodynamics
The entropy change of liquid 2, In the previous three examples (example 84, 85, 86), we
saw that change in entropy is positive in all the three
Tf
AS2 =m2C2In- irreversible processes. The generalised statement of second
T, law of thermodynamics is:
Since Tj > T2, this entropy change is positive. The total The total enrropy change of an isolated system is always
entropy change is the sum of the entropy changes. greater than or equal to zero.
&Stolal = IiSI + AS2 AS 2:OJ/K
Tj T1 •.. Note that this sta£ement involves an isolated system.
= mlC1ln- +m2C2In- If an ice block is kept open to atmosphere, heat is
T) T2
transferred from environment to system.

www.puucho.com
Study Anurag Mishra with www.puucho.com

THERMODYNAMICS

~Stotal = h,Sice + f.Senvironmcnl t..S rcirig. = a J/K


l1StOlal must be positive. Change in entropy of cold reservoir,
If a heated metallic block is kept open to atmosphere, IQI
lI\1eatis transferred from system to environmenl. AS cold rc'C",.oil = - Tc

f.StOla1 = i\Sbl",'k + i\Scnvironmcm Change in entropy of hot reservoir,


Although h.Sb1od:is negative, but i\Scnvironmt'm
is positive
and it must be greater than i\Sblock so that <.\SlOtal is positive. t..S hot reservoir
~+ I<lj
TH
lIFor any irreversible process "'"SlOtalmust be positive.

Environment H Environment t..SI<ltal


~ o-I<lj + I<lj
Tc TH
0".-----/ '" transfer ."-'..
".- Healed -', ./
Heat
transfer
, /,
: Ice
.'
;
'.metallic block! olQI[Tc -TH]
'. .'- p.- .' TcTH
T
System Since Tc <TH, <\Stoml is negative, which is violation of
Fig. 2.65
second law of thermodynamics. Thus perfect heat engine
• The total entropy change is the algebraic sum of the and refrigerator do not exist.
individual entropy change of the components of the
system . Example 89y--
.•. The total entropy change is zero for a reversible
process. The system shown in the Fig. 2£.89 is in equilibrium. The
piston is massless, frieeionless and insulated. All walls of the
Example chamber are also insulated. Whet! heat is generated inside the
lower chamber the piston .~lowly moves upward by 3 m and
Show that Q peifeer heat engine and a peifect refrigerator the liquid comes out through an orifice so that it can rise to a
cannot exist. It is violation of second law of thermodynamics. maximum height of 5 m above the orifice level. The lower
chamber contains 2 moles of an ideal monatomic gas at
Solution: For a heat engine, SOaK. Find the heat generated in the lower chamber, if
t..SrOlal = t..Sengine + t..Shot reservoir + t..Scold reservoir ~orifice
After cycle of the heat engine it returns to its initial
.;tate, entropy is a state variable; therefore

t..S engine = a J/K


The entropy change of hot reservoir,
IQH 1 H,=5m
t..Shot rc.,e"lOir = - -T-
Liquid r
H

which is negative as heat is transferred from the system.


For a perfect heat engine, I Q, I = 0; hence
GasT-500K
t..S mid resenll>ir = a J/K

t..Stmal = --~
IQH 1

T"
Total entropy change is negative, which violates second
Fig. 2E.89 (a)
aw of thermodynamics.
For a perfect refrigerator, area of the piston A = 1m 2
= t..Srefrig. + t..Sculd reservoir + AShOl reservoir
""S rmal
density of the liquid, p = 103 kgl m3
After one cycle of the refrigerator, it returns to its initial g ~ lOm/,'
-;tate; entropy is a state variable. Atmospheric pressure, P"'m = 105 N 1m2

www.puucho.com
Study Anurag Mishra with www.puucho.com

Solution: Assume datum at the position of the orifice. Pt VI pzVz


Since --=--
Initial energy of the liquid Tt T2
1 2 T = PZV2 T = (1.8)(4)
Ei = -"2PgAHl z t
P, V, (1.5)(2)
=.!. (103)(10)(5)2 = -1.25 x 105 J T = (1.8)(4) (500)
2 I (1.5)(2)
and final energy
Increases in internal energy. !J.UnCv!J.T
1
Er =--pA(H) _h)2 +rgAhho 3R
2 here. n=2,C =2;Y

= -~ (10' )(10)(1)(5 -I)' + (103)(10)(1)(2)(5) !J.T = 1200 - 500 = 700K


2
= -0.45 x 105 + 105 .. 8U =(2)(3(2)(8.3)(700) = 17.43kJ

Work done by the expanding gas is First law of thermodynamics


W =Er -E = 1.25)( 105 -0.45)( 105 + 105 Q =!J.U+W
j

= 17.43 + 180 = 197.43 kJ


= l.8x 105J
""",,, Example
"""".
,,"',,"
'"''''''
'"''''''
"'"''''
,,,",,"
",,",,' n-moles of an ideal gas at 2rc are expanded isothermally to
",,","
II,,,,,,, five time its volume and then heated at this constant volume
""","
""'""
'" "."
,,"",,
". '"''
,'"''''''
.. Ho=5m to raise its pressure equal to initial pressure (before
~ansion). If heat given to the system in the process is
"""",
"""
"""'"
... 83.14 kJ then find the value of n. The r( = ~: ) of the gas is
""'."
'."."
'.'"''
' •.....
..p.a.t~!:'lr-;:::==j'::::'t==::;-lt
.", 1.42 [ln5 = 1.61J

Solution:':!J.Q = 6U +!J.W
Liquid p
For isothermal process l'1U = 0 and
!J.W = nRTo In(Vj IV;), where To = 300K
=> !J.QI = nRTo In 5J
For isochoric process !J. W = 0
=> 6Qz =nCy!J.T,6T=-T-To,
PV =Px5V :::>T=1500K
To T
R
=> 6Qz = n -- x 1200J
r-I
Fig. 2E.89 (b) .. 6QI + 6Qz = 83.14J
On solving, we get n = 3
Initial pressure of the gas, Pt = Palm + pgH I
= 10; + (10')(10)(5) Example
= 1.5x lOsPa
P-V diagram of cyclic process is given. The process is
Final pressure of the gas. P2 = Pmm + pr(HI -h + ho) performed on one mole of monoatomic gas. During the
= 105 + (103)(10)(5 - 2 + 5) process ABC, hear is absorbed by the system till poinr Band
= 1.8 x lOs Pa after point B sysrem rejects heat. Then rhe efficiency of the

Initial volume. VI = AH z = (1)(2) = 2m 3 cyc Ite In percenrage).100tn


IS ~ h. t en m IS _ .

Final volume, V2 = A(H z + h) = (1)(2 + 2) = 4m 3

www.puucho.com
Study Anurag Mishra with www.puucho.com

THERMODYNAMICS

aL--'__
o
'-_'-'v
oV o 2,5V 4V

Fig.2E.91

Solution: Percentage efficiency


= Tl = Wcycle x 100
Heat supplied
9 T = PoVo
- PoVo " nR
= 2 x 100
nC,,(TA -TD)+nC,,(TB ~TA)+WAB
9
-Povo
=3 2 x 100
-R(TB -TD)+areaunderAB
2
9
-Povo
2 x 100
~(2: PoVo -PoVo )+ 6PoVo V, ::VO'Pb ::P,::-
Po
8
12 From b to c process is isobaric V :: constant
~-xl00=12
37 Vi T1 V,
-::-,T, ::-.T,
V2 T2 Vb
Exornp.le.
__ =>0 __, ..=~
.
Vb :: Tb T:: Vo x PoVo _ PoVo
V, T,
, 4Vo 2nR
-
8nR
A system containing n moles of an ideal gas (y = 1.5) is taken
through the thermodynamic cycle shown in figure. Let the T :: PoVo
initial pressure Po and volume Vo at point a be given, and let , 8nR
Vb = 4Vo' Assume step a 4' b is an adiabatic process. Find the
Work done during the complete cycle is area of the
pressure, volume and temperature of the system at points b
and c in terms of the given quanti/)' and also the total work
closed cycle
done by the system during one complete cycle. WI ::work from a to b ::[P1V1 -P2V2
y-1
J
Po • a

~nD(T, -T'J~nD(T" -TbJ~nR[!'9~ -~]


'\ y-1 '\ 1-1 1.5-1

PoVo
::nR-- x 2 ::PoVo
v, 2nR
Fig.2E.92 Work from b to c (isobaric process)
Po 3
W2 :: Fev, - Vb):: - [Vo -4Vo] :: -- VoPo
Solution: Given Va Vo' P" = Po.y = 1.5
= B 8
Vb =4VO.Pb =? 3 5
Wntl =PoVo --PoVo ::-PoVo
P" V} = Pb V~~ B B

www.puucho.com
Study Anurag Mishra with www.puucho.com

226

Example (b) Examine the following diagrams :

A movable heavy piston is supported by a spring inside a


vertical cylindrical container as shown. When all air is
pumped out of the container, the piston is in equilibrium as
shown in Diagram 1, with only Q tiny gap between the piston
and the bottom of the container. When a portion of gas at
vL2:v~ TL2
•.••. T v P
(I) (II) (iii)
temperature T is introduced under the piston, the later rises to
a height h as shown in Diagram 2. What would be the height Fig,2E.94
of the piston above the bottom of the container if the gas is
then heated to a temperature Zf ? Assume that the piston and fill in the table:
moves without friction and that the spring obeys Hookes law.
> or <
..
',':~ ,. (i) PI P2
(ii) TI T2
(iii) V, V2

Diagram1 Diagram2 Solution: (a) We have m = VI 'PI and


Flg.2E.9J m - Lim = V2 ,p 2 where VI and V2 are volume of the vessels
at t} °C and t 2°C and P I and P 2 are densities of {he liquid at
Solution: Initially there is no gap between piston and ttOC andt2°C
cylinder because initially all the air pumped out. Thus spring m VI "PI m V 'P2[I+y(t2-tt)J
I
---~ --- => ---- ---------
force balance weight of piston. m-Lim V2'P2 m-Lim VI[I+3u(t2-tl)]P2
Now after introducing gas in container, the decrease in => .6.m=(t2-td[(m-t.m)y-3u'm]
extension of spring is because of the pressure of gas because
due to gas pressure spring begins to compress let the => t2 =t} +-- "m _
(m-t.m).y-3a.m
pressure of gas is PI' A -l' Area of cross-section of piston
P, . A ~ kh ... (1) = 0+ 15
After raising the temperature let new pressure is P2 and (700-15)(18)( 10-5) - 3)( 9)( 10-6)( 700
new height h' = 143.7°C~
P2.A==kh' ... (2)
(b) In (i)VOCT,ButPV=:RT => V==(:~}T
P, h
-==-
P2 h'
So slope of V - T plot is mR
Conserving the number of moles MP
PIAh P2Ah' Here slope of 1 < slope of 2.
--~-- mR mR
T zr => --<--=>P2 <PI orFI >P2
MP} MP 2
h2 1
(h,)2 ~ 2 In (ii) we notice that corresponding to a given value of
P,V1 <V2
J2h~h' PVI < PV2
Example mRTt mRT2.
A:1<A:1 '
(a) A 'weight thennometer's is simply a hollow thin walled
vessel of glass or any other suitable material used in the past
In (iii) T ocF, butPV =<: RT => T =(:}p
to measure temperature. It holds m = 700g of mercury a~
tl = O°C. When it is heated Lim == 15g of mercury flows out.l
:. Slope of T _ P line is MV
m.R
'calculate the final temperature if Y = coefficient of cubical
expansion of mercury = 18)( IO-SOC -1 and a = coefficient oJ. Here slope of 1 < slope of 2
linear exv.ansion of glass = 9)( lO-6oC -I. l'JVt MV2
-- <--=> V2 > VI or VI < V2
mR mR

www.puucho.com
Study Anurag Mishra with www.puucho.com

THERMOOYNAMICS 221

Problem

Only One Alternative is Correct


ePeve I
1. If 2 moles of an idee'll monoatomic gas at temperature isothermal. Let W] and W2 be the respective work
To is mixed with 4 moles of another ideal monoatomic done, then:
gas at temperature 21'0, then the temperature of the W,
(a)W2=Wjln2 (b) W2 =--
mixture is : 1n(2)
5 (b) ~To (c) W2
w,
=- (d) data is insufficient
(a) - To
3 2 2
4 (d)"T 5. The energy density U I V of an ideal monoatomic gas
(e) -To
3 4 ' is related to its pressure P as :
2. A spherical soap bubble (surface tension =- 5) encloses U U 3
(a)-~3P (b)-~-P
11 moles a monoatomic ideal gas. The gas is heated V V 2
U pUS
slowly so that t..hc sUlface area of the hubble increases (e) - ~- (d) - ~-P
V 3 V 2
by 3::SR per unit increment in temperature. The
6. One mole of an ideal gas is taken from state A to state
specific heat for this process is : H by three different processes:
(a) 3R (b) SR p
2 2
(e) 7R (d) 9R
2 2
3. Pressure versus temperature graphs of an ideal gas are
as shown in Fig. Choose the wrong statement :

L,~,J~,
A
v
co ACB (ii) ADB (iii) AEB as shown in the P-V
diagram. The heat absorbed by the gas is :
(a) greater in process (ij) than in (i)
(i) (ii) (iii) (b) the least in process (ii)
Ca} Density of gas is increasing in graph (i) (e) the same in CO and (iii)
(b) Density of gas is decreasing in graph (ii) (d) less in (iii) than in (ii)
(c) Density of gas is constant in graph (iii) 7. Sixty per cent of given sample of oxygen gas when
(d) None of the above raised to a higher temperature dissociates into atoms.
4. A gas is expanded to double its volume by two Ration of its initial heat capacity (at constant volume)
different processes. On is isobaric and the OTher is to the final heat capacity (at constant volume) will be :

www.puucho.com
Study Anurag Mishra with www.puucho.com

THERMODYNAMICS
8 (b) 25
(a) - water in a 0.45 kg copper cup from 298 to 373K ? esp.
7 28 heat of copper = 389 J I kg K)
(e) 10 (d) 25
7 27
(a) 95 (b) 31
8. In a cyclic process shown in the
(e) 39 (d) 45
Fig. An ideal gas is adiabatically p
taken from B (0 A, the work 13. Pressure versus density graph of p
done on the gas during the an ideal gas is shown in Fig.
process B -+ A is 30 J, when the
gas is taken from A -+ B the heat
absorbed by the gas is 20 J. The
v
(a) During the process AB work
done by the gas is positive
(b) During the process AB work
~Jl..
.'
I J '
....
change in internal energy of the gas in the process
done by the gas is negative
(c) During the process BC
/"A 6n p
A-+Bis: internal energy of the gas is
(a) 20 J (b) -30 J "
increasing.
(e) 50 J (d) -10 J (d) None of the above
14. Ideal gas is taken through the process shown in the
9. Three moles of an ideal Fig.
monoatomic gas performs a p (a) In process AB, work done by p
cycle 1-+2-+3-+4-+1 system is positive
(b) In process AB, heat is rejected

j:
shown. The gas temperatures
in differem states are (c) In process AB, internal energy
T1 :=: 400K, T2 = 24DOK and increases
T (d) In process AB, internal energy
T41200K. The work done by
decreases and in process BC, T
the gas during the cycle is :
intef!lal energy increases
(a) 1200R (b) 3600R
15. A process I -+ 2 using diatomic p
(e) 2400R (d) 2000R
gas is shown on the P-V diagram
10. Three moles of an ideal monoaromic gas perform a below. --.-;-71 2

cycle shown in Fig. The gas temperatures in different P2 =2P1 =106N/m2, --_.V
:,
:3'
states are Tt = ZOOK, T2 = 400K. T3 = 1600K, and V2=4\)=0.4m3. The molar ,
T4 = BOOK.The work done by the gas during the cycle v
heat capacity of the gas in this
is: (Take R = 25/3 J/mol-K)
process will be :
(a) 35R/12 (b) 25R/13
(c) 35R/ll (d) 22R17
16. Select the correct statement from among the
following:
(a) A monoatomic gas has three degrees of freedom
because it undergoes translational as well as
(a) 5 kJ (b) 25 kJ rotational matiGn
(e) IS kJ (d) 20 kJ (b) A diatomic molecule undergoes both translational
and rotational motion and has six degrees of
11. Unit mass of liquid of volume VI completely turns into freedom
a gas of volume V2 at constant atmospheric pressure (c) A diatomic molecule is capable of rotating
Po and temperature T. The latent heat of vaporization energetically about each of three mutually
is L.Then the change in internal energy of the gas is : perpendicular axes
(a) L (b) L+PO(V2 -VI) (d) The molecule of a polyatomic gas is capable of
(c) L - PO(V2 - VI) (d) zero rotating energetically about each of three
mutually perpendicular axes and undergoes both
12. If all the heat produced were used, how many Iitres of
translational and rotational motion
natural gas at NTP (heat of combustion of the gas
6 3 17. A gas is expanded from volume Vo and 2Vo under three
= 37.3 x 10 MJ 1m ) is needed to heat 4.54 kg of
different processes. Process 1 is isobaric, process 2 is

www.puucho.com
Study Anurag Mishra with www.puucho.com

JHERMODYNAMICS

isothermal and process 3 is adiabatic. Let AU1• L'!.U2 21. A cylinder of ideal gas is dosed by an 8 Piston
and AU3 be the change in internal energy of the gas in kg movable piston (area 60cm2) as rz,";;~;;"zz1
these processes. Then: shown in Fig. Atmospheric pressure is Gas
P 100 kPa. When the gas is heated from
30°C, to 100~C the piston rises by
p ,.---- ,
1
2 20cm. The piston is then fixed in its place and the gas
is cooled back to 30°C. Let .6QI be the heat added to
3
the gas in the heating process and I.6Q21 the heat lost
during cooling. Then the value of [.6Q2 -1l'.Q21l will
v, v be:
(a) zero (b) 136 J
(a) AU) > ilUz > t.U3
(e) -136 J (d) -68 J
(b) flU) < AU2 < AV3
(e) D.U2 < AUl < L\U3 22. P-V diagram of an ideal gas is as shovvn in Fig. Work
(d) IlU2 < tJ,U3 < L'!.U} done by the gas in the process ABeD is :
P
18, A cylindrical tube of uniform cross-sectional area A is
fitted with n\'o air tight frictionless pistons. The
pistons are connected to each other by a metallic wire.
Initially the pressure of the gas is Po and temperature
2P,
..._-:r-----: D

of To. Atmospheric pressure is also Po, Now the P, -- ••• .8


,,
LA, ~
,• :
temperature of the gas is increased to Zfo• the tension ,, ,, ,,
, ,, ,,
in the wire will be : V
----_.---- v, 3V,
:.:.:.:Gas:-:':'
-:-:-:-:-:-:-:. :.:-: (a) 4PoVo (b) 2PoVo
:.:.:-:-:-:-:-:-:-:- (c) 3Po Vo (d) PoVa
::::::::::::::::::::
-:-:-:-:-:.:-:-: -:-: 23. A vessel of volume 20 litres contains a mixture of
hydrogen and helium at temperature of 2rc and
(a) 2PoA (b) P,A
pressure 2.0 atm. The mass of the mixture is 5g.
ee) PoA (d) 4PoA Assuming the gases to be ideal, the ratio of the mass
2
of hydrogen [0 that of helium in the given mixture will
19. Let V denote the root mean square speed of the
be:
molecules in an ideal diatomic gas at absolute (b) 2 , 3
(a) 1 : 2
temperature T. The mass of a molecule is 'm'. (d) 2 , 5
(c) 2: 1
Neglecting vibrational energy terms, which is false?
24. Pressure versus temperature graph of an ideal gas of
(a) A molecule can have a speed greater than .../2 t'
(b) V is proportional to JT equal number of moles of different volumes are
(c) The average rotational kinetic energy of a plotted as shown in Fig. Choose the correct
molecule is mv 2 / 4 alternative:
(d) The average kinetic energy of a molecule is tan 9 =.nB.
Smv2 / 6 V

20. One mole of an ideal gas undergoes a process


P,
Hence, Po and Vo are constants.
P~I+[~r T

(a) VI = V2, V3 = V4 and V2 > V3


Change in temperature of the gas when volume is
(b) VI = V2, V3 = V4 and V2 < V3
changed from V = Vo to V = 2Vo is :
(c) Vj =V2 =V3 =V4
(a) _ 2PoVo b) llPoVo
SR (lOR Cd) V4 > V3 > V2 > VI
(c) _ SPoVo (d) PoVo 25. Volume versus temperature graph of two moles of
4R helium gas is as shown in Fig. The ratio of heat shown

www.puucho.com •
Study Anurag Mishra with www.puucho.com

in Fig. The ratio of heat absorbed and work done by 3 ,


(a) fuT;] (b) -uTo
the gas in process 1-2 is : 2
V (c) 2aTi Cd) 3uTo2

30. Pressure versus temperature


. graph of an ideal gas is as
shown in Fig. corresponding
density (p) versus volume (\I)
(a) 3 (b) 5/2
graph will be :
Ce) 5/3 Cd)7/2
26. The P-V relation for a monatomic ideal gas undergoing t
an adiabatic change is :
(a) PVS'3 "" Constant (b) PV2 "" Constant,'
, C
p

(c) PVZ.'3 = Constant (d) PV7/3 "" Constant ~, B,C

.:./00
27. Pressure versus temperature graph of an ide'll gas is as
(a) (b)
shown in Fig. Density of the gas at point A is Po'
~A,O
Denshy at B will be :
P ---,---
V v
p

3
3Po
------7
Po •••••• 'A
,

to
"0
B

:
:,
,,
t
Ce)
B~A

v
Cd) 'b: V

(a) -Po
3
(b) "Po 31. The Fig. shows the graph of logarithmic reading of
4
4 pressure and volume for two ideal gases A and B
(c) -Po Cd) 2po
3 undergoing adiabatic process. From Fig. it can be
28. In the p.v diagram shown in Fig. ABC is a semicircle. concluded that:
The work done in the process ABC is :
P(atm)

3 ..•• ~.rc-~.:c ~A
,, '' B
--~ ••• t ••. - :A loV
,
,,
,, (a) gas B is diatomic
V(L)
(b) gas A and B both are diatomic
(c) gas A is monoatomic
IT (d) gas B is monoatomic and gas A is diatomic
(a) zero (b) -atm.L
2 32. A thennodynamic system undergoes cyclic process
(c) -~atm.L (d) 4 atm.L ABCDA as shown in Fig. The work done by the system is :
2 p
29. Pressure P, volume V and temperature T of a cerrain

material are related by P = aT


2
• Here, a is a constant. 3PJ ~,
V ,
The work done by the material when temperature
2P ...... ~_.
,
,,
changes from To to ZTo while pressure remains ,
constant is : A'
P,
V
v,

• www.puucho.com
Study Anurag Mishra with www.puucho.com

mRMODYNAMICS

(a) Po Vo (b) 2PoVo p

(e) PoVo (d) zero


2
33. Certain amount of an ideal gas are contained in a
dosed vessel. The vessel is moving with a constant
v
velocity v. The molecular mass of gas is M. The rise in
temperature of the gas when the vessel is suddenly (a) > 1
stopped is ('f ==Cp ICv) (b) < 1
Mv' Cb) Mv'C ..,-I) (e) I
Ca) ---
2R(y -I) 2R (d) data insufficient
Mv' Mv y 2 37. A sample of an ideal gas is taken through a cycle as
Ce) Cd) - . shown in Fig. It absorbs 50 J of energy during the
2R(y -I)
process AB, no heat during Be, rejects 70 J during
34. A cylinder contains He and 02 of equal volume. They 0\.40 J of work is done on the gas during BC. Internal
arc separated by massless freely moving piston as energy of gas at A is 1500 J, the intemal energy at C
shown. If the gas is adiabatically compressed by
would be :
moving the piston so that volume of He becomes half. p
Finally:
L L
t •• .1

v
0, H.
Area A
Ca) 1590 J (b) 1620 J
(e) 1540 J Cd) 1570 J
38. A liquid of density 0.85 g I cm 3 flows through a
massless massless calorimeter at the rate of 8.0 em 3/S. Heat is added by
Adiabatic wall piston
means of a 250 W electric heating coil and a
(a) pressure in He chamber will be equal to pressure temperature difference of 15"C is established in
in O2 chamber steady-state conditions benveen the inflow and the
(b) pressure in He chamber will be less than pressure outflow points of the liquid. The specific heat for the
in 02 chamber
liquid will be :
(e) volume of He chamber will be will be equal to
(a) 0.6 kcaVkgK (b) 0.3 kcaVkgK
volume of 0 2 chamber
(d) volume of O2 chamber will be LA I (2)25121 (c) 0.5 kcaVkgK Cd) 0.4 kcallkgK
39. One mole of a monoatomic ideal gas udergoes the
35. The ratio of specific heat of a gas at constant pressure
process A --+ B in the given P-V diagram. The specific
to that at constant volume is y. The change in internal
heat for this process is :
energy of a mass of gas when the volume changes from
p
V to 2V at constant pressure P is :
R
Ca)
y -1
(b) PV 6P o •.•.... / .•••
.
•B

(e) PV Cd) yPV 3Po :
"".f... '
'f -1 "f -1 •.
••
36. The Fig. shows two paths for the change of state of a v
Vo 5Vo
gas from A to B. The ratio of molar heat capacities in
path 1 and path 2 is : Ca) 3R Cb) 13R
2 6
(e) 5R Cd) 2R
2

www.puucho.com '.
Study Anurag Mishra with www.puucho.com

40. An ideal monoatomic gas is taken round the cycle (a) W2 >W} >W3 (b) W2 >W3 >W1
ABCDA as shown in the P-V diagram (Fig). The work (c) WI > W2 > Wt (d) WI > W3 > W2
done during the cycle is : 46. An ideal gas is initially at temperature T and volume V.
p
It~ volume is increased by .6.V due to an increase in
i"o.V 2~.2V temperature liT, pressure remains constant. The

, ,
quantity 8 = .6.V/V.6.T veries with temperature as :

A D
P,V P,2V
v
Cb) /
(a) PV Cb) 2PV
ee) 112 PV Cd) zero T
T T+dT
41. If M1.M2,M3 •••••••• are the molecular weights of
different gases, R1,R2,RJ ••••••••• are the gases
constants respectively then which of the following is
valid?
(a) M1 +M2 +M3 + .•........ =1 (d) ~
R1 +R2 +R3 + " T
(b) M1 = M2 = MJ = =Constant
R1 R2 RJ
47. 1\vo monoatomic ideal gases 1 and 2 of molecular
(e) MIR} =M2R2 =MJRJ =..... =Constant
masses mt and m2 respectively are enclosed in
Cd) none of these
separate containers kept at the same temperature. The
42. 1\vo identical containers A and B with frictionless
ratio of the speed of sound in gas 1 to the gas 2 is given
pistons contain the same ideal gas at the same

r' r'
by:
temperature and the same volume V. The mass of the
gas in A is rnA and that in B is mB" The gas in each Ca) (b)
m2 mt
cylinder is now allowed to expand isothermally to the
same final volume 2V. The changes in the pressure in (c) ~ (d) m2
A and B are found to be /1P and 1.5/1P respectively, m2 ml
then: 48. P-V plots for two gases during adiabatic processes are
(a) 4mA =
9mB (b) 2mA = 3mB shown in the Fig. Plots 1 and 2 should correspond
(c) 3mA = 2mB (d) 9mA = 4mB respectively to :
43. A monoatomic ideal gas initially at temperature Tt, is p
enclosed in a cylinder fitted with a frictionless piston.
The gas is allowed to expand adiabatically to a
temperature T2 by releasing the piston suddenly. If L}
and L2 are the lengths of the gas column bfare and
~, 2
after expansion respectively, then T1 / T2 is given by: v
(a) (LI / L2)2/3 (b) L1 / L2
(a) He and O2 (b) 02 and He
(c) L2 / L} (d) {L2 / L} )2/3
(c) He and Ar (d) 02 and N 2
44. The C p IC v ratio for a gas mixture consisting of 4 gIns 49. An ideal gas is taken through the cycle A -+ B -+ C
helium and 32 gms of oxygen is : -+ A, as shown in the Figure. If the net heat supplied
(a) 1.45 Cb)1.6 to the gas in the cycle is 5 J, the work done by the gas
(e) 1.5 Cd)1.66 in the process C -+ A is :
45. Starting with the same initial conditions, an ideal gas V(m3)
expands from volume V} to V2 in three different ways,
the work done by the gas is W} if the process is purely
isothermal, W2 if purely isobaric and W3 if purely
2
Cf',t
. .
• A
adiabatic, then:
1
.

P(Nlm2)
10

www.puucho.com
Study Anurag Mishra with www.puucho.com

TH!RMODYNAMICS 233

(a)-5 J (bl -10 J


., (a)"Rf"tn 2 (b) ~ RT
(e) -15 J (d) -20 J 2

50. Which of the following graphs correctly represent the (el RT (d) ~RT
2
variation of P = -dV I dP with P for an ideal gas at
54. In the P-V diagram of Fig. shown, the gas does 5 J of
V
constant temperature: work along isotherm ab and 4 J along adiabatic be.
What is the change in the internal energy of the gas if
p p the gas traverse the straight path form a to c ?
p

(a)

~ p
(b)
\ p
a b

~,

r-
p p
v
J
(a) I (b) 4 J
(el (d)
~ (e) 5 J (d) 9 J

p p 55. An adiabatic change in a perfect gas in one:


(a) which should be carried alit very rapidly III
thin-walled, perfectly-conducting vessel
51. At a temperature of To = 273K, n\lo moles of an ideal (b) in which the temperature T and pressure pare
gas undergoes a process as shown. The total amount of related by the expression Py-IT" constant
;=

heat imparted to the gas equals Q = 27.7kJ. (c) in which internal energy remains constant
Determine the ratio of molar specific heat capacities. (d) in which no heat enters or leaves the system
T 56. For twO different gases X and Y, having degrees of
/c freedom II and 12 and molar heat capacities at
A,"
constant volume C VI and C V2 respectively, for
273K
.~
.'.
,
.' ,
' B adiabatic process, the In P versus In V graph is plotted
as shown:
v
I,P
(a) IAI (b) 1.63
(e) 1.33 Cd)None of these
52. An air bubble of volume V o is released by a fish at a
depth h in a lake. The bubble rises to the surface.
Assume constant temperature and standard I,V
atmospheric pressure above the lake. The volume of (a) II >12 (b)/2>fl
the bubble just before touching the surface will be (e) CVz >CV1 (d)Cv1 >C"z
(density) of water is p :
57. The V-T diagram of an ideal gas for the process
(a) Vo (b) Vo(pgh/p) A ~ B ~ C is as shown in the Fig. Select the correct
ee) t'o (d) vo( I + P~h) altemative{s) :

(l+P~~) v

53. One mole of an ideal gas is contained within a cylinder


by a frictionless piston and is initially at temperature
T. The pressure of the gas is kept constant while it is o T
heated and its volume doubles. If R is molar gas (a) Pressure of the gas first increases then remains
constant, the work done by the gas in increasing its constant
volume is : (b) Pressure of the gas first decreases then remains
constant

www.puucho.com
Study Anurag Mishra with www.puucho.com
234
(c) Pressure of the gas remains constant throughout 62. One mole of an ideal gas with heat capacity at
Cd) Nothing can be said about the pressure of the gas
constant pressure Cp undergoes the process
from this graph
T = To + aV where To and a. are constants. If its volume
58. An ideal gas, contained in a cylinder by a frictionless increases from VI to V2 the amount of heat transferred-
piston, is allowed to expand from voltime VI at to the gas is :
pressure PI to volume V2 at pressure P2' Its
temperature is kept constant throughout. The work
done by the gas is :
(a) CpRTo 11~~r
(a) zero, because it obeys Boyles's law and therefore (b) a.CP(V2-Vl)-RToln~1
P2V2-PIVl =0
(b) negative, because the pressure has decreased and
so the force on the pistOn has been diminishing (c) aCp(V2 -VI)+RTo InIV,[
lV1
(c) zero, because it has been kept at constant
temperature
unchanged
and so its internal energy is
(d) RTo lnl~- aC p (V2 - VI)

Cd) positive, because the volume has increased


59. The molar heat capacity of an ideal gas at constant
63. Fig. shows a parabolic graph between T and -!. for a
V
pressure is greater than that at constant volume mixture of a gas undergoing an adiabatic process.
because
What is the ratio of VmLS and speed of sound in the
(a) work has to be done against intermolecular forces mixture?
as the gas expands
T
(b) work has to be done against external pressure as
the gas expands
(c) the molecules gain rmational kinetic energy at the
2T
o
-7---- ~
gas expands To • :
.'
Cd) the molecules move faster when heat is supplied at (1NoJ (4NoJ (1N)
constant pressure than when supplied at constant
volume Ca) 13 .J2
60. The equation of a state of a gas is given by
V~ (b)

P(V -b) = nRT. If 1 k mol of a gas is isothermally (e) f2 (d) .fj


expanded from volume V to 2V, the work done during V3
the process is : 64. The density p versus temperature T graph of an ideal
(a) RT Inl
2V -
'1
V-b
bl V
(b) RTlnl ~ bl gas is shown in Fig. Choose the incorrect statement(s):
p

V-b
(c)RTln--
I2V-b I (d)RTlnl~1
v=bI
61. An ideal gas has temperature T and volume V. The
o T
quantity is = dVjVdT varies with temperature under
isobaric conditions as:
(a) The curve represents an isobaric compression
(b) The internal energy of the gas decreases
, (c) The system absorbs heat during the process
(d) The curve shifts upwards at higher pressures
(al (b)
0
65. fig. shows three isotherms at temperatures 300 K,
T T+AT T T+AT
Temperature K sao K and 700 K respectively on P-V diagram. lWo
Temperature K
thermodynamic process (A and B) are performed on
one mole of monoatomic ideal gas as shown in Fig.
, ,
(e) ~ (d) ~
0 0
T T+t.T T T+AT
Temperature K Temperature K

www.puucho.com
Study Anurag Mishra with www.puucho.com

THERMODYNAMICS 235
p 68. Liquid oxygen at 50 K is heated to 300 K at constant
pressure of 1 atm. The rate of heating is constant.
Which of the following graphs represents the variation

TL TE-t
of temperature with time?

T, Ca) (b)
T,
T
V

en The gas in process A is heated while the gas in


process B is cooled.
Oi) The work done in process B is less than the work
done in process A.
(iii) The amount of heat is absorbed by the gas in both
processes.
Which of the above observations must be correct?
(a) (ii) only (b) (ij) and (iii) only
69.
(e) T

Lt Lt Cd)T

When an ideal diatomic gas is heated at constant


pressure the fraction of the heal energy supplied
ec) (i) and (ii) only Cd) (i) and (iii) only which increases the internal energy of the gas is :
(a) (2/5) (b) (3/5)
66. The poT diagram of thermodynamic heat engine cycle
is shown in Fig. Two curved processes arc adiabatic. (e) (7/5) (d) (517)

The work done for one mole of monoatomic gas in one 70. During the adiabatic expansion of 2 moles of a gas, the

.
cycle is given by : change in internal energy was found to be equal to
p (-IOOJ). The work done during the process will be
8 C equal to :

U
3T, ST' (a) zero (b) 200 J
(e) -100J Cd)100J
To 2T 71. A container contains n moles of oxygen molecules
:~,0 0
,>- (02), The molecules get dissociated if the temperature
a T T of the gas is greater than some minimum
(a) 1.25RTQ (b) 2RTo temperature Td• The percentage dissociation into
(c) -2RTo Cd) -1.25RT, atomic oxygen is given by (~ - I). The ideal gas

67. A cyclic process ABeD is shown in a V-I' diagram, the equation can be written as (for Td ::::;T ::::;2Td):
2
corresponding P-V diagram is : (a) PV == nRT (b) PV == nRy
Td
V

:[78 72.
Ce) PV = 2nRT Cd) PV = nR(2T -Td)
In a thermodynamic process for an ideal gas, the

... T
system and the surrounding can influence each other
only through work. Which of the following curves best
represents this process?
P P
p

Ca) A'CJ c
Cb)
A~=J c
'V

~I
III (isothermal)
II
v v
+----_v
p p
c (a) I (b) II
(c) III Cd) IV
ee)
.LJ: v
Cd)
8~A v

www.puucho.com
Study Anurag Mishra with www.puucho.com

THERMODYNAMICS
73. PV curve for the process whose vr curve is:
T

adiabatic
.
C7'
-,' b

p~v
V 78. A rigid tank contains 35 kg of nitrogen at 6 atm
Sufficient quantity of oxygen is supplied to increases
the pressure to 9 atm. while the temperature remains
(a) P~y (b) constant. Amount of oxygen supplied to the tank is:
(a) 5 kg (b) 10 kg
(e) 20 kg (d) 40 kg

Is~~vIt="
79. A perfect gas of a given mass is heated first in a small
vessel and then in a large vessel, such that their
(e) (d)
volumes remain unchanged. The P-T CUlVesare:
, (a) parabolic with same curvature
(b) parabolic with different curvature
74. Find the approx. number of molecules contained in a (c) linear with same slopes
vessel of volume 7 litres at O°C at 1.3 x lOs pascal: (d) linear with different slopes

J;
(a) 2Ax 1023 (b) 3.0x 1023 80. At a temperature 11<,the pressure of 4.0 g argon in a
Ce) 6.0x 1023 Cd) 4.8 x 1023 bulb is P. The bulb is put in a bath having temperature
75. One mole of an ideal gas is kept enclosed higher by 50 K than the first one. 0.8 g of argon gas
under a light piston (area : IO-2m2) had to be removed to maintained original pressure.
connected by a compressed spring (spring The temperature T is equal to:
constant IOON/m). The volume of gas is (a) 510 K (b) 200 K
a.83m3 and its temperature is lOOK. The (e) 100 K (d) 73 K

gas is heated so that it compresses the spring further 81. When 2 g of a gas are introduced into an evacuated
by 0.1 m. The work done by the gas in the process is flask kept at 25°C the pressure is found to be one
(Take R = &3J/mole and suppose there is no atmosphere. If 3 g of another gas added to the same
atmosphere) : flask the pressure becomes 1.5 atmospheres. The ratio
(a) 3J (b) 6J of the molecular weights of these gases will be:
(e) 9J (d) 1.5J (,)1:3 (b)3:1
76. An ideal gas mixture filled inside a balloon expands (e) 2 : 3 (d) 3 : 2
according to the relation PV2l3 = constant. The 82. An ideal gas foHows a process PI' = constant. The
temperature inside the balloon is: correct graph between pressure and volume is:
(a) increasing
(c) constant
(b) decreasing
(d) can't be said
77. An ideal gas undergoes a thermodynamics cycle as
shown in Fig. Which of the following graphs
(a) t2 v
(b) p~

p~ plL
represents the same cycle?

(e) (d)
v~" ••• __ y, v

1~1:."
83. The process AB is shown in the Fig. As the gas is taken
from A to B, its temperature:

(a) p~" (b)

www.puucho.com
Study Anurag Mishra with www.puucho.com

THERMODYNAMICS
89. The ratio of average translational kinetic energy to
2P A
rotational kinetic energy of a diatomic molecule at
P .... 0,..
,,
,,
,
temperature Tis:
(n) 3 (b) 7/5
, , (e) 5/3 (d) 3/2
90. One mole of an ideal gas at STP is heated in an
insulated closed container until the average speed of
its molecules is doubled. Its pressure would therefore
increase by factor:
(a) 1.5 ./2
(b) ..
84, (e) 2 (d) 4
91. Three particles have speeds of 2u, 1011 and 11 u. Which
of the following statements is correct?
(a) The r.m.s. speed exceeds the mean speed by about

"
(b) The mean speed exceeds the r.m.s. speed by about
u
(c) The r.m.s. speed equals the mean speed.
85,
(d) The r.m.s. speed exceeds the mean speed by more
than 2u
92. The ratio of specific heats of a gas is ~, then the
7
number of degrees of freedom of the gas molecules for
translational motion is:
86. (a) 7 (b) 3
(c) 6 Cd) none
93. A diatomic gas of molecules weight 30 glmole is filled
in a container at 27°C. It is moving at a velocity 100
mls. If it is suddenly stopped, the rise in temperature
of gas is:
(a) 60 (b) 600
R R
4
(c) 6)( 10 Cd) 6)( lOs
Temperature R R
(n)C (b) A
94. One mole of an ideal diatomic gas is taken through the
cycle as shown in the Fig. ?
(e) B Cd) none
1 2 : isochoric process
---)0

87. A vessel contains 1 mole of O2 gas (molar mass 32) at


2 ~ 3 : straight line on P-V diagram
a temperature 7'. The pressure of the gas is P. An
3 ~ 1 : isobaric process
identical vessel comaining one mole of He gas (molar
The average molecular speed of the gas in the states
mass 4) at a temperature zr has a pressure of;
1, 2, and 3 are in the ratio:
(n) P/B (b) P
(e) 2P (d) BP P 2{V04Pol

88. A container X has volume double that of container Y


<lnd both are connected by a thin tube. Hath contains
~3(4VO'POI
same ideal gas. The temperature of X is 200 K and that 1 (Vo.Po)
of Y is 400 K. [f mass of gas in X is m then in Y it will
v
(a) I : 2 : 2 (b) 1 0 .fi 0 .fi
be:
(a) m/8 (b) 111/6 (c)I:l:l (d)lo204
ee) m/4 (d) m/2

www.puucho.com
Study Anurag Mishra with www.puucho.com

238 THEIlMOOYNAMlCS
95. (N < 100) molecules of a gas have velocities 1, 2, 100. In the above thermodynamic process, the correct
3 N/km/s respectively. Then: statement is:
(a) r.m.$. speed and average speed of molecules is P
same
(b) ratio of r.m.s. speed to average speed is
J(2N + I)(N + 1)/6N
3PO--:2]8
2Po ••
ee) ratio of r.m.s. speed to average speed is
, ,C
, , V
JC2N + I)(N + 1)/6 Vo 2Vo
Cd) ratio of r.m.s. speed to average speed of a molecule
(a) Heat given in the complete cycle ABO\ is zero
i, 2/.J6X x JC2N + I/(N + I)
(b) Work done in the complete cycle ABC4. is zero
96. At temperature T, N molecules of gas A each having (e) Work done in the complete cycle ABO\ is (1/2Po Vol
mass m and at the same temperature 2N molecules of (d) None of these
gas B each having mass 2 m are filled in a container. 101. Pressure versus temperature graph of an ideal gas is
The mean square velocity of molecules of gas B is v2 shown in Fig.
and mean square of x component of velocity of
molecules of gas A is w2 the ratio of W2/V2 is :
Ca)1 (b) 2
P :~:
Ce) 1/3 (d) 2/3
97. Five particles have speeds 1, 2, 3, 4, and 5 mls,the ':--A T
average velocity of the panicles is (in mls) :
Ca) 3 (a) During the process AB work done by the gas in
(b) 0 positive
(e) 2.5 (b) During the process CD work done by the gas in
(d) cannot be calculated negative
(c) During the process BC internal energy of the gas is
98. A given mass of a gas expands from a state A to the increasing
state B by three paths 1, 2 and 3 as shown in T-V (d) None of these
indicator diagram. If WI' W2 and W3 respectively by
102. A reversible adiabatic path on a P-V diagram for an
the work done by the gas along the three paths, then:
ideal gas passes through state A where
T P",0.7x 105 N/m-2 and V ",0,0049 m3. The ratio of

0
..~
I2J" 3
V
specific heat of the gas is 1.4. The slope of path at A is:
(a) 2.ox107Nm-s
(c) -2.0x 107Nm-s
(b) 1.0x107Nm-s
(d) -1.0x 107Nm-s

103. An ideal gas is taken from point A to point C on P-V


diagram through two process AGC and ABC as shown
in the Fig. Process AGC is isothermal.
(a) WI >W2>W3 (b) WI <W2<W3
P
(c) WI "'W2 ",W3 (d)W1 <W2,WI >W3

PO.J~~c
99. An ideal gas undergoes the process 1 2 as shown in
-jo

the Fig. the heat supplied and work done in the


process is l!Q and.1.W respectively. The ratio .1.Q:l!.W
is:
. :,

Vl ....
O~li
7' Vo 2Vo
V

(a) Process AGC requires more heat than process ABC


(b) Process ABC requires more heat than process AGC
(c) Both process AGC and ABC require same amount
(a) y:y-1 Cb) y of heat
Ce) y-1 Cd)'I-I/y (d) Data is insufficient for comparison of heat
requirement for the two processes

www.puucho.com
Study Anurag Mishra with www.puucho.com

THIRMODYNAMICS 239

.04. Heat is added to an ideal gas and the gas expands. In p ,

\:::]:
such a process the temperature:
(a) must always increase
(b) will remain the same if the work done equals the
hear added v
(c) must always decrease
(d) will remain the same if change in internal energy (a) - 40 J (b)-20J
equals the heat added (c) +40 J (d) +20 J
11105. One mole of an ideal gas requires 207 J heat to raise 110. When unit mass of water boils to become steam at
the temperature by 10K when heated at constant 100"C, it absorbs Q amount of heat. The densities of
pressure. If the same gas is heated at constant volume water and steam at 100"C are P I and P 2 respectively
to raise the temperature by 5 K,then the heat required and the atmospheric pressure is Po, The increase in

po(~-~l
may be : internal energy of the water is:
(a) 62.lJ (b) 124J (a) Q (b) Q +
(e) 12.4J (d) 6.2J PI P2
106. A thermodynamic cycle takes in heat energy at a high
temperature and rejects energy at a lower
(e)Q+Po(~-~l (d)Q-Po(~+-11
P2 PI PI P2
temperature. If the amount of energy rejected at the
111. A poly-atomic gas with six degrees of freedom does
low temperature is 3 times the amount of work done
25J of work when it is expanded at constant pressure.
by the cycle, the efficiency of the cycle is:
The heat given to the gas is:
(a) 0.25 (b) 0.33 (a) 100J (b) 150J
(e) 0.67 (d) 0.9 (e) 200 J (d) 250 J
107. Monoatomic, diatomic and triatomic gases whose 112. An ideal gas expands from volume VI to V2. This may
initial volume and pressure are same, are compressed be achieved by either of the three processes: isobaric,
till their volume becomes half the initial volume. isothermal and adiabatic. Let AU be the change in
(a) If the compression is adiabatic then monoatomic internal energy of the gas Q, be the quantity of heat
gas will have maximum final pressure added to the system and W be work done by the
(b) If the compression is adiabatic then tri-atomic gas system on the gas. Identify which of the following
will have maximum final pressure
statements is false for AU ?
(c) If the compression is adiabatic then their final
(a) f..U is least under adiabatic process
pressure will be same
(d) If the compression is isothermal then their final (b) t:,.U is greatest under adiabatic process
pressure will be different (c) AU is greatest under the isobaric process
Cd) AU in isothermal process lies in between the
108. If heat is added at constant volume, 6300 J of heat are
values obtained under isobaric and adiabatic
required to raise the temperature of an ideal gas by processes
150 K. If instead, heat is added at constant pressure, 113. In an isobaric expansion of an ideal gas, which of the
8800 joules arc required for the same temperature following is zero?
change. When the temperature of the gas changes by (a) work done (b) AQ
300 K. the internal energy of the gas changes by: (e) ~u (d) d' V/dT'
(a) 5000 J (b) 12600J
114. A perfect gas is found to obey the relation PV3/2 =
(e) 17600J (d) 22600 J
constant, during an adiabatic process. If such a gas,
109. Three processes form a thermodynamic cycle as shown initially at a temperature 1', is compressed
on P-V diagram for an ideal gas. Process 1 -+ 2 takes adiabatically to half its initial volume, then its final
place at constant temperature (300 K). Process 2 -+ 3 temperature will be:
takes place at constant volume. During this process (a) 2T (b) 4T
40J of heat leaves the system. Process 3 -+ 1 is (c) ",Fl.T (d) 2.'/21'
adiabatic and temperature 1'3 is 275 K. Work done by 115. An ideal monoatomic gas is carried around the cycle
the gas during the process 3 -+ 1 is: ABCDA as shown in the Fig. The efficiency of the gas
cycle is:

www.puucho.com
-
Study Anurag Mishra with www.puucho.com

p
B

0:
120. '!Wo identical vessels A and B contains equal amount
of ideal monoatomic gas. The piston of A is fixed but-

3~:::.. A:,
that of B is free. Same amount of heat is absorbed b}'
A and B. If B' s internal energy increases by 100 J, the
change in internal energy of A is:
V
V, 2V,

Ca) ~ (b) 2 A B
25 21
4
(e) - Cd) 2
31 31
116. A gas takes part in two processes in which. it is -heated ''0
from the same initial state 1 to the. same final
(a) 100 J (b) 500 J
temperature. The processes are shown on the P.V
3
diagram by the straight line 1 -) 2 and 1 -) 3. 2 and 3
(c) 250 J (d) none of these
are the points on the same isothermal curve. Ql and Q2
121. Three processes compose a thermodynamics cycle
are the heat transfer along the two processes. Then:
shown in the P.V diagram. Process 1 -+ 2 takes place
p
at constant temperature. Process 2 -+ 3 takes place at
constant volume, and process 3 -+ 1 is adiabatic
During the complete cycle, the total amount of work
done is 10 J. During process 2 -lo 3, the internal energy
decrease by 20 J and during process 3 -+ 1, 20 J of
(a) Q, =Q, (b) Q, <Q, work is done on the system. How much heat is added
(c) Q1 > Q2 Cd) insufficient data to the system during process 1 -+ 2 ?
117. In thermodynamic process pressure of a flXed mass of p
gas is changed in such a manner that the gas releases
30 joule of heat and 18 joule of work was done on the
gas. If the initial internal energy of the gas was 60
joule, then, the final internal energy will be: V
(a) 32 joule (b) 48 joule (a) 0 (b) 10 J
(c) 72 joule Cd) 96 joule (e) 20 J (d) 30 J
118. A cylinder made of perfectly non-conducting material 122. An ideal gas undergoes an adiabatic process obeying
dosed at both ends is divided into two equal pans by a the relation pv4/3 ""constant. If its initial temperature
heat proof piston. Both parts of the cylinder contain is 300 K and then its pressure is increased upto four
the same masses of a gas at a temperature to"" 27'lC times its initial value, then the final temperature is (in
and pressure Po "" 1 atm. Now if the gas in one of the kelvin) :
pans is slowly heated to t "" 57°C while the (a) 300-/2 (b) 300 1/2
temperature of first part is maintained at to the (e) 600 (d) 1200
distance moved by the piston from the middle of the 123. The adiabatic Bulk modulus of a diatomic gas at
cylinder will be : (length of the cylinder"" 84 cm) atmospheric pressure is:
(a) 3 cm (b) 5 cm (a) Zero (b) 1 Nm-2
(c) 2 cm (d) 1 em 4
(c) lAx 10 Nm~2 (d) lAx 105 Nm-2
119. 1 gm water at 100 e and 105 Pa pressure converts into
D
124. A closed container is fully insulated from outside. One
1841 emJ of steam at constant temperature and half of it is filled with an ideal gas X separated by a
pressure. If latent heat of vaporization of water is plate P from the mher half Ywhich contains a vacuum
2250 J/gm. The change in internal energy of water in as shown in Fig. When P is removed, X moves into Y.
this process is: Which of the following statements is correct?
(a) zero (b) 2250 J
(c) 2066 J (d) none of these

www.puucho.com
Study Anurag Mishra with www.puucho.com

THERMODYNAMICS
127. Consider the thermodynamics cycle shown on P-V
diagram. The process A -+ B is isobaric, B -jo C is
x y
gas vaccum isochoric and C _ A is a straight line process. The
p
following internal energy and heat are given:
P(Nm-3)

::::1\l:.
(a) No work is done by X
(b) X decreases in temperature
ee) X increases in internal energy
(d) X doubles in pressure ,
125. In Ideal gas undergoes a thermodynamics cycle as
shown in figure .. Which of the following graphs
1.5 v(m3)
represents the same cycle?
V 6.U.•._.H =+400kJ and Q/J-oC =-sOOkJ
e
V
B The heat flow in the process QC-->A is:
(a) - 20 kJ
(b) +25 kJ
.
/'A
(e) - 25 kJ
T Cd) Data are insufficient
p p
128. 1 kg of a gas does 20 kJ of work and receives 16 kJ of

(a)
Af7 B

(b)
...
.A: heat when it is expanded between two states. A second
kind of expansion can be found between the initial and

.
. .' e
T T
final state which requires a heat input of 9 kJ. The
work done by the gas in the second expansion is:
(a) 32 kJ (b) 5 kJ
p p

A\]: A~:
(e) -4 kJ (d) 13kJ
129. A vessel contains an ideal monoatomic gas which
(e) (d) expands at constant pressure, when heat Q is given to
it. Then the work done in expansion is:
T T
(a) Q (b) ~Q
5
126. A cyclic process ABC4. is shown in P-T diagram. When
2 (d) ~Q
presented on P-v, it would: (e) -Q
5 3
p

"VA
... .e
130. One mole of an ideal monoatomic gas at temperature
To expands slowly according to the law P/V '" constant.
If the final temperature is 2To• heat supplied to the gas
is:
T 3
(a) 2RTo (b) ~RT,
2
1
(c) RTo (d)~RTo
2
131. A diatomic gas follows equation PV'" '" constant,
during a process. What should be the value of m such
that its molar heat capacity during process'" R ?
(a) 2/3 (b) 1
(e) 1.5 (d) 5/3
132. One mole of an ideal gas at temperature T1 expends
according to the law ~ '" a (constant). The work
V'
done by the gas till temperature of gas becomes T2 is:

www.puucho.com
Study Anurag Mishra with www.puucho.com

ICS
1
(a) -R(T, -T,) twice its volume and then compressed at constant
2
1 pressure to CVo/2) and the gas is brought to original
(e) -R(T2 -TI) state by a process in which P IX: V (Pressure is directly
4
proportional to volume): The correct representation of
133.2 mole of a diatomic gas undergoes the process: process is :
PT2 IV =constant. Then, the molar heat capacity of the I
gas during the process will be equal to : P
(a) sR/2 (b) 9R/2
(a)
(e) 3R (d) 4R Po •••• - ••
..
134. A resistance coil connected to an ~
external battery is placed inside an
adiabatic cylinder fitted with a v m
frictionless piston and containing j . R" i
IP,
an ideal gas. A current i flows , P
through the coil which has a resistance R. At what
(b)
speed must the piston move upward in order that the
temperature of the gas remains unchanged? Neglect
.,
atmospheric pressure .
(a) ~ (b) Rmg
Rg i'
(e) mg (d) i'R
i' mg
135. A long glass tube of length L, sealed at both ends,
contains a small column of mercury (density =p) of
length 'o'(a «L) at its middle and air at pressure P on
both sides. The tube is fixed horizontally. If the
mercury column gets a small displacement, the time

.-?
VQ
period of its oscillations would be (assuming that the
air on the sides undergoes isothermal expansion or
compression) : (d) t
(a) n[pLajP]"' (b) 2n[p La/P]>2
v<f2 ;:. :
(e) n[2pLajP]"' (d) n[pLa/ZP]u,
ToT •...•
136. Two pistons having low thermal conductivity divide an
adiabatic container in three equal parts as shown. An 138. Two completely identical samples of the same ideal
ideal gas is present in the three parts A.B & C having gas are in equal volume containers with the same
initial pressures as shown and same temperatures. pressure and temperature in containers labeled A and
Now the pistons are released. Then the final B. The gas in container A performs non.zero positive
equilibrium length of part A after long time will be: work W on the surroundings during an isobaric
(constant. pressure) process before the pressure is
L
• reduced isochorically (constant volume) to 1/2 its
B c initial amount. The gas in container B has its pressure
2P P reduced isochorically (constant volume) to 1/2 its
initial value and then the gas performs same non-zero

(a) L/B
___
-U3

(b) L/4
positive work W on the surroundings during an
isobaric (constant pressure) process. After the
processes are performed on the gases in containers A
and B, which is at the higher temperature?
(e) L/6 (d) L/s
(a) The gas in container A
137. One mole of an ideal gas at pressure Po and
(b) The gas in container B
temperature To volume Vo is expanded isothermally to
(c) The gases have equal temperature.

www.puucho.com
Study Anurag Mishra with www.puucho.com

THERMODYNAMICS
Cd) The value of the work W is necessary to answer (a) The number of molecular collisions per second on

~I
this question. the common wall
(b) the average speed of the molecules
(c) The pressure exerted by the helium
139.
~\ _~111
(21
(1~:.'1 (d) The density of the helium
143. Under isobaric condition if the temperature of a room
increases then :
v-
, P-
II
(a) the total kinetic energy of the molecules in the
room will increases
(b) the total kinetic energy of the air molecules
contained in the room will decrease.
(e) the total kinetic energy of the air molecules in the
.'~...'. room will remain the same.
(d} the density of air in the room will increase.
v- 144. A thermod}'namic system is taken through the cycle

'" abea as shown in the figure. Then:


P
If (1) represents isothermal and (2) represents 200 kPa •
adiabatic, which of the graphs given above in respect
of an ideal gas are correct?
(a) I and II (b) 11 and III 100kPa ••••• b
':
(e) I and III Cd) I, II and III
140. For an ideal gas the equation of a process for which the 100 cm3
v
300 cm3
heat capacity of the gas varies with temperature as
(a) heat rejected by the gas during the process is 10J
C = afT (a is a constant) is given by : (b) heat absorbed by the gas during the pTQcessis 10J
(a) VlnT = constant (c) work done by the gas is 20 J
,
(b) \lTl(y-lle"RT =constant (d) work done by the gas is -10J
145. Oce mole of an ideal monoatomic gas is taken from
(e) Vy-1Te(/,RI = constant
point A to C along the path ABC. The initial
Cd) Vy-1 = constant
temperature at A is To. For the process A -1> B -1> C:
141. Find the pressure at which temperature attains its
p
maximum value if the relation between pressure and
volume for an ideal ~sP = Po + (1- u)V2; X > 1 A
Po B
(a) 2Po (b) Po P .
3 3 ..J! ~ ,c
2
(e) Po Cd) 4Po
3 v
142. Cubical tanks X and Y have the same volumes and
share a common fixed perfectly conducting wall. (a) Heat absorbed"" RTo/2
There is 1 gm of helium in tank X and 2 gm of helium (b) Heat liberated"" 2R'fo
in tank Y. Which of the following is the same for both (c) Change in internal energy ""0
samples treating them as ideal gases? (d) Work done = 0
146. One of the straight lines in the Fig. depicts the
x y dependence of the work done IWI on the temperature
variations T for an isobaric process. The two of
/ \ following are the adiabatic curves for argon and
.
.:1gm 2gm' ..
. nitrogen. The isobaric process which depicts both the
..
. He He' ". gases is :

www.puucho.com
Study Anurag Mishra with www.puucho.com

'.' process AB is two times the work done in the process


CD then what is the value of T1/T2 ?
T

.._--~ .. •••• B
T T'-'P~,'
2 " •• ' D
.'
(a) 1
(e) 3
(b) 2
(d) 4
.. '
P
147. A same amount of same gas of temperature Tare (aJ 1/2 (b) 1
enclosed in a three identical A, B & C. The temperature (e) 2 (d) 4
of wall of three container is TA,TB &Tc• (TA >Ta >TcJ 150. The P-V diagram shows four different possible
respectively. The pressure on wall of vessel : reversible processes performed on a monatomic ideal
(a) PA >PB >Pc I .'. gas, Process A is isobaric (constant pressure), Process
(b) PA <PH <Pc B is isothermal (constant temperature). Process C is
ee) PA ""ps =Pc adiabatic, Process D is isochoric (constant volume).
Cd) Data's are insufficient to decide For ~hich process(es) does the temperature of the gas
148. The pressure P of a gas is plotted against its absolute decrease?
temperature T for two different constant volumes VI P
A
and Vz, where ~ > V2• P is plotted on the y-axis and T Po
on the x-axis :
(a) The curve for VI has greater slope than the curve ..! Po D~
for V2 2
(b) The curve for V2 has greater slope than the curve V
Vo 2Vo
for V)
(e) The curves must intersect at some point other [han (a) Process A only
T ~O (b) Process Conly
Cd) The curves have the same slope and do not (c) Only Processes C and D
intersect (d) Only Processes B, C and D
149. On a T-P diagram, two moles of ideal gas perform
process AB and CD. If the work done by the gas in the

www.puucho.com
THERMODYNAMICS
-
Study Anurag Mishra with www.puucho.com

245

ANllWERll

3. (0) 4. (a) 5. (b) 6. (d) 7. (0) B. (b)


1. (a) 2. (0)
(b) 12. (0) 13. (d) 14. (b) 15. (d) 16. (d)
9. (0) 10. (0) 11.
19. (0) 20. (b) 21. (b) 22. (0) 23. (d) 24. (a)
17. (a) lB. (b)
(a) 27. (b) 2B. (b) 29. (d) 30. (d) 31. (d) 32. (d)
25. (b) 26.
(b) 37. (a) 3B. (a) 39. (b) 40. (a)
33. (b) 34. (d) 35. (0) 36.
43. (d) 44. (0) 45. (a) 46. (0) 47. (b) 4B. (b)
41. (0) 42. (0)

(a) 51. (b) 52. (d) 53. (0) 54. (a) 55. (d) 56. (b)
49. (a) 50.
(b) 60. (a) 61. (0) 62. (0) 63. (b) 64. (0)
57. (a) 5B. (d) 59.
67. (a) 6B. (0) 69. {d) 70. (d) 71. (b) 72. (b)
65. (0) 66. (a)
(d) 76. (a) 77. (a) 7B. (0) 79. (d) BO. (b)
73. (a) 74. (a) 75.
(a) B5. (b) 86. (0) B7. (0) BB. (0)
B1. (a) B2. (0) B3. (a) B4.

(a) 92. (b) 93. (a) 94. (a) 95. (d) 96. (d)
B9. (d) 90. (d) 91.
99. (a) 100. (0) 101. (0) 102. (0) 103. (a) 104. (b)
97. (d) 98. (a)
(a) 108. (b) 109. (a) 110. (b) 111. (al 112. (b)
105. (a) 106. (a) 107.

115. (a) 116. (b) 117. (b) 118. (0) 119. (0) 120. (b)
113. (d) 114. (0)

123. (d) 124. (a) 125. (a) 126. (0) 127. (0) 128. (d)
12l. (d) 122. (a)

131. (d) 132. (b) 133. (d) 134. (d) 135. (a) 136. (b)
129. (0) 130. (a)

139. (d) 140. (b) 141. (a) 142. (b) 143. (0) 144. (b)
137. (0) 138. (b)
147. (a) 148. (b) 149. (0) 150. (0)
145. (0) 146. (d)

www.puucho.com
Study Anurag Mishra with www.puucho.com --
Solution

Level-I: Only One Alternative is Correct


21. [b) The gas pressure
2. [el 4S Weight of piston
P-Po ""-
r = --------+ atm. pressure
Area of cross - section
4S
dp",,-dr
r' = 8><9.8 +1.00xlOsN/m2
60 x 10-4
dw
C ""Cv +-- = 1.13>< 105N / m2
ndT

""Cv +[R + 16.511: rdr] "" 7R During the heating process, the internal energy is
3 ndT 2 changed by AU1 and work liWI• is done.
12. [el Let V be the volume of natural gas needed. Therefore, liQ1 = AUt + liWI = .1U1 + p. dV
Then [37.3 x 106]V =AU\ +(1.13)( lOs)(U20x 60x 10-4)
= [(4.54)(4184) + (145)(389)]75
:= toUl + 136J
Solving we get V "" O.039m2 "" 39L
During the cooling process, no work is done as
15. [dl P2 =2P], V2 =:4V} , n =1 volume is constant, AW = 0.
PdV
C =:Gv +_ Hence, .6.Q2 =dU2" But tJ.U2 is negative as the
dT
temperature decreases, and since the gas returns
dw =: Pdv =: Area to its original temperature, tlU2 := -I1U1"
1
=: -[(PI + P2)(V2 - VI) Hence, [.1Q\ -lliQ21l
2
= (troUI + 136-IlU1) = +136 J
1 9
",-(3PI x3VI)"'-PIVI 23. [d) Suppose there are n1 moles of hydrogen and n2
2 2
moles of helium in the given mix(Ure. Then the
dT ",T2 -TJ '" P2V2 _ PJVI pressure of the mixture will be,
R R
nlRT n2RT RT
_ 2PJ x 4V) p) VJ TPI VI P =--+-- =(nt +n2)-
~~~---=-- V V V
R R R
=> 2xI01.3x103""(n +n )(0.3)(300)""1.62
c =~R+.2x PIV]R ] 2 20xIQ-3
2 2 7P]VI
The mass of the mixture is,
'" ~R + 9R '" 44R '" 22R nj x2+n2x4",S
2 14 14 7 ... (1)
(nJ + 2n2) = 2.5 ...(2)

www.puucho.com
Study Anurag Mishra with www.puucho.com

IHERMOD (

Solving equation (1) and (2), 2Cv ",,(lx%+lX~)R=4R


"1 0: 0.74, "2 '" 0.88

H coce, --
mH
~
0.74)( 2 2C ~(IX~+ h
p 2
Z)R
2
~ 6R
mHe 0.88x 4
Cp 6R
= 1.48 """,*2: 5 - ~- ~1.5
3.52
Cv 4R
26. [a] The first law of thermodynamics gives with E as 75. [d] kxo = PA [xo is initial compression]
internal energy, dE = -dE for the adiabatic
process. The energy of monoatomic ideal gas is
100xo =( n~T)x 1O~2

given by x ~~I_xlxa3xl00x_1_=o.l
E ~~NkT o 100 0.83 100
2 0.2 (Total compression)
Xl =

dE ~ ~NkdT 1
Wgas =-xl00x(0.2) 2 1
--xl00x(0.1) z
2 2 2
Moreover using the ideal gas law 100 1
~_x_ [4-1]=1.5
dW ~PdV ~ NkTdV 2 100
V 105. [a] 207=CpxlO
~NkdT ~ _ NkT dV 6.Q=Cyx5
2 V
6.Q '" 207 x.s:.. = 103.5
dT +3:dV =0 2 Cp Y
T 3 V
Minimum value of 6.Q is for maximum
38. [aJ This is a problem of 'Flow calorimeter' used to
measure specific heat of a liquid. y = Ymax ="35 for monoatomlc. gas
Amount of heat supplied to the water per second
by the heating coil", Q.• = 250 joule ~Q ~ 103.5 ~ 61.5
(5/3)
250
~--kcal.
4186 (61.5$ 6.Q< 103.5)
the volume of liquid flowing out per second 135. [a]
= 8.0em:'! = 8x IO-6m3
-,-
(L-a)
-,-
(L-a)

Mass of this liquid = (0.85) x 1000 x 8 x lO-6kg


Temperamres rise of this mass of liquid = 15°C
I

l-.
-,--'
p

• •
l-a
-,-+'
p

I
Hence, 250

Hence,
4186

S
= mst = 0.85 x 8x 10-3 x sx 15

= -------

=
250x 103
4186>< 0.85)( 8x 15
0.6 kcaVkg K
I

P1V1 "'PZV2

p(L;a)A ~P2(L;a -X)A


• •
p

I

41. [c) The product of molecular weight and gas


_ pel-a)
constant is Rw(universal gas constant) P 2 -
(L-a-2x)
Hence
MjRj ""M2R2 ""Constant p(L;a)A =(L;a + X )AP2
44. [c] The molecule contains 1 mole of helium and one
mole of oxygen P. _ (L-a+
2 -
pel-a)
2x)
4 gm of helium (monatomic weight and
molecular weight) corresponds to 1 mole 32 gm _P' _ peL -a) peL -a)
P22-
of oxygen (diatomic molecular weight 32) [(L-a)-(2x)J [(L-a)+2x]
corresponds to one mole.

t www.puucho.com
Study Anurag Mishra with www.puucho.com

tV' = 4P(L -a) 140. [bJ '


[(L _all _4x1]
~Q=dU+PdV
F = tV'A = 4PA(L - a)
[(L_a)2 _4x2] => ~dT=CvdT+PdV
T

~=~L:=2~L: =>
a
-dT=CvdT+-dV
T
RT
V
T =~(L~r2
= ~ =>
I
C = VTy-1e<liRT
PV 141. [a]
136. [bl n=-
RT PV = nRT
Final pressure & temperature is same hence
=> Po+(1-aV2)V=nRT
volume will be in ratio of number of moles.
137. [e] For isothermal process VI = 2Vo T = (Po + (1 - a)V2)V
PI = Po/2 nR
For isobaric process 2
dT = Po + 3V (I-a) ",,0
Vo dV nR nR
VI = Vo/ 2, Tf = ---To.
2x 2Vo Po = 3(a -1)V2
To
=- V2 = PJ
4
3(a-l)
For P oc V process
p
p - V must be straight line P=Po+(l-a) 0
2 3(a -1)
T oc V =;. V - T must be parabolic
=;. Po 2P
p2 ocT P - T must be parabolic =Po--=- o
3 3
138. [bl e.g .•
143. [eJ

u = nf RT, where f is degrees of freedom and U is


2
internal energy.
also PV = nRT
(P0f2,VO,ToJ2 W 1
v, V
v => U=PV-
2
Po Since the total volume of the room is constant
W = Po' Vo = 2" (VI - Vol
and pressure is also constant therefore energy of
VI =- 3Vo => T, = '3[0/2 the room remains constant. Same molecule will
TB > TA leave the room.
139. [dl Asy>1 144. [b] Work done = area under the CUlVe

for 1Vy-1 = constant d'T


dT < 0 -->0 =!2 x 200x 100 ""lOx 103 kPacm3
dV dV'
and for T1 =KPy-l L\Q "" L\W (in a cyclic process)
dT > 0 --<0 d'T 146. [dl For adiabatic process, "" nf RL\T and for
L\Wa
dP dP' 2
isobaric process L\Wj "" nR.L\T role of the line is
less for isobaric process.

www.puucho.com
Study Anurag Mishra with www.puucho.com

149. [c:] Let slope of line AG is m} & that of line DB1s'm2', 160. [el PV = nRT
then For isobaric conditions
T PdV =nRdT
A •••
T, "' •• 'B
dV nR
-=-
C ,':
T .... ~O
2 ••
: .'::
;,'.
•••• :
:
:
. dT
dV
P
nRP 1
1
,,'
.... ::
'~' :
p
--=--=-
VdT PnRT T t
.!L!l .!l. 11
m m m2 m2 D=-
1
1 1
T
m2. m2. hence rectangular hyperbola.
nRT}ln- ~ 2nRT2.ln-
m} m}
.- ,.... _ , f'

T1 = 2 , ',' ,
T,

www.puucho.com
Study Anurag Mishra with www.puucho.com

250 THlIMllIlilWllCS

Problem ~I

9?eve I f2\ More than One Alternative is/are Correct


~'-----------'
1. In the arrangement shown in Fig. (c) Heat absorbed/released by p
Gas is thermally insulated. An the gas is maximum in path
ideal gas is filled in the cylinder k 1 1
having pressure Po (>
m, , (d) Temperature of the gas first A~B
atmospheric pressure Po)' Spring increases and then decreases '-..;.7
of force constant k is initially continuously in path 1 3
unastretched. Piston of mass m 4. Two moles of a monoatomic L--- 'V
and area S is frictionless. In V'
equilibrium piston rises up a ideal gas undergoes a thermodynamic process - =
distance xo, then: r'
constant, ifthe temperature is raised by 300 Kthen;
(a) final pressure of the gas is Pa + kxo + mg (a) work done by the gas is 400R
S S
(b) change in internal energy is 900R
(b) work done by the gas is .!. kx5 + mgxo (c) molar heat capacity of the gas for the process is
2
13(6 R
(e) decrease in internal energy of the gas is
1 , Cd) molar heat capacity of the gas for the process is
- kxo + mgxo + PaSxo 3/2 R
2
5. During the process A-B of an ideal gas:
Cd) work done by the gas is 12 kx5 p
2
2. P.V diagram of a cyclic process ABC4, is as shown in
Fig. Choose the correct statement.(s) :
p

T
(a) Work done on the gas is zero
(b) Density of the gas is constant
v (c) Slope of line AB from the T-axis is inversely
proportional to the number of moles of the gas
(a) .6.QA-->B = negative (b) .6.UB-oC = positive
(d) Slope of line AB from the T-axis is directly
(e) tiUC ...•A = negative Cd) i'1WCA.B ••• negative proportional to the number of moles of the gas
3. A gas undergoes the change in its state from position A 6. Temperature versus pressure p
to position B via three different paths as shown in Fig. graph of an ideal gas is shown in
select the correct alternative (s) : Fig. During the process AB :
(a) Change in internal energy in all the three paths is (a) internal energy of the gas A--0--8
equal remains constant
(b) In all the three paths heat is absorbed by the gas (b) volume of the gas is T
increased

www.puucho.com
Study Anurag Mishra with www.puucho.com

THERMODYNAMICS 251
(e) work done by the atmosphere on the gas is (c) C I' < 5/2 R for a diatomic gas
positive (d) Cv >R for a diatomic gas
(d) pressure is inversely proportional to volume 11. A barometer is faulty. When the true barometer
7. One mole of an ideal monoatomic gas is taken from A reading are 73 and 75 em of Hg, the faulty barometer
to C along the path ABC. The temperature of the gas at reads 69 em and 70 em respectively:
A is To' For the process ABC : (a) The total length of the barometer tube is 74 em
p (b) The true reading when the faulty barometer reads
69.5 em is 73.94 em
(c) The faulty barometer reading when the true
barometer reads 74 em is 69.52 em
(d) The total length of the barometer tube is 69.5 em
12. The density (p) of an ideal gas p
varies with temperature T as
'-----',---'---V
Vo 2V shown in Fig. Then:
B
o
(a) the product of P& Vat A is
Ca) work done by the gas is RTo
equal to the product of
Cb) change in internal energy of the gas is .!2 RTo P& V at B
2 (b) pressure at B is greater than
A
(e) heat absorbed by the gas is .!...! RTo the pressure at A
2 (c) work done by the gas T
(d) heat absorbed by the gas is 13 RYo during the process AB is
2 negative
(R = universal gas constant) (d) the change in internal energy from A to B is zero
8. Select the correct statements about ideal gas: 13. A gas is found to obey the law P2V == constant. The
(a) Molecules of a gas are in random motion colliding initial temperature and volume are To and Vo. If the
against one another and with the walls of the gas expands to a volume 3Vo' then:
container (a) final temperature becomes J3 To
(b) The gas is not isotropic and the constant 0/3) in (b) internal energy of the gas will increase
equation p = .! pu2 r.m.s. is result of this properry C
3 (c) final tempemture becomes ~
V3
(e) The time during which a collision lasts is
negligible compared to the time of free path (d) internal energy of the gas decreases
between collisions 14. For an ideal gas :
(d) There is no force of interaction between molecules (a) The change in internal energy in a constant
among themselves or between molecules and the pressure process from temperature T1 and T2 is
wall except during collision equal to nC ~ (T 2 - 'Ii), where Cv is the molar heat
capacity at constant volume and 11 the number of
9. For two different gases X and InP moles of the gas
Y, having degrees of freedom
(b) The change in internal energy of the gas and the
I, and 12 and molar heat work done by the gas are equal in magnitude in an
capacities at constant adiabatic process
volume Cv and C v2 (c) the internal energy does not change in an
respectively, the In P versus In isothermal process
V graph is plotted for '------~I" V (d) no heat is added or removed in an adiabatic
adiabatic process, as shown: process.
(a) I, > 12 (b) 12 > /, 15. An ideal gas is taken from the state A(Pressure P,
(c) CV2 >CVj (d) CYj >C1'2
volume V) to the state B (pressure PI2, volume 2\1)
along a straight line path in the P~Vdiagram. Select the
10. At ordinary temperature, the molecules of an ideal gas
correct statements from the following:
have only translational and rotational kinetic energies.
(a) The work done by the gas in the process A to B
At high temperatures they may also have vibrational
exceed the work that would be done by it if the
energy. As a result of this at higher temperatures:
system were taken from A to B along an isothermal
(C~, == molar heat capacity at constant volume) (b) In the T-V diagram, the path AB becomes a part of
(a) C v = 312 R for a monoatomic gas a parabola
(b) C Y == 312 R for a diatomic gas

www.puucho.com
Study Anurag Mishra with www.puucho.com

252 THERMODYNAMICS i
ee) In the poT diagram, the path ABbecomes a part of (c) change in internal energy of the gas is ..!. kx~
a hyperbola 2
Cd) In going from A to B, the temperature T of the gas (d) temperature of the gas is decreased
first increases to a maximum value and then 20. n moles of an ideal gas p
decreases undergo an isothermal
16. An ideal gas can be expanded from an initial state to process at temperature T.
certain volume through two different processes p-v graph of the process is
(i) PV2 "" constant and (ii) P = KV2 where K is a shown in the Fig. A point
positive constant. Then: A(V"P,) is located on the
(a) final temperature in (i) will be greater than in (ii) P-V curve. Tangent at point
(b) final temperature in (ij) will be greater than in (i) A, cuts the V-axis at point D. 0
D
v
(e) total heat given to the gas in (0 case is greater AD is the line joining the
than in (ii) ,'I \'"1 point A to the origin D of PV diagram. Then :
Cd) total heat given to the gas in eii) case is greater
than in (i)
(a) coordinate of points D is ( 3~1 ,oJ
17. The total kinetic energy of translatory motion of all the (b) coordinate of points D is (2V,,0)
molecules of 5 Htres of nitrogen exening a pressure P (c) area of the triangle ADD is nRT
is 3000 J :
(d) area of the triangle ADD is ~ nRT
(a) the total KE. of 10 litres of N2 at a pressure of 2P 4
is 3000J 2). A gas may expand either adiabatically or isothermally.
(b) the toral KE. of 10 litres of He at a pressure of2P is A number of P-Vcurves are drawn for the two process
3000J over different ranges of pressure and volume. It will be
(c) the total KE. of 10 litres of O2 at a pressure of 2P found that:
is 2000J
(a) two adiabatic curves do not intersect
(d) th~ total KE. of 10 litres of Neat a pressure of 2P is
(b) two isothermal curves do not intersect
1200J
(c) an adiabatic curve and an isothennal curve may
18. Two spheres A and B have same radius but the heat intersect
capacity of A is greater than that of B. The surfaces of (d) the magnitude of the slope of an isothennal curve
both are painted black. They are heated to the same for the same value of pressure and volume is less
temperature and allowed to cool. Then: than that of adiabatic curve
(a) A cools faster than B .
22. During the melting of a slab of ice at 273 K at
(b) both A and B cool at the same rate
atmospheric pressure :
(c) at any temperature the ratio of their rates of
(a) positive work is done by the ice-water system on
cooling is a constant
the atmosphere
(d) B cools faster than A
(b) positive work is done on the ice-water system by
19. A thermally insulated the atmosphere
chamber of volume 2Vo (c) the internal energy of the ice-water increases
B
is divided by a (d) the internal energy of the ice-water system
frictionless piston of decreases
area 5 into two equal
23. A bimetallic strip is fanned out of two identical Strip
parts A and B. Pan A
one of copper and the other of brass. The coefficients
has an ideal gas at pressure Po and temperature To and
of linear expansion of the (\NO metals are ar and UB'
in part B is vacuum. A massless spring of force constant
On heating, the temperature of the strip goes up by fiT
k is connected with piston and the wall of the
and the strip bends to form an arc of radius of
container as shown. Initially spring is unstretched. Gas
curvature R. Then R is :
in chamber A is allowed to expand. Let in equilibrium
(a) proportional to l!.T
spring is compressed by xo' Then:
(b) inversely proportional to fiT
(a) final pressure of the gas is kxo (c) proportional to lOB -ucl
S
1 , (d) inversely proportional to laB -ucl
(b) work done by the gas -kxo 24. The Fig. shows a (lIp) versus T graph for an ideal gas.
2
Select the correct statement(s) :

www.puucho.com
Study Anurag Mishra with www.puucho.com

THERMOOYNAMICS 253
(lIp) (b) is infinite for an isothermal process

2
(c) depends only on the nature of the gas for a process

.,//
/1
in which either volume or pressure is constant
(d) is equal to the product of the molecular weight
and specific heat capacity for any process
28. During an experiment, an ideal gas is found to ooey a
o'
T condition p2 = constant [p = density of the gas]. The
p
(a) The graph represents an isobaric expansion
gas is initially at temperature T, pressure P and density
(b) L..,rgethe slope of straight line higher the pressure
p. The gas expands such that density changes to 1'/2 :
ee) Internal energy of the gas increases
(a) the pressure of the gas changes to .J2 P
Cd) Work done during the process is positive
(b) the temperature of the gas changes to ..Pi T
25. The Fig. shmvs PV versus T graph of ideal gas. Choose (c) the graph of the above process on the P.T diagram
the correct alternative(s) : is parabola
PV (d) the graph of the above process on the P-T diagram
is hyperbola .
29. Monoatomic, diatomic and triatomic gases whose
initial volume and pressure are same, each is
compressed till their pressure becomes tvvice the initial
pressure. Then :
(a) if the compression is isothermal, then their final
T
volumes will be same
(b) if the compression is adiabatic, then their final
(a) The slope of the straight line is independent of the
volumes will be different
monoatomic or diatomic nature of the gas (c) if the compression is adiabatic, then triatomic gas
(b) The slope of the straight line depends on the molar
will have maximum final volume
mass of the gas
(d) if the compression is adiabatic, then monoatomic
ee) The straight line denotes a process in which the
gas will have maximum final volume
gas is heated up
Cd) The slope of straight line represents the universal 30. Two gases have the same initial pressure, volume and
temperature. They expand to the same final volume,
constant
one adiabatically and the other isothermal. Then :
26. A rectangular narrow V-tube has equal arm lengths
(a) th~ final temperature is greater for the isothennal
and base length, each equal
process
to I. The venical arms are
(b) the final pressure is greater for the isothermal
filled with mercury up to l!2
process
and then one end is sealed. (c) the work done by the gas is greater for the
By heating the enclosed gas
isothennal process
all the mercury is expelled. If (d) all the above options are incorrect
atmospheric pressure is Po,
the density of mercury is p
1----------.-- 31. Some of the thermodynamic parameters are state
and cross. sectional area is S, 1/2 variables while some are process variables. Some
grouping of the parameters are given. Select the
then: • ••
(a) Work done by the gas correct:
. (a) State variables : Temperature, No. of moles;
agamst t h e atmosp henc. pressure IS. -51 PoS Process variables: Internal energy, work done by
2
the gas
(b) Work done by the gas against the gravity is ~ Spg/2 (b) State variables : Volume, Temperature; Process
variables: Internal energy, work done by the gas
(c) Work done by the gas against the atmospheric
(c) State ••.. ariables : work done by the gas, heat
pressure is Post
rejected by the gas; Process variables
(d) Work done by the gas against the gravity is ~ Spg/2 Temperature, volume
(d) State variables: Internal energy, •...olume; Process
27. The molar heat capacity for an ideal gas: variables: Work done by the gas, heat absorbed by
(a) is lero for an adiabatic process the gas

www.puucho.com
Study Anurag Mishra with www.puucho.com

THERMODYNAMICS
During an experiment, an ideal gas found to obey a
condition vp2 = constant. The gas is initially at a
temperature T, pressure P and volume V. The gas
expands to volume 4V :
(a) The pressure of gas changes to ~
2
(b) The temperature of gas changes to 4T
(c) The graph of above process on the P.T diagram is
parabola
(d) The graph of above process on the PoT diagram is
hyperbola
During an experiment, an ideal gas is found to obey a
". p2
can d ltIon - -= constant [p -= density of the gas]. The
p
gas is initially at temperature T, pressure P and density
p. The gas expands such that density charges to pl2 :
(a) The pressure of the gas changes to .J2 P
(b) The temperature of the gas changes to .j2 T
(c) The graph of above process on the P-T diagram is
parabola
(d) The graph of the above process on the PoT diagram
33. An ideal gas expands in such a way that PV2 = is hyperbola
constant throughout the process: 38. According to kinetic theory of gases,which of the
(a) The graph of the process of T-V diagram is a following statement will be true:
parabola (a) Ideal gases can not be liquified
(b) The graph of the process of T-V diagram is a (b) The molecules of ideal gas do not obey ne'Ntons
straight line laws of motion
(c) Such an expansion is possible only with heating (c) Pressure of gas is always inversely proportional to
(d) Such an expansion is possible only with cooling its volume
34. A gas expands such that its initial and final (d) Molecules of gas never move in straight line
temperature are equal. Also, the process followed by 39. According to kinetic theory of gases:
the gas traces a straight line on the P-V diagram: (a) the velocity of molecules decreases for each
(a) The temperature of the gas remains constant collision
throughout (b) the pressure exerted by a diatomic gas is
(b) The temperature of the gas first increases and then proportional to the mean velocity of the molecule
deceases (c) the K.E. of the gas decreases on expansion at
(c) The temperature of the gas first decreases and constant temperature
then increases (d) the mean translational KE. of a diatomic gas
(d) The straight line has a negative slope increases with increase in absolute temperature
35. Fig. shows the 40. fuo vessel of the same volume contain the same gas at
pressure P versus same temperature. If the pressure in the vessels be in
volume V graphs for the ratio of 1 : 2, then:
two different gas (a) the ratio of the average kinetic energy is 1 : 2
sample at a given (b) the ratio of the root mean square velocity is 1 : 1
temperature. M A and (c) the ratio of the average velocity is 1 : 2
M B are masses of two (d) the ratio of number of molecules is 1 : 2
samples, nA and nB 41. A vertical cylinder with heat-conducting walls is closed
are numbers of at the bottom and is fitted with a smooth light piston.
moles. Which of the It contains one mole of an ideal gas. The temperature
following must be of the gas is always equal to the surrounding's
incorrect. temperature, To. The piston is moved up slowly to
(a) MA >M8 (b)MA <MB increase the volume of the gas to 11 times. \Vhich of the
(c) nA >nB (d)nA<nB following is incorrect?

www.puucho.com
THERMODYNAMICS
-
Study Anurag Mishra with www.puucho.com

255
(a) Work done by the gas is RTo In '1 p
(b) Work done against the atmosphere is RTo(lj-1)
(e) Thre is no change in the internal energy of the gas
Cd)The final pressure of the gas is _1~ times its
(11 ~ 1)
initial pressure (O,O)L
V
42. A dosed vessel contains a mixture of two diatomic
(a) QI/ -Wn >Qc-Wc
gases A and B. Molar mass of A is 16 times that of B
and mass of gas A contained in the vessel is 2 times (b) QA -QD =WA -WI)
that of B. The following statements are given (c) WA <Wn <We <WD
CO Average kinetic energy per molecule of A is equal (d) QA >Qn >Qc >QD
to that of H. 47. A student records tiQ, .o.U and .o.W for a thermody
Oi) Root mC,1n square value of translational velocity of -namic cycle A -Jo B -Jo C -t A. Certain entries are
B is four times that of A. missing. Find correct entry in following options :
(iii) pressure exerted by B is eight times of that exerted
AB BC CA
by A.
(vi) Number of molecules of B in the cylinder is eight 'W 40J 30J
time that of A.
(a) (i), 00 and (iii) are true "u SOJ
(b) (ji), (iii) and (iv) are true
"Q 150J lOJ
ee) (i), (ii) and (jv) are true
(d) None are true (a) Wnc = -70J (b) .o.QO\. = 130J
43. A mixture of ideal gases 7 kg of nitrogen and 11 kg of (e) !'J.U"B =190J (d) !'J.UO\. =-160J
CO2, Then: 48. fuo moles of mono-atomic gas is expanded from
(Take y for nitrogen and CO2 as 1.4 and 1.3 (Po, Vol to (Po, 2Vo) under isobaric condition. Let .o.Q],
respectively) be the heat given to the gas, .o.W] the work done by the
(a) equivalent molecular weight of the mixture is 36 gas and tiU} the change in internal energy. Now the
(b) equivalent molecular weight of the mixture is 18 monoatomic gas is replaced by a diatomic gas. Other
(c) "(for the mixture is 5/2 conditions remaining the same. The corresponding
(d) y for the mixture is 47/35 values in this case are !'J.Q2' !'J.W2, t:l.U2 respectively,
44. A piston is slowly pushed into a metal cylinder then:
containing an ideal gas. Which of the following (a) .o.Q] -.o.Q2 = .o.U] -.o.U2
statements is/are incorrect? (b) !'J.U2 + .o.W2 > .o.U] + !'J.W]
(a) The pressure of the gas increases (c) !'J.U2 > t:l.U]
(b) The number of the molecules per unit volume (d) None of these
increases
49. For an ideal gas:
(c) The average speed of gas molecules increases
(a) the changing in internal energy in a constant
(d) The frequency of collision of the gas molecules
pressure process from temperature Tj to T2 is
with the piston increases
equal to nCv{T2 -T]) where Cv is the molar
45. In case of hydrogen and oxygen at N.T.P., which of the specific heat at constant volume and n is the
following quantities is/are the same? number of the moles of the gas
(a) Average momentum per molecule (b) the change in internal energy of the gas and the
(b) Average kinetic energy per molecule work done by the gas are equal in magnitude in an
(c) Kinetic energy per unit volume adiabatic process
(d) Kinetic energy per unit mass (c) the internal energy does not change in an
46. An ideal gas is taken from state 1 to state 2 through isothermal process
optional path A,B,C and D as shown in P-V diagram. Cd) Cal. (b) and (0)
Let Q, Wand U represent the heat supplied, work done
and internal energy of the gas respectively. Then:

www.puucho.com
Study Anurag Mishra with www.puucho.com
- THE CS

50. An enclosed ideal gas is taken 54. A 'cyclic process ABCD is shown in

t~
through a cycle as shown in the the p. V diagram. Which of the
figure. Then: following curves represents the
(a) a long AB, temperature same process if BC and DA are
decreases while along BC isothennal processes :
temperature increases A' V-

'"
Q
(b) along AB, temperature
increases while along BC the temperature
decreases.
(c) along CAwork is done by the gas and the internal
energy remains constant.
(a)
l~ (b) V
t~
A
D8
(d) along CA work is done on the gas and internal T- T-
energy of the gas increases

to
51. In the figure shown. the amount :of hihit' sup~lfed co
one mole of an ideal gas is plotted:or{thitfiorizontal
axis and the amount work performed by the gas is
drawn on the venical axis. One of the straight lines in
(e)
l~ D C
(d)

the Fig. is an isothenn and the other tWo are isobars of T~ T~


two ,gases. The initial states of both gases are same.
80 55. 2 moles of a mono-atomic gas are expanded to double
70 its initial volume, through a process P/V = constant. If
60 its initial temperature is 300 K. then which of the
W(J)! 50 following is not true.
40 (a) ~T = 900K (b) ~Q = 3200R
32 (e) ~Q = 3600R (d) W = 900R
30
20 56. An ideal gas has molar heat
10 capacity at contant pressure
C p = 5R/2. The gas is kept in a
o 20 40 Q~Jt 80 cylindrical vessel fitted with a
piston which is free to move. Mass
Select the correct statements. of the frictionless piston is 9 kg.
(a) Curve 3 corresponds to isothermal process Initial volume of the gas is
(b) Curve 1 corresponds to a polyacomic gas 0.002701 3 and cross-section area of the piston is 0.09
(c) Curve 2 corresponds to a monatomic gas m 2. The initial temperature of the gas is 300 K.
(d) Process 1 and 2 are isobaric process. Atmospheric pressure Po' = l.05 x lOs N/m 2. An
4
52. In the previous question 31, if the two gases are amount of 25 x 10 J of heat energy is supplied to the
compressed to the same final volume : gas, then:
(a) the final temperature is greater for the adiabatic (a) Initial pressure of the gas is l.06x 105 N/m Z
process (b) Final temperature of the gas is 1000 K
(b) the final pressure is greater for the adiabatic (c) Final pressure of the gas is 1.06x lOs N/m2
process
(d) Work done by gas is 9.94 x 103 J
(c) the work done on the gas is greater for the
. adiabaric process 57. On the poT graph of an ideal gas:
(d) all the above oprions are incorrect (a) adiabatic process will be a straight line
53. The first law of thermodynamics can be written as (b) isochoric process will be a straight line passing
6.U = 6.Q + 6.W for an ideal gas. Which of the
through the origin
following statements is correct? (c) adiabatic curve will have a positive slope
(a) 6.U is always zero when no heat enters or leaves (d) the slope of adiabatic curve will decrease with
the gas increase in T
(b) 6.W is the work done by the gas in this written law 58. A gas is trapped in a cylinder closed by a frictionless
(c) t::..U is zero when heat is supplied and the movable piston having some mass m.
temperature stays constant (a) pressure of the gas will be equal to that of the
(d) 6.Q = -t::..W when the temperature increases very surrounding if axis of the cylinder is horizontal.
slowly

'''''''''. www.puucho.com
Study Anurag Mishra with www.puucho.com

THERMODYNAMICS
(b) pressure of the gas cannot be less than that of the (b) Root mean square value of translational velocity of
surrounding if axis of the cylinder is not gas B is four times that of A
horizontal. (c) Pressure exerted by gas B is eight times of that
(e) pressure of the gas may be equal to that of exerted by gas A
surrounding if axis of the cylinder is not (d) Number of molecules of gas B in the cylinder is
horizontal. eight times that of gas A
Cd) pressure of the gas may be less than that of 62. A partition divides a container having insulated walls
surrounding if axis of the cylinder is not into twO compartments I and II. The same gas fills the
horizontaL two compartments whose initial parameters are given.
59. At ordinary temperatures, the molecules of a diatomic The partition is a conducting wall which can move
gas have only translational and rotational kinetic freely without friction. Which of the following
energies. At high temperatures, they may also have statements is/are correct, with reference to the final
vibrational energy. As a result of this, compared to equilibrium position?
lower temperatures, a diatomic gas at higher r;===;===;-,
temperatures will have: ,I P,y,T 1 'Trv.rl
2P

(a) lower molar heat capacity


(b) higher molar heat capacity (a) The pressure in the two compartments are equal
ee) lower isothermal compressibility (b) Volume of compartment I is 3V/S
Cd) higher isothermal compressibility (c) Volume of compartment II is 12V/S
60. The initial state of an ideal gas is represented by the (d) Final pressure in compartment I is SPj3
point a on the P - V diagram and its final state by the 63, In Fig. here shows four paths traversed by a gas on a
point e. The gas goes from the state a to the state e by p-v diagram. If AU l' aU z' ilU 3 and t:.U" are the change
three quasi stationary processes represented by (j) abe in internal energies in their respective path, then
(ij) ace (ij) ade. The heat absorbed by the gas is : choose the incorrect relation :
p

,
Volume
T,
(a) the same in all the processes. T,
(b) the same in processes (i) and (ii) T, V
(c) greater process (i) than in (iii)
(d) less in process (ii) than in (iii) (a) t:.U1 =aUz =t:.U" =ilU3
61. A closed vessel contains a mixture of two diatomic (b) t:.U1+t:.Uz =ilU3 +tl.U"
gases A and B. Molar mass of A is }6 times and that of (c) ilUI > tl.Uz > ilU3 > ilU"
B and mass of gas A, contained in the vessel is 2 times (d) aUI <tl.Uz <L'!.U3<tl.U"
that of B.
(a) Average kinetic energy per molecule of gas A is
equal to that of gas B

www.puucho.com
Study Anurag Mishra with www.puucho.com

_cs

= Level-2: More than One Alternatives is/are CorreCt


~~-----~---
ANSWERS

1. (a, c) 2. (a, b, d)" 3. (a, b, c) 4. (a, b, c) 5. (a, b, d)


6. (a, c, d) 7. (a, c) 8. (a, d) 9. (b, c) 10. (a, d)
11. (a, b, c) 12. (a, b, c, d) 13. (a, b) 14. (a, b. c, d) 15. (a, b, d)
16. (b. d) 17. (c, d) 18. (c, d) 19. (a, h, C, d) 20. Ch, c)
21. (b, c, d) 22. (h, c) 23. (b. d) 24. (a, c, d) 25. (a, h, C, d)
26. (a, b) 27. (a, b, c. d) 28. (b. d) 29. (a, b, d) 30. (a, b, c)
31. (d) 32. (0) 33. (d)
. 34. (b. d) 35. (0)
36. (a, d) 37. (b. d) 38. (,) 39. (d) 40. (b. d)
41. (d) 42. (d) 43. (a, d) 44. (b) 45. (0)
46. (a, d) 47. (b. d) 48. (d) 49. (b, c) 50. (,)
51. (a, b. c, d) 52. (a, b, c) 53. (0) 54. (a, b) 55. (b)
56. (a, c) 57. (b, c) 58. (a, d) 59. (b, c) 60. (c, d)
61. (a, b, C, d) 62. (a, b. c, d) 63. (b, c, d)

www.puucho.com
Study Anurag Mishra with www.puucho.com

s [] ria t i [] n

LeveI-2: More than One Alternatives Islare CoI'T'fd


Adiabatic: PVY = constt.:::> p( R:r = constt.

l
51. [a b, c, d]
:::::> pI-YTY =constt.(csay):::>P=c,Ty-1
(a) For isothermal process Q = W.
Curve 3 corresponds to isothermal process, and
,
Curve 1 and 2 corresponds to isobaric processes.
.. slope: dP =.!:.:L Ty-l > a (not constt.)
dt y-1
(b) For isobaric process d'P
W =p.6.V =RliT -->0

Q =Cp6T =
, + 2)
( -2- Rl>T 61,
dT'
[a, b, c, d]

(where i = degrees of freedom) ~KT avg. KE is same ~ a


2
W 2
~3RT 3RT
Q i+2 vm\.lA = M' vnnsfl = (M/16) = 4VnnsA -) b
__
2W,201.
=_=~=_:::::>1=6
For curve 1,
i + 2 Q, 80 4 nA = 2m n,
M •
= 2m
(Mj16)
= 16mjM )
(polyatomic gas)

For curve 2, _2_=Wz =32=~:::>i=3


i+2Q280S
P, =("An:n,)PO'PA =("A":",)PO .-H,d

(monatomic gas) 62. [a, b. c, d]


56. [a. c] PV = PI VI Temp. will not change as internal energy of
C, = SRj2, C" = 3Rj2 T T
.6,W=n(Cp-Ct')=l_~=~ the system will remain unchanged
.6.Q nC p 5 5 For 2nd compartment
(2P)2V P,(3V - V,)
---=
.6,W = UQ = 2 x 2S x 104 = 104 J T T
5 5 PV
Pressure is constant and equal to ~ 4PV=-(3V-VI)
V,
5 9 x 10
P=Po+mg!A=l.05xlO +--
0.09
= 1.06)( 105 N/m'l
57. [b, cJ
RT
Ideal gas PV=RT-:::>V=-
P

www.puucho.com
Study Anurag Mishra with www.puucho.com

Problem

Pevel 3 . ComRrehension Based Problems

Indicator diagram for 1 mole of a monoatomic ideal gas is =The Fig. shows an insulated cylinder of volume V
shown in Fig. it consist of two isobar and two isotherm. The containing monoatomic gas in both the compartments. The
temperature of isotherms are 300 K and 600 K. The piston is diathermic. Initially the piston is kept fixed and
minimum volume of gas is given VI and maximum volume the' system is allowed to acquire a state of thermal
is 41), heat absorbed is process (1), (2), (3) and (4) are Q"
Q2• Q3 and Q4 respectively and area of cycle is A. Now
answer the following question. --
equilibrium. The initial pressures and temperatures are as
shown in the Fig. Calculate:
,~

P....:Q,V' P,V,
4 2

.•••••••• P 1. The final temperature:


DP.V43 (a) (Pt -P2)TIT2 (b) (PI +P2)TIT2

v ~~-~~ ~~-~~
(c) (PI + P2)TIT2 (d) (PI + P2)T1T2
1. Efficiency of cycle : ~~+~~ ~~-~~
A
Ca) - (b) A 2. The final pressures of each companments :
Q, Q1 +Q2 (a) PI PI (PI +,P2)T2 'P _P_,_CP_,_+_P_,_lT_,
2
Ce) A Cd) Q, +Q2 +Q3 +Q4 PIT2 +P2TI (PIT2 +P2TI)
Q1 +Q2 +QJ +Q4
2. The value of Q1 :
Q,
,
Cb)P Pl(PI+P2)T2
PIT2 +P2TI
.,
P P2(PI-P2)TI
(P1T2 +P2TI)
Ca) 450R (b) 750R
(e) 200R Cd) zero (c) PI PI (PI +P2)T2 'P P2(P1 -P2)TI
2
3. Change in internal energy in process '3' : PIT2 -P2TI (PIT2 +P2TI)
Ca) '-450R Cb) 200R d) P PI(PI +P2)T1 P P2(PI +P )T
Ce) -200R Cd) 750R
C
, P]T2 +P2T2 , 'c PIT2 +P22TI)1

www.puucho.com
Study Anurag Mishra with www.puucho.com
261
THERMODYNAMiCS
3. The heat that flows from RHS to LHS, given '1"2> Ti• 1. The initial pressure of the gas is :
(a) IN/m2 (b) l02N/m2.
Ca) ~PIP2V (12 +T1) (b) ~PIP2V tTl -TI} (c) 1.l0N/m 2. (d) L12N/m 2.
4 PITz - PZT} 4 PITz + P2Y}
2. Wo:k done by the gas in t = 5s is :
(e) ~PIP2V {Tl-Tll (d)~P1P2V tTz-Tll 1 Ca) 300 J Cb) 400 J
3 PJTZ +PZT] 4 PJTZ +P2TJ V
(e) 114.3 J Cd) 153.6 J
3. Increase in temperature of gas in 5 s is :
Ca) 6.9 x 1O-3K Cb) 6.9 K
Two mole of a monoatomic ideal gas follows the process (c) 83 X 10-4 K (d) 96 x lO-2.K
PV4/3 "" const.
1. which of the following statement is correct about this
process? The Fig. shmvs tvo/Oidentical cylinders A and B which
(a) It is an adiabatic process contain equal amounts of an ideal gas with adiabatic
(b) It is a process in which t.Q = - ~ ~U exponent g. The piston of cylinder A is frce and that of
5
cylinder B is fixed.
ee) It is a process in which dQ = dU (ahvays) B
A
Cd) It is a process in which dQ = - dU (always)
2. If this process is possible:
(a) its molar heat capacity is negative
(b) its molar heat capacity is positive
(e) its molar heat capacity is zero , Heat , Heat
Cd) molar heat capacity can't be defined until volume
or pressure is constant 1. If the same amount of heat is absorbed by each
3. If initial temperature of system is To and it is cylinder, then in which cylinder the temperature rise is
compressed to '1 times its initiCll volume then work more?

m'"
done by system is : (a) Temperature of A will be more
(bl Information is not sufficient to reach the
Cb) 6R1ol' - J conclusion
(c) The temperature of both A and B will be same

(c) 3RTo(l- (11)-11,) Cd) 3RTo[I-(~r3) (d) Temperature of B will be more


2. If the temperature rise in cylinder A is To, then
determine the temperature risc in cylinder B.
T
(a) .....Q. (b) To
y
2 k-mol of an ideal diatomic gas is enclosed in a vertical (e) 11" Cd)Zfa
cylinder fitted with a piston and spring as shown in the Fig_ 3. An ideal gas with adiabatic exponent y is heated at
Initially, the spring is compressed by 5 em and then the constant pressure. It absorbs Q amount of heat.
electric heater starts supplying energy to the gas at constant Detennine the fractions of heat absorbed in raising the
rate of 100 J/s and due to conduction through walls of internal energy and performing the work :
cylinder and radiation, 20 J/s has been lost to Ca) y + 2 Cbl'i
surroundings. 2y
(c) y+1 Cd) "(-2
Y Y

Fig. shows variation of internal energy U with the density p


[Take k = 1000 N/m, g = 10m/s2, Atmospheric pressure,
2
of 1 mole of ideal monoatomic gas for a thermodynamic
Po = lOs N/m 2.. Cross-section area of piston Ao = 50cm • cycle ABO\. Here process AB is a part of rectangular
Mass of piston m = 1kg, R = 8.3kJ/mol-K) hyperbola.
Based on above information, answer the following
questions:

www.puucho.com
Study Anurag Mishra with www.puucho.com
262
THERMODYNAMICS
u

.:::KJ.
, ,
,,
, p 1. When the lower piston is moved upwards a distance
h!2, the compression :
1. The P-V diagram of above cycle: (a) leads to cooling
p p (b) takes place isothermally
(c) takes place adiabatically
(d) leads to heating
(a) (b) 2. Which of the following statements is correct?
(a) Heat given in compression is used to do work
v v against elastic force only
(b) Heat given in compression is used to do work
p
against elastic force and atmospheric force
(c) No work is done during compression
(e) (d) There is no change in internal energy
Cd) none of these
3. The value of h is :
. , (a) 1 m (b) 1.4 m
c....c-==;>- 'V
(c) 1.6 m (d) 2 m
2. The total amount of heat absorbed by the cycle is :
(a) C30!n2S-2)Uo (b) Cs O!nOA-2)Uo

(e) sauo Cd) None of these An horizontal insulated cylindrical vessel of length 21 is
3. The work done in process AB is : separated by a thin insulating piston into two equal pans as
(a) -Va (b) -2Uo
shown in Fig. Each part contains n moles of an ideal
monoatomic gas at temperature T. The piston is connected
ee) -SUo Cd) None of these
to end faces of the cylinder with the help of two springs
4. Efficiency of the cycle is :
each of spring constant k. Initially both the springs are in
(a) 1Oln2.5-6 (b) 5In2.5-6 relaxed state. The temperature of left part of cylinder is
10102.5+9 5102.5+9 kept constant by some external means while heat is
(e) 10102.5-3 (dlNoneofthese supplied to right pan with the help of filament. Based on
5102.5+9 above information, answer the following questions:


k k
M ideal gas at NTP is enclosed in an adiabatic vertical
cylinder having an area of cross section A 27 em 2 = o. T o. T
between mo light movable pistons as shown in the Fig.
Spring with force constant k = 3700N/m is in a relaxed
state initially. Now the lower position is moved upwards a
1. If energy (heat) Q is supplied to right part so that the
distance hl2, h being the initial length of gas column. It is
observed that the upper piston moves up by a distance piston moves to the left by i, then the change in
4
'h / 16. Final temperature of gas = 413 x 273 K. Take y for:
the gas to be 3/2. ._ internal energy of right part of cylinder is :
(a) 15 kl2 + nRT
16
(b) 3nR[5kl' +
2 4nR 3
zr]

www.puucho.com
Study Anurag Mishra with www.puucho.com

THERMODYIIAMICS
2. If 100 gm of ice is melted during the cycle ABQ\, how
ee} ~k12 +2nRT much work is done on the gas?
16
Cd) None of the above (a) S kcal (b) 5 kcal
2. The heat given by left part of cylinder to external (e) 2.1 kJ (d) 4.2 kJ
arrangement is (for Q. 1) : 3. If the change in the volume is (VI - Vl) = Vml, the
(a) Q - kl2 - nRT work done (in N/m 2) during the cycle is, with P being
(b) Q - kl2 - 2nRT the atmospheric pressure acting on the piston.
(a) PV J (b) 2PV J
ee) Q_15 kl2 -nRT 2 3
16
(c) PV J (d) None of these
Cd) None of the above
3. The temperature of the gas in right part in final
equilibrium is (for information of Q. 1) :
!P" " " " "I ~i(j1
__ Vo.i_lJ_~ i.I

2 2
(a) Skl + ZT (b) 15k/ + 51' An ideal diatomic gas undergoes a process in which its
BnR 3 16nR 3 internal energy relates to its volume as, U = kJV, where k is
2 ,
(e) 15kl + zr Cd) Ski + 51' a positive constant.
16nR 3 8nR 3 Based on given information, answer the following
questions:
1. The molar heat capacity of gas for this process is :
(a) 2.5R (b) 2R
A cylinder containing an ideal gas (see diagram) and closed
by a movable piston is submerged in an ice-water mixture. (e) 4,5R (d) 3,SR
The piston is quickly pushed down from position (1) tD 2. The amount of heat transferred between system and
position (2) (process AB). The piston is held at position (2) surrounding so that the internal energy of system
until the gas is again at 0" C (process Be). Then the piston is increases by 300 J is :
slowly raised back to position (1) (process CA) : (a) 300 J (b) 240 J
(e) 420 J (d) 540 J
3. If 2 moles of gas is taken in mentioned process, then
o 000 the change in temperature of gas for Q. 12 is :
00
~o o 0 [R o8.314J/mol-k]
:: 2 0 0 (a) 7.2 K (b) 10K
o -----~:..----- 0 (d) 100 K
------------ 0 (c) 72K
o O'oooooe.o 0
o 0 0 o
o 00 0 0 0
o A container of volume 4Vo made of a perfectly
non-conducting material is divided into two equal parts by
1. Which of the following P-V diagrams will correctly a fixed rigid wall whose lower half is non-conducting and
represent the processes AB, BC and CA and the cycle upper half is purely conducting. The right side of the wall is
ABCA ? divided into equal parts (initially) by means of a massless
, non-conducting piston free to move as shown. Section A
ANC
A' : '
contains 2 mol of a gas while the section Band C contain 1
B~
f--2>.lc P
mol each of the same gas (y = 1.5) at pressure Po. The
,, (b) 'B
,, heater in left part is switched on till the final pressure in
(a) ,, ,, section C becomes.125/27 Po, Calculate:
, ,
v, V, V V, V, V

Nc
p p
A s' :

C~A
' ,
(e) : :8 (d)
,, '
'
,, ',
,
v, v, V v, v, v

www.puucho.com
Study Anurag Mishra with www.puucho.com

~'j I'

A B V, •.P." Vol_tJ
," \ "',.'1 .•..
{JI ~
~
An ideal gas undergoes a change in its state from 1 to 2
P, through three difference processes, A, Band C. as shown in
~

I
P, , Fig.
P

P,

1. Final temperature in part A : v


(a) 12PoVo (b) PaVo
13R R 1. If the work done during the three processes are WA,
ee) IOSPoVo d) 20SPoVo WB and We, respectively, then:
13R (27R (a) WA =WB =We (b)WA >WB >Wc
2. Final temperature in pan C : (e) WA =WC>WB (d)We>WA >WB
(a) PoVo (b) PaVo 2. If the heat supplied to the gas are QA' QB and Qe
R 3R respectively, then :
(e) SPaVo Cd) SPaVo (a) QA =QB =Qc (b) QA < Q8 < Qe
3R R (e) QA > QB > Qe (d)QA =Qn >Qc
3. The heat supplied by the heater:
Ca) 405 p ~ (b) 113 p ~ tfP" ,~,~" 1'1 ~ r.TI@
__ vv_;::;v~
800 500
315 368 5 mole of helium are mixed with 2 mole of hydrogen to
(e) -PaVo Cd) -PaVo
9 9 form a mixture. Take molar mass of helium M I 4g and=
that of hydrogen M:2 = 2g
"f) v~
__ vv_,.
4{> ti'V ~,~ \~
1. The equivalent molar mass of the mixture is:
An ideal diatomic gas is expanded so that the amount of. (a) 6 g Cb) 13g
heat transferred to the gas is equal to the decrease in its 7
internal energy. (e) lBg (d) none
7
1. The molar specific heat of the gas in this process is 2. The equivalent degree of freedom f of the mixture is:
given by C whose value is : (a) 3.57 Cb)1.14
(a) -5R (b) -3R (c) 4.4 (d) none
2 2 3. The equivalent value of y is:
Ce) 2R Cd) 5R (a) 1.59 Cb)1.53
2 (c) 1.56 (d) none
2. The process can be represented by the equation 711" -= 4. If the internal energy of He sample of 100J and that of
constant where the value of n is : the hydrogen sample is 200 J, then the internal energy
Wn=! (b)n=! of the mixture is:
5 5 (a) 900 J (b) 128.5 J
(e) n =~ (d) n =~ (e) 171.4 J (d) 300 J
2 5
3. If in the above process, the initial temperature of the
gas be To and the final volume be 32 times the initial
volume, the work done by the gas during the process 'A monoatomic ideal gas of two moles is taken through a
will be, in joules : cyclic process starring from A as shown in Fig. The volume
(a) RTo (b) 5RT, ratios are VB = 2 and VD = 4. If the temperature TA at A is
2 VA VA
27:.J:" then _
(e) 2RTo (d) RT,
2

, www.puucho.com
Study Anurag Mishra with www.puucho.com

THERMODYNAMICS 265

--tJ:!..
.•.
"" vv
\~ " f'/ ./'f ., ~
•.•.O-~

A massless piston, which can move without friction, doses


a sample of Helium in a vertical, thermally insulated
cylinder, which is dosed ac its baHam, and the cross-section
of which is A = 2 dm 2. Above the piston there is a fixed
stand to which an unstretched spring of spring constant
2000N/rn is attached, whose bottom end is at a distance of
2 drn from the piston when the volume of the gas is
Vo = 8dm 2. The external pressure is
1. The temperature of the gas at point B is :
(a) 1200 K Cb) 600 K Po = lOs Pa, g = 10m/ S2. The gas confined in the cylinder is
Ce) 400 K Cd) 3001n2K heated with an electric heating element. (1 dm = 10-1 m)
,
2. Hear absorbed by the gas in the process A to B is :
Ca) 600 R
ee) 1500 R

cycle is :
Cb) 900 R
Cd) 1200Rln2
3. The total work done by the gas during the complete
4JF:t VO
d

Ca) 600 R Cb)ISOOR


Ce) 1200 R (d) 900Rln2

The dot in Fig. represents the initial state of a gas. An 1. How much heat is supplied by the elemcnl till it
adiabat divides the P.V diagram into regions 1 and 2 as reaches the spring:
(a) zero (b) 600 J
shown.
p (e) 8000 J Cd) 1000 J
2. How will process look like on a P-V diagmm :

B
(a) P)-'~B , v (b) P -.-/_A_ v

v
L
1. For which
corresponding
positive:
of the following processes,
heat supplied to the system Q is

(a) the gas moves up along the adiabat


(b) it moves down along the adiabat
the

(e) Pl-'~B .v
Cd)

ee} it moves to anywhere in region 1


3. What is the ratio of work done by gas in moving the
Cd) it moves to anywhere in region 2
piston from initial position to A, and from there to B:
2. As the gas moves down along the adiabatic, the
Cal 10 (b) ~
temperature: 11 10
(a) increases
(e)~ (d)~
(b) decreases 3 5
ee} remains constant
Cd) variation depends on type of gas

www.puucho.com
Study Anurag Mishra with www.puucho.com
266 THEIlJIIOD'/NAMI •

4. Choose the correct P-T diagram for the given cyclic


t!"_ •.••.•
•. ,","":.':{:,
-t,; - ~ ~ process:
The Fig. shows p~vdiagram of a thermodynamic cycle.

2P.
P

..~ c
(a) PIl?".
a T
(b) PI;lt'"
JC_ T

p.
"A: o
0
v. 3Vo V
Cd) none of these
(e) P ~c

1. The work done by the cycle is : a T


(a) 2PoVo (b) 3PoVo 5. Choose the correct V.T diagram for the given cyclic
(e) PoVo Cd) 6PoVo process:
2. IfTA,TB,Tc and To are the respective temperature at
A, B,C and D. Then, choose the correct statement if
TA = To : (a) VI,~lt (b)
(a) The maximum
occurs at C.
(b) To = 3To
temperature during the cycle
J=_r
ee) Ts = 2To
(d) aU the above
3. Identify the diagram which correctly represents the
(e) v A\\ Cd) none of these

'
heat inflow and outflow of the system: ~c
P aL~=---.T

t:] a, c
(a) a, ~"
.....," ," -tl
_•.••.• ",:~/;' ~
•...
~
A 0
a. Two cylinder A and B having piston connected by massless
0'--,,-----,;'-;;;,-. v rod (as shown in Fig.). The cross-sectional area of two
cylinders are same and equal to •S' . The cylinder A contains
P
m gm of an ideal gas at pressure Po and temperature To. The
cylinder B contain identical gas at same temperature To but

(b)

0
t:l
a,

A 04 D
a,
V
has different mass. The piston is held at the state in the
position so that volume of gas in cylinder A and cylinder B
are same and is equal to Vo'

'H:'::.1.....i:;~ i(
::::
P

AOa,
:.'.
A B
B
a,
(e) ifhe walls and piston of cylinder A are thermally insulated,
a, whereas cylinder B is maintained at temperature To
o 04 C reservoir. The whole system is in vacuum. Now the piston is
V slowly released and it moves towards left and mechanical
equilibrium is reached at the state when the volume of gas
P
in cylinder A becomes Vo . Then (here y for gas:::: 1.5).
2

Cd)

0
W
a,

o 04 C
a,

V
1. The mass of gas in cylinder B :
(a)
(c)
2./2 m
J2 m
(b) 3./2 m
(d) none of these

, www.puucho.com
Study Anurag Mishra with www.puucho.com

THERMODYNAMICS -----_. 267


2. The change in internal energy of gas in cylinder A :
(a)
(e)
(12 -l)PoVo
PoVo
(,/2-1)
(b) 2(../2 -l)PoVo
Cd) none of these

3. If work done by gas in cylinder B is WB and work done


(a) 1>7".
a T
by gas in cylinder A is WA' then :
(a) WA =-WE
(b) IW,i>iW,1 (e) VIp"
(e) IW,I< IWBI
Cd) we can't say anything
4. What will be the compressive force in connecting rod
at equilibrium:
aLT
(a) PS (b) ,/ZPS
32
ee) 2 pS Cd) none of these

A sample of ideal gas is taken through the cyclic process


shown in the Fig. The temperature of the gas at state A is
The Fig. represents a cyclic process in which the process CA T A = 200 K. At states Band C the temperature of the gas is
is adiabatic the same.
P P

aL-----_'v V
3V,
1. Select the correct statement(s) :
(a) Positive work is done during the isobaric process 1. Net work done by the gas in the process is :
(b) Negative work is done during the adiabatic (a) 2PAVA (b)4PAVA
process (c) 6PAVA (d) 8PAVA
ee) The system cools dovl'D. during the process Be 2. Which of the following graphs best represent the cyclic
Cd) The system reject heat during the process CA process in T-V diagram:
2. If TA,TB and Tc are the temperatures at A,B and C
respectively, then identify the correct statement : I
T T
I
(a) TA <T8 <Tc (b)TA =,Te
(c) TA >Tc (d)TA <TB >Tc (a) (b)
3. Select the diagram showing the correct representation
of the thermodynamic cycle on at P-T diagram:

(a)

~~T
(b) lv".
a T (e)
I
T

(d)
I
T

(e) PI.y" (d)P~A/ "

aLT a'
..
T
3. What is the greatest temperature of the gas during the
cyclic process ?
(a) 600 K
(c) 2400 K
(b) 1200 K
(d) none of these
4. Select the diagram showing the correct representation
of the thermodynamic cycle on at V.T diagram:

www.puucho.com
Study Anurag Mishra with www.puucho.com

268 THERMODYNAMICS
Column I Column II
MATCHING TYPE PROBLEMS
(p) Both Wand.6U are
1. An ideal gas (molar specific heat C v = SR/2) is taken positive
along pams acb, adb, and abo P2 = 2PI, V2 = 2V}. Along
ab, P = kV where k is a constant. The various
(a) V ...
0:,....
A B
parameters are shown in the Fig. Match Column I with L ~p
the corresponding options of Column II.
P
(q) Both W andNl are
P negative
b
P,
T, [b)
P, d
L T
T,

(r) W is positive whereas.6U


T
v, v, V
A is negative

[e)
Column I
"- "- Column II I ~B
"-
(a)
W,'" (p) 15RT1/2 I
v
[b) w••• (q) -lSRTll2
(s) W is negative whereas
[e) V •• [.) RT, aU is positive
[d) Voc- (5) 2RT1
(d) V

2. In Column I some actions performed on a body are


mentioned, followed by an incomplete statement, P
while in Column II phases are mentioned which can
complete the statements of Column I. Match the
entries of Column I with the entries of Column II. 4. In Column I some statements or expressions related to
first law of thermodynamics are given and in Column
Column I Column II II some processes are mentioned. Match the entries of
(a) When a person rubs his hands, (p) Increases Column I, with the entries of Column II.
they become warm. The internal
Column I "-
energy of the hands
[b) A cylinder containing gas is \fig- [q) Increases because "-
[aJ dU = nCvdT is valid for " Column II
(p) Adiabatic ptocess
orously rubbed from outside, as heat is supplied to (b) Temperature of the system can (q) r-Isothermal process
a result, the cylinder and gas the system change in
both get heated up. The internal
energy of the cylinder. [e) Q '" dU + W is valid for [.J Polytropic process
[el A cylinder containing gas is vig- [.) Increases because [d) The process in which heat ex- (s) Free expansion
orously rubbed from outside, as mechanical work is change between the system and
a result the cylinder and gas done on the system surroundings is zero
both gets heated up. The inter-
nal energy of the gas 5. An ideal monoatomic gas undergoes different types of
processes which are described in Column I. Match the
[d) A closed bottle containing water [s) Remains constant corresponding effects in Column 11. The letters have
is shaken vigorously, as a result
their usual meanings.
the water gets warm. The inter-
nal energy of water "-
"- Column I
"- Column II I
3. A sample of gas goes from state A to state B to in four [a) P=2V2 (p) If v~lume increases then tempera-
different manners, as shown by the graphs. Let W be ture will also increase
the work done by the gas and ilU be change in internal (b) pv2 '" constant (q) If volume increases then tempera-
energy along the path AB. Correctly match the graphs ture decreases
with the statements provided.

www.puucho.com
Study Anurag Mishra with www.puucho.com

THERMODYNAMICS

-(e) C=Cv+2R (,j Fur expansion, heat will have to


- Column I
-~-
Column 11
269

be supplied to the gas (aj Heat given to system (i.e. (p) Positive
(,j If temperature increases then ideal gas sample) during iso-
-(d) C=CI'-2R
work done by gas is positive thermal expansion
(b) Work done by gas during adi. (q)
6. X and r are two equal size containers. X contains 5 abatk compression
mol of 1I2 and Y has 10 mol of 02 .It the same
temperature. Assuming that the gases are ideal, match
(c) Change in internal energy of (r)
Columns J and II :
gLlssample during adiabatic
Column I Column II process

(a) 1n the container X (p) Pressure of the gas is more. (d) Change in internal energy of (s) Negative
gas sample from most initial
state to the final state
(b) in the container Y (q) nns speed of gas molecules is
more 10. h kmol of a monochromarir ideal gas is taken
quasistatically from state A to state C along the
(e) Since the number of (r) Average thermal is more energy straight line shown in the Fig. Alternalivcly, the same
molecules of a molecule is the same as that
gas is taken quasistatically from A to C along the path
of a molecule in the other
container ARC, Express all answers in terms of PA and VA'
p
Cd) Since the gas has a (5) Internal energy of the system is
smaller (5) molecu- more than that of the system in P =2P .•. 6 c
the other container. c
lar mass

7. Match the column I with Column II


\ Column I
"- "- Column II A

(aj lsochoric process (p) DU '" 0 v, v


(b) Isobaric process (q) 8Q ",0
Column I Column II
(oj Isothermal process (,) 8W",Q

(d) Adiabmic process (,) DW ~ 0

8. Find the charge in the internal energy of the system in (b) Chang~ in inrernal energy of the (q) 13 P \l
each of situation in talumn 1. gas along the straight line path AC 2 A A

Column I Column II (e) The work done by the gas along (,)
the path ABC
(aj
(d) The heatM/' added to the gas
along the path ABC

11. An ideal gas at pressure P and volumeV is expanded to


volume 2V, Column I represents the thermodynamic
processes used during expansion. Column II
(e) Twel\'e hundred calories is rl'- (r) 1680J represents the work during these processes in the
moved from a gas held at con- random order.
stant volume
" Column I "''' Column II
9. There is an ideal gas sample. The ratio of C p and C v
(a) isobaric (p) pv(1-i-Y)
for gas sample is y. In its initial state its pressure is PI
1-1
and volume is VI Now it is expanded isothennally
from volume V1 to V2. Then it is compressed (bj isothermal (q) PV
adiabatically from volume V2 to VI again.
(0) adi:lbatic (,j PVjln 2
Hegarding the <lbove situation, match the following
(dj The heat MI' added to the gas (,) 9
~PAVA
along the path ABC 2

www.puucho.com
Study Anurag Mishra with www.puucho.com

270 IHERMODYNAM
12. When a sample of a gas is taken from state i to state f 14. An ideal gas undergoes two processes A and B. One 0
along path iaf. heat supplied to the gas is SO cal and these is isothermal and the other is adiabatic.
work done by gas is 20 cal. If it is taken by path ibf,
then heat supplied is 36 cal. Column I Column I[
(aj (p) Heat supplied during curve

~'0f A is positive

'v
p~

~
; b

Volume (b) (q) Work done by gas in both


processes positive
Column-I Column-II
p~
(a) Work done by the gas along path ibf is (p) 6 cal : B
(b) If work done upon the gas is 13 cal for (q) 18 cal . V
the return path ft, then heat rejected
by the gas along path if is (e) (t) Internal energy increases

l~v
in adiabatic process
(el If internal energy of the gas at state i is (r) 40 cal
10 cal, then internal energy at state f
;,
(d) If internal energy at state b is 22 cal (5) 43 cal
and at i is 10 cal then heat supplied to
the gas along path ib is (d) (,) Temperature of gas in pro-

l~v
cess 8 is consamt
13. Respective graphs for adiabatic process taking first
tenn on Y-axis.

15. For an ideal gas a process P-V diagram is a circle. An


adiabatic from A passes through C. An isotherm from
A passes through B. We take a part of the circular cyclic
process. Comment on the sign of the quantity of
Column I Column II column-I.
(a) Pressure-Temperature (p) p
\.
(3)...•...•• B
C' :
'•
(b) Volume-Temperature (qj
O
....
"
V

Column-I '\ \ Column-II I


(aj Heat given to the gas in going from A (p) Positive
(e) Pressure-Voloume
to C along circle
(bj Heat given to the gas in going from B (q) Negative
to C along circle

(d) Pressure-Internal Energy (,j (e) Heat given to the gas in going from C (t) Zero
to D along circle
(,j can't be said

www.puucho.com
Study Anurag Mishra with www.puucho.com

THERMOOYNAMICS
16. An ideal gas is taken along the reversible processes as 2. Statement~l: In an adiabatic process, change in
represented by the adjoining diagram. internal energy of a gas is equal to work done on or by
P{Nlm<) the gas in the process.
Statement.2 : Temperature of gas remains constant
c in a adiabatic process.
15
3. Statement-I: Air quickly leaking out of a balloon
becomes cooler.
Statement-2: The leaking air undergoes adiabatic
expa:lsion.
5- -----_ .._ .._----~ 4. Statement-I: In cyclic process, initial and final state
:A
, are same. Therefore net work done is zero.
,
V(m~) .
2 6 Statement-2: Initial and final temperature is equal,
therefore change in internal energy is zero.
"- Column-I "- "- Column-II
5. Statement-I: Systems are in thermal equilibrium,
(a) For processB ---+ C (pJ L\Q .> 0 when their temperature are same or average kinetic
(b) For processA ---+ B (qJ L\W > 0
energy per molecule is same.
Statement-2: Total kinetic energy or internal energy
(e) For cycleA ---+ B --Jo C ---+ A (,J L\U .> 0
or tetal energy does not determine the direcdon of
(dJ ForprocessC -t A (,) L\W = 0 flow of heat.
(0 L\Q <0
6. Statement-I: When a boule of cold carbonated
drink is opened a slight fog forms around the opening.
Statement-2: Adiabatic expansion of the gas causes
lowering of temperature and condensation of ,vater
vapours.
7. Statement-I: A quasi-static process is so called
because it is a sudden and large change of the system.
ASSERTION AND REASON Statement-2: An adiabatic process is no quase-statk
becCluseit is a sudden and large change of the system.
Direction: In the questions that follow rvvostatements 8. Statement-I: The work done on an ideal gas in
are given. Statement.2 is purported to be the explanation for changing its volume from V1 to V2 under any
statement 1. Study both the statements carefully and then polytrophic process is given by the integral "f P. dV
select your answers, according to the codes given below:
(a) If Statement-l is true, Statement-2 is true;
taken along the process.
"
Statement-2 is the correct explanation for
Statement-2: No work is done under an isochoric
Statement. I.
(b) If Statement-l is true, Statement.2 is true, process of the gas.
Statement-2 is not a correct explanation for 9. Statement~l: When an ideal gas is taken from a
Statement-I. given thermodynamic state A to another given
(c) If Statement.l is true; Statement-2 is false. thermodynamic state B by any polytrophic process, the
(d) If Statement-l is false; Statement-2 is true. change in the interval energy of the system will be the
1. Statement-l : An ideal gas is enclosed within a same in all processes.
container fitted with a piston. When volume of this Statement-2: Internal energy of the gas depends
enclosed gas is increased at constant temperamre, the only upon its absolute temperature.
pressure exerted by the gas on the piston decreases. 10. Statement-I: Experimental results indicate that the
Statement.2 : In the above situation the rate of molar specific heat of hydrogen gas at constant
molecules striking the piston decreases. If the rate at volume below 50 K is equal to 5/2 R, where R is the
which molecules of a gas having same average speed universal gas constant.
striking a given area of the waU decreases, the Statement-2: A diatomic hydrogen molecule
pressure exerted by gas on the wall decreases. possesses three translational and two rotational
degrees of freedom at all temperatures.

www.puucho.com
Study Anurag Mishra with www.puucho.com

272 1M RMOllYJIAMICS

AN9WEIttl

= level-3: Comprehension Based Problems

Passage.1: 1. (a) 2. Cb) 3. (e) Passage-2: 1. (a) 2. Cd) 3. Cb)

Passage.3: 1. (e) 2. (a) 3. (a) Passage-4: 1. Cd) 2. (e) 3. (a)

Passage-5: 1. (d) 2. (a) 3. (a) Passage-6: 1. (a) 2. (a) 3. Cb) 4. (a)

Passage-7: 1. (e ,d) 2. Cb) 3. (e) Passage-8: I. (a) 2. (a) 3. Cd)


4. (a)

Passage.9: 1. Cd) 2. (a) 3. Cd) Passage-10: I. (e) 2. Cd) 3. (a)

Passage-11 : 1. Cd) 2. (e) 3. (d) Passage-12: 1. (a) 2. (h) 3. (b)

Passage.13: 1. (h) 2. (e) Passage-14: 1. Cb) 2. (a) 3. (e)

Passage-15: 1. Cb) 2. (e) 3. (a) Passage-16: 1. Cd) 2. Cb)

Passage-17: 1. (d) 2. (b) 3. (a) Passage-18: 1. (a) 2. Cd) 3. (h) 4. (a)

Passage-19: 1. Cb) 2. Cb) 3. (e) Passage-20: 1. (a. h, e) 2. (e) 3. (e) 4. (b)


4. (e)

Passage-21 ; 1. Cd) 2. Cd) 3. (e)

=: Matching Type Problem

I. (a)- ,; (b)- r; Cc)- p; (d) -q 2. (a)- p. r; (b)- P. r; (c)- P. q; (d)- r

3. (a) -" Cb) -q; (c) or; (d) .q 4. (a)- p, q, r, s; (b)- p, r; (e)- P. q, r. s; (d)- p. s
5. (a) -p, r, s; (b) .q; (e) -p, r, s; (d) -q, r 6. (a)- q. r; (b). p, r. ,; (c). P. ,; (d)- q
7. (a) -r; (b) -s; (e) -p, s; (d) -q. s 8. (a)- q; (b). r; (c). p
9. (a) -p; (b) or.,; (c) -p, q; (d) -poq 10. (a)- r; (b)- ,; (c)- p; (d)-q

11. Ca) -q; (b) -" (c) -p; (d) -r 12. (a)- p; (b)- ,; (c)- r; (d)- q

13. (a) or; (b) -p; (c) -p; (d) .r 14. (a)- p, q ; (b)- r, s; (e). P. q; (d)- r, s
15. (a) -p; (b) .q; Cc) -q 16. (a)-p. q. r; (b).p, r. ,; (c)-p, q; (d)-,

== Assertion and Reason

I. (a) 2. (c) 3. Ca) 4. Cd) 5. Cb) 6. Ca) 7. Cb) 8. (c)

9. Ca) 10. (d)

www.puucho.com
Study Anurag Mishra with www.puucho.com

IHEAMODYNAMICS

Solution

= Level-3: Comprehension Based Problems

Passage-4
say Q' is transferred from gas to external
arrangement (which is used to keep its
temperature constant).
As temperature of left pan is constant so change
1. [d] At t = 0,
let p be the in internal energy of left part is zero.
pressure of gas, then free
Body diagram of piston
would be as follows : c¥l====O From first law of thermodynamics
Q _ Q' = Total work done by the gas against
surrounding in both the chambers (W) + change
poA+kxo+rng=pA
in internal energy of right chamber (6U).
kxo mg
P=Po+-+- W = work done against the spring force only
A A 2
= k{l/4)2 x 2= kl
P = 1.12 N/m2 2 16
2. [c] From first law of thermodynamics, t.U= nCvaT= nC,,(T,-T)
dQ=dU+dW Where Tf is the final equilibrium temperature of
dW = dQ-dU fight part.
__ 31/4__ 5//4_
= n(Cp -C,,)dT = nRdT
W = nRxaT = 114.3J
k k
3. [a] Net heat supplied in time tis,
J
Q = (100- 20)dr = BOr. As heat has been P" T

supplied to the gas, it expands under constant


pressure, thus increasing the internal energy.
Using ideal gas equation
dQ = nCpdT 31
PIxA x-
Q = nCpt:.T ____ 4_= PoAI
RT RT
t:.T= BOxS
2x 103 x?. x 8.3 51
P2Ax- PoAI
2 4 =--
= 6.9x 1O-3K RT
Passage-B From eqm'j'b '
I num 0 f' pIston, P2 - PI = -KI
2A
1. [aj, 2. [aj, 3. [d]
As heat Q is supplied to right part, the piston T = ST[~ + ~] where Po = nRT
f 4 2nRT 3 Al
moves towards left thus compressing the left
spring and elongating right spring. To keep the
temperature of left parts constant some energy

www.puucho.com
Study Anurag Mishra with www.puucho.com
274 THIIIMOD\'NAMICS
2
Sk1 5T Passage-12
=--+-
BnR 3
1. [a] It is given that there is decreases of the internal
3R energy while the gas expands absorbing some
t.U=nx-xCTj -T)
2 heat, which numerically equal to the decrease in
internal energy.
= 3nR[Skl' + zr]
28nR3 Hence dQ = -dU
Thus, dQ = dW +dU= -dU
So. Q'=Q-W -flU
2 dW =-2dU
=Q_kI2 _15k1 -nRT
16 16 (remembering that both dW and du are
negative)
=Q_kl2 -nRT.
'.. PdV = 3CdT [dU = -edT]
Passage-10
Where C is the molar specific heat of the gas
1. [el. 2.[dl. 3.[al
Since the gas is ideal, PV = RT
Using U ::::nC II T where n is numb~r of moles of
gas and T is temperature of gas \ . From (2) P =-
RT
V
nCl,T = kV1!2
.
Hence (1) gives RT
-dV = XdT ~ _dV = __
2CdT
1V-I12 = _k_ = constant V V R T
nC,
Using PV=nRTT=_
,
PV
nR
Solving, we get \IT
(-'C) R = constant

PV
nR
X v-1I2 = constant Also since, dQ = -dU:C = (dQ)
dT v
COns!
pv1/2 = constant

S0,the process U = kJV is equivalent to pv1I2 =


constant which is a standard form of polytropic
process. Hence R
C=-Cy =---
R 1-1
C=Cv+--
I-x R SR
SR R
=-7-~-2'since the gas is diatomic.
=-+---=4.5R --1
2 1-1/2 2
-R
du = nCvdT = 300J
2. [bl From equation (3), TV2C = constant
SR
= n x-dT= 300J -')
2
Putting C = -- SR2 •IV --( s = constant
9
.1Q = nCdT = n x -RdT
2 3. tal Under the given process, the final volume V is 32
times the initial volume Yo'
= 2. x nRdT = 2. x ~ x 300J
2 2 5 nyo PdV 32Vo RT
= 540)
Thus, w = f -2
"
= f - dV
\( 2V
(see note below)
'0 0

dU= 2x SR xdT= 300 J 32Vo RT


2 f -TdV
dT = 300 = 300
V
o 2V
SR 5xR314 T v:l!S
S
Now, 1VI/ = T. v:l/S given T = -.2.....2....-
= 7.2 K o 0 VlIS

www.puucho.com
Study Anurag Mishra with www.puucho.com

THIRMODYNAMICS
Tf
32Vo R T v.lIS J J dT -= -2C[TI
Hence,W= f -~dV
2V V,IS
Integrating, W -= dW -= -X
o
- To]
v,
Putting 1'/ -= To C = _ SR
2' 2
SR
W -= --To
2
Passage.16
1. [d] If the gas goes into the region 2 the work done
by the gas increases as the area under curve
increases so mole heat is needed to the supplied.
W = _ 5Rl'o 2. [b] The temperature of the gas decreases as the gas
= 2 does work at the expense of internal energy.
SRT Passage.17
Work done by the gas:::; __ 0
2 1. [d] Since from begining to A process is isobaric hence
Note: In a normal polytropic process, the work 6.Q = nC p6.T
done dW changes the temperature by dT and
volume by av. Here heat dQ entries or levels the 5
6.Q = -nR x ----
[PoV, - PoV,]
system, dQ = dW + dU, and molar specific heat 2 nR
(= C v) is positive. Consider the adiabatic
Vi = 8x 10-3 Vg = (8+ 2x 2)x 10-3
process, in which dQ = 0. The work done dW
goes entirely to change temperature by dT and "'Q: 1000J
volume by dV. Here dW = -dU, and molar
specific heat = 0. 2. [b] Initially Po -= canst. and then P increases linearly
with V.
Consider a polytropic process in which the molar
kx
specific heat, C is negative. The relation 3. [al Pa =Po+-
dQ = dW + dU still holds. But the heat dQ A
entering the system, cools the system, lowers the 1
105 + 2000 x 2 X10- '" 1.2 x lOS
temperature T by dT and volume by dV directly. 2x 10-2
In addition, the work done dW also changes
temperature T and volume V. Thus, in such a upto A WI = lOs x 4x 10-3 = 400J
process, a given change of temperarure dT 3
involves a change of volume dV1 due to the work h At
= 2.Zx lOS x 4x 10- =: 440J
fromA~B 2
done dW and a funher change of volume dV2 2
due to heat dQ (as an effect of the negative W1
-:--:
400 10
specific heat), the total change of volume being W2 440 11
dV -= dV1 + dV2• The work done dW contributes
to only the part dV1 of this change. Passage-19
m ...(1)
In the polytropic process in the given problem PoVo = -RT in A cylinder
M
dQ -= -dU and C -= -e~,and I dV11=I dV21. Thus in
a given change dV of the system, the work done PBVO = rna RT in B cylinder ...(2)
dW contributes to only dV12 of that change, and M
hence dW -= P. dV [instead of p. dV as in a Adiabatic
2

Aliter:
normal process]
Let Tf be final temperature.
PoVJ=PI(~r
T_oo_oO
T. ViiS T VIIS To Pf = zJ2po = Ps
Then, :-
I - vj!s - [32V
o]I/S 2 Divide eqns. (1)/(Z),
PoVo rn
Also, dW : -2dU : -2CdT --:-
PaVs rna

www.puucho.com
Study Anurag Mishra with www.puucho.com

(d) i\H = 0, W = -ve ~ .&V=+ve


PoVo m
m,
2f2Po(~Vo) 2, [c, d]
Since ITom A --+ B
rna '= 3J2m
Since.&Q "" 0 & W = -ve & C --+ B, PV is increasing
Passage-21
.&U=--W=-
pI VI -p:v.) =_~2
2Po-x2."r2-PoVo
Vo r;;
_
( y-1
I I

(3'2)-1
Sol. [el Area of triangle =.! (Base) x
2
Height

= 2(./2 - I)Po Vo
'. , ,p,'p ",[:>,r,
Frompressive = PI Area = 2F2PS , ,..
3P ••••••••••••••• 9PA
W = (P;Vi -PI VI) = PoVo - 2FzpoCYo/2) TA :
A
y-l (3/2)~~,
PA "'..
" . •

= 2PoVo(l-./2)
= PoVo(2(l-1.414) = Po Vo(-O.B2B) = -O.B2BPoVo
I= Q.B2BPoVo
IW..•..
::;SP..•..
V..•..
T =C = nRTo 1n(V/) = P/V/ In(3Vo) For BC P.V eq. is P = crV + b
~ 2Vo

= 2f2Poe~0 H~)
= 3F2PoVo In(%)
= [min (1.5)PoVol = (4.242 x OAO)PoVo
WB = 1.719PoVo 9PA = crV•.•..+b

IWal= 1.719PoVo 3PA = JaV ..•..+ b


6P..... p..•..
IWAI< IW,I 0=--=-3-
2V..... V..•..
Alternative Fig.
.....
p~---_'"" b= 9P..•..-crV ..•..
=9PA+3P ..•..
b =- 12P..•..
3P
p=--AV+IZP ..•..
VA
PV = - 3PA V2 + lZPA V
o "--;;->;;;-:r-:=:- VA '
_ VrJ2 Vo 3V"'''''
••••••••
nRT = -3PA/VA V2 + 12PA V ... (1)

Passage.20 nRT dT = _ 3P (2V) + 12P


A
dV VA
1. [a, b, e]
(a) Since volume is increasing dTjdV=O ,
(b) Since volume is decreasing
(c) Since PV is deceasing

www.puucho.com
Study Anurag Mishra with www.puucho.com

!HERM DYNAMICS
By equation (1) 14. (a) Isothennal process, 6,V = 0, W = +ve ~ 8Q = +ve
Since volume is increasing in both processes
3PA 2
nRT = --(2VA) + 12PA(2VA) (b) A is adiabatic with -ve work done => tlU = +ve
VA
B is isothennal
= -l2PA VA + 24PA VA (c) Same as a
nRT=l2PAVA (d) Same as b
nRT = 12nRTA .' > 15. (a) A to C along circle net work done is +ve and tlV is
-ve
T = 12TA
But (tlW - tlV) > 0 SO, 6Q = tlW - tlU
=12x200 tlQ>O
= 2400K (b) B to C along circle there is compression so work
done is(-ve) and tlU is (-ve) So tlQ is -ve
(c) Same as B to C along circle
Matching Type Problem

Assertion and Reason


8. (a) dQ = dU + dW
Here dQ = + SOOx 4184 J = 2092
1000 7. It is true that for real gases, and for the real procedures
dW = +400J, since the work is done by the system in the laboratory, an adiabatic process is a sudden and
large change of the system obtained in a small time.
Hence, dU = dQ - dW However, for an ideal gas, and for an ideal procedure
= 2092-4000= 1700J we assume for such a gas, an ideal adiabatic process is
(b) ~U=~Q-~W considered as quasi.static. in which the system passes
= (300)(4.184) - (-420) = 1680J through a continuous succession of equilibrium states,
(e) ~U= (~Q- ~W) which we can plot on a P-V diagram. Such a process is
= (-1200)(4.184) - 0 = -5000J
also assumed to be reversible. though of course. a
quasi-static process may not be reversible. Such ideal
[Note that dQ is positive when heat is added to the
conditions as we assume may not be obtained in
system and .:i,W is positive when the system does the
particle but almost all real systems deviate little from
work} the results of our ideal procedures, which may
12. Wittl + Wif = Riaf therefore considered as approximations to real
20+uij = 50 processes.
uif = 30 8. Work is done only expansions or compressions of the
and Wibj +Uif = Hib/ gas where volume changes occur. In a quasi-static
Wibj + 30 = 36 condition, a small change of volume dV under pressure
P involves the work dW = p.dV. In a succession of
Wibf = 6
quasi-static equilibrium states, total work done can be
(a) Wibf = 6
obtained by integration.
(b) Hji=Wfi+Ufi 9. Any given thennodynamic state has a definite
=13+30=43 temperature T. The internal energy of a system in an
(e) Vif =30 U;-=10 Vf =40 ideal gas is a function of only the absolute temperature
Cd) Hib"",Wib+Uib and is independent of its pressure and volume. The
= 6+(22-10) = 18 internal energy per mole of a gas is directly
13. PV1 = constant, V(y-I)T = constant, proportional to its absolute temperature.
P(l-Y)T"f = constant
p~ _1_. P oc T1'/(y-ll. V <X TL'{y-ll
V'
P <>< TYi(l-l) oc (L E. )yi{y-I)

www.puucho.com
Study Anurag Mishra with www.puucho.com

,'; ,',

Previous Year 'Problems

Only One Alternative is Correct 5. Two metallic sphere 51 and 52 are made of the same
material and have got identical surface finish. The
1. A wall has two layers A and B, each made of different mass of 51 is thrice that of 52' Both spheres are heated
material. Both the layers have the same thickness. The to the same high temperature and placed in the same
thermal conductivity of the material of A is twice that . room having lower temperature but are thennally
of B. Under thermal equilibrium, the temperature insulated from each other. The ratio of the initial rate
difference across the wall is 36°C. The temperature of cooling of 51 ofthatof52 is: (lIT 1995)
difference across the layer A is :
(a) 6°C (b) 12°C (e) 18°C
(lIT 1980)
(d) 24°C
2. Aconstant volume gas thermometer works on:
6.
(a) ~ (b) 3J (e) ~ (d) m'
1

Three rods of identical cross-sectional area and made


(lIT 1980)
(a) the principle of Archimedes . from the same metal form the sides of an isosceles
(b) Boyle's law " : triangle ABC, right-angled at B. The points A and Bare
- maintained at temperatures T and (J2)T respectively.
(c) Pascal's law (d) Charle's law
In the steady state, the temperature of the point C is
3. An ideal monoatomic p ,
Te. Assuming that only heat conduction takes place,
gas is taken round the DB C Tc/Ti" (lIT 1995)
cycle ABCDA as shown (2P, (2P,2\1)
in the P-V diagram (see (a) 1 (b)_3_
figure). The workdone 2(,/2-1) ,/2+1
during the cycle is : A D (e) 1 (d)
_1_
(lIT 1983; 1M) (p, 11) (p, 211) .J3 (,/2 -I) ,/2 + 1
(a) PV '----------v 7. The average translational kinetic energy and the root
(b) 2PV mean square speed of molecules in a sample of oxygen
(e) PV gas at 300 K are 6.21 x 10-21 J and 484 m/s
2 respectively. The corresponding values at 600 K are
(d) zero nearly (assuming ideal gas behaviour)
4. From the following statements, concerning ideal gas (lIT 1997)
at any given temperature T, select the correct one(s). 12.42 x 10-21 J, 968 mls
(a)

(lIT 1995) (b) 8.78 X 10-21 J, 684 mls


(a) The coefficient of volume expansion at constant (c) 6.21 X 10-21 J, 968 mls
pressure is the same for all ideal gases Cd) 12.42 x 10-21 J, 684 mls
(b) The average translational kinetic energy per 8. The intensity of radiation emitted by the sun has its
molecule of oxygen gas is 3kT, k being Boltzmann maximum valUe at a wavelength of 510 nm and that
constant emitted by the Nonh Star has the maximum valUe at
(c) The mean-free path of molecules increases with 350 nm. If these stars behave like black bodies, then
increase in the pressure the ratio of the surface temperatures of the sun and
(d) In a gaseous mixture, the average translational the North Star is : (lIT 1997)
kinetic energy of the molecules of each component (a) 1.46 (b) 0.69 (e) 1.21 (d) 0.83
is different

www.puucho.com
Study Anurag Mishra with www.puucho.com
279
PREVIOUS YEARPROBlfMS
9. The average translational kinetic energy of O2 (molar 18. A block of ice at _10° C is slowly heated and converted
mass 32) molecules at a particular temperature is to steam at 100°C. Which of the following curves
0,048 eV. The translational kinetic energy of N2 represents the phenomenon qualitatively?
(molar mass 28) molecules in eV at the same (lIT 2000)
temperature is : (liT 1997)
(a) 0,0015 (b) 0,003 (e) O,04S (d) O,76S'
10. A vessel contains 1 mole of 0 2 gas (molar mass 32) at
a temperature T. The pressure of the gas is P. An
()~ ()~
identical vessel containing one mole of He gas Hem supplied _ Heat supplied_

(relative molar mass 4) at a temperature 2T has a

()lL )~
pressure of : (lIT 1997)
(a) PIS (b) P (e) 2P (d) SP
11. A spherical black body with a radius of 12 em radiates
450 W power at 500 K. If the radius is halved and the Heal supplied _ Heat supplied_
temperature doubled, the power radiated in watt
would be : (lIT 1997) 19. Starting with the same initial y
(a) 225 (b) 450 (e) 900 (d) IS00 conditions, an ideal gas 1
12. One mole of a monoatomic gas is heated at a constant expands from volume Vi.to Vz p
pressure of 1 atmosphere from a K to 100 K. If the gas in three different ways. The
constant R = 8.32 J/moVK, the change in internal work done by the gas is W1 if the
energy of the gas is approximately: (lIT 1998) process is purely isothermal, 0
(a) 2,3 J (b) 46 J W,2 if purely isobaric and W3 if
(c) 8.67 x 103 J (d) 1.25 x 103 J purely adiabatic. Then
(lIT 2000)
13. An ideal gas heat engine is operating between 227°C
(a) Wz > WI > W3 (b) W2 > W3 > WI
and 127°C. It absorbs 104 J of heat at the higher
temperature. The amount of heat converted into work (c) WI > Wz > W3 Cd) WI > W3 > W2
is : (lIT 1998) 20. An ideal gas is initially at temperatureT and volume V.
Its volume is increased by.1.V due to an increase in
(a)2000 J (b) 4000 J (e) SOOO J (d) 5600 J
temperature ~T, pressure remaining constant. The
14. A gas mixture consists of 2 mole of oxygen and 4 mole
of argon at temperature T. Neglecting all vibrational quannry, u, = --~v vanes
, Wit' h temperature as :
modes, the total internal energy of the system is : V~T
(liT 1999)
(a) 4RT (b) 15RT (e) 9RT (d) llRT
15. The ratio of the speed of sound in nitrogen gas to that
in helium gas, at 300 Kis: (lIT 1999)
(a) AlI7) (b) J(l/7J (e) (,[3)/5 [d) (.J6)/5
16. Two monoawmic ideal gases 1 and 2 of molecular
masses M1 and M z respectively are enclosed in
separate containers kept at the same temperature. The
ratio of the speed of sound in gas 1 to that in gas 2 is
given by: (lIT 2000)

(a) IMj (b) 1M2 (e) Mj (d) M2


iM, \Mj M2 M[
21. The plots of intensity versus y
17. A monoatomic ideal gas, initially at temperature T1, is
enclosed in a cylinder fitted with a frictionless pistion. wavelength for three black T,
The gas is allowed to expand adiabatically to a
temperature Tz by releasing the piston suddenly. If L}
and Lz are the lengths of the gas column before and
bodies at temperature TI,Tz
and T3 respectively are as
shown. Their temperature are
) f<)ct
after expansion respectively, then T11Tz is given by: such that: L~A-_~--X
(lIT 2000)
2'3 (liT 2000)
(d)(~:l

www.puucho.com I
Study Anurag Mishra with www.puucho.com

---~TH:::ERMOO=;::_:::::::::lc"'r
(a) T1 >T2 >T3 (b) T1 >T3 >T2 27. An ideal black-body at room temperature is thrown
ee) T2 >T3 >T} (d)T3 >T2 >1i. into a furnace. It is observed that: (lIT 2002)
22. Three rods made of same ~900C (a) initially it is the darkest body and at later times the
material and having the same brightest
cross-section have been joined DOC (b) it is the darkest body at all times
as shown in the figure. Each rod (c) it cannot be distinguished at all times
is of the same length. The left 90°C
Cd) initially it is the darkest body and at later times it
and right ends are kept at DOC
cannot be distinguished
and 90"C respectively. The temperature of the
28. The graph, shown in the
junction aCthe three rods will be: (liT 2001)
(a) 45' (b) 60'C Ce)30'C Cd)20'C adjacent diagram, represents the x ~
variation of temperature (T) of
23. In a given process on an ideal gas dW = 0 and dQ < 0.
two bodies, x and y having same T
Thenfor the gas: (lIT 2001) surface area, with time (t) due to
(a) the temperature will decrease the emission of radiation. Find
(b) the volume will increase the correct relation between the
(e) the pressure will remain constam emissivity and absorptivity powers of the two bodies:
Cd) the temperature will increase (lIT 2003J
24. P-V plots for two gases during adiabatic processes are (a) Ex >Ey and ax <ay(b)Ex <Ey andax >ay
shown in the figure. Plots 1 and 2 should correspond (c) Ex >Ey and ax >ay(d)Ex <Ey and ax <ay
respectively to : (liT 2001) 29. 2 kg of ice at -20°C is mixed with 5 kg of water at 20"C
p
in an insulting vessel having a negligible heat
capacity. Calculate the final mass of water remaining
in the container. It is given that the specific heats of
water and ice are 1 kcaVkgOCand 0.5 kcaVkgOCwhile
the latent heat of fusion of ice is 80 kcaVkg :
v (liT 2003J
(a) Heand02 (b) 02 andHe (a) 7 kg (b) 6 kg (e) 4 kg (d) 2 kg
(e) He and Ar Cd)02 and N2 30. The PT diagram for an ideal gas is ~
25. Which of the following graphs correctly represents the shown in the figure, where AC is an t-..
""
vanatlOn fA
0 I-'= ---
"h p lor
dV;dP Wit , an I"dea I gas at adiabatic process. Find the P ~
V corresponding PV diagram. B C
constant temperature? (lIT 2002) (lIT 2003; 2M) T

ca)P~
p
(b)P~
p
(a) plk V
Cb) plk V

Ce) pI ~ (d)pl ~
Ce)P~ Cd)pI ~ Bee B
V V
p p
31. Two rods, one of aluminium and the other made of
steel, having initial lengths 11 and {2 are connected
26. An ideal gas is taken through together to form a single rod of length II + 12, The
the cycle A -+ B -+ C -+ A, C B coefficients of linear expansion for aluminium and
as shown in the figure. If the steel are a II and a s respectively. If the length of each
net heat supplied to the gas ~ rod increases by the same amount when their
in the cycle is 5 J, the work A
temperature are raised by tOC, then find the ratio
done by the gas in the 11/01 + {2): (lIT 2003)
process C -+ A is: P(N/m2) 0
(a) as/all (b) all/as
(liT 2002) (c) as/(as +aa) (d) all/Carl +0.5)
Ca) -5J (b) -IOJ (e) -15J Cd)-20J

t www.puucho.com
Study Anurag Mishra with www.puucho.com

PREVIOUS YEAR PROBLEMS


2B1

32. Three discs A,B and C having radii 2, 4 and 6 em (a) O.4ea AT4 (b) 0.8ccrAT4
respectively are coated with carbon black. (c) 0.6ecrAT4 (d) 1.0ecrA.T4
Wavelengths for maximum intensity for the three
38. In which of the following processes. convection does
discs are 300, 400 and 500 om respectively. If QA' QB
not take place primarily? (lIT 2005)
and Qc are powers emitted by A,B ande respectively,
then: (lIT 2004) (a) sea and land breeze
(a) QA will be maximum (b) QB will be maximum (b) boiling of water
(c) heating air around a furnace
ee) Qc will be maximum Cd) QA = QB = Qc
(d) wanning of glass of bulb due to filament
33. An ideal-gas initially atE\. V1 is expanded to P2' V2 and
then compressed adiabatically to the same volume V1 39. Water of volume 21itre in a container is heated \\'ith a
and pressure P3' IfW is the net work done by the gas in coil of 1 kW at 27'C. The lid of the container is open
complete process which of the following is true? and energy dissipates at rate of 160 J/s. In how much
(lIT 2004) time temperature will rise from 2TC to 77'C? [Given
specific heat of water is 4.2 kJ/kg) (lIT 2005)
(a) W > 0; P3 > Pt (b) W < 0; P3 > 11
(a)7min (b)6min2s
ee) W > 0; P3 < 11 Cd)W < 0; P3 < 11
(c) 8 min 20 s (d) 14 min
34. Two identical rods are connected bet\Neen twO
containers. One of them is at lOOoe containing water 40. Calorie is defined as the amount of hem required to
raise temperature of 1 g of water by 1°C and it is
and another is at DOC containing ice. If rods are
defined under which of the following conditions?
connected in parallel then the rate of melting of icc is
(liT 2005)
ql g!sec. If they are connected in series then the rate is
Q2.TheratioQ2!qlis: (lIT 2004) (a) From 14SC to 15.5"C at 760 mm of I-lg
Ca) 2 Cb)4 Ce)1/2 Cd)1/4 (b) From 98.5"C to 99SC at 760 mm of IIg
35. If liquefied oxygen at 1 atmospheric pressure is heated (c) From 13SC to 14SC at 76 mm of Hg
from 50 Kto 300 Kby supplying heat at constant rate, (d) From 3SC to 4SC at 76 mm of Hg
the graph of temperature vs time will be : 41. An ideal gas is expanding such that PT:! = constant.
(lIT 2004) The coefficient of volume expansion of the gas is :

CO)t, Cb)¥, Ca) 1/1' Cb)2/1'

=:I More than One Alternatives


Ce)3/1'
(liT 2008)
Cd)4/1'

ce)k
is/are Correct

1. At room temperature, the rms speed of the molecules


Cd)~ of a certain diatomic gas is found to be 1930 m/s. The
gasis: (lIT 1984)
(a) H2 (b) F2 (c) 02 (d) CI2
36. Variation of radiant energy EA 2. 70 caloric of heat arc required to raise the
emitted by sun, filament of temperature of 2 mole of an ideal gas at constant
tungsten lamp and welding pressure from 30°C to 35"C. The amount of the heat
arc as a function of its required (in calorie) to raise the [emperalure of the
wavelength is shown in figure. same gas through the same range (30'C to 35"C) at
Which of the following option constant volume is: (liT 1985)
is the correct match?
Ca) 30 Cb) 50 Ce)70 Cd)90
(liT 2005)
3. Stream at 100"C is passed into 1.1 kg of water
(a) Sun-T3' tungsten filament-TI' Welding arc-T2 contained in a calorimeter of water equivalent 0.02 kg
(b) Sun-T2, tungsten filament-T}, Welding arc-T3 at 15"C till the temperature of the calorimeter and its
(c) Sun-T3, tungsten filament-T2, Welding arc-Tl contents rises at 80°G. The mass of the steam
(d) Sun-TI, tungsten filament-T2, Welding arc-T3 cO:1densed in kilogram is : (liT 1986)
37. A spherical body of area A and emissivity e = 0.6 is Ca) 0.130 Cb)0.065 Ce)0.260 Cd)0.135
kept inside a perfectly black body. Energy radiated per 4. A cylinder of radius R, made of a material of thermal
second by the body at temperature T is : conductivity K}, is surrounded by a cylindrical shell of
(lIT 2005)

www.puucho.com
Study Anurag Mishra with www.puucho.com

THI_ICS
inner radius R and outer radius 2R made of a material (a) The workdone by the gas in the process A to B-
of thermal conductivity K2. The two ends of the exceeds the work that would be done by it if the
combined system are maintained at two different system were taken from A to B along the isotherm
temperatures. There is no loss of heat across the (b) In the T-V diagram, the path ABbecomes a part of
cylindrical surface and the system is in steady state. a parabola
The effective thermal conductivity of the system is ;
(c) In the P-T diagram, the path AB becomes a part of a
(lIT 1988)
hyperbola
(a) K1 +K2 (b) K1K2(K1 +K2)
(d) In going from A to B, the temperature T of the gas
(e) (K, +3K,)!4 Cd)(3K, +K,)/4
first increases to a maximum value and then
5. If one mole of a monatomic gas (y = 5/3)is mixed with decreases.
one mole of a diatomic gas (y = 7/5), the value ofy for 10. Two bodies A and B have thermal emissil,iries of 0.01
the mixture is: (lIT 1988)
and 0.81 respectively. The outer surface areas of the
(a) lAO (b) 1.50 Ce) 1.53 Cd) 3.07
two bodies are the same. The two bodies emit total
6. For an ideal gas: (lIT 1989) radiant power of the same rate. The wavelength)" B
(a) the change in internal energy in a constant corresponding to maximum spectral radiancy in the
pressure from temperature 1i to T2 is equal to radiation from B shifted from the wavelength
nC v (T 2 - T1 ), where C v is the molar specific hear corresponding to maximum spectral radiancy in the
at constant volume and II the number of moles of radiation from A, by 1.00 /lm. If the temperature of A
the gas is 5802 K: (lIT 1994)
(b) the change in internal energy of the gas and the (a) the temperature of B is 1934 K
work done by the gas are equal in magnitude in an (b) ).,B = 1.5 /lm
adiabatic process (c) the temperature of B is 11604 K
(c) the internal energy does not change in an (d) the temperature of B is 2901 K
isothermal process 11. The temperature of an ideal gas is increased from
(d) no heat is added or removed in an adiabatic 120 K to 480 K. If at 120 K the root.mean square
process velocity of the gas molecules is v , at 480 Kit becomes;
7. When an ideal diatomic gas is heated at constant (liT 1996)
pressure, the fraction of the heat energy supplied Ca) 4v Cb) 2v Ce) vl2 (d) "/4
which increases the internal energy of the gas is: 12. Two identical containers A and B with frictionless
(liT 1990) pistons contain the same ideal gas at the same
(a)~ (b)~ (e)~ Cd)~ temperature and the same volume V. The mass of the
5 5 7 7 gas in A is rnA' and that in B is mB' The gas in each
8. Three dosed vessels A, Band C are at the same cylinder is now allowed to expand isothermally to the
temperature T and contain gases which obey the same final volume 2 V. The changes in the pressure in
Maxwellian distribution of velocities. Vessel A A and B are found to be M' and l.5M' respectively.
contains only O2, B only N2 and C a mixture of equal Then: (lIT 1998)
quanriries of O2 and N2. If the average speed of the (a) 4mA = 9mB (b) 2mA =- 3mB
02 molecules in vessel A is U I mat the N2 molecules in
°
vessel B is U 2, the average speed of the 2 molecules
(c) 3mA = 2mB (d) 9mA = 4mB
13. A given quantity of an ideal gas is at pressure P and
in vessel Cis: (lIT 1992)
absolute temperature T. The isothennal bulk modulus
(a)Ul+U2 (b)u\ of the gas is : (lIT 1998)
2
Ca) ~P Cb) P (e) ~P Cd) 2P
(c) (vI' v2 l'2 (d) ~3~T 3 2
14. Two cylinders A and B fitted with pistons contain
where M is the mass of an oxygen molecule. equal amounts of an ideal diatomic gas at 300 K. The
9. An ideal gas is taken from the state A (pressure P, piswn of A is free to move, while that of B is held fixed.
volume V) to the state B (pressure P/'4 volume 2V) The same amount of heat is given to the gas in each
along a straight line path in the P-V diagram. Select cylinder. If the rise in temperature of the gas in A is 30
the correct statement(s) from the following: K, then the rise in temperature of the gas in B is :
(lIT 1993) (lIT 1998)
(a) 30 K (b) 18 K Ce) 50 K (d) 42 K

, www.puucho.com
Study Anurag Mishra with www.puucho.com

PRMOUS YIAR PROBLEMS 2B3

15. During the melting of a slab of ice at 273 K at TO remains constant. Which of the following
atmosphericpressure: (lIT 1998) statements is/are true? (lIT 2006)
Ca) positive work is done by the ice-water system on (a) The rate of emission of energy from the black body
the atmosphere remains the same
(b) positive work is done on the ice-water system by (b) The rate of emission of energy from the black body
the atmosphere increases
(e) the internal energy of the ice-water system (c) The rate of absorption of energy by the black body
increases increases
Cd) the internal energy of the ice-water system (d) The energy radiated by the black body equals the
decreases energy absorbed by it
16. A black body is at a temperature of2880 K.The energy 21. C "and C p denote the molar specific heat capacities of
of radiation emitted by this object with wavelength a gas at constant volume and constant pressure,
between 499 om and SODnm is VI, between 999 nm respectively. Then (lIT 2009)
and 1000 om is Uz and bet\veen 1499 om and 1500 (a) C p - C t, is larger for a diatomic ideal gas than
om is U3- The Wien constant b == 2.88 x 106nm K. for a monoatomic ideal gas
Then: (liT 1998) (b) C p + C I' is larger for a diatomic ideal gas than
(a) VI =0 (b)U3 ==0 (c)UI >Uz (d)U2 >UJ for a monoatomic ideal gas
17. A vessel contains a mixture of one mole of oxygen and IC
(e) C p l' is larger for a diatomic ideal gas than for
two mole of nitrogen at 300 K. The ratio of the average a monoatomic ideal gas
rotational kinetic energy per 02 molecule to that per (d) C p . C I' is larger for a diatomic ideal gas than for a
N2 molecule is: (lIT 1998) monoaLOmic ideal gas
(a) 1 , 1 22. The figure shows the P - V plot of

m
A
(b) 1 , 2 an ideal gas taken through a cycle
(e) 2, 1
ABCDA. The part ABC is a
semi-circle and CDA is half of an I
:0 B
(d) depends on the moments of inertia of the twO C
ellipse. Then: o V
molecules 123
18. Let V, vrm5 and "p respectively denote the mean (a) the process during the path
speed, root mean square speed, and most probable A -+ B is isothermal
speed of the molecules in an ideal monoatomic gas at (b) heat flows out of the gas during the path B-+C~ D
absolute temperature T. The mass of a molecule is m. (c) work done during the path A ~ B ~ C is zero
Then: (lIT 1998) (d) positive work is done by the gas in the cycle
(a) no molecule can have a speed greater than ABCDA (lIT 2009)
,/Zl'rms
(b) no molecule can have a speed less than vp/J2
(c) vp <v<Prms

(d) the average kinetic energy of a molecule is ~ mv~


=: True/False
1. The root-mean square speeds of the molecules of
4
19. A bimetallic strip is formed out of two indentical strips differem ideal gases, maintained at the same
one of copper and the other of brass. The coefficients temperature are the same. (lIT 1981)
of linear expansion of the two metals are Uc and uB'
On heating, the temperature ofthe strip goes up by liT 2. The volume V versus temperature T V~
and the strip bends to form an arc of radius of graphs for a certain amoum of a PI
curvature R. Then R is : (lIT 1999) perfect gas at two pressure Pt and P2 / ./ P
(a) proportional to liT are as shown in figure. It follows (,/ 2

(b) inversely proportional to liT from the graphs that Pt is greater T


(c) proportional toluB -ucl than P2' (liT 1982)
3. Two differem gases at the same temperature have
(d) inversely proportional to IUn -Ctc I
equal root mean square velocities. (lIT 1982)
20. Initially a black body at absolute temperature T is kept
inside a closed chamber at absolute temperature To.
Now the chamber is slightly opened to allow sun rays
to enter. It is observed that temperatures T and To

www.puucho.com
Study Anurag Mishra with www.puucho.com
"'2;;'84;-------
THIRMODYNAMICS
4. The curves A and B in the figure I~ 7. A point source of heat of power P is placed at tht
show P-V graphs for an isothermal \::: centre of a spherical shell of mean radius R. ThE
and an adiabatic process for an ideal P A material of the shell has thennal conducti\.ity K. If thE
gas. The isothermal process is B temperature difference between the outer and mne,_
represented by the curve A. V _ surface of the shell is not to exceed T, the thickness oJ
(lIT 1985) the shell should not be less than .
5. At a given temperature, the specific heat of a gas at (JIT 1991)1
constant pressure is always greater than its specific 8. A substance of mass M kg requires a power input of i
heat at constant volume. (lIT 1987) watt to remain in the molten state at its melting point.
6. The root mean square (rms) speed of oxygen When the power source is turned off, the sample-
molecules (02) at a cerrain temperature T (degree completely solidifies in time t second. The latent heatl
absolute) is L'. If the temperature is doubled and of fusion of the substance is .
oxygen gas dissociates into atomic oxygen, the rms (lIT 1992).
speed remains unchanged. (lIT 1987) 9. A container of volume 1 m 3 is divided into two equal~
7. Two spheres of the same material have radii 1 m and parts by a partition. One part has an ideal gas at 300 K
4m and temperatures 4000 Kand 2000 Krespectively. and the other part is vacuum. The whole system is
The energy radiated per second by the first sphere is thermally isolated from the surroundings. When the
greater than that by the second. (lIT 1988) partition is removed, the gas expands [Q occupy the
== Fill in the Blanks
whole volume. Its temperature will now be
(lIT 1993)
.

1. One mole of a monoatomic ideal gas is mixed with one 10. An ideal gas with pressure P, volume V and
mole of a diatomic ideal gas. The molar specific heat of temperature T is expanded isothermally to volume 2 V
the mixture at constant volume is . and a final pressure ~. If the same gas is expanded
(lIT 1984) adiabatically to volume 2 V, the final pressure is Pa.
2. The variation of temperature of The ratio of the specific heats of the gas is 1.67. The
a material, as heat is given to it t ratio Pa is (lIT 1994)
at a constant rate, is shown in T C 11
the figure. The material is in A P D 11. Two metal cubes A
solid state at the point O. The B
and B of same size are
state of the material at the point 0 heatadded- arranged as shown in
Pis . figure. The extreme
(lIT 1985) ends of the combination are maintained at the
3. During an experiment, an ideal gas is found to obey an indicated temperatures. The arrangement is thermally
additional law VP2 = constant. The gas is initially ata insulated. The coefficients of thermal conductivity of
temperature T, and volume V. When it expands to a A andB are 300 W/moC and 200 W/moC, respectively.
volume 2V, the temperature becomes . Mter steady state is reached the temperature T of the
interface will be (lIT 1996)
(lIT 1987)
4. 300 gram of water at 25°C is added to 100 gram of ice 12. A ring shaped tube contains two MJ M2
at O°c. The final temperature of the mixture is ideal gases with equal masses and
..•........... DOC. (lIT 1989) molar masses M1 = 32 and
5. The earth receives, at its surface. radiation from the M 2 = 28. The gases are separated
sun at the rate of 1400 W m -2. The distance of the by one fixed partition and another
centre of the sun from the surface of the earth is movable stopper S which can move
freely without friction inside the
LSx1011 m and radius of the sun is 7xlOs m.
ring. The angle 0: as shown in the
Treating the sun as a black body, it follows from the
figure is degree.
above data that its surface temperature is K.
(lIT 1997)
(lIT 1989)
6. A solid copper sphere (densityp and specific heat s) of 13. A gas thermometer is used as a standard thermometer
radius r at an initial temperature 200 K is suspended for measurement of temperature, when the gas
inside a chamber whose walls are at almost 0 K. The container of the thermometer is immersed in water at
time required for the temperature of the sphere to its triple point 273.16 K, the pressure in the gas
drop to 100 Kis (lIT 1991) thermometer reads 3.0 x 104 N/m 2. When the gas
container of the same thermometer is immersed in

www.puucho.com
Study Anurag Mishra with www.puucho.com

.R!VIOUS YEARPRO'LEMS 2.5

another system, the gas pressure reads 3.5 x 104 (b) An ideal monoatomic gas expands (q) The temperature of
to twice its original volume such the gas increases or
N/m 2. The temperature of this system is therefore
that its pressure where V is the remains constam
................."C. (lIT 1997) volume of the ~as.
14. Eanh receives 1400 W/m2 of solar power. If all the
(e) An idea; monoatomicgas expands (,) The gas loses heat
solar energy falling on a lens of area 0.2 m 2 is focused to twice its original volume such
on to a block of ice of mass 280 gram, the time taken to that its pressure where V is its
melt the ice will be minute. volume
(Latent heat of fusion of ice = 3.3 x 105 J/kg.) Cd) An ideal monoatomicgas expands C,) The gas gains heat
(lIT 1997) such [hilt its pressure P ,nd

:I Matching Type Problems


volume V follows the behaviour
shown in the graph
P
1. Heat gIVen
process is positive,
to P(atm)
1
ld
=
30 ..
' V
match the

J~LM
VI 2V1
following option of 20
column I with the 10 Assertion and Reason-""'"
corresponding
option of column 10 20 Instructions: Each question contains sratemellt-l
[I. (lIT 2006)
(assertion) and statemellt-2 (reason). O/these
Column 1 Column II statements, mark correct choice if
I') lK [p) 6W > 0 (a) Statements-] and 2 are true and statement-2 is a
correct explanation for statement-]
(b) KL (q) C!.Q < 0
(b) Statemems-] and 2 are true and statement-2 is not a
Ie) LM CO C!.W <0 correct explanation for statement-!
(d) i\-fJ C,) C!.Q>0 (c) Statement-] is true, statement-2 is false
(d) Statement-1 is false, statement-2 is true.
2. Column I gives some devices and column II gives some 1. Statement-]: The total translational kinetic energy
processes on which the functioning of these divices of all the molecules of a given mass of an ideal gas is
depend. Match the device in column I with the 1.5 times the product of its pressure and its volume.
processes in column II. (lIT 2007)
Statement-2 : The molecules of a gas collide with

=
Column] Column II each other and the velocities of the molecules change
due to the collision. (lIT 2007)
(,) BimetallicStrip (p) Radiation from a hot body
Cb) Steam engine Cq) Energyconversion Integer Answer TYpe Peoblems
(e) Incandescent lamp (,) Melting 1. A metal rod ABof length lOx has its one end A in ice at
(d) Eb:tric fuse (,) Thennal expansion of solids we and the other end B in water at 100ue. If a point P
on the rod is maintained at 400"C, then it is found that
3. Column I contains a list of processes involving expansion equal amounts of water and ice evaporate and melt
of an ideal gas. Match this with Column II describing the per unit time. The latent heat of evaporation of water
thermodvnamicchan edwin this rocess. (llT2008) is 540 caVg and latent heat of melting of ice is 80
caVg. If the point P is at a distance of Ax from the ice
Column] Column II end A , find the value of A . {Neglect any heat loss to
(a) An insulated containet has two (p) The temperature the surrounding.] (lIT 2009)
chambers separated by a valve. of the gas 2. A piece of ice (heat capacity:= 2100 J kg -1 "C-1 and
Chamber I contains an ideal gas decreases latent heat:=:3.36 x lOS J kg -1 )of mass m grams is at
and the Ch;]mber II has vacuum.
The valve is opened. _50 Cat atmospheric pressUl"e.It is given 420 J of heat
so that the ice starts melting. Finally when the
...
1:1::\ II
~,
ice-water mixture is in equilibrium, it is found that 1
iaral ga$ vacuum
.I'.~'.:,: gm of ice has melted. Assuming there is no other heat
exchange in the process, the value of TIl is: (fiT 201 0)

www.puucho.com
Study Anurag Mishra with www.puucho.com

=
286

Subjective Problems

1. A room is maintained at 20°C by a heater of resistance


20 and 2S mm of Hg.
Calculate the new pressure in the jar.
THERMODYNAMICS

vapor pressure of water at the n'Vo temperatures are

(lIT 1980)
20 ohm connected to 200 volt mains. The temperature
6. A cyclic process ABCAshown V
is unifonn throughout the room and the heat is
in the V-T diagram (fig.) is V
transmitted through a glass window of area 1 m 2 and
perfonned with a constant
thickness 0.2 em. Calculate the temperature outside.
. mass of an ideal gas. Show the
Thermal conductivity of glass is 0.2 cal m -ls-l(oC)-1
same process on a P-V
and mechanical equivalent of heat is 4.2 J ca1-1 . diagram (In the figure, 0\ is VI
(lIT 1978) parallel to the V-axis and BC is
2. A sinker of weight Wo has an apparent weight WI parallel to the T-axis).
when placed in a liquid at a temperature Tj and Wz (lIT 1981)
when weighed in the same liquid at a temperature Tz.
7. A lead bullet just melts when stopped by an obstacle.
The coefficient of cubical expansion of the material of
Assuming that 25% of the heat is absorbed by the
the sinker is p. What is the coefficient of volume
expansion of the liquid? (liT 1978) obstacle, find the velocity of the bullet if its initial
3. A composite rod is made by joining a copper rod, end temperature is 27'C. (Melting point of lead = 327°C.
to end, with a second rod of different material but of Specific heat of lead = 0.03 callg C, Latent heat of
C

the same cross-section. At 2ScC, the composite rod is 1 fusion ofIead = 6callg, J = 4.2J/cal.)
meter in length, of which the length of the copper rod (lIT 1981)
is 30 em. At 12SeC the length of the composite rod
8. Calculate the work done when one mole of a perfect
increases by 1.91 mm.
gas is compressed adiabatically. The initial pressure
When the composite rod is not allowed to expand by
and volume of the gas are 105 N/m 2 and 6 liue
holding it between two rigid walls, it is found that the
respectively. The final volume of the gas is 2 litre.
length of the two constituents do not change with the
Molar specific heat of the gas at constant volume is
rise of temperature. Find the Young's modulus and the
3RJ2 (lIT 1982)
coefficient of linear expansion of the second rod.
[Given: Coefficient of linear expansion of copper 9. A solid sphere of copper of radius R and a hollow
= 1.7 x 10-5 per"C, sphere of the same material of inner radius r and outer
Young's modulus of copper = 1.3 x 1011 N/m 2) radius R are heated to the same temperature and
(lIT 1979) allowed to cool in the same environment. Which of
4. A solid material is supplied with heat at constant rate. them starts cooling faster? (lIT 1982)
The temperature of the material is changing with the 10. One gram mole of oxygen at 27'C and one
heat input as shown in the graph in figure. Study the atmospheric pressure is enclosed in a vessel.
graph carefully and answer the following questions. CD Assuming the molecules to be moving with l' nm'
find the number of collisions per second which the
C D molecules make with one square metre area of the

i A B
vessel wall.
(ii) The vessel is next thermally insulated and moved
E with a constant speed vo. It is then suddenly
'"
D HeatInput_
stopped. The process results in a rise of the
temperature of the gas by 1cc. Calculate the speed

D
(i) What do the horizontal region AB and CD vo. (lIT 1983)
represent? 11. The rectangular box shown in
(ii) If CDis equal to 2AB, what do you infer? figure has a partition which can
(iiOWhat does the slope of DE represent? slide without friction along the
(iv) The slope of OA > the slope of BC. What does this length of the box. Initially each of
indicate? (lIT 1980) the two chambers of the box has
5. A jar contains a gas and a few drops of water at T K. one mole of a mono-atomic ideal gas ('( = 5/3) at a
The pressure in the jar is 830 mm of Hg. The pressure Po, volume Vo and temperature To. The
chamber on the left is slowly heated by an electric
temperature of the jar is reduced by 1%. The saturated
heater. The walls of the box and the partition are

www.puucho.com
Study Anurag Mishra with www.puucho.com

PREVIOUS YEAR PROBLEMS 2BI


thermally insulated. Heat loss through the lead wires
Open atmosphere
of the heater is negligible. The gas in the left chamber
exp"mrls pushing the partition until the final pressure Heater - Rigid
suppon
in both chambers becomes 243Po/32. DeLermine (i)
the final temperature of the gas in each chamber and
OJ) the workdone by the gas in the right chamber.
(lIT 1984) The gas is heated by a small electric heater until the
12. Two glass bulbs of equal volume are connected by a piston moves out slowly by 0.1 m. Calculate the final
narrow tube and are filled with a gas at GOC and a temperature of the gas and the heat supplied (in joule)
pressure of 76 em of mercury. One of the bulbs is then by the heater. The force constant of the spring is 8000
placed in melting ice and the other is placed in a water N/m. atmospheric pressure is LO x lOs Nm -2. The
bath maintained at 62°C. Whut is the new value of {he cylinder and the piston are thermally insulated. The
pressure inside the bulbs? The volume of the piston is massless and there is no friction berwcen the
connecting tube is negligible. (lIT 1985) piston and the cylinder. Neglect heat loss through lead
13. An electric heater is used in a room of total wall area wires of the heater. The heat capacity of the heater coil
137 m 2 to maintain a temperature of + 20°C inside it, is negligible. A<;sumethe spring to be massless.
when the outside temperature is -10 C. The walls
0
(lIT 1989)

have three different layer materials. The innermost 17. An ideal gas having initial pressure P, volume Vand
temperature T is allowed to expand adiabatically until
layer is of wood of thickness 2.5 em, the middle layer
its volume becomes 5.66 Vwhile its temperature falls
is of cement of thickness 1.0 em and the outermost to Tj2.
layer is of brick of thickness 25.0 em. Find the power (i) How many degrees of freedom do the gas
ohhe electric-heater. Assume that there is no heat loss molecules have?
through the floor to the ceiling. The thermal (ii) Obtain the workdone by the gas during the
conductivities of wood, cement and brick cue 0.125, expansion as a function of the initial pressure P
1.5 and LO watt/moC respectively. (lIT 1986) and volume V. (lIT 1990)
14. An ideal gas has a specific heat at constant pressure 18. Three moles of an ideal gas ( C p = ~R ) at pressure PA
C p = 5R. The gas is kept in a closed vessel of volume
2 and temperature TA are isothermally expanded to
0.0083 m 3, at a temperature of 300 K and a pressure rwice the initial volume. The gas is then compressed at
of L6 x 106 N/m 2. An amount of 2.49 x 104 joule of constant pressure to its original volume. Finally gas is
heat energy is supplied to the gas. Calculate the final compressed at constant volume to its original pressure
temperature and pressure of the gas. PA-
(lIT 1987) (a) Sketch P-V and P-T diagrams for the comp!ete
15. Two moles of helium gas (y = 5/3) are initially at process.
temperature 27°C and occupy a volume of 20 litres. (b) Calculate the net workdone by the gas, and the net
The gas is first expanded at constant pressure until the heat supplied to the gas during the complete
volume is doubled. Then it undergoes an adiabatic process. (liT 1991)
change until the temperature returns to its initial 19. Two moles of helium p

value.
(i) Sketch the process on a P- V diagram.
(ij) What are the final volume and pressure of the gas?
gas undergo a cyclic
process as shown in 2atm
figure. Assuming the
A
f------t B

(iij)What is the workdone by gas? (liT 1988)


gas to be ideal, latm ------,C
calculate the a
16. An ideal monoatomic gas is confined in a cylinder by a
following quantities 300 K 400 K T
spring.loaded piston of cross-section 8.0 x 10-3 m 2. in [his process:
Initially the gas is at 300 K and occupies a volume of (lIT 1992)
2.4x 10-3013 and the spring is in its relaxed
(a) The net change in the heat energy
(unstretched, uncompressed) state, figure.
(b) The net work done
(c) The net change in internal energy
20. A cylindrical block of length 0.4 01 and area of cross-
section 0.04 m 2 is placed coaxially on a thin metal

www.puucho.com
Study Anurag Mishra with www.puucho.com
288 ,"£RMODYNAMI
disc of mass 0.4 kg and the same cross-section. The y(= CpiC v) = 5(3 and another gas B with y = 7(5 at a
upper face of the cylinder is maintained at a constant certain temperature T. The molar masses of the gases
temperature of 400 K and the initial temperature of A and Bare 4 and 32, respectively. The gases A and B
the disk is 300 K. If the thennal conductivity of the do not react with each other and are assumed to be
material of the cylinder is 10 watt/mK and the specific ideal. The gaseous mixture follows the equation
heat of the material of the disc is 600 J/kgK, how long PV19,13 =constant, in adiabatic processes.
will it wke for the temperature of the disc to increase (a) Find the number of moles of the gas B in the
to 350 K? Assume, for purpose of calculation, the gaseous mixture.
thermal conductivity of the disc to be very high and (b) Compute the speed of sound in the gaseous
the system to be thermally insulated except for the mixture at T = 300 K.
upper faceof the cylinder. (lIT 1992) (c) If T is raised by 1 K from 300 K. find the
21. One mole of a monoatomic ideal p percentage change in the speed of sound in the
gas is taken through the cycle gaseous mixture.

U
shown in figure. (d) The mixrure is compressed adiabatically to I(S of
A -+ B: adiabatic expansion its initial volume V. Find the change in its
adiabatic compressibility in terms of the given
B -Jo C : cooling at constant D C 8
volume quantities. (lIT 1995)
C -!' D: adiabatic compression
v 25. At 27'C two moles of an ideal monoatomic gas occupy
D -+ A: heating at constant volume a volume V.The gas expands adiabatically to a volume
The pressure and temperature at A, B etc. are denoted 2V. Calculate 0) the final temperature of the gas, (ij)
by PA,TA,Pn,TB, etc., respectively. Given that change in its internal energy, and (iii) the workdone
TA = 100K,PB = (2,!3)PA and Pc = (l/3)PA,calculate by the gas during this process. (lIT 1996)
the following quantities:
26. The temperature of 100 g of water is to be raised from
(i) The workdone by the gas in the process A ~ B
24°C to 90°C by adding steam to it. Calculate the mass
(ij) The heat lost by the gas in the process B -+ C.
of the steam required for this purpose.
(iii)The remperarureTD. [Given: (2/3)2'5 = 0.85]
(liT 1996)
(lIT 1993)
27. A thin rod of negligible mass and area of cross-section
22. An ideal gas is taken through a cyclic thermodynamic
4 x 10-6 m 2. suspended vertically from one end, has a
process through four steps. The amounts of heat
length of 0.5 m at 100°C. The rod is cooled to DoC,but
involved in these steps are Ql = 5960J, Q2 = -5585J,
prevented from contracting by attaching a mass at the
Q3 = ~2908 J and Q4 = 3645 J, respectively. The
lower end. Find (i) this mass. and (ij) the energy
corresponding quantities of work involved are
stored in the rod. Given for the rod, Young's modulus
WI = 2200 J, W2 =-825 J, W3 =-1100J and W4
respectively. = 1011 N/m 2, c~efficient of linear expansion 10-5 K-1
andg =10m/s2. (lIT 1997)
(i) Find the value of w4
(ij) What is the efficiency of the cycle? 28. A double-pane window used for insulting a room
(lIT 1994) thermally from outside consists of two glass sheets
23. A closed container of volume 0.02 m 3 contains a each of area 1m2 and thickness 0.01 m separated by a
mixture of neon and argon gases, at a temperature of 0.05 m thick stagnant air space. In the steady state, the
27eC and pressure of 1 x 105 Nm -2. The total mass of room glass inter-face and the glass-outdoor interface
the mixture is 28 g. If the molar masses of neon and are at constant temperatures of 27°C and DoC
argon are 20 and 40 g mol-1 respectively, find the respectively. Calculate the rate of heat flow through
masses of the individual gases in the container the window pane. Also find the temperature of other
assuming them to be ideal (Universal gas comant interfaces. Given thermal conductivities of glass and
R = 8.314J/mol-K) (liT 1994) air as 0.8 and 0.08 W m -lK-I respectively.
24. A gaseous mixture enclosed in a vessel of volume V (lIT 1997)
consists of one mole of a gas A with y(= epic v) = 5/3

www.puucho.com
Study Anurag Mishra with www.puucho.com
289
PREVIOUS YEAR PROBLEMS
33. A solid body X of heat capacity C is kept in an
29. A sample of 2 kg of monoatomic p
helium (assumed idea\) is taken atmosphere whose temperature is TA = 300 K.At time
through the process ABC and 10' __
8 C t = 0 the temperature of X is To = 400 K. It cools
another sample of 2 kg of the
same gas is taken through the A 0 P
according to Newton's law of cooling. At time t}> its
temperature is found to be 350 K.
process ADC (see figure). Given At this time (tl), the body X is connected to a large box
3
molecular mass of helium == 4 10 20 V(m ) Y at atmospheric temperature T A, through a
(i) What is the temperature of helium in each of the
conducting rod of length L, cross.sectional area A and
thermal conductivity K. The heat capacity of Y is so
s£ates A, B,C and D?
large that any variation in its temperature may be
(ii) Is there any way of telling afterwards which
neglected. The cross.sectional area A of the
sample of helium went through the process ABC
connecting rod is small compared to the surface area
and which went through the process ADe? Write
of X. Find the temperature of X at time t = 3tl.
yes or no. (lIT 1998)
(iii) How much is the heat involved in each process
34. Two moles of an ideal monatomic gas, initially at
ABC and ADC? (lIT 1997)
pressure 11 and volume VI' undergo an adiabatic
30. One mole of a diatomic ideal gas (y == 1.4) is taken compression until its volume is V2. Then the gas is
through a cyclic process starting from point A. The given heat Q at constant volume V2.
process A ---t B is an adiabatic compression, B -7 C is (a) Sketch the complete process on a P-V diagram.
isobanc expansion, C -+ D is an adiabatic expansion, (b) Find the total workdone by the gas, the total
and D -+ A is isochoric. The volume ratios are change in its internal energy and the final
VA/VB = 16and VelV8 = 2and the temperature at A temperature of the gas. [Give your answer in
is TA = 300 K. Calculate the rem perature of the gas at terms 11 , \1, V2,QandRj. (lIT 1999)
the points B andDand find the efficiency of the cycle. 35. Two moles of an ideal p
monoatomic gas taken 2P, .~C
(lIT 1997)
31. The apparatus :~:~~h ai~yclet:CA.p~; P .1... A
shown in the figure . diagram. During the
consists of four A: : B :c D process AB, pressure and
glass
D

columns 9S C ~.'-:-:-:-:-:-.
~ soc L~. 5°C temperature of the gas
vary such tha't PT =
Tt 2T1 p

connected by
constant. IfTI = 300K, calculate
horizontal sections. The height of two central columns
{al the workdone on the gas in the process ABand
Band Cafe 49 cm each. The twO outer columns A and (b) the heat absorbed or released by the gas in each of
D are open to the atmosphere. A and C are maintained the processes.
at a temperature of 9S C while the columns Band D
Q

Give answer in terms of the gas constant R.


are maintained at SQc. The height of the liquid in A (IlT 2000)
and D measured from the base are 52.8 em and 51 em 36. An ice cube of mass 0.1 kg at DoC is placed in an
respectively. Determine the coefficient of thermal isolated container which is at 227"C. The specific heat
expansion of the liquid. (lIT 1997) s of the container varies with temperature T according
to the empirical relation s = A + BT, where
32. One mole of an ideal
A = 100caVkg.K and B:= 2 x 10~2 caVkg.K2. If the
p
B
monoatomic gas is taken 3Po
round the cyclic process G final temperature of the container. (Latent heat of
ABCA as shown in figure.
Calculate
pO ..'" ...
J--~ fusion of water = 8 x 104 caVkg, Specific heat of
water= 103 caVkg-K) (lIT 2001)
(a) the work done by the 37. A 5 m long cylindrical steel wire with radius 2 x 10-3m
gas. Vo 2Vo V is suspended vartically from a rigid support and
(b) the heat rejected by the gas in the path CA.and the carries a bob of mass 100 kg at the other end. If the
heat absorbed by the gas in the p<lth AB; bob gets snapped, calculate the change in temperature
(c) the net heat absorbed by the gas in the path Be; of the wire ignoring radiation losses. (For the steel
(d) the maximum temperature attained by the gas wire: Young's modulus := 2.1 x 1011 Pa; Density
during the cycle. (lIT 1998)

www.puucho.com
Study Anurag Mishra with www.puucho.com

THERMODYNAMICS
=- 7860kg/m3; Specific heae = 420J/kg-K). "'42. A diatomic gas is enclosed in a
(lIT 2001) vessel fitted with massless ~
38. A monoatomic ideal gas of two moles is taken through
movable piston. Area of cross 1m
2
II m

-'":P
a cyclic process starring from A as shown in figure. The section of vessel is 1 m 2. Initial
volume rarios are f P D C height of the piston 1 m(see the
D D
VB = 2 and V =- 4. If V v figure). The initial temperature of
VA VA , the gas is 300 K. The temperature of the gas is
v'-_ ...- -_ ...., i
the temperature TA at A V
is 27"C. Calculate,
,-
~ jA
.
i
. increased to 400 K. keeping pressure constant.
Calculate the new height of the piston. The piston is
(a) the temperature of 0 TA TB T brought to its initial position with no heat exchange.
the gas at point B, Calculate the final temperature of the gas. You can
(b) heat absorbed or released by the gas in each leave answer in fraction. (lIT 2004)
process, 43. A cubical block of coefficient of linear expansion as is
(e) the total work done by the gas during the complete submerged partially inside a liquid of coefficient of
cycle. volume expansion y /. On increasing the temperature
Express your answer in terms of the gas constant R. of the system IJ.T, the height of the cube inside the
(lIT 2001) liquid remains unchanged. Find the relation between
39. A cubical box of side 1 meter contains helium gas as andYl' (lIT 2004)
(atomic weight 4) at a pressure of 100 N/m 2. During .44.' A cylindrical rod of length I, thermal conductivity K
an obselVation time of 1 second. an atom travelling and area of
cross,section A
with the root mean square speed parallel to one of the
has one end in
edges of the cube, was found to make 500 hits with a
the furnace at Insulation
panicular wall, withom any collision with other temperature T1
atoms. Take R """(25/3) J/mol-K and k = 1.38 x 10-23 and the other
J/K. end in Furnace
(a) Evaluate the temperature of the gas. surrounding at temperature T2. Surface of the rod
(b) Evaluate the average kinetic energy per atom. exposed to the surrounding has emissivity e. Also
(c) Evaluate the total mass of helium gas in the hoox. T2 '"'Ts +IJ.TandTs »IJ.T.
(lIT 2002)
IfTt - Ts ex: IJ.T. find the proportionality constant.
40. An insulated container containing monoatomic gas of (liT 2004)
molar mass M is moving with a velocity va. If the 45. A small spherical body of radius r is falling under
container is suddenly stopped, find the change in gravity in a viscous medium. Due to friction the
temperature. medium gets heated. How does the rate of heating
(lIT 2003) T{= 12'?C depend on radius of body when it attains terminal
41. Hot oil is circulated through velocity? (lIT 2004)
an insulated container with 46. A cylinder of mass 1 kg is given heat of 20,000 J at
a wooden lid at the top
whose
K = 0.149
thickness
emissivity
conductivity _
J/(m.oC-sec),
t = 5 mm,
= 0.6.
U
To Ta=27"C.
-Hot Oil
atmospheric pressure. If initially the temperature of
cylinder is 20"e, find
(a) final temperature of the cylinder.
(b) workdone by the cylinder.
(c) change in internal energy of the cylinder
Temperature of the top of (Given that specific heat of cylinder
the lid is maintained at T1 = 127°C. The ambient = 400J kg -1 °c-1• coefficient of volume expansion
temperature Ta """27°C. Calculate = 9x lO-SoC-I, Atmospheric pressure'"' 105 N/m2
(a) rate of heat loss per unit area due to radiation and Density of cylinder= 9000kgim 3). (lIT 2005)
from the lid.
47. 0.05 kg steam at 373 K and 0,45 kg of ice at 253 Kare
(b) temperature of the oil. mixed in an insulated vessel. Find the equilibrium
(Givena',", 17 xlO-8Wm-2K-4) (lIT 2003) temperature of the mixture,
3 =
Given, Lfusion 80 caVg = 336J/g,
Lvaporisalion= 540 caVg = 2268J/g,
sice = 2100Jlkg K= 0.5 caVg K
andsWatt"r = 4200J/kg K = lcaVg K (liT 2006)

www.puucho.com
Study Anurag Mishra with www.puucho.com
291
PREVIOUS YEAR PROBLEMS
ANSWERS

== Only One Alternative is Correct


(d) 3. (a) 4. (a) 5. (d) 6. (b) 7. (d) 8. (b) 9. (c) 10. (c)
1. (b) 2.
14. (d) 15. (c) 16. (b) 17. (d) 18. (a) 19. (a) 20. (c)
11. (d) 12. (d) 13. (a)

(b) 23. (a) 24. (b) 25. (a) 26. (a) 27. (a) 28. (c) 29. (b) 30. (b)
21. (b) 22.
(b) 34. (d) 35. (c) 36. (a) 37. (c) 38. (d) 39. (c) 40. (a)
31. (c) 32. (b) 33.

=41.

1. (a)
(c)

More than One Alternatives is/are Correct

2. (b) 3. (a) 4. (c)


,", ,"

5. (b)
6. (a. b, e, d) 7. (d) 8. (b) 9. (a, b, d) 10. (a, b)
11. (b) 12. (c) 13. (b) 14, (d) 15. (b, c)
16. (d) 17. (a) 18. (c, d) 19. (b, d) , 20. (a, d)

=21. (b, d)

True/False
1. False
22. (b, d)

2. False 3. False 4. True 5. True

= 6. False

Fill in the Blanks

1. 2R
7. False

2. Panly solid and partly liquid 3 • .JZf 4. 0


4rrR2KT 8. P xt 9. 300 K
5. 5803 6. 1.71 prs second 7.
P M

= 10. 0.628

Matching Typ:e Problems


11. 60"C

1. a~ q; b~ p,s; c--+ s; d--+ q,r


12. 192

2. a-+,s; b-+q; c--+p;


13. 45.6

d-+r
14. 5.5

3. a--+ q: b-l' p,r; c--+ p.s; d -+ q,s

== Assertion and Reason


= 1. (b)

Integer Answer Type Peoblems

= 1. 9

Subjective Problems
2. 8

(W,-W1)
2. --~-~--+..
~(WO-Wl)
.
CWo - Wz)(Tz - T1) CWo - Wz)
5.817mmofHg 6. p. V diagram 7. 12.96 m/s
4. See solution .'
8. -973.1J 9. Hollow sphere

www.puucho.com
Study Anurag Mishra with www.puucho.com

KS
27
10. (i) 1.97 X 10 (ii) 35.6 m/s 11. (i) 12.9[0. 2.25To_ (ij) -15.58To joule
12. 83.75 cm ofHg 13.9000W 14. 67SK,3.6xIQ6N/m2
15. (ii) 113 litre, 0.44 x 105 N/m 2 , (iii) 12450J 16. 800 K, 720 J
17. (i) 5, (ii) 1.25 PV 18. (b) 0.58 RTA, 0.58 RTA
19. (a) 1152 J (b) 1152 J ee) zero 20. 166.38 sec, 21. (i) 1870 J. (ij) -5298J, (iii) 500 K
22. (i) 765 J, (ij) 10.82% 23. Neon:= 4g;Argon = 24g
24. (a) 2 moles (b) 401 mis, (e) 0.167%, Cd)- 8.27 x 10-5 V', 25. (0 189 K, (ii) - 2767 J, (iii) 2767 J
26. 12 g 27. (i) 40 kg (ij) 0.1 J 28. 41.54 J/s. 26.48°C, a.S2°e
29. (OTA = 120.3K, TB = 240.6K,T, = 481.3 K,T = 240.6K; (ij) No; (iii)QABC ='3.25X106J,QADC = 2.75x 106J
v
30. TB =909K,TD = 791.4K,TJ = 61.4% 31. 6.67xlO-5peroC

32. (a) p, v. (b) - SPaVo 3P. Yo 'Ce) PaVo Cd) 2SVoPo 33. [300+ 12.5e ~tl ] kelvin
o o. 2' 0 0- 2 SR

2
3 [
34. (b)-lJ.1\ 1- -'(V, )"3} Q--lJ.1\
V2
3 [1- (1\).3]
_
2
, lJ. v."3v.-.3
V2
' 2
2R
Q
+_
3R
35. (a) + 1200 R, (b) -2100R, 1500R, 831.8R 36. 0.495 kg 37. 0.00457'C
38. (a) 600 K, (b) 1500 R, 831.8 R,- 900R,- 831.8R. ee) 600 R 39. (a) 160 K, (b) 3.312x 10-21 J, ee) 0.3 gram
Mv 0
40. __
2
41. (a) 595 W/m2, (b) 420 K 4
42. _ m, 400 _ (4)2,'5
K
~ 3 3
4ealTs3
43. 'II = 2as 44. 1 + - __ 45. Rate of heat produced oc: r5
K
46. Ca) 70'C, (b) 0.05 J,(c) 19999.95 J . 47. 273 Kor DOC

www.puucho.com
Study Anurag Mishra with www.puucho.com
THERMODYItIMKS

Solution

4
Only One Alternative Is COrrect
or ~T eaAy = or 6T = (ecrT4)~
Mms 6t sm

1. (bl Let a be the temperature of interface, For a sphere, m = ~ 1tr3p or r = ( 3m )1:3

As (~;t(~;t =
:. Area of sphere. A = 41tr2
3

or A =
41tp

4Jt( 4rrp
3m )
2'
fiT1 6.T2
.. K1Ax-=K2A--

(2/(2)(36-0) "" =K ""


(0-0)
.. Rate of cooling

""
72-213=8
2

"" (~~) = (eo;4 )4rrx(4~J' m:'


.. 0=24"C S = (constant) x m-l'3
:. Temperature

2. (d) Charle's law.


difference across layer
A = 36- 24 =' 12°C
.. ;: = (:~ rV3 13

3. (al Work done in a cyclic process =


Area enclosed by cycle or ;: = (3:r =(~r3
:. Work done = AB x Be
Of W = (2P - P)(2V - V) or W = PV
6V 1 dV
.. ~~ = (~r3
4. (al As6V=V.yt.T ~ Y=--=-'-
V.t.T V dT 6. (b) For heat conduction.
At constant pressure,
V
- = constant
T
=C "Q =KA("T)
dV VldVI A(T)
'" I
;::} V=C.T => -=C=-=>-'-=-
dT TVdTT
=> y = 1.. so at a given temperature y for all gases
T
is same.
(b) The average translational kinetic energy per
molecule is ~ kT.
2
(c) When pressure increases, mean free path of 90"
molecule decreases. B c
;f2n (Td
Cd) Average K.E. does not depend on the gas.
Hence each component in a gaseous mixture has
Since B is at higher temperature than A, heat flows
same average translational kinetic energy. So (d)
from B to A, A to C and then C to B, for steady
is correct.
4 state.
5. (d) According to Stefan's law, 6Q = eaAT M
Also, 6Q = ms 6T .'. ms 6T = eaAT4 M

www.puucho.com
Study Anurag Mishra with www.puucho.com

FarAe, ~Q =KA(r-rc)
at .J'2n. or P, =('1/2)' x(Z!j)4
450 l Ii
ForCB, dQ = KA (Tc -
M l
a
-rzr) or P2
r}

=450X(~)x(\6)=450X4
Equate the two equations for steady state. . ., or P2 = laDOW .
... KA(Th:C)=KA(TC-/Zf) 12. (d) dU=CvdT=(3:JdT
or T-Tc =.J2Tc-Zf = ~ x (8.3) x (l00) = 1.25 x 103 J
2
or 3T=TdJ2+1)or Tc = 3
T (.J2+I)
13. (al T,
7. (d) Average K. E. (K) = ~ kT ~=I--
2 '.' 1 I', •
T1
K1 _ 600 ('21 = 1 _ (127 + 273) = 1_ 400 = ~
=- - -1i or K-2 _ T2 K-1 -_ -x
- -x ~ x 10-'1)
K2 T2 1i 300 (227 + 273) 500 5
.. W = llClJ. =.!-5 X 104 = 2000J :. W = 2000J.
or ](2 =12.42xlO-Z1J
14. (d) nfRT
Internal energy = __
Vrrru = /3kT, ~ = fG 2
\ m vI vT; where n = number of mole of the gas
or V2 = {T; XVI = ~600 x484 =
f number of degree of freedom
vIi 300 .. For 2 mole of oxygen, U0 = 2x5xRT - SRT
= J2 x 484 = lA14x 484 = 684mjs 2
8. (bl According to Wien's displacement law, Again, for 4 mole of argon,
AmY = constant (b). Let VA = 4x3xRT =6RT
5 = Sun,NS = North star, 2
.. Total internal energy = U 0 + UA
.. o..m)STs = (Am)NSTNS
= SRT+ 6RT = llRT

9. (e)
..

-
Ts =o.'m)NS
(A'm h
TNS
3
=350=0.69
SlO 15. (e) Velocity of sound in a gas v= flt-
Average K. E. (K) = - kT
2
It depends on temperature and does not depend
for v = JrRT
dV
= ~YRT
M
(':densityxvolume= M)
on molar mass.
For both the gases, average translational VN2 YN2 MHe
o'. --= --x __
kinetic energy will be same viz 0.048 eV. VHe "tHe MN2
10. (e) pv= nRT
IJ. T1 where YN2 = 7/5, "tHe = 5/3
-=- C",'V, nand R are same)
P T z 2 7/5 4 J3
= -x-=_.
P T 5/3 28 5
-=- or Pz =2P
P, Zf
11. (d) According to Stefan's law,
16. (b) Velocity of sound in a gas v :,. flt-
Energy radiated E = eAoy4(.6.t)
or v = ~YRT whered =densityofgas,
•. .!.=ex(41tr2)xor4 dv
at
or Power (P) = (4necr)r2 4 r or v = jrRT (","dV .e M)

r r
'M

i {~ x[~~ .. .:'L =
tl2
JMM}2 as y, R, T are constant factors.

www.puucho.com
Study Anurag Mishra with www.puucho.com
THE

17. (d) For an adiabatic process, IVy-1 = constant


VI =~,V2 =AL2
where A denotes area of cross-section of the gas
cylinder.

.• T1 (V,)3
_ = _
~-1 .
,as y = -5 for monoatomtC. gas.
O"C O'C

Tz VI 3 A 2Q B

.!L = (L, )2'3 Q


or
AB-=BD=BE=/
Tz L}
18. (a) The change of ice at _lOoe into steam at lOO<>C
occurs in four stages: It is represented by curve
(a).
I stage- The temperature of ice changes from Heat will flow by conduction along the rods from
90° e ends to junction and from junction to 0° C
-lOoe roO°C.
end.
IV KA(90-e)
lOO"C Steam Along rod DB, Q = l ... (1)

AlongEB,Q = KA(9~-e) ... (2)

AlongBA,heat=2Q :.2Q=KA(8-0) ...(3)


I
or from eq. (1) and (3) eliminate Q.
2KA(90-e) KA(e)

Ice II
I 1
or 2(90-8) = e or3e = 180 or e = 60°C
23. (a) According to first law of thermodynamics,
II stage- Ice at aoc changes in water at Doe. dQ=dU+dW or dQ=dU+O (':dW=o,
The state changes as heat is supplied. given)
III stage- Water at DOGchange into water at or dQ=dU
100°C. since dQ < 0 as per the question,
IV stage- Water at 100°C changes into steam at :. dQ is negative
:. dU is also negative. Thus internal energy
100°C.
19. (a) The work done is equal to area under the curve decreases.
and volume axis 00 the P-V diagram. :. Internal energy decreases when the
temperature decreases
Obviously W2 > WI > W3.
Hence the temperature will decrease.
20. (e) For a given mass of gas at constant pressure,
V V+.6.V V 24. (b) For adiabatic process, pv"Y = constant
- = constant .. --- =-
T T +.1.T T p
or VT+V6T=VT+T6V or V6T:T6V
1 6V 1
or _= __ or -=0 or oT=l
T V6T T
The equation represents a rectangular hyperbola
ofthefonnxy = ,2.
Hence the &-T variation is represented by graph
(e). V
21. (b) According to Wien's displacement law, V
AT = constant.
For He gas, '{ = 1.67
From!'A graph, Al < A3 < A2'
Helium is a monoatomic gas,
Hence T1 > T3 > T2.
For oxygen gas, '{ = 1.4
22. (b) Let the temperature of junction be 8°e.

www.puucho.com
Study Anurag Mishra with www.puucho.com
PROBIIMS 296,
Oxygen is diatomic gas From graph, consider cooling of x and y from
For a given value of v, temperature T2 to TI.
11 > P2' as is clear from graph. Time taken by x = (t2 - (1)
Since PV'Y = constant. Time taken by y = (£3 -tl)
:. 11 V'YI = P2VY2 or 1.. = V'Y2 = (V)Y2-Yl x takes less time and soEx > Ey ...(1)
P2 V'YI According to Kirchhoff s law, a good emitter is also
since 11 > P2 :. 12 > 11 since YHe > 10
a good absorber.
2
,', Curve 2 represents helium Cmonoatomic) .. ax > ay ...(2)
and Curve 1 represents oxygen (diatomic) 29. (b) Heat lost by water at 20"C = msw liT
H = 5 x 1 x 20 = lOOkcal
,", Plot 1 is for oxygen, Plot 2 is for helium.
25. (a) At first, ice at (-20"C)will take heat to change into
At a constant temperaNre, for a given mass of gas, ice at DOC.
=
PV constant, according to Boyle's law.
H = msiceliT = (2kg) x O.5 x 20 = 20 kcal
,', PV = constant :. Pdv + VdP = 0 :. After this, heat available = (100 - 20) = SOkcal
dV V dV 1
or -=-- or - __ =+_ This heat will now be gained by ice at O"C to melt
dPP VdPP into water at 0" C. Let m kg of ice melt.
1
or ~=- or /3P=l. .'. m x SO = SO :. m = lkg.
P
Out of 2 kg of ice, 1 kg of ice melts into water and
The equation between /3and P is of the form xy =
1 kg of ice remains unmelted in container.
constant which represents a rectangular
hyperbola. :. Amount of water in container = 5 + 1 = 6kg.
,', Graph between p and P will be a rectangular 30. (b) An inspection of the P.T graph reveals that
hyperbola represented by graph (a). (1) AC is an adiabatic process (given)
26. (a) For cyclic process ABCA, (2) AB is an isothermal process at T constant.
Q=WAB+WBC+WD\. (,; (3) BC is an isobaric process at P constant.
WAH = P x.1.V lOx 1) = (i) In all the four options, BC is an isobaric process.
5 = 10+ 0+ W~ (ii) AB is an isothermal process.
or Wo. = 5-10 or WQ\ = -SJ. :. PV = constant
27. (a) According [0 Kirchhoffs law, good absorbers are :. Graph between P and V is a rectangular
good emitters and bad reflectors. While at lower hyperbola of the form .ry = constant.
temperature, a black-body absorbs all the incident Graphs (b) and (d) satisfy this condition.
radiations. Ie does not reflect any radiation Options (a) and (c) are therefore incorrect.
incident upon it when it is thrown in the surface. (iii) AC is an adiabatic process (given).
Initially it is therefore the darkest body. In P-V diagram, slope of AC > slope of AB because
At later times, the black body attains the slope of adiabatic curve > slope of isothermal
temperature of the hot furnace and so it radiates curve. This condition is certainly not satisfied in
maximum energy. Ie becomes the brightest of all. option (d) where AC is a straight line at constant
Option (a) represents the answer. V. Here AC is an isochoric process. Ie is not an
28. (e) The two bodies x and y have same surface area. adiabatic process. (d) is incorrect.
Emissivity (E)oc rate of cooling Hence the ultimate correct process is (b).
T 31. (e) By coefficient of linear expansion,

° =~,
L x liT
whereM = change in length.
.. M =aLliT
,,,
,, y .. Ma =Oa1]dT and M$ =0$12liT
,:' Ma = M$ or 0a1IliT = 0sl2liT
TI ------.----.
:, or 12
-=~ ° 12
=>-+1=~+1 °
, 'I 05 11 0$
,,
or 12+1]
-_=___0a+os 11
.. __ = 0$_

Eoc-(~~)" " " II 05 11+12 ll.a+o$


or 32. (b) According to Wien's displacement law,
AT = b = Wien's constant.

www.puucho.com
Study Anurag Mishra with www.puucho.com ' .•
297,' THERMODYNAMICS ..•,
.. AATA = b or TA = ----
b Further I WHel >1 WAn I ..\•
b X 10 7
3 X 10-7 Net work done = WAn + (-WBC)
W = A negative value (.: WBC > WAB)
..
"

TA = =_ Z
wherez=(bx107)
3 3 W <0
7 From the graph itself, P3 > 11 :. W < Oand P3 > 11
'J I TB =---
S'lffilary b x 10 = Z
an d Hence option (b) represents correct ans\ver.
4 4
34. (d) In parallel combination of rods,
_bXI07 Z
T c- resultant conductivity K p = K} + K 2
S 5 Kp =K+K=2K
Again, According to Stefan's law,
[n series combination, Ks
Q ::::Power radiated by black body = AcrT4
where A = area of disc = 1tR2. K
s =K xK K =
,,
(~r
QA = (7tR~) xa x (TA)4 2K 2
(2K)A(lOO - 0) 2KA x 100
.. q} "" --~--~ = ----
or QA = 1[(2x 10-2)2 xcr x I I 1
_(KJAOOO-O) _ KAxlOO "

2' q2 2 I - 21 '"
.•
'
QA =1(OX10-4xz4x-
34 q2 = KA x 100 x I , ~ = .!. •j
-4 4 4 q} 21 2KA x 100 ql 4
or QA = (rrO"x10 xz )x-
81 35. (el (i) Oxygen supplied is in liquid state. When heat is
,
Put nO"x 10-4 x z4 = k = constant

supplied at constant rate, at first liquid oxygen at 1
or QA
4k
= -81
= o.049k
50 K will be raised to its boiling temperature, say
OK, ..I
Similarly, QB = k x (4)2
(4)4
= l5.. ::::O.062k
16
.. Q = ms(8 - 50) = msTl, where TI = (8 - 50) K
Q msT}
1
j
or -~- =-- •
time r
Qc = kx(6)' = 36k =0.057k
(t)


(5)4 625 g = constant, as heat is supplied at constant rate.
r
Hence QB is maximum.
• msTI = (constant)xt
33. (bl The slope of adiabatic process, at a given state Hence (T. r) graph will be a stra-ight -line in this
(P, V, T) is greater than the slope of isothermal
region. It is inclined to time axis.
process. Hence AB is isothermal and Be is
(ii) At boiling poim, the heat supplied is used in
adiabatic. Work done in a process is given by area
converting liquid oxygen into gaseous state,
under the curve and V-axis.
keeping temperature at e K.
p
Ql=mL
c rr -t)graph vvill be a straight line parallel to taxis
as heat is supplied at constant rate.
(iii) When the vaporisation is complete, the gas at .
"
,
. e will now be raised to 300 K. •
p~---•.••. 1------ B "
Q2 = III X Sgas x (300 - e) l
,,
,, Heat Q2 .
. IS constant. Heat is supplied at constant
1
,, v time i
vi rate. l
VI
.. msgas x(300-8)= (constant)t '.
The work done W is positive if volume increases in '"
',f
the process and W is negative if volume decreases' or msgas x T2 = (constant)r
I
:. (T- t) graphs will again be a straight line :;
in the process. "
inclined to time axis. Hence option (c) represents "
Hence WAB is positive as volume increases from "l
to V2. WBC is negative as volume decreases from
36. (a)
the temperature-time graph.
According to Wien's displacemem law,
••••
V2 coVl.
AT = constant i
,')
:$
,~ •

www.puucho.com
Study Anurag Mishra with www.puucho.com
PREVIOUS YEAR PROBLEMS
E,.
On d'f' ..
I lerennatmg we get 3 _dT = _.
dV
T V
Therefore, the coefficient of volume expansion
I dV 3
~T3 --=-
V dT T
--1:
,, , ' ~T2
T
: I
More than One Alternatives is/are Correct

).
1. (a)': For a gas Urms::: FRT
M
From the graph given, 3RT
:. M:= ---, T :::room temperature = 27 C 0

A3 <A2 <1'1 (vmz.~)2


.. T3 >T2 >T1
= 300K
Temperature of sun is higher than that of welding
M: 3x 8.3x 300 kg : 7470 x 1000 g
arc which is higher than that of tungsten
filamenvlamp. (930)' 37249 x 100
.. Sun = T3, Welding arc = T2 M:2g .. Thus, the gas isHz.
Tungsten filament = T1 2. (b) Q = nCpdT

37. (el According to Stefan's law, energy radiated per Cp :..!L..: 70 :~:7 cal
T
second by a body of emissivity at a temperature ndT 2x (35- 30) 2x 5 mol x K
is eaAT4; irrespective of surroundings cal
Now,Cy :::Cp-R or Cy :7-2:5---
.. E = UfueAT4. mol x K
38. (d) Warming of glass of bulb due to filament is due (Q .. Q':nCydT:2x5x5:50cal.
radiation primarily. 3. (a) Heat lost by steam"" Heat gained by water
For convections process, a medium is required
+ calorimeter
which can move between two points to transfer
heat. Bulb is almost evacuated to have no medium. .. mL+ ms(lOO- 80): 1.12xsx (80-15)
Option (d) represents the answer. or m[540+(lx20)]:112x1x65
39. (e) Energy required in heating water = ms9 or m:
1.12x1x65: 65 kgorm:0.13kg.
Q = 2x 4200x (77 - 27) 560 500
Q=2x4200x50J ...(1) 4. (c) Let K : thermal conductivity of the system.
'j energy available =KMT
F rom
.. Q
COl , --- _
sec
I'
= Power of coil- Power lost Q::: K x n(2R)2 x I1T ::: 4K1tR211T
... (1)
PA lOOOW-160W
::: I I
PA ::: 840W, let time taken::: l For inner cylinder,
:. EnergyQ::: PA Xl::: B40r ... (2) _ K1All1T
QI -
or From (1) and (2), B40r :::2 X 4200 x 50 I
2x 4200 x 50 . 2
:. Q : Kl x 1tR x I1T : Kl1tR2I1T
or l::: -----::: 500 sec::: Bmm 20 sec ...(2)
840 l
I I
.. Time taken::: 8 min 20 sec. For outer cylinder,
40. (a) 1 calorie is the amount of heat required to raise Q2 : K2A211T
temperature of 1 g of water from 14.5°C to 15. 5°C I
at 760 mm ofHg. Hence correct option is (a). 2 tiT
:. Q, = K, x.1(ZR) - (R)2 I-I
41. (c) The gas expands such that PT2 :::C
Using the ideal gas equation PV ::: nRT inPT2 :::C, 3K2nR211T
...(3)
(~T)T2:::c ~T3 o<V. I
Q=<l}+Q,

www.puucho.com
Study Anurag Mishra with www.puucho.com

299
4K1tRZt1T Kl~R26T 3K2nRZdT obviously Ai > A2 :. W1 > W2
~---+----- Option (a) is correct.
I I I
(b) To study V.T diagram.
or 4K=K1+3Kz orK=K1+3Kz
4 In the given process, AB is a straight line. If has a
negative slope and a positive intercept.
5. (b) For a rnonoatomic gas C V, = 3R
...
2
,
The equation ofline is P = -aV + p ... (1)
. . C 5R Where a and P are positive constants .
For d laromle gas, v =-
, 2 .. PV=nRT ... (2)
3R 5R
1x-+1x- .. PnRTp"
= --. ut It In eq. ()1
2 2 =2R V
(GV)mixture
nl +"2 1+1
.. Equation, ~T = -aV +13
(C P )mixture = (C v )mixture + R
2
=2R+R=3R or T = _aV +'W ... (3)
. _(Cp)mixture_3R_15 ,JR nR
Ymixture - - - - . This represents a parabola in terms ofT and V.
(C V )miXlure 2R
:. The path AB becomes a part of parabola.
6. (a. b, c, d)
(a) !lU = nCvD-T = nCv(Tz -TI)
Option (b) is correct.
(b) In an adiabatic process t1Q = a (c) To study P.T diagram
For P.T diagram. eliminate V between (1) and (2)
.. liQ=liU+6W
,JRT 2
or 0= t.U +.dW P = -a-+~ or P -P~ = -Q.nRT
P
or IM~=I~WI _p2 pp
(e) In an isothermal process, tJ.T = 0 T=--+-- ... (4)
.. 6U=O (:.lJ.U=nCv!1T) anR allR
Cd) In an adiabatic process, tlQ = 0 This represents a parabola in terms ofT and P.
Hence all the four options are correct. :. The path AB becomes a part of parabola.
:. The option (c) is not correct as AB does not a part
7. (d) .1U = nCvdT
of hyperbola.
tlQ = nCp!l.T :.
(d) Variation ofT along AB
6U nCvdT Gv 1 5 2
6Q= nCpdT=Cp ='1=7 From (3). T = -aV + pV
oR nR
8. (b) According to Maxwell's distribution of Ted of
dT = -2nV +1- ... (5)
gas, the average speed of molecules v= SRT dV nR oR
rrM
.. v ft, for a definite
gas.
When dT =0, V=1- ... (6)
00: dV 2n
Since temperature of A andC vessels are same, the d2r -2a -2u
--=--+0=--
average speed of oxygen (°2) molecules will be dV2 oR oR
equal in A and C. 2
. . d T.. h
Average speed of O2 molecules in vessel Th e secon d d envatlve -- IS negatlve. It means T as
dV2
C=l'l'
some maximum value.
9. (a, b, d) (a)
p p V = !.is the value of maximaof temperature.
2n
P
A
P ........ A Also PAVA = PBVB ,RTA = RTn orTA = TB
In going from A to B, the temperature of the gas first
P/2 P/2 ........ B
increases to maximum at V -'" 13/2a
Then the temperantre decreases and resorted to
v original value.
Hence option (d) is correct.
Givcn prucess WI lsothennal process W~ 10. (a, b)
(i) (ii)
(a) To calculate temperature of B
Work done = Area under curve and V.axis.

www.puucho.com
Study Anurag Mishra with www.puucho.com

PREVIOUS YIAll PIOILIMS 3011i

Power of bpdy A = eAaT~x (area) Bulk modulus = -(-P.6V) = P


Power of body B = eBaT 84 x (area) ~V
:. Isothermal Bulk modulus.= P.
The cwo powers are equal.
14. (d) The piswn of A is free to move. Volume of gas in
.. eBaT: x (area) = eAaTJ x (area)
cylinder A will change hut pressure •••• ;11 remain-
..
or ri = (:: )rJ or
constant. Heat will be gained by the gas at constant
pressure.

ri = (0.01) x (5802)4 :. (~Q)A = nCp(~TlA ...(1)

(~r
0.B1 The piston of Bis held fixed. Heat will be gained by the
gas at constant volume.
or ri = x (5802)4 or Ta = 58 °2 :. (~Q). = nCy (<lTl.
3
Given (.6Q)A = (.6Q)B :. nCp(.6T)A'= nCV(.1T)B
or Ta = 1934 K :. Option (a) is correct.
(b) According to Wien's displacement law, or (.6T)s .=(~~ )<.6T)A or (.6T)B =lAx30
AT=constant .. AATA =ABTB
AA Ta AA _ 1934 (.: Cp .= 1.4fordiatomicgas)
0' AB ""'TA or AB - 5802 Cy
or (.1T) B = 42 K.
0< AA =.!. or AB =3AA 15. (b, c)
A. 3
(a) Positive work is done hy the system if volume
Given:"-B -AA =l.OxlO-6m increases in the process. When ice at 273 K melts into
or 3AA - AA = 10-6 or 2AA = 10-6 water at 273 K, there is a decrease in volume. Hence
work done by the system is not positive hut negative.
or AA = 0.5 x 10-6 and
Option (a) is not correct.
AB =3xO.5xlO-6
(b) Since the melting of ice involves a decrease in
or Ag=l5Xl0-6m. volume, positive work is done on the ice-water system
Hence option (b) is correct. hy me atmosphere.
Option (h) is correct.
11. (b) vrrns = ~3~T ..
=RF (c) According to first law of thermodynamics,

.. ---v
(vrmsh 480
t20
= --
or (vmlSh = 2v
dQ=dU+dW :. dU=dQ-dlV
dQ is positive as ice absorbs heat in melting.
dW is negative as volume decreases in mehing of ice.
12. (e) For containers A and B, the initial and final :. dU is positive in ice-water system.
volumes are V and ZV, under isothermal conditions. T :. Internal energy increases.
remains constant. Hence option (c) is correct.
According to gas equation, Option (d) is not correct as (c) holds good.
PV= m RT ... (1) 16. (d)
M
According to Wien's displacement law,
P.1V + VM = 0
Am T = Wien's constant (b)
0< M' =_p( 11:) or M= -P(V;2V) = p
0< l1P = :; = m(:) = constant xm

(M)A rnA liP rnA


.. ---=-- 0, --=-
(M)B rnB 151iP rnB
or 3rnA = 2mB,

13. (b) Bulk modulus = ---M'V ... (1)


tN
For an isomermal process, PV = constant
:. P.1V + VM =
a or VIiP -P.6V = ...(2)
or
b 2.88x106nmK
Am .=-.=
From 0) and (2), T 2BBOK

www.puucho.com
Study Anurag Mishra with www.puucho.com

301

or Am 0:: 1000 om or R + d = (I + aBuT)(l- ac6T), by binomial


.. Energy of radiation is maximum at A m == 1000 om R
Thus U2 > VI,U2 > U3 theorem.
d
VI to 0, U2 7;-0 or 1 + - = 1 + (aR - ac )ilT - Smaller terms
.. Option Cd) is correct. R
d d
17. (a) According to law of equilibrium of energy. the or -=(aB-ac)6T or R=-----
R (aB-ac)6T
average kinetic energy per molecule per degree of
1
freedom is..! kn T. Since oxygen and nitrogen, both, arc R~-.
2 l>.T
diatomic gases and they are contained in a vessel at Option (b) is correct.
300 K, they will have the same number of rotational and Roc 1
degree of freedom 1. e., nva. Ian -acl
Hence the average rotational K.E. per molecule of Option (d) is correct.
each gas 20, Ca, d)
kBT
;;:;2x-=k BT From Stefan's law, the rate of emission of energy from
2 a black bo~y is proportional to (T - To) 4 . Since T and
Thus the ratio will be 1 : 1 .
To remain constant, the rate of emission of energy
18. (c, d)
remains the same. Since the temperature of the black
Ii = ~8kBT, Pml.\ = )3knT, v p = \/2kBT body remains constant, the energy radiated by it =
1m, m m energy absorbed by it. Hence the options (a) and (d)
where kB is the Boltzmann's constant are correct.
Obviously, v p < v < V rms. Option (c) is correct. 21. (b, d)
3kBT
Again, E=.!.mv~ or E='!'m Formanoatamicgas,Cp =~R,C!, =~R.
2 2 TIl 2 2
3 2kBT 3 2 Cp-C" =R
or E=-11I-- or E=-mv .
4 m 4 p
Far diatomic gasCp =~R,C(,=~R.
Option Cd) is correct. 2 2
Again, according to kinetic theory of gases, a molecule Cp-C1,=R
of a gas can have speed such that it lies between 0 and C p - C I' is same for both
""'.Hence the options (a)and (b) can never be correct. Cp +Cp = 6R (fordiatamic)
19. (b, d) C p + C I' = 4R (far monoatomic)
Coefficient of linear expansion of brass is greater than So (Cp +Cv)dia > (Cp +Cl!)mono
thatofcopperi.e.,aB >ac.
C p = ~ = 1.4 (far diatomic)
Ct, 5
C 5
-f... = _ = 1. 66 (for monoatomic)
CI' 3
(Cp)(C,,)= 35R2 =8.75R2 (for diatomic)
4
(C p HC l!) = 15 R 2 (far monoatomic)
4
So (Cp.C,,)diatomic > (Cp,C1')monoatomic
22. (b, d)
(a) process is not isothermal
(b) volume decreases and temperature decreases
.. LB = LoO +aBuT) ilU = negative, 6W = negative
or (R + d)O = LoO +aHuT) 6Q = negative
Again, Lc = LoO + ac6T) (c) work done in process A -7 B -7 C is positive
or RO=Lo(l+acilT)
(R+d)fJ l+aBllT (d) cycle is clockwise, so work done by the gas is
positive.
RB 1 +ac.1..T

www.puucho.com
Study Anurag Mishra with www.puucho.com

PREVIOUS YEAR PROBUMS 302

=:I True/False For atomic oxygen O. V,=-~3RXzr


M/2
=-~3RT
M
1. False v'= 2v.
According to kinetic theory of gases, 7. False

vrrm '= \
/3RT
M :,vrms depends on molar mass.

It will be different for different ideal gases.


2. False
v

v,
v, T
1. C
v =- nlCV1 +n2CV2 =- {W)+I(~) =-2R.
nl +n2 1+1
2. A to B represents a change which occurs at constant
For a gas, -
PV = constant rem perature when heat is added to the system.
T
From 0 to A =- Temperature increases at solid state.
For a given temperature T1,
A = It represents solid state.
11. VI
= Pz V2
AB = It represents change of state from solid to liquid
1\
-=- 2
V
when heat is added.
Pz \.'t
Since P lies on AB, the material is partly solid and
From graph, Vi. > V2 partly liquid. At B, the state is liquid.
3. False 3. GiVen: VP2 =- constant ...(1)

Vrms = \/3~T. It means that Vrms depends on M. According to gas equation, PV =- RT ...(2)
Eliminate P from eq. (1) and (2)
Hence different gases have different Vrms' RT)2 T2
4. True .. V( V =- constant => V =- constant
The slope of P-V graph is more for adiabatic process
than for isothermal process. From the graph it is clear m (T2)2 = (V2 ) => T2 = T, rv;
that slope for curve B is greater than the slope for l1i l \.l vV;
curve A. Hence the isothermal process is represented
by the curve A. T2 = T ~ = Fzr :. T2 = Fzr.
5. True
4. Ice at 0° C gains heat first melt into water at DOC and
Cp > Cv.
then this water gains heat for rise of its temperature if
When volume is kept constant, the heat supplied is heat energy is available.
used only for one purpose of increasing temperature
Heat required ice for melting =- mL
of gas.
.. mL=-100x80x4.2J=-3360DJ ...(1)
When pressure is kept constant, the heat is required
Heat lost by water at 25°C to fall to DOC =- m.s6.t
for two purpose viz for increasing temperature and for
.. m.s6.T=-3DDx4.2x2S=315DDJ ...(2)
adding work against constant pressure. Hence
Cp > Cv. Water does not provide adequate energy and so the
6. False whole of ice does not melt, as (2) is less than (1).

vnm =- J3~T :. Final temperature will be DOC.


5. According to Stefan's law, sun at temperature T
radiates energy.

www.puucho.com
Study Anurag Mishra with www.puucho.com

303 THERMOOYNAMICS

7. Let inner radius of shell is a and outer radius is b. If


thickness of shell isc,a =R _:., b =R +:.
2 2

..- , . -'
..- ...
..
,
..
, I'
b
,

.i
'. .'
Energy radiated per unit time per unit area = cry4 ' .._.-
.. Energy = (crT4)x (area 47tR2) ... (1)
time
.. Energy received on earth = 1400 x 41'1:r2 ... (2)
Equate (1) and (2)

or r4 = 1400xr
,
oR'
r4 = 1400 x (1.5 x lOll )2
or
CS.67 X 10-8)(7 x 108)2
1
24 =--
or . T4 = 14x2.25xl0 or T=5803K. 4,K
5.67x49xl08
6. The solid copper sphere is losing heat by radiation
when its temperature falls from 200 K to 100 K.
,
If R»c, R2_~",R2
According to Stefan's law, 4
energy radiated per second = aAy4 ... (1) RTh = t 2; H = dQ = L\ T [in steady state]
Heat lost = -msdr. 4nKR dt RTh
2 2
The negative sign means loss, p = 4nKR T ~ t = 4rtKR T
or Heat lost =: -(volume x density)sdT t P
OC
Heat lost
----=--TCT
4 2 dT
ps- ... (2) =
8. Let L latent heat of fusion of substance
time 3 dt Heat energy lost in solidification = ML
From (1) and (2), .. (Power P) x (time t) = ML
3 3
aAT4 = _ 4Jtr ps dT or dt == _ 41tr ps dT L = Pc
3 dt 3Acr T4 M
3 9. The whole system is thermally isolated from the
dt = _ 4n:r ps dT
OC surroundings. Hence the expansion is adiabatic:
3(41t£2)0" T4 obviously dQ = O.The other part of the container is
or I r dt = _ prs 100 1 .E!- = prs I zoo dT vacuum. Work done (dW) the gas is zero.
o 3cr 200 r4 3cr 100 T4 .. dQ=dW+dU dU=zero
Temperature remains constant.
or t = prs [ __ 1_] 200 = 7ps x 10-6
The temperature will now be 300 K.
3cr 3T 3 100 120"
10, When expansion is isothermal
7prs x 10-6 P
= PV=~ x2V or ~ =- ... (1)
72 x (5.67 x 10-8) 2

t = prs x
9cr
(_I100 )'[1_.1]8 When expansion is adiabatic;
PVY = Pa X (2V)Y or Pa =-
p
... (2)
2'
P P 2 2 2
6 -a =-x- =- = -- = 0.628
c = 7ps x 10-6 = .7prs x 10- = l.71prs ~ 2Y P ZY 21.67
7'21J 72x(S.67xlO-8)
c = 1.7lprssec.

www.puucho.com
Study Anurag Mishra with www.puucho.com

," . PREVIOUS YEAR PROBLEMS 304



t .. Pa "" 0.628. =1400xO.2xt ,..(2)
P, Q =280,
11. In steady state, rate of flow of heat is same at all 2BOt =9.24xl04
, (lOO°C-T) 4
~ cross-sectIOns. For rod A, H = KIA 1

=
or t = 9.24xl0 = 330sec or t = 5.Smin .
t" 280

.
(T - O°C)
forradB, H=K2A I
".' ,
Matching Type Problems
~ KIA (lOO"C-T) = K A.!:.
2
I I

r =>
=>
=>
300(lOO"C - T) = 200T
300°C-3T=2T
T = 60"C
=> ST=300"C
1. (a) --. (q)
JK is an isochoric process when volume
constant at 10m 3 and pressure decreases.
remains

~ :. Work done.<'!.W '= P;iV = P x 0 = zero


12. The movable stopper S moves irside the ring till the

t
"
pressure on both sides are equal. ,'. .<'!.Q'= .<'!.U+;iW = 11U .,' P <>< T,
volume
..
at

When pressure decreases, T also decreases.


constant

:. .<'!.Ubecomes negative. : . .<'!.Qbecomes negative


:. .<'!.Q < 0
~
(b) --. (p,s)
~
KL is an isobaric process at constant pressure 10 atm.
I:" The volume increases from 10m 3 to 20 m 3 .
.. .<'!.W>O [':.<'!.W=Px.1V]
~

f.'
Again, .: V <>< T when pressure is constant
J\=P, :. T increases as V increases along KL
nlRT nzRT m

t or -- = -- where n=- :. Heat is absorbed in the process : . .<'!.Q > O.


- Vj , V2 M (c) ~ (s)
(, m R LM is an isochoric process at constant
or --=--
~: MJ \1 M2VZ volume 20 m 3, when pressure increases

" V2 M1
-=-=-=-
VI M2
32
28
8
7
.<'!.w= P x.<'!.V = P x 0= zero
.. .<'!.Q= .<'!.W+.<'!.U = 0 +.<'!.U or toQ = toU

ft V1 =Z P <>< T at constant volume

., V,
.. \1 +Vz = 7+8=~
8
Add 1 to each side
.. T increases, along with pressure, along LM.
.. toQ>O

, V2 8 8 (d) --. (q, r)

.'
t~
,.
Here, as per diagram of ring, (Vi + V2) corresponds
with 360" and V2 with a.
MJ is a process in which volume decreases
pressure increases.
and

,.~. .. 360" = 15 =>0."= 360" x 8 _ (24 x 8)0


toW < 0, as volume decreases.
1\ \.1 _ P2 V2 . T2 _ P2 V2 _ 30 x 10

..,
3
(x" 8 15 and

<- 1i-r;-" T1 - 1\\'J. - 20x20=4
• ,
13,
.. 0.=192" .
A gas thermometer is a constant volume thermometer. .. T2 <T1 :. 6.Uisnegative.

" 11 Pz
-=-
T1
or T2 =-1j
T2
P2

4
1\
.. .<'!.Q< 0
2. (a)--. (s); (b)--. (q); (c)--. (p); (d)--. (r)
3. (a) As boundary is non conducting, 6.Q = 0.
or T2 ~ 3.5 X 10 X 273.16 In the case of free expansion W = 0.
4
3.0 x 10
From the first law of thermodynamics Q = .1.U + W
or T2 = 318.6 K = 45.6°C.
0= .1.U + 0 or .<'!.U= 0 or U is constant ~ T is constant.
14. Heat required to melt ice = mL
(a)~ (q) (As temperature remains constant).
or Q = (0.280)(3.3 x 105) 1 2
(b) P-- or PV =C
or Q=9.24x104 J ... (1) V'
Solar energy received on earth nRTV = C TV=C'

www.puucho.com
Study Anurag Mishra with www.puucho.com

Since when volume increases, the temperature Now, m x C x (5) = 420 - 336
m x 2100 x 5 x 10-3 "" 84 ~ m = 8g
decreases.

constant,
R
x
Now Q = nC6T, for polytropic process, pv =
'= Subjective Problems
1. Heat produced by the heater per second is
C =Cv +--
1- x
R 3 g = v2 = (200)2 = 2000Js-1 = 2000 cal s-I ... (1)
C=Cv+--=Cv-R=-R-R t R 20 4.2
-2+ 1 2
R R Heat transmitted through the window per second is
or C;:; -~Q = Il-D.T
2 2 Q KA(T,-T,)
-"-~-~
6T is negative so Q is negative, Means heat is lost. r d
(b) -; (p.r) where, T2 = temperature outside.
4 2
(c) PV4'3 = c, rvI, 3 =C Given TI = 20°C,d = 0.2 em, A = 1m2 = 10 cm
So when volume increases temperature decreases. and
R 3 3 K ::; O.2eal m -1 s-1 (cC)-1 = 0.002 cal em -1 s-I (0 C)-I.
Now C =Cv +-- = -R -3R or C = --R
-~+1 2 2 4
3 Thus _Q = 0.002x10 x(20-T~c 2)
t 0.2
Hence, Q = n( -%R )C6T) = 100(20 - T2) cal sec-I ,..(2)
As hoT is negative Q will be positive. Equating (1) and (2), we have
2000 0
(e) -; (P.s) 100(20-T2)=-- .-. T2 =15.24 C.
Cd) As product of P and V increases, temperature 4.2
. b ecause T = -PV 2. Loss of weight of sinker in liquid at temperature
mcreases
nR 1i. = W0 - WI' Therefore, weight of liquid displaced at
Q = 6U+W temperatureTI = Wo - WI' If PI is the density of liquid
!:J.U = +vc (.1.T = +ve). at temperature TI, then the volume VI of liquid
W = +ve (since volume increases) displaced at temperature TI is given by
So Q = +ve, Hence the gas gains heat. WO-WI =VIpg

= (d) -; (q. s)

Assertion and Reason

1. (b)
or

Similarly, volume
temperature T2 is
Wo -WI
VI =~~~
gPl
V2 of liquid displaced
...(1) ..

at

'= Integer Answer Type peoblems


V ,-
_WO-WI
gp,
where P2 is the density of the liquid at temperatureT2.
...(2)

Volume VI and V2 are related as


1. 0" 100' V," V,[l+~(T, -T,)] ...(3)
(lOx- Ax) where p is the coefficient of cubical expansion of the
material of the sinker.
. 400-0 . 400-100 Using eq. (1) and (2) in (3), we have
11 = 12 =
(AX. kA) (lO-A)XkA (Wo - W,) " (Wo - W,) x[l +~(T2 - T,)]
gP2 gPI.
i= (~}f =_LI_ or
£L=(WO-W1)x(l+!3(T2-TI)]
P2 (Wo -W2)
...(4)
p
(2 (~~)" L If M is the mass of the liquid, then
400 'A.\'" 80
M M
PI =- andp2 =-
-----"- \1 V2
300 (lO-A)X 540
2. (8) Heat absorbed during melting of 1 gm of ice,
5
Q =: mL "" 0.001 x 3.36 x 10 =: 336J

www.puucho.com
Study Anurag Mishra with www.puucho.com
PIOBLEMS
306
PI V
-=- 2 ... (5)
,-,' constant for a longer period of time. Now the solid
Pz \)
is a mixture of different two phases.
If Yis the coefficient of cubical expansion of the liquid, (ii) If CD = 2AB, this means that the amount of heat
then \1 and Vz are related as Vz = \1 [l + y(T2 -Ii)] required to conven the solid completely from one
or V2 = 1 + y(T2 - 'Ii) ...(6) phase to another is double the amount of heat
V, required to conven the solid from one phase to
Using eq. (6) in (5), we get another (different phases).
£L=1+Y(T2-1jJ ... (7)
(iii) The slope of DE represents that the temperature of
P2 the solid in this phase begins to rise.
Using eq. (7) is (4), we have (iv) The slope of OA > slope of BC indicates that there
(Wo-W,) is a rise in specific heal.
1+)'(T2 -1]):= -~-~x[I +P (T2 -Ii)]
(Wo - W2) 5. Total pressure in jar T K = pressure of gas + saturated
which gives ...... vapour pressure of water at T K
(W2 -WI! ~(wo-WI) .'. Pressure of gas along (~ ) at T K
y = ---~-~---+-----
(WO -W2) (T2 -T,) (WO -W2) = 830- 30 = SOD mm of Hg.
Let pressure of gas alone (T -~) K be P2
3. Let A be the area of cross-section of each rod 100 .
Length of the composite rod at 25cC Applying the formula Ii = P2
= 1metre , T1 T2
Length of the copper rod at 25° C /25 =: 30 em SOO
We get -- P2
= -_~_
Length of the second rod at 2Sl>C L25 =
70 em
T (T _-!..-)
Length of the copper rod at 125 C, 0
100
1125 = 125[1 +0:(125- 25») 99
or P2 =SOOx-=792mmofHg
Where (l is the coefficient of linear expansion of copper 100
=
and is 1.7 X 10-5 per "C
, .', Saturated vapour pressure of water at
:. 1125 = 30(1 + 1.7 x 10-3) = 30.051 em
=
Increase in length of copper rod 0.051 ern 100 (T _..I-)
K "" 25mm ofHg

L12S = L25 (l + P x 100) =


70(1 + 100/3) .. Total pressure in the jar at this new temperature
where J3 is the coefficient of linear expansion of the = 792+ 25= 817 mm ofHg.
second rod. 6. A -+ B. AB is parallel to V.axis. A straight line between
Increase in length of second rod "" 7000~ A and B in p.v graph indicates that at constant
Total increase in length = 0.051 + 7000p = 0.191 em pressure Ii, volume increases from Vi to V2. Obviously
0.191 = 0.051+ 7000~ this is possible if V oc T at constant pressure Ii.
p
7000~ = 0140
P = 2 x lO-speroC P, A
B
,
The force required to maintain the copper rod at its ,,,
initial length when the rod is heated, is given by ,,
F = YAaT = 13 x l(P A x 1.7 X 10:,"5x 100 ,,,
,
=2.21X108Adyne" .. P2 - •• ---i .. --•... -...
, ,C
where A is the area of cross-!>ection : ':
: V
.. Force required to maintain the second rod = YApT
V, V,
.. YxA x 2x 10-5 x100= 2.21 x 108 A
.. Y = 1.1xl011 N/m2 B -+ C. BC is parallel to P-axis. Volume remains
4. (i) The region AB represents that the temperature of constant at V2• temperature decreases from T2 to 1i
and pressure decreases from Ii .to P2.
the solid material stops rising for a while inspite of
the fact that heat input is being pumped. This C -+ A. CA represents an" isothermal process. At
happens when the solid is a mixture of two constant temperature 1i.. volume decreases from V2 to
phases. Along CD, the temperature remains Vi while pressure increases from P2 to 11.

www.puucho.com
Study Anurag Mishra with www.puucho.com
THERMODYNAMICS
307
7. The kinetic energy of bullet, when stopped by an Q ~ nu6 .. dQ =
dt
ms(de)
dt
obstacle, is converted into heat. 75% of the heat
generated is retained by the bullet so that it is heated where _de '" Rare of fall of temperature.
till it melts down. dr
de
:. _"' __ IdQ de
or _<x I
.. 75 [..!:.mpZ]= ms~e+mL
dt ms dt dt
__
mass
100 2
Hollow sphere has less mass.
or v2 =~(S6.8+L) :. Hollow sphere will have faster fall of temperature.
3
:. Hollow sphere starts cooling faster.
or vZ =~[(O.03x4.2x300)+(6x4.2)]
3 10. (D One gram mole of oxygen'" 32 g.
According to Avogadro number, 6x 1023 molecules
or v2:::; ~ (37.8+2S.2] = 8x63 = 168
3 3 will be contained in a gram-mole of a gas.
1 32
or v = 12 .96 m sec- .' Mass of one molecule of oxygen =: 3
8. The compression of one mole of a perfect gas is an 6x102
adiabatic process. =: 5.3xlO-23g

...(1)
.. Mass of one molecule of oxygen =: 5.3 x 10-23g
m =: 5.3 x 10-26kg ...(1)
.. Cp -Cv =R Let n =: number of collisions per sec per m 2.
Cp :::;R +Cv whereCv = 3R/2 (given) Momentum of incident molecule =: mu
3R SR Momentum of reflected molecule =: -mu
or Cp =R+-=-
2 2
Cp SR:'Z 5 ... (2) .. Change in momentum per collision ~ mu - (-mu)
Y =- ory=-- =-
Cv 3R,'2 3 =: 2mu
From (2) and (1), .. Change in momentum in n collisions ~ 2mnu
Pz = 105(~r'3 = 6.24xl0sNm-
2 ... (3)
.. Change in momentum per sec per m 2 ~ 2mnu
Change
.. __ ~ in momentum ~ 2m nu or pressure
PVI = constant K :. P = 1(1,,-1 Time x area
~ 2mnu
Work done dW = PdV, dW = KV-Y dV
or 105 ~ 2mnu ...(2)
or Ii;' dW = KI~2 V-YdV. u ~ v,ms
W = _K_[V-Y+1]v2 or U ~ /3RT ~ 3x 8.3x 300 =: 483m sec-1
-"(+1 vj or
\ M 32"1000
W =~ [V-r+l _ \1.-y+l] u ~ 483 m sec- 1 ... (3)
1-)' 2 1 or
K,rY V - j(\1,-y \!,
.. Put (1) and (3) in (2) to obtain n
or W = 2 2 1 1 or W = P2 V2 -11 \!,1 105 105
1-)' 1-)' n = --. or n::: ---------- 26
3 2mu 2x(S.3xlO- ) x (483)
or W = (6.26xlOS)x(2xlO-3)-(lOS)(6xIO- )
or n ~ 1.97 x 1027 ...(4)
. 1-1.67
or
W = _ (1252 - 600) '" -652 (ij) To calculate va:
0.67 0.67 K.E.of 1 g mole of oxygen == ~ MV5 ... (5)
or Work done'" -973.1 J
Work done is negative because the volume of gas Heat gained by 1 g mole of oxygen ~ 1 xC y x!l.T
decreases due to adiabatic compression. or Heatenergygained~ CydT ~ (Cy x 1) ..(6)
9. Rate of cooling of a hot body by radiation, in an The K.E. is actually converted into heat energy.
environment depends upon its area and temperature. I 2
-Mvo ~ Cv
For the solid and hqUow spheres, these factors are the 2
same. Hence the rate of cooling dQ will be the same.
_. . dt
1 .2
-mvo ~--
2
R
y-I [."CV =~]
y-1

www.puucho.com
Study Anurag Mishra with www.puucho.com
BIIMS

or 2 2R 2x&3 2x &3x 1000


vO=---= Adiabatic work done = P2 V2 - PaVo
M(y-I) ~(I.4I-I) 32 x 0.41 (1- y)
1000
or Va = -----
J 2X&3XlOOO
32 x 0.41
or Va = 35.6 m sec
-1 .. W=(I~~f:3:ox~;_povo]
11. (i) For left chamber, the process of heating is slow.
Initially, the parameters are Po, Vo, To or W=-~[~-l]PoVa
Finally, the parameters are 243 ,Po, \1, TI. 3 5 15
32 or W=--X-X(RTo)=--X&3To
2 4 8
PoVo :(Po X243)x '" or W = -15. 58Tojoule
To 32 T1
Work done is negative as the volume in right chamber
or ", _ 243 Vi T. decreases.
- 32 V 0 ...(1)
o 12. Let the tight bulb placed in water bath at 62°C and the
Adiabatic compression occurs in the right chamber. left bulb be placed in melting ice at DoC.
The initial quantities are po. VOl y, To
.. Po x 243 If
T h e fima I quantities are ~--.
32

Po x 243
1'2. y,
T

(V,)' 32
e 'n;",II, ~

C
n "y _
rOlla ----Xv2
,,'( or - =
32 Vo 243
,;,,11, :]
or (~:r=:4~=m5 0<:=GJ'=2 8
7
OOC 6ZOC

or V2=-Vo 8
...(2)
Let x molecules
bulb.
be transferred from right bulb to left

But '1 + V2 = 2Vo (Given)


27
:. \.1 + -8Vo = 2Vo
..
, .. For left bulb, using PV = nRT,
Initially, 76 x V = nR x 273
27
46 Finally, P'xV = (n + x)R x 273
or Vi = -Vo ...(3) P'=(n+x) .
27
... (1)
From eqs. (1) and (3), 76 n
r; 243 46Vo T, Now consider the right bulb.
1 = 32 x 27V
o x 0 Initially, 76 x V = nR x 273
.207 1 Finally, P'xV = (n - x)R x (273 + 62)
or T1=-IO=
T
2.9To ...(4)
16 or P' (n-x) 335
-=--x- ... (2)
Again, for the right chamber, use P.T equation for an 76 n 273
adiabatic operation. From eq. (1) and (2),
'. TY = (constanr)xp'H n+x n-x 335
--=--x_
TJ = (constant) x PJ-l n n273
or 273(n + x) = 335(n - x)
Ti = (constant) x (243P.
~
),.,
or 273x + 335x = 335n - 273n
or 608x = 620 or n = 608 x ...(3)
.. (~~r (~4;~2r-l = Put (3) in (1) to obtain P'
62

or (~~ r =(23423)~.'=U4~r or
P' n+x
:. -=--=-+-=1+-
76 n

( 62)
n x
nn

76x670
x
n

or (~~r3 =m'Q3 or ~~ =GJ' =~ P'= 76 1+-


608
= --_
608
:. New pressure = 83.75 cm of Hg.
=83.75 em ofHg.

9 13. According to principle of heat-conduction,


or T2 = -To or T2 =2 .25To ... (5)
4
(ii)Work done by the gas in tight chamber.
Q = KA(9r-82)< or g
= A(9r -8,)
d "t diK

www.puucho.com
Study Anurag Mishra with www.puucho.com
THIRM AMICS

or f\ x (20x 10-3) = 2x B.3x 300


K, K2 K,
\Vood Cement Brick or Pt =2.49xl05Nm-z ...(1)
d, d, d,
Again, consider the process AB.
II v,
__ 1
p V,
=_'_,~=Pz =PforAB.
or power (P) = ACSl - 82)
elK Ii T2
Px20xlO-3 Px(40xlO-3)
P= 137[20-(-10)] 0' '-'-'-'-'-- = ------
300 T2
.. [__
2.5 +_+_
1 25]x 10-'
0.125 1.5 1 or T2 = 600 K. ... (2)
(b) For final volume V3, consider the adiabatic Be
or p=137x30xlOO or P=9000W.
137/3 .' Tzvr1 =T3Vrl or (V,)Y-l
V3
= T3
T2
gas equation, PV = nRT
14. According [Q

(l.6xI06)xO.0083
PV 16 where T3 = Initial temperature 300 K.
" n=-=~--~---=- 5
RT 8.3x300 3

'" n =-
16 ... (1) 0'
(~~f(~~~J = =~

(~~r
3
5R 1
"Cp-Cv=R .. --Cv =R
2 oc =-
2
3R

('1" ( "
Cv =- ... (2)
2
Amount of heat = nCvdT
4 16 3R
40
0' V
3
="2
'J
="8 = 0.354

2.49 x10 = - x- x dT 0'


V3 = __40 = 113 litre ... (3)
3 2 U354
4
.. dT = 2.49 x 10 x3x 2 = 375 K ...(3) (c) For final pressure P3 :P3 V3 = nRT3
16x3xB.3 .. P = nRT3 where T3 = Initial temperature 300 K
3
.. Final temperature = 300 + 375 V3
T, 675 K = ...(4)
or
_ 2x8.3x300
P 3 ------
Volume of the gas is constant as it is kept in a dosed l13xlO-3
vessel. or P3 = 0.44 x 10SNm-2 ...(4)
.. IIV=P,V
(iii) Work done by the gas: W = WA8 + WBC
T1 T2
6 or W=l\(V _Vi)+nR(T2-T3)
2
or p = 1\ T2 = (1.6 x 10 ) x 675 y-l
2 T1 300
0'
... (5)
or P2 = 3.6x l06Nm-Z W = (2.49x105)(40-20)l0-3 + (2X8.3 (600-300)
.. Final temperature (Tz) = 675 K
Final pressure (Pz) = 3.6 x 106 Nm -2.
~-1)
or W = 4980 + 7470 or W = 12450 J
15. (i) P-V diagram is sketched.
p 16. Final pressure = Patm + Pspring
kx 5 8000 x 0.1
or P2 =Po+- or P2 = (1.0 x 10 )+---- 3
2.49 (PI AV\) A 8xlO-
Pl"'P2"'P •• ----. or P =2xl0 N/m5 2 ...(1)
•,, 2
,, Final volume = Initial volume + expansion
0.44 : , V2 =V1 +xA
p _•• .:.. •• ~ •• _ c
3
.
', .,,
• ,, or 3
V2 = (2.4x 10-3)+ (O.l)x (8x 10- )
,
,, '' .
,,
, v or V2 =3.2xl0-3m3 ...(2)
According to gas equation,
1\ Vi = P2V2 or T = P2V2T1 2
eii) (a) For a gas, PV = nRT ,', 1\ VI = nRTl T1 T2 l\V1

www.puucho.com
Study Anurag Mishra with www.puucho.com

or

or T2 =: 800 K
(2 x lOs) x (3. 2 x 10-3)
T2 = '-'_-'--'--'- -'- __
(10 x 105) x (2.4 x 10-3)
X 300

...(3)
Heat supplied is used for three purposes. They are
: 2
p

_._._-c~ B
.
2.....~zr
p
310

(1) For expanding gas against pressure = PaY ,,


2
.. .. :
"'---.
'
:
:
B

'. spnng = _I kx'


(2) For compressmg , v " , T
2
(3) For raising temperature of gas = nC v.dT
.'. Q = P.1.V+~kx2 +nCvdT
(i) Consider process A to B.
2
where n =0 <9.\1 Cv = 3R It is an isothermal process of expansion of gas.
R1i ' 2 . From P.V graph, PA VA =: PB VB
PA
or Q=105 X(USxlO-3)+..!.XBOOOx(O,I)2 or PA VA =: PB X (2VA) or Pa =: 2 ...
(1)
2
(ii) Consider process B to C.
+(~~ )(~)500 It is an isobaric process of compression of gas.
VB =: Vc
TB TC
80+40+ 105 x lAx 10-3 x SOOx 3 2V VA T
Q= 300x2 --,-A or Tc =_A ... (2)
TA Tc 2
or Q = 720 J
(iii)Consider process C to A.
Hence final temperature = 800 K,
Heat supplied = 720 J. It is an isochoric process of increase of pressure.
17. (i) Let P, V and T denote initial conditions/quantities. Pc = PA .. PB = PA or ~ = PA
Tc TA Tc TA 2xTc TA
Then the final quantities aTe P', S.66V, T12.
TA
.. For adiabatic operation 'Ii Vji'-l = T2Vr1 .. Tc=- ... (3)
2
or 1111-1 = (~)(S.66V)'Y-l or 2= (5.66)1-1 (b) To calculate network done and net heat supplied:
Wnet = W AB + WBC + Wo;.
or log 2 = (y - 1) log 5.66
or (13010= (y -1)(0.7528) or Wnet = nRTA In(~; )+nR(Tc -TA)+O
0.3010
or (y - 1) = -- = 0.4 or y = 1.4 ... (1)
0.7528 or Wnet=(3)RTAln(~A)+(3)R(T;_TA)
2
I+-=y
2
.. 1=-=--=-=5
2 2
...
(2 " .
1 y-I 1.4-1 0.4 or Wnet = 3RTA X2.30310glO 2-- 3RT
- A
) 2
Degrees of freedom of gas molecules = 5. or Wnet =3RTA x2.303xO.301O-1.5RTA
(ii) To calculate work done in adiabatic process: or Wnet = 2.0BRTA -1. 5RTA
or Wnet =0. S8RTA
In a d.la batlc
. process, W = ~-'--'~
R(T, -T,) ...(4)
y-I In a cyclic process, tJ..U = 0
.'. W = R[T - (Tf2)] = ~ =.1.25RT .. Qnet =Wnet+O or Qn~t =O.S8RTA ...(5)
(1.4-1) 2x(14 19. Given: Number of moles =2
or W == 1.25x1\ Vi (-:1\, \1 are P, V) PA = PB = 2 atm , AB is isobaric process
Hence, degrees of freedom = 5, Workdone= 1.25PV Pc = PD = 1 atm, CDis isobaric process
Note: The gas is a diatomic gas as its y '" IA,f = 5. TB Tc = =
400K, BC is isothermal process
18. (a) For the complete process, the P-Vand P-T diagrams T D = T A == 300 K, DA is isothermal process.
are shown below: ABCD is a cyclic process. Hence dU O. =
WAR = nR(TB -TA) =
2x a,31 x (400- 300) == 1662 J
WBe = 2.~03nRTlOglO(;~)

www.puucho.com
Study Anurag Mishra with www.puucho.com
THERMOIlYNAM1C51

== 2.303 x 2 x 8.31 x 400x 10glO( i) or I [(eo - 300)] KA


og (8 _ 350) == msI t
0
== 2.303 x 2 x 8.31 x 400 x 0.3010 or t == msl x 2.303 x 10 [(400 - 300)]
KA g10 (400-350)
"4608 J
P(alm)
or time == (0.4)(600)(0.4) x 2.303 x 0.3010
2 __
;A,--_+-_~ B (10)(0.04)
c.:
log10 2 == 0.3010)
or time == 166.38 sec.
2 1
D+-__ e--_~ 'c 21. Given: TA == 1000K, Pa == -PA, Pc == -PA
T 3 3
400 K Cp 5 .
300K n == 1, '( = - :0 - f or monoatomlC gas.
Cv 3
WCD == nR(TD -Tel A --+ B: Adiabatic expansion.
== 2 x 8.31 x (300- 400)
B --lo C: Cooling at constant volume.
"-1662 J An isochoric process.
WDA == 2.303nRTIOglO(~:) C --+ D: Adiabatic compression
D --lo A: Heating at constant volume
== 2.303 x 2 x 8.31 x 300x IOglO(~) An isochoric process.
2/5

== _ 2.303 x 2 x 8.31 x 300 x 0.3010 == -3456J ( ~) "085


:. Wnel == WAR + Wac +WCD +WDA (i) To calculate work done (W), along A --+ B
== 1662 + 4608 - 1662- 3456 == 1152 J.
t. C., WABdenotes an adiabatic process.
(a) To find Qnel
According to first law of thermodynamics,

..
Q==W+I3.U
Q == 1152 + 0, as I3.U== 0 in a cyclic process
"G:r TB"TA(~;f(1)
..(~J' or

.. Qnel == 1152 J or TB "(1000)(~r~ "(1000)(~r5


(b) W" 1152 J
Cc) I3.U== 0, since ABCDA is a cyclic process. or Ta == 1000 x 0.85 == 850 K
20. The upper face of cylinder is at 400 K R &31
WAR ==--(Ta -TA)== ( )(850-1000)
The initial temperature of disc = 300 K 1-'( 1-~
The final temperature of disc == 350 K 3
Heat flows by conduction from upper surface of the == 8.31 x 150 x 3 = 1870 J
cylinder maintained at 400 K to disc. 2
Let temperature of disc be 8 at any instant of time. (ii) To calculate heat lost (Q) along B --+ C 1. e., Qae
By principle of heat-conduction, For an isochoric process,
dQ KA(eo - B) f r d Ta PB
d't== I ' orcym er. -"- ...(21
Tc Pe
Upper surface
(PA3 X~l850
dQ dB .
_
dt
== ms-, for dIsc.
dt B, K Te == (PcP lT B ==
2PA
K
dB KA(Bo -B) a
.. ms-==---- 400 K
Te==425K
dt I E or
Cylinder 1 dO
or __dB KA
"_dt dt Qae ==nCVI3.T==(l)(3:)(Tc-TB)
80 -8 Iml
or J350K d8 KA J' d 0.4 kg == _3 x 8.31 x (425- 850)
300K (9 _ 8) "'" msl 0 t 2
0
350K KA == _ 3 x 8.31 x 425 == -5298 J
or [- I og(80 -8) ] 300K =-t BK 2
m,l
(iii) To calculate TD

www.puucho.com
Study Anurag Mishra with www.puucho.com

312
C to D is an adiabatic compression

..(:~r=(;~r
or P = (m + 28) RT or (m + 28) ==40PV
40 V RT
.•.(3)
or (m+ 28) = 40x (l xlOs)x 0.02
D to A is an isochoric process. 8.314x 300
. Pv P PA TA or m+28=32 orm==4g
-=- A or-=_ ... (4)
Tv TA Po Tv '. Massofneon=4g
From eq. (1) and (3) Mass of argon = 24 g.
24. (a) To find number of moles of gas B.
(
PCPA ))'-1 = (Tc x TA)'
... (5) Let the number of moles of gas B = n
FVPB Tv TB
Given: The number of moles of gas A ==1
From eq, (2) and (4),
S.lnce U = --.
nRT F or mixture,
. U = U m, 1 = _19
y-I 13
(;~~:) = (~:~;) ... (6)
.. Um ==UA+UB
From eq. (5) and (6), eliminate P terms (nA + nB )RT nART nBRT
~~~--=-+-
Yrn-1
TcTA
( TaTD
)Y-l = (TcTA)Y or 1 = TcTA l+n
YA-l
1
18-1
n
TBTv TBTv or ~-~ + --- = ---
or Tv = TcTA
TB
or Tv = (425)(1000)
850
(;~-I) G-I) G-l)
or 13(1+n)=~+5n orI3+13n=9+15n
or Tv""SOOK 6 22
22. (i) To calculate W4,
or 4=211 or n=2 ... (1)
l1U = 0 as the process is given to be cyclic. According (b) To compute speed of sound
to first Jaw of thermodynamics,
l1Q "" .1U+dW = O+.1.W or l1Q = 6W Speed of sound v = J1~T
or Q1 +Q2 +Q3 +Q4 =WI +W2 +W3 +W4
.
For mixture, "AMA + "8MB_
M ==_~~
or 5960-5585-2980+3645= 2200-825-1100
nA + nB
+W, M = (1x4)+{2x32) = 68 gram
or W4 = 765 J
(ii) To calculate efficiency of cycle fl. 1+2 3 mole
work done 19 8.31x300x3
~=-----
Heat supplied
:. v== -x-----oru=40Imlsec
13 68xlO-3 '

or '1::::WI +W2 +W3 +W4 = 1040 10.82% = (c) To find percentage change in speed of sound.
ClI +Q, 9605 Velocity of sound v o<.ff :. v = (constam k)T1 '2
23. Temperature of mixture = 2rC = 300K or dl' = .!.kT-I/2
Let m = mass of neon gas in the mixture dT 2
kdT
.. Mass of Argon in the mixture = (28 - m) or dl' = -- where dr = lK, T = 300 K.
2.fi
.. No. of gram moles of neon = _m (n})
20 or dv"" kdT x_I_='!'(dT)
. . No. of gram moles of argon (n 2) = _28-m v 2.fi k.fi 2 T
40 dv IdT
:. - x 100 = -- x 100
According to Dalton's law of partial pressures, v 2 T
P=11+P2
where P = Pressure of mixture of gases.
or % change in v =.!. x
2 300
(_1_)
x 100 =.! ""0.167
6
11 = Pressure due to neon gas in mixture. .. % change in v = 0.167%
P2 = Pressure due to argon gas in mixture. (d) To find change in adiabatic compressibility
nlRT n2RT RT For adiabatic change, PV'f = constam
p=--+-- or P=(nl +n2)-
V V V .". V(dP) + PyV,-1 (dV) = 0
0' p=(~+~)RT
20 40 V or
dP
-=--
yP dPv'
or -_==yP
dV V dV
or Bulk modulus = E "" yP'

www.puucho.com
Study Anurag Mishra with www.puucho.com

THERMODYNAMICS

26. Let m be the mass of the steam required to raise the


Compressibility ee) = 1
temperature of 100 g of water from 24CCto 90cC
Bulk modulus eE)

.. C=- 1
yP
Heat lost by steam = Heat gained by water
mLT m xs xf',T1 = M xs X f',T2
mx 540+ m x 1 x[100- 901 = 100x 1 x[90-24]
or ~C=~~2-Y~=~(;2-~) ... (1)
:. 550m = 6600 :. m = 12 g
27. (i) \Vhen the rod is cooled, it will contract in length.
. n (nA +nB)RT
A gam, 'I = -- --- When a mass is attached at its lower end, its length
V increases.
or I\ =(l+Z)xR31x300 or 1\ =7479 ...(2) For no change in length,
V V

Again, ~ =(~:r r =(5~ =5


1
contraction when cooled= extension when stretched.

.. 0.
Contraction in length
.. 0. = ---------------
Original length x change of temperature
= --
l>L
or M... = LwiT ...(1)
or P2 = 11 X 51 ( where 'I = ~~ giVen) ... (3)
Lx f',T

Put (3) and (2) in (1), Again,Y= F/A or M=!-~ ...(2)


M,'L A Y

.. ~C=HI\~5,-~J=Y~[5~-I] From eq. (1) and (2),

or .1C = 13V
19x7479
[_1_ -1]
51913
Lo.tlT = -
AY
=--
AY
FL MgL

= 13V [0.1 _ 1) = _ (13 x D.9D)V or M = Lo.tlTAY = o..1TAY


19x7479 19x7479 gL g
(13 x I)V .. M = (10-5) x (100) x (4 x 10-6 )(101l)
=-
19x8310 10
= -o.0000827V = -8.27 x lO-sV orM=40kg
25. Given T} = 27°C = 300K, n = 2 moles. y = 5/3, (ii) To find energy stored in rod.
Vi. = V, Vz = 2V,Cv = 3R for monoatomic gas. Energy = ~ x stress x strain x volume
2 2
(i) To find final temperature: T2- or
In adiabatic process, PVY = constant, Energy =.!x 40xlO xO.5XlO-3 xO.5x4xlO-6
.. IV"(-l = conStant, pl-1T1 = constant 2 4x 10-6 0.5
or Energy = 0.1 J.
:. T} \.)"(-I= T2V21-1 or T2 = T1 ('"
V )'-1 28. Forfirstglasssheet,~ 2
=0.01m,A=lm ,K1 =0.8
2
For stagnant air, d2 = 0.05 m, A = 1 m 2, K 2 = 0.08
or
V)S-1
T2 = 300 ( 2V 3 =
(I)'"
300 2: = 300xo.63 For second glass sheet,
d3 = O.Olm,A = Im2,K3 = 0.8
or T2 = 189 K Temperature difference, SA = 27cC, 8B = OCC
(ii) To find change in internal energy = .1U. (outside)
aU = nCvll.T
Stagnant
Glass Glass
or f:.U=CZ)x(3:)XCT2-T1) Air
K1 K, K, DOG
or aU=3x8.31x(189-300)
or 6U = -2767 J Room d2 d, Outdoor
~
(iii) Work done
8A 81 82 6B
The process is adiabatic, so f',Q = 0
According ro first law of thennodynamics Heat KA(~8)
At steady state, -- (H) =
f',Q=f',W+f',U sec d
0= f',W + (-2767) or f',W = 2767 J.

www.puucho.com
Study Anurag Mishra with www.puucho.com

GUS lUI PROBLEMS

.', At first glass sheet, H = K1A(SA -61) t.W = (20 - 10)(10)x 104 = 10' J
dl
.. llQABC = llU + llW = (2.25xI06)+(106)
Hd
or SA-S, :::- l ...(1) =3.25xl06J ",(5)
AKj
Process ADC:
At air, H = K2A(61 -62)
f:.U = As in the process ABC = 2.25 x 106 J
d,
Hd, f:.W = Area under AD
or 61 -62 = -- ...(2) or f:.W=(20-10)(5x104)=0.5x106J
AK,
Hd :. f:.QADC = f:.U + f:.W = 2.25 x 106 + US x 106
For second glass sheet,62 -SB = __ 3 ... (3)
AK, =2.75xl06J ... (6)
Add up (I), (2), (3) 30. Given: n = 1 mole
y = 1.4 for diatomic gas.
SA -OB = H(~+!2..+!2.) p
A K1 K2 K3
B
or (27 _ 0) = H [0.01 + 0.05 + 0.01] = H[0.52]
1 0.8 0.08 0.8 0.80
or H = 27 x 0.80 = 27 x ax 10 = 41.54 J/s ...(4)
0.52 52
From eq. (4) and (I),
27-6 =(27X8XlO)x~
I 52 lxO.B v
'. 61 = 27 - 0,52 = 26.48°C ...(5)
Vo 2Vo 16Vo
From eq. (4) and (3),
VA Vc
B2_0=(27XSXIO)x 0.01 TA =300K, -=16, -=2
VB . VB
52 Ixo.S
A --+ B: adiabatic compression
"' 82 = O.52°C B -) C: isobaric expansion
29. Number of g mole of He = n C --+ D: adiabatic expansion
m 2000 D --+ A: isornoric process.
0=-""--=<500
M 4 TofindTB.
CDTo find TA, TB, Te• Tv Process A --+ B is adiabatic
T = PV
nR
4
T =PAVA =(5x10 )XlO=120.3K ... (1)
.. A nR SOOx&31 or TB = (16)1.4-1= (16)2:5 = (256)1/5 = 3.03
TA
. TB -_PBVB
Agam, _CIOX104)XlO_2406K (2)
-- - ----- _ • ••• .. TB = TA x 3.03 or Ta = 300 x 3.03
nR SOOx&31
or TB =909 K ... (1)
. Tc -_PcVc_UOXI04)X20_4813K
Agam, -- - _. ...(3) To find To.
nR SOOx8.31 B --+ C is an isobaric process.
4
. TD _PDV
Agam, ----
v _(SxlO )X20-2406K
-. . ..
(4) •. VB = Vc , according to Charles' law
nR SOOx 8.31 TB Tc
(ii) No.
(iii) Process ABC: or Tc = TB(~~) Tc = or 909 x 2

liU = nCvdT = n(3:}Tc -TA) or Tc = 1818K ... (2)


C --+ D is an adiabatic process.
or I:iU = (500) + 3 x (8.31) (481.3-120.3) 1
2 .. TD = (VC)"Y- or' To =TcX(VC)'-1
or Tc Vo VD
DJJ.500X3x&31X361 or llU=2.25x1Q6J
:I or To = 1818XL~r4-1 = 1818(~r'5

www.puucho.com
Study Anurag Mishra with www.puucho.com

315 THERMODYNAMICS

or TD =1818><0.435 or To =791.4K ...(3) "1+5'( 51+49 100 1


0< --~-----=--=--
To find efficiency of the cycle. 1 + 95y 52.8 + 49 101.8 1.018
% efficiency(l1) = Net work done x 100 or 1+95y=1.018+5xlOI8y
Heat absorbed or 95y - 5.090'( = 0.018 or y = 0.018 = 2 x 10--4
~=Q,-Q2XI00 89.91
0'
QI .. Coefficient of linear/thennal expansion
where Q2 = Heat released in the cycle. y 2xlO-4
U=-=---
QI = Heat absorbed in the cycle. 3 3
5
For adiabatic process AB and CD,6Q = a .. u=6.67xlO- percC
or QAB=QCD=0 32. Number of mole of ideal monoatomic gas = n = 1mole
" 5R 3R
NowQoA=nCvdT orQDA=(l{S:)(TA-TD) For monoatomlc gas: Cp = -,Cv =-
2 2
(-: Cv = 5: for diatomic gas) ABCA. is a cyclic process.
A --7 B: An isochoric process at constant volume Vo.
C --7 A: An isobaric compression at constant pressure
Qz = ( 5 x :.31 )c300 _ 791.4) = -10208.8J
Po.
Negative sign show heat is lost by the system (a) Work done by the gas: dW;
Again QBC = nC ptiT or dW = Area of triangle ABC
1
dW=-(2Vo-Vo)(3Po-Po)
QI =Cl)(7:)cTc-T B)
or
2
or dW = Po Ya ... (1)
C4 =lXC X
:
31
)X(l818-909) (b) Heat rejected along CA.:dQCA:
p
Q = 7 x &31 x 909 = 26438.3J

::"':1~~
1
2

(I - ~: )100
..

0'
~=

= (1- 1020&8)100
11 26438.3 or Tl=
614%
.
3~: , :
c
,,
HenceTB =909K, Tv =791.4K=791K ,, v
'1 = 61.4% Vo 2\'0
31. Density of a liquid decreases as temperature rises
according to relation do = dt (l + yt) where dl> do = dQCA = nCpdT = 1 x Cp(TA - TC>
densities at tOC and aoc, Y = coefficient of cubical Po x 2Vo
Tc=~--
expansion of liquid. lxR
Po T _ Po x Va
A - lxR
A
.. dQCA=lX5R[PoVo_2PoVo]
2 R R
SPaYo ... (2)
or dQCA=- 2
h
95°C Again, heat absorbed along AB: dQAB:
~~=====:=:=~~ dQAB =nCvdT=lxCv(TB -TA)
. Base Base T = (3Po )VA T = Po x Va
hI = 52.8 em, h2 = 51cm, h = 49cm B lxR'A lxR
Pressure at B = Pressure at C .. dQAB = 1 x 3R (3Po Va _ PoVo ) = 3Po Vo ...(3)
Po + d9S"CghJ - dsocgh = Po + dSQcgh2 - d9socgh 2 R R
or d9S0cChJ + h) = dS0cCh2 + h) (c) Net heat absorbed along Be:
d9S"c h2+h do.(l+95y) h2+h In a cyclic process, dU = 0,
0' -- =-- 0< ~---~ =--
dS"C hJ+h " doi(l+ 5y) hJ+h dQ = dW or dQAB + dQBc + dQCA = dW

www.puucho.com
Study Anurag Mishra with www.puucho.com

--- or 3PoVo +dQBC ---

From eq. 0), (2) and (3)


SPaVo
2
=POVo or

or
~=-kdt
T-TA
[In(T - TA )]~5g= -kt}
or J350 ~=-kJtl
400 T-TA 0
dr
316

SPaVo
or dQBC =PoVo-3PoVo+--
2 or In ( 350 - 300) = -kt or kt, = In 2 (1)
400 _ 300' ...
or dQ Be _
---
PaVo ( )
... 4
2 During time £}, the temperature falls from 400 K to
Cd) The maximum temperature of gas: Tmax:
350K.
Be is a straight line. The maximum temperature of the Stage II: Cooling from time t} to 3t}
gas Tmax vviil lie on Be at some point. Cooling from 350 K down is caused by conduction and
radiation.
For a straight line y mx + C =
2R By conduction,
For Be, slope m = ----.Q., Intercept C = SPo
Vo
dQ = KA(T-TA) or .'!..(-msT) = KA(T-TA)
.. p=_(~o )V+5Po .,' p= ~T andn=l dt dtL L
dT KA(T-TA)
or -ms- = ---~
.. nRT =_(2Po)V+SPo dt L
V Vo or _CdT =KA(T-TA),wherems=c.
2 dr L
or T = ..!.[-2POV
R Vo
+ SPa v] ...(5) C denotes heat capacity of body X.
dT KA
--=-(T-TA) ...(2)
d2T
For T to be maXimum, - = 0, --
.
= +ve.
dT dr a
dV dv2 By radiation, cooling under Newton's law,
dT
.. -4PoV +SPo = 0 --k(T-TA) = ... (3)
dt
Vo
.. Total rate of cooling (2) + (3)
or V = SVo • Put this value in (5) to obtain Tmax' _dT = KA(T-TA) +k(T-T )
4 A
dr CL
. Tm,,=M-~ox(5~O)\5Po(5~O)] _dT =(KA+k)(T-TAJ
dr a
or T
max
=..!:.[
R
-2SPoVo +lOXSPoVo]
8 •• - J 350
T dT
(T-T )=
(KA
CL +
k)J r1", dr
A

or T max = 2SP
8 R
OVo
... (6) or -[In(T _ TA)]T
350
= (KA + k)[t]3r
CL
1

r1

33. In first stage, the solid body' X cools according to or In(350-300)=(KA+k)(2t)


Newton's law of cooling. Temperature falls from 400 K T-300 CL 1
to 350 K.
In second stage, the body X cools by conduction as
or In(
T-300
50 )= 2lG4tl + 2ktl
CL
well as by radiation under Newton's law.
or In( 50 ) = 2lG4t} .+- 2In2 [Fromeq. (1)]
TA""'JOOK TA"'JOOK T-300 CL
_ 21n 2 = 2K.4.tl
I BodY.\OO K I I X
BOc.ly. J50K I Rod
L,A, K
Y
80'
or In 50
T-300 CL
1st slage lind stage or In 50 = 2KAtl
(T-300)x4 CL
2KAl:1 -2lV\t1
Stage I: Cooling from time t = 0 to t = £}: or 12.5 = ea. or T _ 300 = 125e a
According to Newton's law of cooling. (T - 300)

- dT
dr
= k(T - TA). where k = cooling constant
or T = [300 + 12 .5. -~r, ] K

www.puucho.com
Study Anurag Mishra with www.puucho.com

317

34. (a) P.V diagram of the process:


AB denotes the adiabatic curve of compression.
or Q_ 3'1 "\
2
[1_(V1 )"3] V2
= 2R [Tc _ 'I
2'3
VI]
IIR
Coordinates of A =: (VI ,11.)
Coordinates
P
of B = (V2,P2)
c
"' .9.- _ 'I"\
3R 2R
[1- (.:i)23] =
V2
Tc _ 'I"\ [.,'n
2R
= 2]

11 Vi (.:i)23
Q
P2 ••• ------- B or Te =- ~ +
,, 3R 2R V2
,,
,
,,
Q P v:53v:-23
or Final temperature =-_.... 1 1 2
PI •• _ ••
.
~
,
•••• A
3R 2R
35. Number of mole, n =-2
:, .
: V
TI = 300K, gas is monoatomic.
V2 VI ABO\ is a cyclic operation,
HeatQ is given at B, at constant volume v2. Be denotes p
this process. The complete process is sketched on the B
p-v diagram. 2PI •••••••• _., c
(b) (i) Total work done W:
W =W
AB
+ WBC or W = 11VI -P2V2 +zero
(y -I) PI •••......•. __ •••••. A

w = 11VI -PZV2 ... (1)


(y -I)
T
By adiabatic law along AB,
TI 2T1
P2 = 11 V.
_1 )' where y = -5 for monoatomic gas. (a) Work done in process AB.
( V 3
2 '.' PV =-nRT, for a gas,

W _I [1\"\-'1("\)' V2]
=
(y - I) V2
... PdV + VdP = nRdT
or pdV + (n~T)dP =- nRdT

W - 11VI
(y-l) [ 1
( )'( )]
VI
V2
V2
VI
or pdv + nRTdP
P
=- nRdT
• PT =-constam, as given in the question
... (1)

0' W = 3P~VI [1-(~: r] ...(2)


.. PdT + TdP = 0
.. :rom eq. (1) and (2) PdV:= 2nRdT
... (2)

., WA~B =-JPdV
(ij) Total change in internal energy: ,1U
... (3)
JiB
.. W (by gas) =- , 2nRdT =-2nR(TB -TA)
tiU = LWAB +l'iUBC
',' AB is an. adiabatic process, :. I'iQ = O. or W (by gas) = 2 x 2 x R(300 - 600)
.. O=I'iUAB +I'iW or /'i.UAB =-/'iW ...(4) or W (by gas) -1200R =
Again, Be is an isochoric process,:. I'i W = 0 .. Workdoneonthegas=-+1200R
.. Q=LlUBC+O ...(5) (b) Heat absorbed or released in different process.
.. 6.U=-I'iW+Q [Fmmeq. (3), (4), (5)] , 3R 5R 5
For a monoatomlc gas, Cv =--,Cp =--, y =--
or tiU = Q - Work done 2 2 3

0' 6U = Q _ 3P~VI [1- ( ~J


3] ... (6)
Process A ~ B: ilU = nCvilT
or ilU=- (2{3:)(TB -TA)

(iii) Final temperature of gas: Tc: or ilU =-2 x 3R x (300 - 600) or ilU =--900R
ilU =- IIC yilT :c 2(_R_)
)'-1
(Te - TA)
..
2
QA--+B = W A--+B + ilU
= -1200R + (-900)R =- -2100R
= Heat is released in the process:

www.puucho.com
Study Anurag Mishra with www.puucho.com

PRMOUS PRO lEMS 318-


Process B ---+ C: An isobaric process: M2g2/
or U = -~- ...(1)
.. QB-'Oc: nCp.1.T = 2(S:)CTc -TB) 2nr2y
Heat gained by wire = msl'.T
= 2x SR (600- 300) = 1500R Q = (nr2/p)sdT ... (2)
2
M2g2/
= Heat is absorbed in the process Q =U :. nr2/psl'.T = ---
Process C -+ k An isothermal process: 2nr2y
In an isothermal process, I1U = 0 M2g2
or l'.T=----
2(nr2)2ysp
.. QC-+A=WC-+A =nRTcln(:~)
(l00 x 10)2
=
3
2 x [3.14 x (2 x 10- )2]2 x 2.1 x 1011x 420 x 7860
=nR(21i)ln(~ )=2XRX(2X300)(1n2)
or l'.T = 0.00457°C.
= 1200R x 0.6932 = 831.84R 38, (a) Temperature of gas at B: Tn:
= Heat is absorbed in the process
AB' IS a stralg
, h t I'me. -V = constant
36. According to principle of calorimetry, T
Heat lost by container = msdT or dQ = meA + BT)dT V V
.. --!l = ...1!.at constant pressure.
or J~dQ=m!;gg (A+BT)dT TA Tn

.. TB = VB xTA where VB = 2 (given)


or Q=+T+B~'[ VA VA
or TB =2x300-=600K
2 (b) Heat absorbed/released in each process:
or Q = m[lOOT+ 2xlO- T2]300
(i) Process AB. AB is an isobaric process.
2 500
or .. Q=nCpdT or QAB=2X(~)RX300

Q = m[100(300- 500)+ (300)' - (500)'] or QAB= 1500R.


100 Heat is absorbed.
or Q = m [-20000 - 1600] calorie (ii) Process Be. Be is an isothermal process.
or Q = -21600mcalorie ... (1)
Heat gained by ice in melting = mL
Q = 2.303 x nRT 10g10(~~ )

.. !lI =
Ut x 80000 = 8000 cal [.: Q=W+U=W+O=W}
Heat gained by water of above ice msll. T = or QBC = 2.303 x (2) x R x 600 x 10&102
.. =
Q2 = 0.1 x 1000 x 27 2700 cal or QBC = 831.8R.
.. Total heat gained = 8000+ 2700 Heat is absorbed. Volume increases .
Ql +Q2 = 10700 cal ,.. (2) (iii) Process CD. CD is an isochoric process.
" Heat lost = Heat gained or 21600m -=10700 In isochoric process, dw ::: O.
. 10700
.. Mass of contamer m = -- = 0.495 kg
21600
Q=nCvl'.T or QCD=2X(3:)<-300)=-900R.
37. The elastic potential energy of the steel wire is
converted into heat in wire: when the bob gets Heat is released. Volume decreases.
snapped.
(iv) Process DA. DA is an isothermal process.
'
EIasuc potentia, 1 energy m
"I WIre = - ScrV = -_,
S'V
2 2Y .. Q=2.303xnRTXIOglO(~;)
where S = stress = Mg , cr = strain = ~
nr2 y or QVA =2.303X2~.~X300XIOglO(~)
V = volume of wire = nr2l
or QVA = -831.8R. .,
Y = Young's modulus of material of wire
Heat is released.
.. EnergyU=-x I (Mg)2
- x--
2
nr /
2 nr2 Y

www.puucho.com
Study Anurag Mishra with www.puucho.com

319 -==JTEHEmRM!O~Di!lYN!!AMilIlCsg
(e) Work done in complete cycle: (1) ~ (2), .. KA(To -Til - 595A
work done = QAB + QBC + QeD + QDA thickness (t)
= 1500R + 831.8R - 900R - 831.8R = 600R or T _TI=595xl
o
39. (a) Temperature of helium gas: K
2
Time for one hit on a wall = _1_ ,eo or T _ 400 = 595 x (0.5 x 10- )
500 a 0.149
. distance or To - 400 = 20 or To = 420 K.
:. Velocity of atom = --.~-
time
Onward + Backward journeys 42. When pressure is kept constant, T1 = T2
Distance = VI V2
=1+1=2m
2x SOD Volume = Area x height = Ah
= -- = lOOOm/s
.• vrm.>
1 ,. -2L =~ or lIZ = T2h1
Ah1 Ah2 T1
BY k.meuc. t heory, vrms = ~3RT
-- .'. \13RT
M
= 1000
.... or h _400xl_~m
M
2- 300 3
T = 1000 x 1000 x M
or The process is adiabatic. When the gas is compressed
3xR
without exchange of heat,

:. ;~=(~:r- T'=400(~r'S
T= lOOOxlOOOx4xlO~3 x3 1
or
3 x 25 :. K
or T=160K
(b) Average kinetic energy per atom of helium: 43 . When the temperature is increased by !!T, the volume
of the cube increases but the density of liquid
.. K.E.=~kT arK.E. =~x(1.38xl0-23)(l60)J
2 2 decreases, The depth upto which the cube is
or K.E.= 3.312 x 10-21 J submerged in the liquid remains the same. Obviously
(e) Total mass of helium in the box. PV = nRT
the upthrust remains the same.
Initial upthrust = Final upthrust
or PV= m R'I' or m= PVM
M RT Vpg = V'p"g
(100)(1)(4) 3 where V,V' = volume of cubical body immersed
or m=----=- or m=o.3g
p,p' = densities of liquid at tvo/Otemperatures
25 x160 10
3 (Ah)p = IA(I + 2n,"T)h]( p )
40. Decrease in kinetic energy I+YI!!T
= Increase in internal energy or (l + y,!!T) = 1+ 'liJ..sfiT
or ~mv6=nCv~T or ~mv6=(:)(3:)~T or y,fiT = 'liJ..s!!T or Yl = 'liJ..s.
44. The lateral surface of the cylindrical rod does not lose
2 heat either by conduction or by radiation, Energy is
or liT = Muo
3R
radiated by the circular cross section at temperature
T2. Heat flows by conduction along the rod.
41. (a) The rate of heat loss per unit area due to radiation
from the lid is given by Stefan's law. .'. Heat conducted Q = KA(T1 - T2 )tJ.r
I
.. E = eo(Tl4 - T04)
Heat radiated E = eoA(T24 - Ts4)M
= (Q6)e: x 10-8jr(400l4 - (300)4] For equilibrium, E = Q
~4 T4). _ KA(1} -T2)tJ.r
or ea:A\' 2 - s o.t - -~-~-
8 x 108[256- I
= O.6x 17 x 10- 81] = 595 w/m2
or (Ii _T2)=eO(Tz4-Ts4)l
3
K
(b) Temperarure of oil: To:
or T) - (Ts + fiT) = eol [(Ts + fiT)4 - Ts4]
Rate ofheat-conduetion= KA (To - TI) ... (1) K
t
Rate of heat-radiation = 595 x area of lid-top
= 595A ... (2)
or Tt-Ts-!!T=KTs eol 4[( l+r:;
"T)4 -1 1

www.puucho.com
Study Anurag Mishra with www.puucho.com

--- or 1i-Ts ealT' [ 1+---1


=--'-
K
4~T
T,
] +t1T .. 'li + Q2 = Heat supplied
400(T - 20) + O.OOl(T - 20) = 20000
or 400.00l(T - 20) = 20000
4ealTs3 tJ.T T
or T}- Ts =----+~
K or T-20= 20000 ""50
400.001
or 1j -T, =[ 4e~T,3 +1]~T or T-SQ+20 or T=70°C
(b) Work done by the cylinder:
...(3)

3 W =Q, = 0.0.0.1(70.-20.) =0.0.5 J


.. !lropol1ionality constant = 1 + 4ecrlT.
K ee} Change in internal energy
45. Rate of heat produced ~u= 20.000. - 0..0.5= 19999.95 J
= (viscous force F) x (velocity v) 47. When steam at 373 K is mixed with ice at 253 K, steam
dQ , loses heat and gets condensed into water at 373 K.
or dt = (61tl1rv)(v) = 6m"'1J While ice gains heat to rise from 253 K to 273 K and
then to melt in water at 273 K.
CD Heat lost by steam during condensation = msLv
= 61tll{~ (0 - p)r2g]' = (fir[ x 4g2 x (0 - p)2 ) ,,5 :. C2I = msLv or C2I = (0.05) x (2268 x 1000)J
9 11 81xll or !2J = 113400J
::: (constant) x rS or Rate of heating oc: ,5 (ii) Heat lost by water of steam to cool to 273 K
Hence the rate of heating is proportional to fifth power = mssw9
of radius of body when it attains terminal velocity. :. Q, = (0..0.5) x (420.0.) x (lo.o.) = 21o.o.OJ
46. (a) Heat supplied to the cylinder is used up in (iii) Total heat available from steam = !2J + Q2
(0 raising temperature of cylinder = msliT :. Q=113400+21000 orQ=134400J
(ii) doing work = Pann (V2 -\1) (iv) Heat gained by ice at 253 K = Msice6T
(i) <lI =
ms~T :. Q', = (0.45) x (2100.) x (273 - 253) = 18900. J
or <lI = 1 x 400(T - 200) ... (1) (v) Heat gained by ice during melting = MLF
(ii) Work done = Patm x (Vz - Vi) :. Q', = (0..45) x (336 x 100.0.) = 15120.0. J
Q2 = 105 X ('1 'fl!.T) (vi) Total heat required by ice = Q'l +Q' 2
.. et= 18900+ 151200 or etl = 170100J
Qz = 105 x ma~s x y x (T - 20) .. Q' > Q, whole of the ice will not melt.
denSity
.. Some ice at 273 K remains unmelted.
Q2 = 105 x_1_X9xlO-S x(T-20) .. Equilibrium temperature of mixture
9000
=273 K=O°C.
Q, = 0.001 x (T - 20) ... (2)

www.puucho.com
Study Anurag Mishra with www.puucho.com

www.puucho.com
Study Anurag Mishra with www.puucho.com

www.puucho.com

You might also like